Вы находитесь на странице: 1из 92

TOPNOTCH MEDICAL BOARD PREP BIOCHEMISTRY SUPEREXAM

For inquiries visit www.topnotchboardprep.com.ph or email us at topnotchmedicalboardprep@gmail.com


DEAR TOPNOTCH FRIENDS:

PLEASE FOLLOW THESE INSTRUCTIONS:

1. These questions are previous diagnostic, midterm, and finals exams of Topnotch, almost all of them made by Topnotch Board Exam Topnotchers.
2. Answer this Topnotch Superexam seriously 100-items at a time. Cover the “Explanations” Column. Do not immediately look at the answers from the
answer key. That’s not the correct way of answering sample exams. You need to treat these MCQs as exercises and not as handouts.
3. Time yourself. 1.5 hours per 100-item block.
4. After answering each 100-item block, refer to the Topnotch Answer Key for the correct answers. Please be careful of “frameshift mutations” when
checking your answers – check every 10 items. (the format of the answer key was designed for you to practice against “frameshift mutations”)
5. The Topnotch Superexams are EXERCISES for the actual med boards. They will not appear verbatim in your future exams. More than knowing what’s
the correct answer, it’s more important for you to:
a. Know why the other choices are wrong
b. Know why the other choices were included in the first place
c. Know the explanation to the correct answer
6. Sharpen your mind by answering the Topnotch Superexams. Most of these questions based on past feedback are more difficult than the actual questions
in the med boards. In these exams made by Board Exam Topnotchers, if you’re getting a score of 60/100 , that’s already a good score. More than 80/100
is outstanding.


Item QUESTION EXPLANATION AUTHOR TOPNOTCH
# EXAM
1 The pH at which majority of amino acid exists in a form A zwitterion is a molecule with a positive and a KRISTEL DIAGNOSTIC
where it has an equal number of positive and negative negative charge in different locations yielding to a net TANHUI EXAM -
charges and thus is electrically neutral is called the: charge of zero for the whole molecule. The pI, or the (TOP 3 - MARCH 2016
A. pK1 isoelectric pH is the pH at which this AUG 2015
B. pK2 isoelectric/zwitterion form of the amino acid MED
C. pK1 + pK2 predominates. Source: Harper 27th ed p.17 BOARDS;
D. pIE. pH TOPNOTCH
MD FROM
LA SALLE)
2 Glutathione is an important molecule for the clearance of Remember the 3 amino acids which make up KRISTEL DIAGNOSTIC
free radicals in the body. It consists of only 3 amino Glutathione. (Similar principle/concept to previous TANHUI EXAM -
acids. Which of the following polypeptides is the primary exam questions) The most important amino acid is (TOP 3 - MARCH 2016
structure of glutathione? cysteine which contains the active -SH group. Source: AUG 2015
A. Glutamyl – cysteinyl – glycine Harper 27th ed p. 19 MED
B. Glutamine – cysteinyl – glycine BOARDS;
C. Glutamine – threonyl – methionine TOPNOTCH
D. Glutamine – cysteinyl – methionine MD FROM
E. None of the above LA SALLE)

3 Which of the following amino acid is incorrectly paired Tyrosine does not contain a thiol group. Tyrosine KRISTEL DIAGNOSTIC
with its side chain? contains a phenol group. Source: Harper 27th ed p. 18 TANHUI EXAM -
A. Tyrosine – Thiol group containing (TOP 3 - MARCH 2016
B. Tryptophan – indole group containing AUG 2015
C. Proline – imino group containing MED
D. Arginine – guandino group containing BOARDS;
E. Histidine – imidazole group containing TOPNOTCH
MD FROM
LA SALLE)
4 A neonate, born at home, with no newborn care This is a classic case of hemorrhagic disease of the KRISTEL DIAGNOSTIC
administered is rushed to the ER for heavy bleeding newborn. They can present with seizures for TANHUI EXAM -
from the umbilical stump and recent onset generalized intracranial hemorrhage. Source: Harper 27th ed p (TOP 3 - MARCH 2016
seizures. What vitamin deficiency is responsible for this? 495 AUG 2015
A. Vitamin A MED
B. Vitamin D BOARDS;
C. Vitamin E TOPNOTCH
D. Vitamin K MD FROM
E. Vitamin C LA SALLE)

5 What is the rate limiting enzyme of ketogenesis? HMG CoA reductase is the rate limiting enzyme for KRISTEL DIAGNOSTIC
A. HMG-CoA synthase cholesterol synthesis. Source: Harper 27th ed p 191 TANHUI EXAM -
B. HMG-CoA reductase (TOP 3 - MARCH 2016
C. HMG-CoA lyase AUG 2015
D. HMG-CoA transferase MED
E. All of the above BOARDS;
TOPNOTCH
MD FROM
LA SALLE)
6 An infant presents with hepatosplenomegaly and failure Glycogen storage diseases: enzyme deficienct: signs KRISTEL DIAGNOSTIC
to thrive. A liver biopsy was done which revealed and symptoms/buzzwords TANHUI EXAM -
accumulation of polysaccharide with few branch points (TOP 3 - MARCH 2016
because of a deficiency for an important enzyme in - Von Gierke’s disease: Glucose 6 phosphatase AUG 2015
carbohydrate metabolism. What is the diagnosis for this deficiency: severe hypoglycemia (because glucose MED
patient? generated from all pathways (gluconeogenesis and BOARDS;
A. Von Gierke Disease glycogenolysis) is trapped in the hepatocyte and TOPNOTCH
B. Andersen’s Disease cannot be released into the blood) MD FROM
C. Tarui’s Disease - Pompe’s disease: acid maltase: heart failure LA SALLE)
D. Fabry’s Disease - Cori’s disease: debranching enzyme: accumulation of
E. Her’s Disease branched polysaccharide
- Anderson’s disease: branching enzyme:
accumulation of polysaccharide with few branch
points
- McArdle syndrome: muscle phosphorylase: poor
exercise tolerance, accumulation of muscle glycogen
TOPNOTCH MEDICAL BOARD PREP BIOCHEMISTRY SUPEREXAM Page 1 of 92
For inquiries visit www.topnotchboardprep.com.ph or email us at topnotchmedicalboardprep@gmail.com
TOPNOTCH MEDICAL BOARD PREP BIOCHEMISTRY SUPEREXAM
For inquiries visit www.topnotchboardprep.com.ph or email us at topnotchmedicalboardprep@gmail.com
Item QUESTION EXPLANATION AUTHOR TOPNOTCH
# EXAM
- Her’s disease: liver phosphorylase: accumulation of
liver glycogen, mild hypoglycemia (because only
glycogenolysis is impaired, gluconeogenesis can
replenish glucose)
Source: Harper 27th ed p. 166

7 A patient who presented with mild jaundice had work up Try to master direct and indirect hyperbilirubinemia. KRISTEL DIAGNOSTIC
done, which revealed increased direct bilirubin, absent It’s a favorite theme and there are many question TANHUI EXAM -
urine urobilinogen, (+) urine bilirubin. Liver biopsy variations that can be done across many subjects. (TOP 3 - MARCH 2016
showed liver hyperpigmentation. Which of the following AUG 2015
is the most likely diagnosis? This is a case of conjugated hyperbilirubinemia (note MED
A. Crigler- Najjar syndrome sometimes, the examinar can be unforgiving and may BOARDS;
B. Gilbert syndrome not put increased direct bilirubin so be familiar with TOPNOTCH
C. Dubin Johnson syndrome other lab findings too). Rotor syndrome and dubin MD FROM
D. Rotor syndrome Johnson syndrome present similarly except dubin LA SALLE)
E. Hepatitis Johnson has liver hyperpigmentation (dubin – dark
liver)

Conjugated hyperbil – increased direct serum
bilirubin, absent urine urobilinogen, (+) urine
bilirubin
Ex. Obstructive jaundice, dubin Johnson, Rotor
syndrome

Unconjugated – increased indirect serum bilirubin, (+)
urine urobilinogen, (-) urine bilirubin
Ex. Crigler-Najjar, Gilbert

Source: Harper 27th ed p292
8 If 20% of DNA consists Guanosine nucleotides, how According to Chargaff’s rule, the concentration of A KRISTEL DIAGNOSTIC
many percent are thymidine nucleotides? nucleotides equals T nucleotides while C nucleotides TANHUI EXAM -
A. 20% equals G nucleotides. (TOP 3 - MARCH 2016
B. 30% AUG 2015
C. 40% 100% - 20% (G) – 20%(C) = 60% (A+T) MED
D. 50% 60% / 2 = 30% BOARDS;
E. Cannot be identified from given information TOPNOTCH
MD FROM
LA SALLE)

9 Covalent modification is one of the most rapid ways of Acetylation and methylation are covalent KRISTEL DIAGNOSTIC
regulating enzyme activity in the cell. During fasting modifications on histones to activate or inactivate TANHUI EXAM -
which of the following is expected to occur? transcription. Metabolic enzymes are regulated via (TOP 3 - MARCH 2016
A. Phosphorylation of Glucose-6-phosphatase phosphorylation or dephosphorylation.Glucagon AUG 2015
B. Dephosphorylation of Glycogen synthase phosphorylates, insulin dephosphorylates. (My MED
C. Acetylation mnemonic is DIG-P) This results to either activation or BOARDS;
D. Methylation inactivation of particular enzymes in certain TOPNOTCH
E. None of the above pathways. MD FROM
Make sure you master this concept and which LA SALLE)
enzyme/pathways are activated or inactivated by
insulin and glucagon. They can easily ask 5-10
different varieties of these sort of questions.
10 After a meal, insulin levels in the blood become elevated Glucagon phosphorylates, insulin dephosphorylates. KRISTEL DIAGNOSTIC
to effect which of the following changes in enzyme (My mnemonic is DIG-P) This results to either TANHUI EXAM -
activity? activation or inactivation of particular enzymes in (TOP 3 - MARCH 2016
A. Glucokinase is dephosphorylated and inactivated certain pathways. AUG 2015
B. Glycogen synthase is dephosphorylated and activated Make sure you master this concept and which MED
C. HMG CoA synthase is phosphorylated and activated enzyme/pathways are activated or inactivated by BOARDS;
D. Acetyl CoA carboxylase is phosphorylated and insulin and glucagon. They can easily ask 5-10 TOPNOTCH
inactivated different varieties of these sort of questions. MD FROM
E. A and B LA SALLE)

11 Which is the main source of NADPH for Lipogenesis? NADPH is utilized in many pathways in the cell such as KRISTEL DIAGNOSTIC
A. Pentose Phosphate Pathway lipogenesis, nucleic acid metabolism, and even in the TANHUI EXAM -
B. Krebs Cycle pathways utilized by the innate immunity. There are 2 (TOP 3 - MARCH 2016
C. Urea Cycle sources of NADPH – the Pentose phosphate pathway AUG 2015
D. Gluconeogenesis and the malic enzyme. The PPP is most significant. MED
E. A and C Source: Harper 27th ed p199 BOARDS;
TOPNOTCH
MD FROM
LA SALLE)
12 Cyanide inhibits which of the following complex of the Inhibitors: KRISTEL DIAGNOSTIC
electron transport chain? Complex I – Barbiturates TANHUI EXAM -
A. NADH Q oxidoreductase Complex II – Malonate (TOP 3 - MARCH 2016
B. Q cytochrome c oxidoreductase Complex III – Antimycin A and dimercaprol AUG 2015
C. Cytochrome C oxidase Complex IV – H2S, Cyanide, CO MED
D. Succinate Q reductase BOARDS;
E. None of the above Source: Harper 27th ed p107 TOPNOTCH
MD FROM
LA SALLE)

TOPNOTCH MEDICAL BOARD PREP BIOCHEMISTRY SUPEREXAM Page 2 of 92


For inquiries visit www.topnotchboardprep.com.ph or email us at topnotchmedicalboardprep@gmail.com
TOPNOTCH MEDICAL BOARD PREP BIOCHEMISTRY SUPEREXAM
For inquiries visit www.topnotchboardprep.com.ph or email us at topnotchmedicalboardprep@gmail.com
Item QUESTION EXPLANATION AUTHOR TOPNOTCH
# EXAM
13 Which of the following is not an essential fatty acid? Both are essential FA. KRISTEL DIAGNOSTIC
A. Linoleate TANHUI EXAM -
B. Alpha-linolenate Source: Harper 27th ed p204 (TOP 3 - MARCH 2016
C. Both AUG 2015
D. Neither MED
E. Fatty acids are not essential in diet and can be BOARDS;
synthesized from carbohydrates and proteins TOPNOTCH
MD FROM
LA SALLE)

14 Starch is a polymer of which of the following? Source: Harper 27th ed 116 KRISTEL DIAGNOSTIC
A. Glucose TANHUI EXAM -
B. Galactose (TOP 3 - MARCH 2016
C. Fructose AUG 2015
D. Tryptophan MED
E. Cellulose BOARDS;
TOPNOTCH
MD FROM
LA SALLE)
15 If the enthalpy change is negative and the entropy Thermodynamics is still chemistry! Don’t sacrifice this KRISTEL DIAGNOSTIC
change is positive in a particular reaction, will this topic, they can still ask a few questions on this. Know TANHUI EXAM -
proceed spontaneously? when a reaction is spontaneous, the definitions of (TOP 3 - MARCH 2016
A. Yes, always entropy and enthalpy and the basic laws of AUG 2015
B. No, always thermodynamics. Source: Topnotch handout in MED
C. Maybe, but only at high temp biochemistry BOARDS;
D. Maybe, but only at low temp TOPNOTCH
E. Cannot be determined with the given information MD FROM
LA SALLE)

16 A 6 month old infant presented with hyperacusis Be familiar with the clinical presentation as well as KRISTEL DIAGNOSTIC
(exaggerated startle response), cherry red spot on the enzyme deficiency and substance which accumulates TANHUI EXAM -
macula and froglike position. What substance is expected for lysosomal storage diseases. (favorite cases are (TOP 3 - MARCH 2016
to accumulate in the brain? taysach’s, neimann pick, gaucher’s) They can ask AUG 2015
A. GM2 Ganglioside similar 2 or 3 step analysis types of question in MED
B. Hexoseaminidase A biochemistry or pediatrics/IM or even patho. BOARDS;
C. Sphingomyelin TOPNOTCH
D. Glycosphingolipids This is a case of Tay Sach’s. The enzyme deficient is MD FROM
E. No accumulation Hexoseaminidase A and the substance GM2 LA SALLE)
ganglioside accumulates in the brain. Note the
buzzwords - exaggerated startle response and cherry
red spot in the macula. They do not present with
hepatomegaly.

Source: Harper 27th ed p.215
Clinical case adapted from first aid for the pediatrics
clerkship 2nd ed.
17 Patient presents with arthritis and connective tissue This is a case of alkaptonuria. KRISTEL DIAGNOSTIC
pigmentation. On letting his urine stand, his urine is TANHUI EXAM -
noted to darken. Which enzyme is deficient in this Source: Harper 27th ed p 259 (TOP 3 - MARCH 2016
patient? AUG 2015
A. Tyrosine aminotransferase MED
B. Phenylalanine hydroxylase BOARDS;
C. Homogentisate oxidase TOPNOTCH
D. Branched chain ketoacid dehydrogenase MD FROM
E. A and C LA SALLE)

18 The following is ATP, Biotin and CO2 requiring: These 3 enzymes need 3 things to function (ATP, KRISTEL DIAGNOSTIC
A. Pyruvate dehydrogenase Biotin, CO2): Acetyl CoA carboxylase, propionyl TANHUI EXAM -
B. Acetyl CoA Carboxylase carboxylase, pyruvate carboxylase. (TOP 3 - MARCH 2016
C. HMG CoA reductase My mnemonic is the ABC enzymes. AUG 2015
D. SGPT MED
E. All of the above Source: Topnotch lecture BOARDS;
TOPNOTCH
MD FROM
LA SALLE)
19 Patient presents with urine smelling of maple syrup or The patient is deficient of branched chain ketoacid KRISTEL DIAGNOSTIC
burnt sugar and is diagnosed with maple syrup urine dehydrogenase. TANHUI EXAM -
disease. Which of the following is not necessary for the (TOP 3 - MARCH 2016
function of the enzyme deficient in this disease? BCKD, like pyruvate dehydrogenase and succinate AUG 2015
A. Thiamine dehydrogenase require the following for function: MED
B. Lipoic acid Thiamine, Lipoic acid, CoA, FAD, NAD BOARDS;
C. FAD TOPNOTCH
D. Biotin Familiarize yourself with these 3 enzymes and the 5 MD FROM
E. None of the above components required for their function. It’s also a LA SALLE)
popular question.
Source: Topnotch lecture

TOPNOTCH MEDICAL BOARD PREP BIOCHEMISTRY SUPEREXAM Page 3 of 92


For inquiries visit www.topnotchboardprep.com.ph or email us at topnotchmedicalboardprep@gmail.com
TOPNOTCH MEDICAL BOARD PREP BIOCHEMISTRY SUPEREXAM
For inquiries visit www.topnotchboardprep.com.ph or email us at topnotchmedicalboardprep@gmail.com
Item QUESTION EXPLANATION AUTHOR TOPNOTCH
# EXAM
20 Patient has an autosomal recessive genetic disease This is a case of xeroderma pigmentosum. KRISTEL DIAGNOSTIC
which makes him markedly susceptible to sunlight with TANHUI EXAM -
subsequent formation of multiple skin cancers and Source: Harper 27th ed p345 (TOP 3 - MARCH 2016
premature death. The inherited defect involves repair of AUG 2015
damaged DNA, particularly thymine dimers. Cells MED
cultured from the patient has low activity of which of the BOARDS;
following DNA repair mechanisms? TOPNOTCH
A. Mismatch repair MD FROM
B. Base excision repair LA SALLE)
C. Nucleotide excision repair
D. Double strand break repair
E. All of the above

21 Chemical added to natural gas that confers its distinctive SIMILAR TO BOARD EXAM CONCEPT. Natural gas is LESTER MIDTERM 1
odor? odorless. To detect leaks, mercaptan is added to the BRYAN CO EXAM -
A. Sulfur gas to make it smell like rotten eggs. (TOP 10 - MARCH 2016
B. Chlorine AUG 2015
C. Methane MED
D. Mercaptan BOARDS;
E. Phenol TOPNOTCH
MD FROM
UST)
22 Marize did not take any food for eight hours in SIMILAR TO BOARD EXAM CONCEPT. Fasting LESTER MIDTERM 1
preparation for a lipid profile test. Which of the increases the glucagon:insulin ratio. Glucagon acts via BRYAN CO EXAM -
following is likely to be occuring inside her cells right a G-protein coupled receptor whose alpha subunit (TOP 10 - MARCH 2016
now? adenylate cyclase, increasing intracellular levels of AUG 2015
A. Glycogen phosphorylase b is unphosphorylated. cAMP. cAMP activates protein kinase A. This enzyme, MED
B. cAMP levels are increased. in turn, activates phosphorylase kinase, which then BOARDS;
C. Pyruvate is converted to lactate. phosphorylates glycogen phosphorylase b, converting TOPNOTCH
D. PFK-2 is unphosphorylated. it into the active form called phosphorylase a. Protein MD FROM
E. Fructose 2-6 bisphosphatase is unphosphorylated. kinase A will also phosphorylate the enzymes UST)
fructose-2,6-bisphosphatase and
phosphofructokinase-2. This covalent
phosphorylation initiated by glucagon activates the
former and inhibits the latter. Conversion of pyruvate
in lactate happens in anaerobic metabolism, which
does not necessarily occur only in fasting. There were
about 10 questions involving insulin, glucagon,
feeding and fasting in Biochemistry during our
Biochemistry exam
23 Ryan experiences bloating and abdominal cramps SIMILAR TO BOARD EXAM CONCEPT. Lactase LESTER MIDTERM 1
whenever he eats dairy-containing products. Which of intolerence is caused by a deficiency of lactase (a BRYAN CO EXAM -
the following is true about his condition? disaccharidase) in the intestinal brush border. Lactose (TOP 10 - MARCH 2016
A. Galactose accumulate in his small intestine. is unable to be broken down into glucose and AUG 2015
B. Increased levels of hydrogen may be detected in his galactose, and thus accumulates in the gut. It is MED
breath. eventually fermented by bacteria to form hydrogen, BOARDS;
C. Stool pH is increased. methane and carbon dioxide, leading to various TOPNOTCH
D. There is a deficiency of a specific disaccharide from abdominal symptoms. 3 questions pertaining to MD FROM
the brush border. lactase deficiency were asked during our UST)
E. Administration of lactulose will lead to improvement Biochemistry exam
of his symptoms.
24 Which of the following amino acids contain a phenol SIMILAR TO BOARD EXAM CONCEPT. Aromatic amino LESTER MIDTERM 1
ring? acids: Phenylalanine - benzene ring; Histidine - BRYAN CO EXAM -
A. Tyrosine Imidazole ring; Tryptophan - Indole ring; Proline (TOP 10 - MARCH 2016
B. Phenylalanine contains a ring structure, but is not classified as an AUG 2015
C. Histidine aromatic amino acid MED
D. Tryptophan BOARDS;
E. Proline TOPNOTCH
MD FROM
UST)
25 The term forensic pertains to: SIMILAR TO BOARD EXAM CONCEPT. This seems to LESTER MIDTERM 1
A. Crime be the only none-Biochemistry related question asked BRYAN CO EXAM -
B. Investigatory during our Board Examination in Biochemistry. (TOP 10 - MARCH 2016
C. Autopsy AUG 2015
D. Law MED
E. Criminal BOARDS;
TOPNOTCH
MD FROM
UST)
26 The following electrophoresis technique is used in the For the different blotting techniques, remember the LESTER MIDTERM 1
study of DNA-binding proteins: mnemonic SNoW DRoP: Southern - DNA, Northern - BRYAN CO EXAM -
A. Northern blot RNA, Western - Protein (TOP 10 - MARCH 2016
B. Western blot AUG 2015
C. Southern blot MED
D. Western blot BOARDS;
E. Southwestern blot TOPNOTCH
MD FROM
UST)

TOPNOTCH MEDICAL BOARD PREP BIOCHEMISTRY SUPEREXAM Page 4 of 92


For inquiries visit www.topnotchboardprep.com.ph or email us at topnotchmedicalboardprep@gmail.com
TOPNOTCH MEDICAL BOARD PREP BIOCHEMISTRY SUPEREXAM
For inquiries visit www.topnotchboardprep.com.ph or email us at topnotchmedicalboardprep@gmail.com
Item QUESTION EXPLANATION AUTHOR TOPNOTCH
# EXAM
27 Characteristic of the genetic code that can account for Non-ambiguous/specific - a certain codon codes LESTER MIDTERM 1
silent mutations? always codes for the same amino acid; Universal - BRYAN CO EXAM -
A. Non-ambiguous codons always code for same amino acid from (TOP 10 - MARCH 2016
B. Universal generation to generation and between species; AUG 2015
C. Non-overlapping Redundant/degenerate - A given amino acid may have MED
D. Degenerate more than one triplet codon coding for it; Non- BOARDS;
E. Commaless overlapping/commaless - DNA is read from a starting TOPNOTCH
point as a continuous sequence of bases, taken three MD FROM
at a time without any gaps UST)
28 Which of the following is a nucleoside? SIMILAR TO BOARD EXAM CONCEPT. LESTER MIDTERM 1
A. Hypoxanthine Bases/Nucleobases: Nitrogenous bases = guanine, BRYAN CO EXAM -
B. Xanthine adenine, thymine, uracil, cytosine, xanthine, (TOP 10 - MARCH 2016
C. Inosine hypoxanthine; Nucleosides: base+sugar = guanosine, AUG 2015
D. cAMP adenosine, thymidine, cytidine, uridine, inosine; MED
E. Adenine Nucleotides: base+sugar+phosphate = ATP, GMP, UDO, BOARDS;
cAMP. 3 similar questions were asked during our TOPNOTCH
Biochemistry exam MD FROM
UST)
29 What is the rate-limiting step of ketogenesis? Rate limiting steps: Urea cycle - Carbamoyl phosphate LESTER MIDTERM 1
A. Carbamoyl phosphate synthetase I synthetase I; De novo pyrimidine synthesis - BRYAN CO EXAM -
B. Carbamoyl phosphate synthetase II Carbamoyl phosphate synthetase II; Fatty acid (TOP 10 - MARCH 2016
C. Acetyl CoA carboxylase synthesis - Acetyl-CoA carboxylase. Only 1 rate- AUG 2015
D. Acetyl CoA decarboxylase limiting enzyme type question appeared during our MED
E. HMG CoA synthase Biochemistry exam BOARDS;
TOPNOTCH
MD FROM
UST)
30 Alex is a thin 24-year old male who began avidly taking 2 B3 - niacin; B5 - pantothenic acid; B6 - pyridoxine; B7 LESTER MIDTERM 1
eggs every morning in order to build up his muscle bulk. - biotin; B9 - folic acid BRYAN CO EXAM -
This might put him at risk of a deficiency of which of the (TOP 10 - MARCH 2016
following vitamin? AUG 2015
A. B3 MED
B. B5 BOARDS;
C. B6 TOPNOTCH
D. B7 MD FROM
E. B9 UST)

31 Serum levels of which ketogenic amino acid are I - Isoleucine; L- Leucine; K - Lysine; V - Valine; Y - LESTER MIDTERM 1
increased in Maple Syrup Urine Disease? Tryptophan BRYAN CO EXAM -
A. I (TOP 10 - MARCH 2016
B. L AUG 2015
C. K MED
D. V BOARDS;
E. Y TOPNOTCH
MD FROM
UST)
32 True of alpha-thalassemia: In alpha thalassemia, one or more genes involved in LESTER MIDTERM 1
A. Beta chain accumulates and precipitates. alpha-globin production are defective, leading to BRYAN CO EXAM -
B. Gamma chain production is decreased. relative excess of beta globin and gamma globin (TOP 10 - MARCH 2016
C. It is transmitted in an autosomal dominant fashion. chains. Despite being a structural disease, it is AUG 2015
D. Hemoglobin C disease results if 3 alleles are affected. transmitted in a autosomal recessive manner. MED
E. Definitive diagnosis may be done through Hemoglobin H (beta globin tetramers) may result BOARDS;
karyotyping. from 3 deleted alleles. Hemoglobinopathies are TOPNOTCH
usually diagnosed by hemoglobin electrophoresis. MD FROM
UST)
33 Lipase whose activity is increased by glucagon? Pancreatic lipase - activated by trypsin; Lipoprotein LESTER MIDTERM 1
A. Pancreatic lipase lipase - activated by insulin; Hepatic lipase acts on the BRYAN CO EXAM -
B. Lipoprotein lipase remaining lipids carried on lipoproteins in the blood (TOP 10 - MARCH 2016
C. Hormone-sensitive lipase to regenerate LDL. AUG 2015
D. Hepatic lipase MED
E. None of the above BOARDS;
TOPNOTCH
MD FROM
UST)
34 Which of the following is a omega 3 fatty acid? The other choices are omega-6 fatty acids. LESTER MIDTERM 1
A. Linoleic acid BRYAN CO EXAM -
B. Linolenic acid (TOP 10 - MARCH 2016
C. Arachidonic acid AUG 2015
D. Oleic acid MED
E. All of the above BOARDS;
TOPNOTCH
MD FROM
UST)
35 A man manifests with short stature. His wife has normal The disease described fits the profile of Vitamin-D LESTER MIDTERM 1
stature, but all his daughters are short and bow-legged. resistant rickets, which is transmitted in a x-linked BRYAN CO EXAM -
All his sons are normal. The probable mode of dominant fashion. 1 genetic question in our (TOP 10 - MARCH 2016
inheritance of this disease is? Biochemistry exam. AUG 2015
A. Autosomal dominant MED
B. Autosomal recessive BOARDS;
C. Mitochondrial TOPNOTCH
D. X-linked dominant MD FROM
E. X-linked recessive UST)

TOPNOTCH MEDICAL BOARD PREP BIOCHEMISTRY SUPEREXAM Page 5 of 92


For inquiries visit www.topnotchboardprep.com.ph or email us at topnotchmedicalboardprep@gmail.com
TOPNOTCH MEDICAL BOARD PREP BIOCHEMISTRY SUPEREXAM
For inquiries visit www.topnotchboardprep.com.ph or email us at topnotchmedicalboardprep@gmail.com
Item QUESTION EXPLANATION AUTHOR TOPNOTCH
# EXAM
36 The Enzyme Commision and IUBMB classifies enzymes LESTER MIDTERM 1
based on their: BRYAN CO EXAM -
A. Functional groups (TOP 10 - MARCH 2016
B. Reactions catalyzed AUG 2015
C. Tertiary and Quaternary structure MED
D. Metabolic pathways they belong to BOARDS;
E. Locations in the body TOPNOTCH
MD FROM
UST)
37 Which of the following transporter systems utilize GLUT transporters facilitate diffusion, while CFTR use LESTER MIDTERM 1
secondary active transport? the primary active transport mechanism. BRYAN CO EXAM -
A. SGLT-1 transporter (TOP 10 - MARCH 2016
B. GLUT-2 transporter AUG 2015
C. GLUT-4 transporter MED
D. Cystic Fibrosis Transporter Receptor (CFTR) BOARDS;
E. GLUT-5 TOPNOTCH
MD FROM
UST)
38 Alcohol metabolism produces large amounts of NADH NADH shifts oxaloacetate to malate. LESTER MIDTERM 1
which inhibit gluconeogenesis by? BRYAN CO EXAM -
A. shifting the pyruvate-lactate equilibrium towards (TOP 10 - MARCH 2016
lactate AUG 2015
B. Favoring the producion of oxaloacetate from malate MED
C. Inhibiting the electron transport chain BOARDS;
D. All of the above TOPNOTCH
E. None of the above MD FROM
UST)
39 A ganglioside may contain all of the following except:
Glycosphinglipids do not contain phosphate. LESTER MIDTERM 1
A. a ceramide structure
Gangliosides, by definition, contain sialic acid BRYAN CO EXAM -
B. glucose or galactose (TOP 10 - MARCH 2016
C. phosphate AUG 2015
D. Sialic acid MED
E. Sphingosine BOARDS;
TOPNOTCH
MD FROM
UST)
40 Arsenic does not inhibit an enzyme in this metabolic Arsenic inhibits the pyruvate dehydrogenase enzyme LESTER MIDTERM 1
pathway? in the pyruvate decarboxylation pathway (Swanson BRYAN CO EXAM -
A. Pyruvate decarboxylation conversion. It also inhibits alpha-ketoglutarate (TOP 10 - MARCH 2016
B. Citric acid cycle dehydrogenase in the TCA cycle. AUG 2015
C. Oxidative phosphorylation MED
D. Pyruvate dehydrogenase pathway BOARDS;
E. None of the above TOPNOTCH
MD FROM
UST)
41 Which of the following is true? Brain cells prefer glucose as fuel source. Cardiac GEORGE MIDTERM 2
A. Brain cells prefer fatty acid as fuel source muscle prefer fatty acid as fuel source. Adipose cells MICHAEL EXAM -
B. Cardiac muscle prefer glucose as fuel source prefer fatty acid as fuel source. 3/4 to 4/5 of glycogen SOSUAN MARCH 2016
C. Adipose cells prefer ketone bodies as fuel source are stored in skeletal muscles. (TOP 5 -
D. One-half of the glycogen stores is in the muscles AUG 2015
E. NOTA MED
BOARDS;
TOPNOTCH
MD FROM
UST)
42 True 5-Fluorouracil GEORGE MIDTERM 2
A. Causes thymineless death MICHAEL EXAM -
B. Used as a chemotherapeutic agent for colonic SOSUAN MARCH 2016
carcinoma (TOP 5 -
C. Inhibitis thymidylate synthase AUG 2015
D. Both B and C MED
E. AOTA BOARDS;
TOPNOTCH
MD FROM
UST)
43 Tangier's disease is characterized by: Tangier's Disease is a rare genetic condition due to a GEORGE MIDTERM 2
A. Hypo-alphaprotenemia deficiency of HDL. This disease is also known as MICHAEL EXAM -
B. Deficiency of LDL cholesterol familial alpha lipoprotein deficiency. People affected SOSUAN MARCH 2016
C. Affected people have xathomas often have mild hypertrigylceridemia, neuropathy and (TOP 5 -
D. Common genetic condition premature atherosclerosis AUG 2015
E. NOTA MED
BOARDS;
TOPNOTCH
MD FROM
UST)
44 Most potent counter-regulatory hormone that stimulates GEORGE MIDTERM 2
gluconeogenesis MICHAEL EXAM -
A. Insulin SOSUAN MARCH 2016
B. Cortisol (TOP 5 -
C. Growth hormone AUG 2015
D. Epinephrine MED
E. Thyroxine BOARDS;
TOPNOTCH
MD FROM
UST)

TOPNOTCH MEDICAL BOARD PREP BIOCHEMISTRY SUPEREXAM Page 6 of 92


For inquiries visit www.topnotchboardprep.com.ph or email us at topnotchmedicalboardprep@gmail.com
TOPNOTCH MEDICAL BOARD PREP BIOCHEMISTRY SUPEREXAM
For inquiries visit www.topnotchboardprep.com.ph or email us at topnotchmedicalboardprep@gmail.com
Item QUESTION EXPLANATION AUTHOR TOPNOTCH
# EXAM
45 The normal glycogen stores last for how long? GEORGE MIDTERM 2
A. 6 hours MICHAEL EXAM -
B. 18 hours SOSUAN MARCH 2016
C. 26 hours (TOP 5 -
D. 36 hours AUG 2015
E. 48 hours MED
BOARDS;
TOPNOTCH
MD FROM
UST)
46 Gamma-glutamyl-cysteinyl-glycine is Gamma-glutamyl-cysteinyl-glycine is glutathionine. It GEORGE MIDTERM 2
A. A potent anti-oxidant is potent anti-oxidant and needs NADPH from PPP to MICHAEL EXAM -
B. Needs pentose-phosphate pathway to be be regenerated. SOSUAN MARCH 2016
regenerated (TOP 5 -
C. Similar to Vitamin E AUG 2015
D. Both A and B MED
E. AOTA BOARDS;
TOPNOTCH
MD FROM
UST)
47 Final common pathway by which electrons are passed to GEORGE MIDTERM 2
the mitochondria to generate ATP MICHAEL EXAM -
A. Glycolyis SOSUAN MARCH 2016
B. TCA (TOP 5 -
C. ETC AUG 2015
D. B-oxidation MED
E. Uronic pathway BOARDS;
TOPNOTCH
MD FROM
UST)
48 Which of the following is a complex 4 inhibitor Amobarbital - Complex 1 inhibitor, CO - complex 4 GEORGE MIDTERM 2
A. Amobarbital inhibitor, Antimycin A and Dimercaprol - complex 3 MICHAEL EXAM -
B. CO inhibitor. 2,4 dintrophenol - uncoupler SOSUAN MARCH 2016
C. Antimycin A (TOP 5 -
D. Dimercaprol AUG 2015
E. 2,4 dintrophenol MED
BOARDS;
TOPNOTCH
MD FROM
UST)
49 True of Central Dogma of Molecular Biology GEORGE MIDTERM 2
A. Translation --> Transcription --> replication MICHAEL EXAM -
B. Replication --> transcription --> translation SOSUAN MARCH 2016
C. Transcription --> translation --> replication (TOP 5 -
D. Translation --> replication --> transcription AUG 2015
E. NOTA MED
BOARDS;
TOPNOTCH
MD FROM
UST)
50 Most of dietary fat are composed of 90% of dietary fats are composed of triacylglycerol GEORGE MIDTERM 2
A. TAG MICHAEL EXAM -
B. Free fatty acid SOSUAN MARCH 2016
C. Cholesterol (TOP 5 -
D. Phospholipid AUG 2015
E. Glycerol MED
BOARDS;
TOPNOTCH
MD FROM
UST)
51 Lactic acid is considered to be a weak acid because: Weak acids like lactic acid never completely GEORGE MIDTERM 2
A. It is insoluble in water at standard temperature and dissociate. The indicated pKa of 5.2 is consistent with MICHAEL EXAM -
pressure. the idea that the lactate anion retains a strong affinity SOSUAN MARCH 2016
B. It fails to obey the Henderson-Hasselbalch equation. for protons, a hallmark of a weak acid. The lactate (TOP 5 -
C. The equilibrium between the acid and its conjugate anion is highly water-soluble. All weak acids obey the AUG 2015
base has a pKa of 5.2.. Henderson- Hasselbalch equation. MED
D. The lactate anion has minimal tendency to attract a BOARDS;
proton. TOPNOTCH
E. NOTA MD FROM
UST)
52 A noncompetitive enzyme inhibitor A noncompetitive inhibitor binds to the enzyme at a GEORGE MIDTERM 2
A. Decreases Vmax and increases Km. site other than MICHAEL EXAM -
B. Decreases Vmax and has no effect on Km. the substrate binding site, so it has little measurable SOSUAN MARCH 2016
C. Has no effect on Vmax or Km. effect on the enzyme’s affinity for substrate, as (TOP 5 -
D. Has no effect on Vmax and increases Km. represented by the Km. However, the inhibitor has the AUG 2015
E. NOTA effect of decreasing MED
the availability of active enzyme capable of catalyzing BOARDS;
the reaction, which manifests itself as a decrease in TOPNOTCH
Vmax. MD FROM
UST)

TOPNOTCH MEDICAL BOARD PREP BIOCHEMISTRY SUPEREXAM Page 7 of 92


For inquiries visit www.topnotchboardprep.com.ph or email us at topnotchmedicalboardprep@gmail.com
TOPNOTCH MEDICAL BOARD PREP BIOCHEMISTRY SUPEREXAM
For inquiries visit www.topnotchboardprep.com.ph or email us at topnotchmedicalboardprep@gmail.com
Item QUESTION EXPLANATION AUTHOR TOPNOTCH
# EXAM
53 Aspartame is composed of what amino acid? Aspartame: Phenylalanine + Methanol + Aspartic Acid GEORGE MIDTERM 2
A. Phenylalanine MICHAEL EXAM -
B. Asparagine SOSUAN MARCH 2016
C. Glutamic acid (TOP 5 -
D. Alanine AUG 2015
E. Tyrosine MED
BOARDS;
TOPNOTCH
MD FROM
UST)
54 Humans can't synthesize Vitamin C due to the absence of The inability of humans to synthesize L-ascorbic acid GEORGE MIDTERM 2
this enzyme is known to be due to a lack of L-gulono-gamma- MICHAEL EXAM -
A. Gulonolactone reductase lactone oxidase, an enzyme that is required for the SOSUAN MARCH 2016
B. Gulonolactone oxidase biosynthesis of this vitamin. (TOP 5 -
C. Gulonolactone ligase AUG 2015
D. Gulonolactone synthase MED
E. NOTA BOARDS;
TOPNOTCH
MD FROM
UST)
55 Certain drugs are thought to increase membrane fluidity Anesthetics are highly lipid-soluble and experiments GEORGE MIDTERM 2
directly, resulting in impaired neurotransmission that with isolated MICHAEL EXAM -
may be the basis for their therapeutic effects. Which membranes indicate that these molecules can dissolve SOSUAN MARCH 2016
class of drugs acts by this direct mechanism? in the hydrophobic center of the membrane bilayer. (TOP 5 -
A. Hallucinogens This causes a measurable increase in the membrane AUG 2015
B. Stimulants fluidity by disrupting MED
C. Sedatives the packed structure of phospholipids tails; thus, It BOARDS;
D. Opiates inhibits neurotransmission (pain sensations) in TOPNOTCH
E. Anesthetics neurons. Hallucinogens and opiates may also affect MD FROM
membrane fluidity, but their effects occur by indirect UST)
mechanisms, resulting from changes in the protein or
lipid composition of the membranes.
56 Rate-limiting step in urea cycle CPS 1 synthase - urea cycle GEORGE MIDTERM 2
A. CPS 1 synthase CPS 2 synthase - pyrimidine synthesis MICHAEL EXAM -
B. CPS 2 synthase Isocitarate dehydrogenase - TCA SOSUAN MARCH 2016
C. Isocitarate dehydrogenase (TOP 5 -
D. Both A and B AUG 2015
E. AOTA MED
BOARDS;
TOPNOTCH
MD FROM
UST)
57 Which of the following organs or tissues does NOT need Only the liver and kidneys can synthesize glucose by GEORGE MIDTERM 2
to be supplied with glucose gluconeogenesis. All the other organs listed are MICHAEL EXAM -
for energy production during a prolonged fast? dependent on provision of glucose from blood, either SOSUAN MARCH 2016
A. Lens supplied by the diet or by gluconeogenesis in liver and (TOP 5 -
B. Brain the kidneys. AUG 2015
C. RBCs MED
D. Liver BOARDS;
E. Cornea TOPNOTCH
MD FROM
UST)
58 The major pathway of amino groups disposal in human GEORGE MIDTERM 2
is by: MICHAEL EXAM -
A. Urea SOSUAN MARCH 2016
B. Ammonia (TOP 5 -
C. Creatinine AUG 2015
D. Uric acid MED
E. AOTA BOARDS;
TOPNOTCH
MD FROM
UST)
59 Bloatedness in lactose intolerance is due to GEORGE MIDTERM 2
A. Increased production of gas by the gut MICHAEL EXAM -
B. Lactose reduced to lactitol and converted to lactone SOSUAN MARCH 2016
C. Bacterial fermentation of undigested lactose in the (TOP 5 -
gut AUG 2015
D. Osmotically driven gfluid shift in the gut MED
E. AOTA BOARDS;
TOPNOTCH
MD FROM
UST)
60 Severe combined immunodeficiency are sensitive to dATP-induced allosteric inhibition of ribonucleotide GEORGE MIDTERM 2
allosteric inhibition of which of the following enzymes of reductase, which catalyzes reduction of the 2′- MICHAEL EXAM -
purine nucleotide metabolism? hydroxyl groups on ADP and GDP to form dADP and SOSUAN MARCH 2016
A. Xanthine oxidase dGDP (TOP 5 -
B. Dihydrofolate reductase AUG 2015
C. Adenosine deaminase MED
D. Ribonucleotide reductase BOARDS;
E. Hypoxanthine-guanine phosphoribosyltransferase TOPNOTCH
MD FROM
UST)

TOPNOTCH MEDICAL BOARD PREP BIOCHEMISTRY SUPEREXAM Page 8 of 92


For inquiries visit www.topnotchboardprep.com.ph or email us at topnotchmedicalboardprep@gmail.com
TOPNOTCH MEDICAL BOARD PREP BIOCHEMISTRY SUPEREXAM
For inquiries visit www.topnotchboardprep.com.ph or email us at topnotchmedicalboardprep@gmail.com
Item QUESTION EXPLANATION AUTHOR TOPNOTCH
# EXAM
61 Vitamin B1 or Thiamine pyrophosphate is a cofactor in Remember the mnemonics ATP. Vit B1 is a co factor of JAN MIDTERM 3
several of these enzymes/steps EXCEPT: Alpha-ketoglutarate DH of the TCA cycle, CHRISTIAN EXAM -
A. Pyruvate to Acetyl-Coenzyme A Transketolase (Phase 2 of the HMP shunt) and FELICIANO MARCH 2016
B. Pyruvate to Oxaloacetate Pyruvate DH complex (pyruvate to Acetyl-CoA). (TOP 2 -
C. Alpha-Keto Glutarate to Succinyl CoA Additonally it is also a co-factor of the the Branched AUG 2015
D. Hexose Monophosphate Shunt Phase 2 chain ketoacid DH step. Pyruvate to oxacocetate is MED
E. Branched chain ketoacid dehydrogenase faciltitated by pyruvate carboxylase which requires BOARDS;
biotin instead. TOPNOTCH
MD FROM
UST)
62 This amino acid is weakly basic but uncharged at SIMILAR TO PREVIOUS BOARD EXAM JAN MIDTERM 3
physiologic pH hence its important role in the CONCEPT/PRINCIPLE. The 3 basic amino acids are CHRISTIAN EXAM -
functioning of hemoglobin. lysine, arginine and histidine. Histidine is a weak base FELICIANO MARCH 2016
A. Proline but largely uncharged at physiologic pH. This is an (TOP 2 -
B. Arginine important property in the functioning of hemoglobin. AUG 2015
C. Lysine MED
D. Phenylalanine BOARDS;
E. Histidine TOPNOTCH
MD FROM
UST)
63 Which statement is true regarding the Krebs Henseleit The urea cycle occurs exclusively in the liver because JAN MIDTERM 3
cycle? it has arginase to cleave arginine into urea. The first 2 CHRISTIAN EXAM -
A. 90% occurs n the liver and 10% in the kidney steps occurs in the mitochondria whereas the FELICIANO MARCH 2016
B. The rate limiting step has an absolute requirement for remaining enzymes are located in the cytosol. The (TOP 2 -
N-acetylglutamate (NAG) as a positive allosteric second nitrogen of urea is from aspartate NOT AUG 2015
activator glutamate. 2 molecules of ATP are req'd to synthesize MED
C. The first 2 steps occurs in the cytoplasm and the rest carbamoly phosphate. Remember: An important BOARDS;
occurs in the mitochondria concept in the urea cycle is that NAG is an essential TOPNOTCH
D. The first nitrogen of urea is from free ammonia while activator of CPS I, the rate limiting step. MD FROM
the second nitrogen is donated by gluatamate UST)
E. Formation of carbamoyl phosphate is driven by
cleavage of 1 mol of ATP
64 The ff is true of proline EXCEPT? Cysteine and not proline contains a sulfhydryl group. JAN MIDTERM 3
A. It is considered a secondary amino acid Proline has a unique 5 membered ring structure CHRISTIAN EXAM -
B. It is an imino acid making it a secondary and an imino acid. Its unique FELICIANO MARCH 2016
C. It contains a sulfhydryl group that is an active part of and sturdy geometry interrupts the alpha helices of (TOP 2 -
many enzymes collagen. AUG 2015
D. It has an aliphatic side and a non polar side chain MED
E. Its unique geometry contributes to formation of BOARDS;
collagen TOPNOTCH
MD FROM
UST)
65 This complex of the electron transport chain is the site Complex IV or cytochrome oxidase is the site where JAN MIDTERM 3
where one-half of O2 is reduced to water. the the final electron acceptor which is 1/2 of oxygen CHRISTIAN EXAM -
A. Cytochrome oxidase is reduced to water FELICIANO MARCH 2016
B. ATP synthase (TOP 2 -
C. NADH dehydrogenase AUG 2015
D. Succinate dehydrogenase MED
E. Ubiquinol ferricytochrome oxidoreductase BOARDS;
TOPNOTCH
MD FROM
UST)
66 A patient is suspected of salicylate toxicitiy. Which of the SIMILAR TO PREVIOUS BOARD EXAM JAN MIDTERM 3
ff statement is true regarding its effects on the ETC? CONCEPT/PRINCIPLE.. Salicylates/aspirin are CHRISTIAN EXAM -
A. There is reduced permeability of the inner uncouplers that dissociates oxidation in the FELICIANO MARCH 2016
mitochondrial membrane to protons hence impaired respiratory chain from phosphorylation. There is (TOP 2 -
ATP synthesis increased permeability of the mitochondiral AUG 2015
B. Respiration dissociates from phosphorylation membrane to protons therefore ATP sysnthesis stops MED
C. The electron transport chain terminates along with but the respiratory chain continues prducing BOARDS;
ATP synthesis excessive heat hence hyperpyrexia. Oligomycin and TOPNOTCH
D. There is direct inhibition of Complex V hence ATP not aspirin is a Complex V inhibitor. MD FROM
synthesis terminates UST)
E. All of the above
67 Which of the ff statements regarding the carbohydrate Glucose, galactose, fructose and mannose are all JAN MIDTERM 3
structure is true? isomers having the chemical formula C6H12O6. CHRISTIAN EXAM -
A. Epimers are compounds that have the same chemical Epimers are compounds that differ around only 1 FELICIANO MARCH 2016
formula different only in structure specific carbon atom hence galactose and mannose (TOP 2 -
B. The L enantiomer is the major sugar of the human are NOT epimers bec they differ on 2 carbons atoms. AUG 2015
body The D enantiomer and not the L is utilzed by the body. MED
C. Galactose and mannose are epimers Anomers can mutarotate without need for enzymes. BOARDS;
D. Glucose and Fructose are isomers TOPNOTCH
E. Anomers are convertible between a linear form and MD FROM
ring form through specific enzymatic reactions UST)
68 The ff statement is true regarding glycosis EXCEPT Fructose 2-6 BP is actualy the most potent allosteric JAN MIDTERM 3
A. It is present in the cytoplasm of all cells activator of PFK-1, the rate limiting step of glycolysis. CHRISTIAN EXAM -
B. Phosphorylation of glucose is an irreversible step All the rest of the statements are true. FELICIANO MARCH 2016
C. 2 ATP molecules are initially utilized (TOP 2 -
D. Fructose 2,6 Bisphosphate is a potent inhibitor of the AUG 2015
rate limiting step MED
E. Only 2 molecules of ATP per glucose is produced BOARDS;
under anaerobic conditions TOPNOTCH
MD FROM
UST)

TOPNOTCH MEDICAL BOARD PREP BIOCHEMISTRY SUPEREXAM Page 9 of 92


For inquiries visit www.topnotchboardprep.com.ph or email us at topnotchmedicalboardprep@gmail.com
TOPNOTCH MEDICAL BOARD PREP BIOCHEMISTRY SUPEREXAM
For inquiries visit www.topnotchboardprep.com.ph or email us at topnotchmedicalboardprep@gmail.com
Item QUESTION EXPLANATION AUTHOR TOPNOTCH
# EXAM
69 Characteristics of glucokinase are the ff: SIMILAR TO PREVIOUS BOARD EXAM JAN MIDTERM 3
A. It is used as basal absorption of glucose by most CONCEPT/PRINCIPLE. Glucokinase despite its name CHRISTIAN EXAM -
tissues can also phosphorylate other hexoses. It has high Km FELICIANO MARCH 2016
B. It can also phosphorylate hexoses other than glucose (low affinity for glucose) and high Vmax and is (TOP 2 -
C. It is saturated under normal conditions present only in liver and pancreas. It is most active AUG 2015
D. It has high affinity for glucose compared to post-prandially. It is the first step but not the rate MED
hexokinase limiting step of glycolysis. Review the table BOARDS;
E. It is the rate limiting step of glycolysis highlighting the differences between hexokinase and TOPNOTCH
glucokinase. MD FROM
UST)
70 In glycogenolysis, the reason why the muscle tissue In the muscle, the enzyme, glucose-6-phosphatase is JAN MIDTERM 3
cannot produce free glucose for release into the blood ABSENT hence glucose 6 phosphate from CHRISTIAN EXAM -
stream is: glycogenolysis enters glycolysis instead. Remember FELICIANO MARCH 2016
A. The cell membrane is selectively impermeable to free that muscle can perform glycogenolysis but can only (TOP 2 -
glucose utilze the glucose for its own use. The other AUG 2015
B. Increased calcium levels in muscle tissue prevents statements are erroneous explanations. MED
exocytosis of free glucose BOARDS;
C. Absence of glucose-6-phosphatase TOPNOTCH
D. Glycogen phosphorylase is allosterically inactivated MD FROM
by increased glucose 6 phosphate levels UST)
E. Debranching enzyme is inacitve in metabolically
active msucles hence glucose is prefentially utilized
intracellularly
71 What step of the Krebs cycle utilizes substrate level The Krebs cycle has only 1 step that uses substrate JAN MIDTERM 3
phosphorylation? elvel phosphorylation to produce 1 mol of GTP namely CHRISTIAN EXAM -
A. Succinate to Fumarate Succinyl-Coa to Succinate by succinate thiokinase. FELICIANO MARCH 2016
B. Succinate to Oxaloacetate (TOP 2 -
C. Alpha keto glutarate to Succinyl CoA AUG 2015
D. Fumarate to Malate MED
E. None of the above BOARDS;
TOPNOTCH
MD FROM
UST)
72 In the fasting state which of the ff occurs? Expect PLENTY of questions regarding the JAN MIDTERM 3
A . Glycogen synthase is phosphorylated feeding/fasting cycle. Effects of insulin/glucagon? CHRISTIAN EXAM -
B. Decreased cAMP What enzymes are active when phosphorylated/ FELICIANO MARCH 2016
C. Increased phosphofructokinase-2 (PFK-2) dephosphorylated? Please master it by heart. In the (TOP 2 -
D. Increased fructose 2-6 Bisphosphate (F2,6 BP) fasting state, glucagon predominates to raise blood AUG 2015
E. Decreased activity of hormone senstive lipase glucose levels. Glucagon increases cAMP and protein MED
kinase A. It acts generally by PHOSPHORYLATING BOARDS;
enzymes. Take note: Glycogen synthase when TOPNOTCH
phosphorylated is INACTIVE hence glycogenesis is MD FROM
inhibited. There is also decreased activity of PFK2 and UST)
its product F2,6-BP which is the most important
alloesteric activator of glycolysis and inhibitor of
gluconeogenesis. Hormone senstive lipase is activated
by glucagon to release FFA for oxidation
73 A person ate a heavy meal 30 mins ago, which of the ff In the fed state, insulin predominates. In contrast to JAN MIDTERM 3
occurs? glucagon, insulin generally activates enzymes by CHRISTIAN EXAM -
A. Increased activity of carnitine acyltransferase DEPHOSPHORYLATION. Glycogen phosphorylase, in FELICIANO MARCH 2016
B. Decreased fructose 2-6 Bisphosphate (F2,6 BP) contrast to glycogen synthase, is active when (TOP 2 -
C. Most rare limiting enzymes are in the phosphorylated. Insulin upregulates F2,6BP to AUG 2015
dephosphorylated state stimulate glycolysis increasing NADH. Carnitine MED
D. Glycogen phosphorylase Is phosphorylated acyltranferase is the rate limiting step of fatty acid BOARDS;
E. Increased NAD/NADH ratio oxidation and is deactivated TOPNOTCH
MD FROM
UST)
74 All of the the ff are effects of glucagon EXCEPT? Again, glucagon (fasting state) generally acts by JAN MIDTERM 3
A. Activation of hormone sensitive lipase phosphorylating enzymes. Hormone senstive lipase is CHRISTIAN EXAM -
B. Increased activity of both ketogenesis and activated to release FFA for oxidation. Both FELICIANO MARCH 2016
ketogenolysis ketogenensis and ketogenolysis is active in the fasting (TOP 2 -
C. Allosteric activation of pyruvate carboxylase by state.Pyruvate carboxylase, an irreversible step in AUG 2015
acetyl-CoA gluconeogenesis is activated to increase free glucose MED
D. Glycogen phosphorylase Is phosphorylated levels. Glycogen phosphorlyase is active when BOARDS;
E. Activation of protein phosphatase phosphorylated. Protein phosphatase is the key TOPNOTCH
effector enzyme of insulin and acts to MD FROM
DEPHOSPHORYLATE other enzymes. UST)
75 What lipoprotein is responsible for reverse cholesterol HDL or the good cholesterol mediates reverse JAN MIDTERM 3
transport? cholesterol transport from periphery to liver. CHRISTIAN EXAM -
A. Chylomicron FELICIANO MARCH 2016
B. VLDL (TOP 2 -
C. LDL AUG 2015
D. HDL MED
E. IDL BOARDS;
TOPNOTCH
MD FROM
UST)
76 NAPDH produced from the HMP shunt is needed in the ff NADPH from HMP shunt is used for reductive JAN MIDTERM 3
pathways EXCEPT? reactions for FA and steroid synthesis, reduction of CHRISTIAN EXAM -
A. Fatty acid synthesis glutathione, NO synthesis, respiratory burst and FELICIANO MARCH 2016
B. Nitric oxide syntehsis CYP450 reactions. (TOP 2 -
C. Cholesterol synthesis AUG 2015
D. Heme synthesis MED
E. Regeneration of Glutathione BOARDS;
TOPNOTCH
MD FROM
TOPNOTCH MEDICAL BOARD PREP BIOCHEMISTRY SUPEREXAM Page 10 of 92
For inquiries visit www.topnotchboardprep.com.ph or email us at topnotchmedicalboardprep@gmail.com
TOPNOTCH MEDICAL BOARD PREP BIOCHEMISTRY SUPEREXAM
For inquiries visit www.topnotchboardprep.com.ph or email us at topnotchmedicalboardprep@gmail.com
Item QUESTION EXPLANATION AUTHOR TOPNOTCH
# EXAM
UST)

77 A patient has mental retardation and hypotonia. Further SIMILAR TO PREVIOUS BOARD EXAM JAN MIDTERM 3
examination revealed a cherry red spot on the macula. CONCEPT/PRINCIPLE. For some reason, Tay sachs CHRISTIAN EXAM -
Traube's space is obliterated. You are suspecting a type disease with characteristic cherry red spot is a FELICIANO MARCH 2016
of sphingolipidosis. What enzyme is probably deficient? favorite topic. Differentiate it from Niemann pick (TOP 2 -
A. Hexosaminadase A diease which also has cherry red spot but with AUG 2015
B. Hexosaminadase B hepatosplenomely as well. Enzyme dieficiencies are: MED
C. Glucocerebrosidase Tay sach's- Hexosaminidase A. Gaucher's- BOARDS;
D. Galactocerebrosidase Glucocerebrosidase. Krabbe's- Galactocerebrosidase. TOPNOTCH
E. Sphingomyelinase Nieman Pick- Sphingomyelinase MD FROM
UST)
78 This intermediate is common to both ketogenesis and SIMILAR TO PREVIOUS BOARD EXAM JAN MIDTERM 3
cholesterol synthesis? CONCEPT/PRINCIPLE. 3-hydroxy 3 methylglutaryl- CHRISTIAN EXAM -
A. 3-hydroxy 3 methylglutaryl-CoA CoA is termed simply as HMG-CoA and and its FELICIANO MARCH 2016
B. 3 hydroxybutyrate production serve as the first step of both cholesterol (TOP 2 -
C. Mevalonate 5-phosphate synthesis and ketogenesis. The other intermediates AUG 2015
D. Farnesyl pyrophosphate are part of ketogenesis or choelsterol synthesis but MED
E. Lanosterol not both. BOARDS;
TOPNOTCH
MD FROM
UST)
79 This is an example of a nucleoside. SIMILAR TO PREVIOUS BOARD EXAM JAN MIDTERM 3
A. Adenine CONCEPT/PRINCIPLE. Memorize the names of CHRISTIAN EXAM -
B. Cytidine nitrogenous bases with their corresponding FELICIANO MARCH 2016
C. Thymine nucleoside and nucleotide. Bases are adenine, (TOP 2 -
D. Adenosine monophosphate guanine, thymine, uracil, cytosine. Nucleoside (base AUG 2015
E. Uracil plus sugar) are adenosine, guanosine, thymidine, MED
uridine and cytidine. Nucelotide (nucleoside plus BOARDS;
phopshate group) are AMP, UMP, TMP etc. TOPNOTCH
MD FROM
UST)
80 It is a characteristic of eukaryotic mRNA. mRNA in eukaryotes are extensively modified: Exons JAN MIDTERM 3
A. Introns are spliced together are spliced together and introns are removed, a long CHRISTIAN EXAM -
B. Long sequence of adenine moelcules at the 5' end sequence of poly A tail at the 3' end and a 7 methyl- FELICIANO MARCH 2016
C. 3'-CCA sequence guanosyl group at 5' end is added. It also makes up (TOP 2 -
D. 7 methyl-guanosyl group at 3' end only 5% of the proportion of the RNA genome. 3'-CCA AUG 2015
E. Makes up only 5% of the proportion of the RNA sequence is a unique property of tRNA. MED
genome BOARDS;
TOPNOTCH
MD FROM
UST)
81 Which of the following is NOT true on how Calcium glycogen phosphorylase is activated by muscle ANDREW FINAL EXAM -
synchronizes the activation of glycogen phosphorylase? phosphorylase kinase TIU (TOP 1 MARCH 2016
a. cytosolic calcium is responsible for both contraction Harper’s Biochemistry 28th edition p. 160 - AUG 2015
and glycogenolysis MED
b. muscle phosphorylase kinase activates glycogen BOARDS;
phosphatase TOPNOTCH
c. phosphorylase kinase is made up of 4 subunits MD FROM
d. binding of calcium activates catalytic site of gamma CIM)
subunit
e. phosphorylated form is only fully activated in the
presence of calcium
82 2. Which of the following amino acids are not found in ANDREW FINAL EXAM -
glutathione? TIU (TOP 1 MARCH 2016
a. cysteine - AUG 2015
b. glutamic acid MED
c. glycine BOARDS;
d. aspartate TOPNOTCH
e. all of the above MD FROM
CIM)
83 3. Which of the following is added to LPG tanks to detect ANDREW FINAL EXAM -
leaks? TIU (TOP 1 MARCH 2016
a. propane - AUG 2015
b. ethyl mercaptan MED
c. methane BOARDS;
d. butane TOPNOTCH
e. acetone MD FROM
CIM)
84 Which of the following amino acids does not contain a Harper’s Biochemistry 28th edition p. 16 ANDREW FINAL EXAM -
basic group? TIU (TOP 1 MARCH 2016
a. arginine - AUG 2015
b. proline MED
c. lysine BOARDS;
d. histidine TOPNOTCH
e. none of the above MD FROM
CIM)

TOPNOTCH MEDICAL BOARD PREP BIOCHEMISTRY SUPEREXAM Page 11 of 92


For inquiries visit www.topnotchboardprep.com.ph or email us at topnotchmedicalboardprep@gmail.com
TOPNOTCH MEDICAL BOARD PREP BIOCHEMISTRY SUPEREXAM
For inquiries visit www.topnotchboardprep.com.ph or email us at topnotchmedicalboardprep@gmail.com
Item QUESTION EXPLANATION AUTHOR TOPNOTCH
# EXAM
85 5. According to the International Union of Biochemists ligases - joining together of 2 molecules coupled with ANDREW FINAL EXAM -
(IUB) which of the following refers to an enzyme that hydrolysis of ATP TIU (TOP 1 MARCH 2016
catalyzes cleavage of C-O bond through atom hydrolases - hydrolytic cleavage - AUG 2015
elimination? transferases - transfer of glycosyl, methyl, or MED
a. oxidoreductases phosphoryl groups, etc BOARDS;
b. transferases Harper’s Biochemistry 28th edition p. 52 TOPNOTCH
c. ligases MD FROM
d. lyases CIM)
e. hydrolases
86 6. An 8 year old child ingests Full cream milk. After 2 refers to lactase deficiency ANDREW FINAL EXAM -
hours, patient was noted to have bloating, flatulence, and Harper’s Biochemistry 28th edition p. 113 TIU (TOP 1 MARCH 2016
soft watery stools. Reduced sugars were found in stools. - AUG 2015
Which of the following is most likely true? MED
a. disaccharidase enzyme deficiency BOARDS;
b. inability to absorb lactose TOPNOTCH
c. inability to break down maltose MD FROM
d. autoimmune destruction of villi CIM)
e. none of the above
87 7. Which of the following is TRUE on regulation of Harper’s Biochemistry 28th edition p. 168 ANDREW FINAL EXAM -
glycolysis and gluconeogenesis in the liver? TIU (TOP 1 MARCH 2016
a. most potent positive allosteric activator of PFK 1 and - AUG 2015
inhibitor of fructose 1,6 bisphosphate is fructose 2,6 MED
bisphosphate BOARDS;
b. fructose 2,6 bisphosphate is formed by PFK -2 TOPNOTCH
c. PFK 2 and fructose 2,6 bisphosphatase are different MD FROM
enzymes CIM)
d. both A and B
e. both B and C
88 8. Which of the following is NOT induced by insulin in Harper’s Biochemistry 28th edition p. 168 ANDREW FINAL EXAM -
the regulatory and adaptive enzymes associated with TIU (TOP 1 MARCH 2016
carbohydrate metabolism? - AUG 2015
a. PFK 1 MED
b. pyruvate carboxylase BOARDS;
c. pyruvate kinase TOPNOTCH
d. glucokinase MD FROM
e. pyruvate dehydrogenase CIM)
89 9. Which of the following processes is NOT increased by Harper’s Biochemistry 28th edition p. 172 ANDREW FINAL EXAM -
glucagon? TIU (TOP 1 MARCH 2016
a. protein synthesis - AUG 2015
b. glycogenolysis MED
c. gluconeogenesis BOARDS;
d. ketogenesis TOPNOTCH
e. lipolysis MD FROM
CIM)
90 10. Which of the following is NOT TRUE of regulation of Free fatty acids are the precursors of ketone bodies ANDREW FINAL EXAM -
ketogenesis? Harper’s Biochemistry 28th edition p. 190 TIU (TOP 1 MARCH 2016
a. the factors regulating mobilization of free fatty acids - AUG 2015
are important in controlling ketogenesis MED
b. triacylglycerols are the precursors of ketone bodies in BOARDS;
the liver TOPNOTCH
c. regulation of entry of fatty acids into the oxidative MD FROM
pathway is through CPT-1 CIM)
d. impaired oxidation of fatty acids leads to diseases
associated with hypoglycemia
e. decreased insulin/glucagon ration promotes B -
oxidation of fatty acids
91 11. Which of the following is the regulatory step of bile Harper’s Biochemistry 28th edition p. 229 ANDREW FINAL EXAM -
acid synthesis? TIU (TOP 1 MARCH 2016
a. HMG Coa synthase - AUG 2015
b. HMG CoA reductase MED
c. 7 alpha - hydroxylase BOARDS;
d. sterol 27 hydroxylase TOPNOTCH
e. none of the above MD FROM
CIM)
92 12. A 6 year old African American presented with Here, glutamic acid, the normal amino acid in position ANDREW FINAL EXAM -
dyspnea, painful digits, splenomegaly, anemia and 6 of the B chain has been replaced by valine. Clearly, TIU (TOP 1 MARCH 2016
jaundice. Which of the following mutations is most likely this hinders normal function. - AUG 2015
present in this patient? Harper’s Biochemistry 28th edition p. 357 MED
a. nonsense BOARDS;
b. missense TOPNOTCH
c. frameshift MD FROM
d. silent CIM)
e. none of the above
93 13. WHich of the following is not a vitamin B12 Harper’s Biochemistry 28th edition p. 476 ANDREW FINAL EXAM -
dependent enzyme? TIU (TOP 1 MARCH 2016
a. methionine synthase - AUG 2015
b. methylmalonyl CoA mutase MED
c. leucine aminomutase BOARDS;
d. alpha ketoglutarate dehydrogenase TOPNOTCH
e. none of the above MD FROM
CIM)

TOPNOTCH MEDICAL BOARD PREP BIOCHEMISTRY SUPEREXAM Page 12 of 92


For inquiries visit www.topnotchboardprep.com.ph or email us at topnotchmedicalboardprep@gmail.com
TOPNOTCH MEDICAL BOARD PREP BIOCHEMISTRY SUPEREXAM
For inquiries visit www.topnotchboardprep.com.ph or email us at topnotchmedicalboardprep@gmail.com
Item QUESTION EXPLANATION AUTHOR TOPNOTCH
# EXAM
94 14. A neonate was found to have poor feeding with Harper’s Biochemistry 28th edition p. 493 ANDREW FINAL EXAM -
seizures. Biochemical tests revealed accumulation of TIU (TOP 1 MARCH 2016
very long chain fatty acids and marked reduction of - AUG 2015
plasmalogen. Mutations in genes encoding peroxisome MED
biogenesis has also been found. Which of the following is BOARDS;
the most likely diagnosis? TOPNOTCH
a. adrenoleukodystrophy MD FROM
b. Refsum’s disease CIM)
c. Zellweger syndrome
d. rhizomelic chondrodysplasia punctata
e. none of the above
95 15. Which of the following is NOT a feature of N - transfer can occur cotranslationally in the ANDREW FINAL EXAM -
glycosylation? endoplasmic reticulum TIU (TOP 1 MARCH 2016
a. the transfer of oligosaccharide occurs in Asn residues Harper’s Biochemistry 28th edition p. 516 - AUG 2015
b. protein bound oligosaccharide partially processed by MED
glucosidases and mannosidases BOARDS;
c. transfer of oligosaccharide occurs post translationally TOPNOTCH
in the golgi MD FROM
d. the enzyme which catalyzes the transfer of CIM)
oligosaccharide is inhibited by tunicamycin
e. none of the above
96 16. Which of the following is NOT true in the metabolism Harper’s Biochemistry 28th edition p. 562 ANDREW FINAL EXAM -
of skeletal muscle? TIU (TOP 1 MARCH 2016
a. skeletal muscle has receptors for glucagon - AUG 2015
b. glucose is resynthesized from lactate through Cori MED
cycle BOARDS;
c. free fatty acids are the major source of energy during TOPNOTCH
marathons MD FROM
d. alanine from muscle is destined for gluconeogenesis CIM)
e. skeletal muscle does not contain glucose 6
phosphatase
97 17. A 32 year old female presented with hepatic Harper’s Biochemistry 28th edition p. 573 ANDREW FINAL EXAM -
encephalopathy and Kayser Fleischer rings. Which of the TIU (TOP 1 MARCH 2016
following enzymes contain copper? - AUG 2015
a. amine oxidase MED
b. superoxide dismutase BOARDS;
c. cytochrome oxidase TOPNOTCH
d. tyrosinase MD FROM
e. all of the above CIM)
98 18. A neonate was seen in the NICU for delayed NADPH oxidase is used for oxygen dependent killing ANDREW FINAL EXAM -
separation of umbilical cord. Which of the following is of microbes TIU (TOP 1 MARCH 2016
not a biochemical feature of neutrophils? Harper’s Biochemistry 28th edition p. 604 - AUG 2015
a. active pentose phosphate pathway MED
b. contains CD11/CD18 integrins in plasma membrane BOARDS;
c. uses NADPH oxidase for oxygen independent killing of TOPNOTCH
microbes MD FROM
d. active glycolysis CIM)
e. moderately active oxidative phosphorylation
99 19. Which of the following is NOT true in the metabolism glycogenolysis is not the reverse of glycogenesis, but ANDREW FINAL EXAM -
of glycogen? is a separate pathway TIU (TOP 1 MARCH 2016
a. CAMP promotes simultaneous activation of Harper’s Biochemistry 28th edition p. 157 - AUG 2015
phosphorylase and inhibition of glycogen synthase MED
b. insulin inhibits glycogenolysis and stimulating BOARDS;
glycogenesis TOPNOTCH
c. glycogenesis and glycogenolysis are the same pathway MD FROM
d. glycogen biosynthesis involves UDP glucose CIM)
e. branching of glycogen involves detachment of existing
glycogen chains
100 20. Which of the following vitamins is given to a child to Harper’s Biochemistry 28th edition p. 469 ANDREW FINAL EXAM -
prevent metaplasia of surface epithelium? TIU (TOP 1 MARCH 2016
a. A - AUG 2015
b. B MED
c. C BOARDS;
d. D TOPNOTCH
e. E MD FROM
CIM)
101 Smith-Lemli-Opitz syndrome is a common autosomal SIMILAR TO PREVIOUS BOARD EXAM ANGELA DIAGNOSTIC
recessive multiple malformation syndrome. In this CONCEPT/PRINCIPLE. Pathway on page 24 of PAULINE P. EXAM - AUG
syndrome there is a deficiency in what enzyme?
Topnotch Handout. Smith-Lemli-Opitz syndrome CALIMAG- 2015
A. HMG-CoA reductase (SLOS) is a well-known malformation syndrome with LOYOLA
B. Acetyl-CoA acetyltransferase principal characteristics of psychomotor and growth (TOP 8 -
C. 3-hydroxy-3-methylglutaryl-CoA reductase retardation, cleft palate, hypospadias, postaxial FEB 2015
D. 7-dehydrocholesterol reductase polydactyly, and a distinctive craniofacial appearance MED
E. 3-hydroxy-3-methylglutaryl-CoA synthase consisting of microcephaly, ptosis, inner epicanthal BOARDS;
folds, anteverted nares, and micrognathia. Mutations TOPNOTCH
in the DHCR7 gene cause Smith-Lemli-Opitz MD FROM
syndrome.The DHCR7 gene provides instructions for UST)
making an enzyme called 7-dehydrocholesterol
reductase. This enzyme is responsible for the final
step in the production of cholesterol.

TOPNOTCH MEDICAL BOARD PREP BIOCHEMISTRY SUPEREXAM Page 13 of 92


For inquiries visit www.topnotchboardprep.com.ph or email us at topnotchmedicalboardprep@gmail.com
TOPNOTCH MEDICAL BOARD PREP BIOCHEMISTRY SUPEREXAM
For inquiries visit www.topnotchboardprep.com.ph or email us at topnotchmedicalboardprep@gmail.com
Item QUESTION EXPLANATION AUTHOR TOPNOTCH
# EXAM
102 Which of the following is the rate limiting step in SIMILAR TO PREVIOUS BOARD EXAM ANGELA DIAGNOSTIC
catecholamine synthesis?
CONCEPT/PRINCIPLE. Pathway on page 31 of PAULINE P. EXAM - AUG
A. DOPA decarboxylase Topnotch Handout. The rate limiting step is Tyrosine CALIMAG- 2015
B. Dopamine hydroxylase hydroxylase. LOYOLA
C. Tyrptophan hydroxylase (TOP 8 -
D. Phenethanolamine N-methyltransferase FEB 2015
E. Tyrosine hydroxylase MED
BOARDS;
TOPNOTCH
MD FROM
UST)
103 Sanfilippo syndrome is one of the autosomal recessive Page 44 of Topnotch Handout. Iduronate sulfatase is a ANGELA DIAGNOSTIC
mucopolysaccharidoses. It is characterized by mental deficiency in Hunters syndrome not in Sanfilippo. PAULINE P. EXAM - AUG
retardation, hyperactivity and eventual loss of motor CALIMAG- 2015
function. Each of the 4 types has a specific deficiency in LOYOLA
the following enzymes, EXCEPT:
(TOP 8 -
A. Iduronate sulfatase FEB 2015
B. heparan sulfamidase MED
C. N-acetylglucosaminidase BOARDS;
D. N-acetylglucosamine sulfatase TOPNOTCH
E. N-acetylglucosamine MD FROM
UST)
104 Which of the following is part of the Electron transport SIMILAR TO PREVIOUS BOARD EXAM ANGELA DIAGNOSTIC
chain and is also found in the Tricarboxylic acid CONCEPT/PRINCIPLE. Page 6 of Topnotch Handout. PAULINE P. EXAM - AUG
pathway: A. NADH dehydrogenase Succinate dehydrogenase is the enzyme in converting CALIMAG- 2015
B. Succinate dehydrogenase succinate to fumarate in the Kreb's cycle and its LOYOLA
C. Ubiquinol:ferricytochrome oxidoreductase product is FADH2. It is also known as Complex II of (TOP 8 -
D. Cytochrome oxidase the ETC. FEB 2015
E. Cytochrome a/a3 MED
BOARDS;
TOPNOTCH
MD FROM
UST)
105 A 27 y/o female patient went in for an executive check Page 27 of Topnotch Handout. LDL binds to specific ANGELA DIAGNOSTIC
up. Laboratory results showed elevated LDL. Which receptors on extrahepatic tissues and on the liver PAULINE P. EXAM - AUG
statement is true regarding LDL.:
where they are endocytosed. A,B and D are functions CALIMAG- 2015
A. Transports cholesterol from the peripheral tissues to of HDL. E is a function of Apo-E LOYOLA
the liver. (TOP 8 -
B. Delivers cholesterol to steroidogenic tissues via SR- FEB 2015
B1 MED
C. Taken up by tissues through receptor mediated BOARDS;
endocytosis TOPNOTCH
D. Shuttles apo-CII and apo-E in blood MD FROM
E. Mediates uptake of chylomicron remnant UST)
106 Further studies showed that the patient in the above Page 28 of Topnotch Handout. In Type II a Familial ANGELA DIAGNOSTIC
case suffers from Familial Hypercholesterolemia Type hypercholesterolemia there is a deficiency or defect in PAULINE P. EXAM - AUG
IIa this defect is brought about by which mechanism:
the LDL-receptors causing increased LDL. A and E are CALIMAG- 2015
A. LCAT deficiency defects in Familial a-Lipoprotein deficiency/ Tangiers LOYOLA
B. Defect in LDL-receptor disease. C is seen in Type I Familial lipoprotein lipase (TOP 8 -
C. Lipoprotein lipase deficiency deficiency. D is seen in Abetalipoproteinemia. FEB 2015
D. Decreased production of apo-B48 and 100 MED
E. Inefficient reverse cholesterol transport BOARDS;
TOPNOTCH
MD FROM
UST)
107 A 70 y/o male was brought to the ER presenting with Page 35 of Topnotch Handout. The patient is in ANGELA DIAGNOSTIC
severe congestive cardiac failure. He has been unwell for Respiratory alkalosis in response to the dyspnea PAULINE P. EXAM - AUG
about a week and has been vomiting for the previous 5 hence there will be a shift to the left of the oxygen CALIMAG- 2015
days. He was on no medication. He was hyperventilating binding curve. pH: pH is greater than 7.44 so an LOYOLA
and was very distressed. His vital signs are: BP 80/60, alkalaemia is present. The cause is an alkalosis (TOP 8 -
HR 85, RR 30. ABG showed pH 7.58, pCO2 21 mmHg, Pattern: pCO2 & bicarbonate are both low suggesting FEB 2015
pO2 154 mmHg, HCO3 14 mmol/l How will his current either a metabolic acidosis or a respiratory alkalosis. MED
condition affect the Oxygen saturation curve of As we already know an alkalosis is present then the BOARDS;
hemoglobin :
primary disorder is a respiratory alkalosis. Asessing TOPNOTCH
A. The oxygen binding curve will assume the shape of a the compensation for a respiratory alkalosis (using MD FROM
hyperbola similar to myoglobin rule 4 - "the 5 for 10" rule): The expected HCO3 is (24 UST)
B. There will be a rightward shift of the oxygen binding - 10) = 14.
curve Formulation: respiratory alkalosis
C. The position of the oxygen binding curve will not be
altered in any way
D. There will be a leftward shift of the oxygen binding
curve
E. Both A and C
108 Respiratory fatigue will eventually set in and bring Page 35 of Topnotch Handout. P50 is the pO2 at which ANGELA DIAGNOSTIC
about acidosis. This will affect the P50 value in the Hemoglobin is 50% saturated. PAULINE P. EXAM - AUG
oxyhemoglobin dissociation curve. The P50 value CALIMAG- 2015
represents the oxygen pressure at which hemoglobin: LOYOLA
A. is almost fully saturated with oxygen (TOP 8 -
B. releases oxygen at the capillary level FEB 2015
C. binds oxygen to 50% saturation of its binding sites MED
D. binds oxygen released by tissues BOARDS;
E. Both A and C TOPNOTCH
MD FROM
UST)

TOPNOTCH MEDICAL BOARD PREP BIOCHEMISTRY SUPEREXAM Page 14 of 92


For inquiries visit www.topnotchboardprep.com.ph or email us at topnotchmedicalboardprep@gmail.com
TOPNOTCH MEDICAL BOARD PREP BIOCHEMISTRY SUPEREXAM
For inquiries visit www.topnotchboardprep.com.ph or email us at topnotchmedicalboardprep@gmail.com
Item QUESTION EXPLANATION AUTHOR TOPNOTCH
# EXAM
109 Alpha-tocopherol has the following functions:
SIMILAR TO PREVIOUS BOARD EXAM ANGELA DIAGNOSTIC
A. Protects membrane lipids from peroxidation CONCEPT/PRINCIPLE. Page 47 of Topnotch Handout. PAULINE P. EXAM - AUG
B. Promotes epithelial cell proliferation and Alpha tocopherol is also known as Vitamin E. It CALIMAG- 2015
differentiation functions as an antioxidant in the lipid phase, protects LOYOLA
C. Increases calcium reabsorption in kidneys and bone membranes from peroxidation and prevents oxidation (TOP 8 -
resorption of LDL. B is a function of Vitamin A. C is a function of FEB 2015
D. Used as co-factors in redox reactions Vitamin D. D is a function of Vitamin B2/Riboflavin. E MED
E. Used as cofactor for acyl transfers is a function of Vitamin B5/Panthotenate. BOARDS;
TOPNOTCH
MD FROM
UST)
110 The importance of the Glucose Alanine cycle in the Page 39 of Topnotch Handout. The glucose alanine ANGELA DIAGNOSTIC
muscle in relation to amino acid metabolism is that:
cycle is used for the removal of excess nitrogen from PAULINE P. EXAM - AUG
A. It is the mechanism by which excess nitrogen from peripheral tissues through alanine. The excess CALIMAG- 2015
muscles is transfered to the liver by transamination of nitrogen will reach the liver through transamination LOYOLA
pyruvte of pyruvate to produce alanine (occurs in the muscle) (TOP 8 -
B. It is used for the synthesis of glucose in the muscle to then in the liver, alanine is converted back to pyruvate FEB 2015
replenish energy supply of the liver which may undergo gluconeogenesis and is transorted MED
C. The carbon skeletons of alanine will be back to the muscles. BOARDS;
transaminated to pyruvate for glycolysis TOPNOTCH
D. Pyruvate is reduced to lactate which is eventually MD FROM
transported to the liver UST)
E. All of the above

111 Which of the following is a true statement regarding the Page 39 of Topnotch Handout. Krebs-Henseleit cycle is ANGELA DIAGNOSTIC
Krebs-Henseleit cycle :
also known as the Ornithine or Urea cycle, which PAULINE P. EXAM - AUG
A. It is the final common pathway for the aerobic functions in the removal of nitrogenous waste CALIMAG- 2015
oxidation of ALL nutrients products in the body. A and B are both true for Kreb's LOYOLA
B. It provides majority of ATP for energy cycle. C is true for the ETC. (TOP 8 -
C. It occurs in the inner mitochondrial membrane FEB 2015
D. It is a pathway for removal of nitrogenous waste in MED
the body BOARDS;
E. Both A and B are correct TOPNOTCH
MD FROM
UST)
112 All of the following non-essential amino acids are Page 40 of Topnotch Handout. Alanine, aspartate, and ANGELA DIAGNOSTIC
synthesized from the transamination of alpha-ketoacids glutamate are synthesized from the transamination of PAULINE P. EXAM - AUG
except: A. Glutamate alpha-ketoacids. CALIMAG- 2015
B. Glycine LOYOLA
C. Alanine (TOP 8 -
D. Aspartate FEB 2015
E. All of the above MED
BOARDS;
TOPNOTCH
MD FROM
UST)
113 A 25 y/o male was diagnosed to have staghorn calculi. Page 41 of Topnotch Handout. Cystinuria is a common ANGELA DIAGNOSTIC
This condition may be due to a common inherited defect inherited defect of renal tubular amino acid PAULINE P. EXAM - AUG
of renal tubular amino acid transporter for the following transporter for cystine, ornithine, lysine and arginine CALIMAG- 2015
amino acids in the PCT of the kidneys, except:
in the PCT of the kidneys. Excess of cystine in the LOYOLA
A. Arginine urine may lead to precipitation of cystine kidney (TOP 8 -
B. Lysine stones and cause staghorn calculi. FEB 2015
C. Ornithine MED
D. Cystine BOARDS;
E. Cysteine TOPNOTCH
MD FROM
UST)
114 Fats are the best storage form of energy because they SIMILAR TO PREVIOUS BOARD EXAM ANGELA DIAGNOSTIC
have ___________ energy content :
CONCEPT/PRINCIPLE. Page 46 of Topnotch Handout. PAULINE P. EXAM - AUG
A. 6 kcal/gram Fats have 9 kcal/gram energy content. Carbohydrates, CALIMAG- 2015
B. 7 kcal/gram proteins and ketones contain 4 kcal/gram, while LOYOLA
C. 8 kcal/gram ethanol contains 7 kcal/gram. (TOP 8 -
D. 9 kcal/gram FEB 2015
E. 10 kcal/gram MED
BOARDS;
TOPNOTCH
MD FROM
UST)
115 A 17 y/o Japanese male who is fond of eating sushi was Page 48 of Topnotch Handout. D. Latum infestation ANGELA DIAGNOSTIC
noted to have diarrhea, abdominal pain, vomiting, which is usually obtained from eating raw fish can PAULINE P. EXAM - AUG
weight loss, and fatigue. He was diagnosed to have D. cause a deficiency in Vitamin B12/cobalamin. CALIMAG- 2015
Latum infestation, causing a deficiency in which of the LOYOLA
following: A. Iron (TOP 8 -
B. Folate FEB 2015
C. Vitamin A MED
D. Vitamin C BOARDS;
E. Cobalamin TOPNOTCH
MD FROM
UST)

TOPNOTCH MEDICAL BOARD PREP BIOCHEMISTRY SUPEREXAM Page 15 of 92


For inquiries visit www.topnotchboardprep.com.ph or email us at topnotchmedicalboardprep@gmail.com
TOPNOTCH MEDICAL BOARD PREP BIOCHEMISTRY SUPEREXAM
For inquiries visit www.topnotchboardprep.com.ph or email us at topnotchmedicalboardprep@gmail.com
Item QUESTION EXPLANATION AUTHOR TOPNOTCH
# EXAM
116 A 50 y/o male patient with colorectal cancer was given Page 51 of Topnotch Handout. 5-FU acts in several ANGELA DIAGNOSTIC
Fluorouracil. This is a pyrimidine analog drug which acts ways, but principally as a thymidylate synthase (TS) PAULINE P. EXAM - AUG
on rapidly dividing cancerous cells and they undergo cell inhibitor. Interrupting the action of this enzyme CALIMAG- 2015
death via thymineless death. Hence this drug is known to blocks synthesis of the pyrimidine thymidine, which is LOYOLA
act on the enzyme in the De Novo Pyrimidine synthesis a nucleoside required for DNA replication. (TOP 8 -
necessary for the:
Thymidylate synthase methylates deoxyuridine FEB 2015
A. Synthesis of carbamoyl phosphate monophosphate (dUMP) to form thymidine MED
B. Methylation of deoxyuridine monophosphate to monophosphate (dTMP). Administration of 5-FU BOARDS;
form thymidine monophosphate causes a scarcity in dTMP, so rapidly dividing TOPNOTCH
C. Formation of orotidine monophosphate cancerous cells undergo cell death via thymineless MD FROM
D. Synthesis of UTP and CTP death. UST)
E. All of the above

117 A 7 month old infant presents with presented with fever Page 52 of Topnotch Handout. Adenosine deaminase ANGELA DIAGNOSTIC
and paralysis of his left arm 3 months after receiving his deficiency leads to severe combined PAULINE P. EXAM - AUG
third oral poliovirus vaccine. Past history included immunodeficiency due to increased levels of dATP CALIMAG- 2015
chronic thrush presenting in the absence of antibiotic inhibiting ribonucleotide reductase and inhibiting LOYOLA
therapy or breastfeeding at 3 months, chronic diarrhea formation of deoxyribonucleotides. B is for Lesch- (TOP 8 -
from 4 months, and recurrent otitis media. He was at the Nyhan syndrome. C is seen in Orotic aciduria. D is for FEB 2015
90th percentile for height and weight, similar to his Menkes syndrome. E is in Tay-sachs. MED
parents. He was noted to have poor feeding and poor BOARDS;
weight gain. This is characteristic of SCID: TOPNOTCH
A. Adenosine deaminase deficiency MD FROM
B. HGPRT deficiency UST)
C. Low activity of orotidine phosphate decarboxylase
D. x-linked recessive disorder caused by deficiency of
ATP-dependent membrane transporter for copper
E. Hexosaminidase deficiency

118 DNA replication is a semi-conservative process where in Page 54 of Topnotch Handout. This occurs during the ANGELA DIAGNOSTIC
each strand serves as a template for the complementary S phase of the cell cycle. PAULINE P. EXAM - AUG
daughter strand and each strand becomes part of the CALIMAG- 2015
daugther strand. This process usually occurs during LOYOLA
which part of the cell cycle?
(TOP 8 -
A. G0 phase FEB 2015
B. G1 phase MED
C. G2 phase BOARDS;
D. S phase TOPNOTCH
E. M phase MD FROM
UST)
119 Which of the following antibiotics used in TB inhibits SIMILAR TO PREVIOUS BOARD EXAM ANGELA DIAGNOSTIC
RNA synthesis:
CONCEPT/PRINCIPLE. Page 59 of Topnotch Handout. PAULINE P. EXAM - AUG
A. Streptomycin Rifampicin binds to the Beta subunit of bacterial DNA- CALIMAG- 2015
B. Rifampicin dependent RNA polymerase and thereby inhibits RNA LOYOLA
C. Isoniazid synthesis. Streptomycin binds to the 30s subunit and (TOP 8 -
D. Pyrazinamide distorts its structure thereby interfering with the FEB 2015
E. Ethambutol initiation of protein synthesis. MED
BOARDS;
TOPNOTCH
MD FROM
UST)
120 This heat stable DNA polymerase used in PCR replicates Page 60 of Topnotch Handout. Taq polymerase is a ANGELA DIAGNOSTIC
the DNA sequence following each primer:
thermostable DNA polymerase named after the PAULINE P. EXAM - AUG
A. Swivelase thermophilic bacterium Thermus aquaticus. As an CALIMAG- 2015
B. Gyrase enzyme able to withstand the protein-denaturing LOYOLA
C. Taq polymerase conditions (high temperature) required during PCR. (TOP 8 -
D. Ligase Taq's optimum temperature for activity is 75–80°C, FEB 2015
E. Helicase with a half-life of greater than 2 hours at 92.5°C, 40 MED
minutes at 95°C and 9 minutes at 97.5°C, and can BOARDS;
replicate a 1000 base pair strand of DNA in less than TOPNOTCH
10 seconds at 72°C. MD FROM
UST)
121 The following amino acids are important in creatine Creatinine is formed in muscle from creatine LYNN MIDTERM 1
biosynthesis except: phosphate by irreversible, non-enzymatic dehydratio, DARYL EXAM - AUG
A. Methionine and loss of phosphate. Glycine, arginine, and FELICIANO 2015
B. Arginine methionine all participate in creatine biosynthesis. VILLAMAT
C. Glycine (Harpers) SIMILAR TO PREVIOUS BOARD EXAM ER, MD
D. Lysine CONCEPT (TOP 5 -
E. None of the above FEB 2015
MED
BOARDS;
TOPNOTCH
MD FROM
EAC)

TOPNOTCH MEDICAL BOARD PREP BIOCHEMISTRY SUPEREXAM Page 16 of 92


For inquiries visit www.topnotchboardprep.com.ph or email us at topnotchmedicalboardprep@gmail.com
TOPNOTCH MEDICAL BOARD PREP BIOCHEMISTRY SUPEREXAM
For inquiries visit www.topnotchboardprep.com.ph or email us at topnotchmedicalboardprep@gmail.com
Item QUESTION EXPLANATION AUTHOR TOPNOTCH
# EXAM
122 Which of the following is NOT a function of Vitamin C? Ascorbic acid role/functions are the following : LYNN MIDTERM 1
A. Antioxidant coenzyme in hydroxylation of proline and lysine in DARYL EXAM - AUG
B. Regulation of cell cycle collagen synthesis, antioxidant, enhances absorption FELICIANO 2015
C. It is necessary for conversion of dopamine to of iron. Regulation of cell cycle is a function of Biotin. VILLAMAT
norepinephrine (Harpers). "Ascorbic acid is the coenzyme for copper ER, MD
D. Coenzyme in hydroxylation of proline and lysine in containing hydroxylases and a ketoglutarate linked (TOP 5 -
collagen synthesis iron containing hydroxylases. Dopamine B- FEB 2015
E. None of the above hydroxylase is a copper-containing enzyme involved MED
in the synthesis of the catecholamines, BOARDS;
norepinephrine, and epinephrine, from tyrosine in the TOPNOTCH
adrenal medulla and central nervous system. During MD FROM
hydroxylation the Cu+ is oxidized to Cu2+; reduction EAC)
back to Cu+ specifically requires ascorbate, which is
oxidized to monodehydroascorbate. Harpers p. 495.
Regulation of cell cycle is a function of biotin. Harpers"
123 Which of the following is INCORRECT regarding Deficiency of lipoprotein lipase will cause severe LYNN MIDTERM 1
hormone-sensitive lipase? chylomicronemia (Harpers, Topnotch handouts) DARYL EXAM - AUG
A. It catalyzes hydrolysis of triacylglycerol stores in FELICIANO 2015
adipose tissue. VILLAMAT
B. Its activity is mainly inhibited by insulin by ER, MD
stimulating phosphodiesterase and lipase phosphatase. (TOP 5 -
C. Deficiency of this enzyme will cause severe FEB 2015
chylomicronemia. MED
D. It is activated by cAMP-dependent protein kinase BOARDS;
and removes fatty acid from carbon 1 and carbon 3 of TOPNOTCH
triacylglycerol. MD FROM
E. Its action yields free fatty acids and glycerol. EAC)
124 What is the rate-limiting step in catecholamine Tyrosine hydroxylase is the rate-enzyme if LYNN MIDTERM 1
synthesis? catecholamine biosynthesis; it uses DARYL EXAM - AUG
A. Phenylalanine hydroxylase tetrahydrobiopterin and molecular oxygen to convert FELICIANO 2015
B. DOPA decarboxylase tyrosine to DOPA. (Pubmed: Archives of Biochemistry VILLAMAT
C. Phenylethanolamine N-methyl transferase and Biophysics, April 2011). SIMILAR TO PREVIOUS ER, MD
D. Tyrosine hydroxylase BOARD EXAM CONCEPT (TOP 5 -
E. Dopamine beta-oxidase FEB 2015
MED
BOARDS;
TOPNOTCH
MD FROM
EAC)
125 Which of the following enzyymes is common to both All the enzymes listed are specific to either glycolysis LYNN MIDTERM 1
glycolysis and gluconeogenesis? or gluconeogenesis. Glucokinase/Hexokinase, DARYL EXAM - AUG
A. Phosphoglycerate kinase Phosphofructokinase, and pyruvate kinase catalyzed FELICIANO 2015
B. Fructose-1,6-biphosphatase irreversible reactions unique to glycolysis. Fructose- VILLAMAT
C. Glucokinase 1,6-biphosphatase and pyruvate carboxylase are ER, MD
D. Pyruvate carboxylase irreversible enzymes in gluconeogenesis. (TOP 5 -
E. Pyruvate kinase FEB 2015
MED
BOARDS;
TOPNOTCH
MD FROM
EAC)
126 The greatest quantitative source of high energy Oxidative phosphorylation occurs only in aerobic LYNN MIDTERM 1
phosphate in aerobic organism is? organism DARYL EXAM - AUG
A. Oxidative phosphorylation FELICIANO 2015
B. Substrate level phosphorylation VILLAMAT
C. Pentose Phosphate pathway ER, MD
D. Citric acid cycle (TOP 5 -
E. Beta-oxidation FEB 2015
MED
BOARDS;
TOPNOTCH
MD FROM
EAC)
127 Which of the following is true regarding myoglobin? Myoglobin follows a hyperbolic curve. Its main LYNN MIDTERM 1
A. It follows a sigmoidal curve. function is for oxygen storage. It is mainly found in DARYL EXAM - AUG
B. Its main function is for oxygen transport. heart and muscle. Its structure is composed of 1 FELICIANO 2015
C. It is mainly affected by allosteric receptors. polypeptide only. In contrast, hemoglobin follows a VILLAMAT
D. It is found in the the muscles and red blood cells. sigmoidal curve and shows cooperativity. It is mainly ER, MD
E. It shows saturation in the oxygen dissociation curve. found in the red blood cells and is affected by (TOP 5 -
allosteric effectors whose interaction with one site of FEB 2015
hemoglobin affects the binding of oxygen to heme MED
groups at other location. It has 4 polypeptides. BOARDS;
TOPNOTCH
MD FROM
EAC)
128 Which of the following is true regarding glycemic index? Carbohydrates that are absorbed less rapidly are food LYNN MIDTERM 1
A. It is described as the increase in blood glucose after that have low glycemic index and are thus more DARYL EXAM - AUG
a test dose of a carbohydrate compared with that after beneficial since it causes less fluctuation in insulin FELICIANO 2015
an equivalent amount of glucose. secretion. Fructose and sugar alcohols are examples VILLAMAT
B. Foods that high glycemic index are considered to be of carbohydrates with low glycemic index. ER, MD
more beneficial since they cause less fluctuation in (TOP 5 -
insulin secretion. FEB 2015
C. Carbohydrates that are absorbed less rapidly have a MED
higher glycemic index. BOARDS;
D. Fructose and sugar alcohols have high glycemic TOPNOTCH
index. MD FROM
TOPNOTCH MEDICAL BOARD PREP BIOCHEMISTRY SUPEREXAM Page 17 of 92
For inquiries visit www.topnotchboardprep.com.ph or email us at topnotchmedicalboardprep@gmail.com
TOPNOTCH MEDICAL BOARD PREP BIOCHEMISTRY SUPEREXAM
For inquiries visit www.topnotchboardprep.com.ph or email us at topnotchmedicalboardprep@gmail.com
Item QUESTION EXPLANATION AUTHOR TOPNOTCH
# EXAM
E. None of the above. EAC)

129 Which statement is true regarding competitive Only option D is correct. All others are description of LYNN MIDTERM 1
inhibition? non-competitive inhibitor. DARYL EXAM - AUG
A. Km is constant. FELICIANO 2015
B. Inhibitor binds to allosteric site of the enzyme and VILLAMAT
halts the enzyme catalysis. ER, MD
C. Vmax is lowered. (TOP 5 -
D. Reversal of the inhibition is through increasing the FEB 2015
substrate. MED
E. Inhibitor changes the shape of the enzyme so it BOARDS;
cannot bind to the substrate. TOPNOTCH
MD FROM
EAC)
130 The most important step in cholesterol excretion: The provision of glycerol 3-phosphate regulates LYNN MIDTERM 1
A. Bile acid secretion esterification. Hormone-sensitive lipase catalyzes DARYL EXAM - AUG
B. Provision of glycerol-3-phosphate hydrolysis of triacylglycerol to form fatty acids and FELICIANO 2015
C. Action of hormone-sensitive lipase glycerol. Biosynthesis of mevalonate (enzyme is HMG- VILLAMAT
D. Biosynthesis of mevalonate CoA reductase is part of cholesterol synthesis. ER, MD
E. Action of lipoproteins Lipoprotein are important in transport About 1 g of (TOP 5 -
cholesterol is eliminated from the body FEB 2015
per day. Approximately half is excreted in the feces MED
after BOARDS;
conversion to bile acids. The remainder is excreted as TOPNOTCH
cholesterol. MD FROM
EAC)
131 What is/are the effects of epinephrine in the body as a Epinephrine, together with glucagon, cortisol, and LYNN MIDTERM 1
result of stressful stimuli? growth hormone counteract the actions of insulin. It DARYL EXAM - AUG
A. Stimulation of phosphorylase via generation of cAMP increases gluconeogenesis and glycogenolysis. Cyclic FELICIANO 2015
B. Increase in glycogenolysis AMP is formed from ATP by adenylyl cyclase and acts VILLAMAT
C. Inactivation of pyruvate kinase as an intracellular 2nd messenger in response to ER, MD
D. All of these. hormones such as epinephrine, NE, and glucagon. (TOP 5 -
E. A & B only. Phosphorylase in muscle is activated in response to FEB 2015
epinephrine acting via cAMP. Pyruvate kinase is an MED
irreversible reaction in glycolysis and is inhibited by BOARDS;
high levels of epinephrine, glucagon and ATP. TOPNOTCH
MD FROM
EAC)
132 The main difference of glucokinase from hexokinase Only option A is correct. The rest are characteristics of LYNN MIDTERM 1
A. It promotes increased hepatic utilization of glucose hexokinase. Glucokinase is present only in liver and DARYL EXAM - AUG
following a meal. islet cells of pancreas, inhibited by fructose 6 FELICIANO 2015
B. It has a low Km and thus, a high affinity for glucose. phosphate, has high Km/low affinity, high Vmax, and VILLAMAT
C. It acts at a constant rate under all normal condition. removes glucose from blood following a meal. ER, MD
D. It has a lower Vmax compared to hexokinase. (TOP 5 -
E. It is saturated under normal conditions and acts to FEB 2015
provide glucose-6-phosphate to meet the cell's need. MED
BOARDS;
TOPNOTCH
MD FROM
EAC)
133 How many ATPs are produced by NADH in the liver? Liver, kidney and heart utilizes the Malate-aspartate LYNN MIDTERM 1
A. 1 shuttle producing 3 ATPs=1 NADH. Skeletal muscle DARYL EXAM - AUG
B. 2 and brain uses glycerol-phosphate shuttle 1 NADH=2 FELICIANO 2015
C. 3 ATP. (SIMILAR TO PREVIOUS BOARD EXAM VILLAMAT
D. 4 CONCEPT) ER, MD
E. 5 (TOP 5 -
FEB 2015
MED
BOARDS;
TOPNOTCH
MD FROM
EAC)
134 The first proponent of the double helix DNA model SIMILAR TO PREVIOUS BOARD EXAM CONCEPT LYNN MIDTERM 1
A. Watson-Crick DARYL EXAM - AUG
B. Wilson-Crick FELICIANO 2015
C. Wilkins-Chargaff VILLAMAT
D. Wilkins-Crick ER, MD
E. Watson-Chargaff (TOP 5 -
FEB 2015
MED
BOARDS;
TOPNOTCH
MD FROM
EAC)

TOPNOTCH MEDICAL BOARD PREP BIOCHEMISTRY SUPEREXAM Page 18 of 92


For inquiries visit www.topnotchboardprep.com.ph or email us at topnotchmedicalboardprep@gmail.com
TOPNOTCH MEDICAL BOARD PREP BIOCHEMISTRY SUPEREXAM
For inquiries visit www.topnotchboardprep.com.ph or email us at topnotchmedicalboardprep@gmail.com
Item QUESTION EXPLANATION AUTHOR TOPNOTCH
# EXAM
135 The process by which a particular segment of DNA is SIMILAR TO PREVIOUS BOARD EXAM CONCEPT LYNN MIDTERM 1
copied by the RNA polymerase? DARYL EXAM - AUG
A. Replication FELICIANO 2015
B. Translation VILLAMAT
C. Transcription ER, MD
D. Initiation (TOP 5 -
E. Elongation FEB 2015
MED
BOARDS;
TOPNOTCH
MD FROM
EAC)
136 The following are effects of insulin except: Effects of insulin: WELL FED STATE. Increased in LYNN MIDTERM 1
A. Stimulation of glycogen synthase. glycogenesis (glycogen synthase), decreased DARYL EXAM - AUG
B. Inhibition of phosphofructokinase-2 glycogenolysis (inhibition of glycogen FELICIANO 2015
C. Inhibition of glycogen phosphorylase phosphorylase), decreased gluconeogenesis VILLAMAT
D. Decreased activity of fructose 1,6 biphosphatase (decreased activity of Fructose-1,6-biphosphatase), ER, MD
E. Activation of carboxylation of acetyl-CoA to malonyl- increase fatty acid synthesis (activation of acetyl CoA (TOP 5 -
CoA carboxylase involved in Acetyl coa --> Malonyl CoA). FEB 2015
Option B is the answer. Insulin activates PFK-2 which MED
activates PFK-1, which catalyzes the rate-limiting step BOARDS;
in glycolysis TOPNOTCH
MD FROM
EAC)
137 A 20-month old child born full terrm via vaginal delivery, β-galactosidase -- Krabbes; Ceramidase --Farber'ss LYNN MIDTERM 1
was brought to the hospital due to inability to sit up. She disease; Sphingomyelinase --Niemann-Pick; α- DARYL EXAM - AUG
was diagnosed to have Tay-Sach's disease. Which of the galactosidase -- Fabry's SIMILAR TO PREVIOUS FELICIANO 2015
following enzymes is deficient in this condition? BOARD EXAM CONCEPT VILLAMAT
A. Hexosaminidase A ER, MD
B. β-galactosidase (TOP 5 -
C. Ceramidase FEB 2015
D. Sphingomyelinase MED
E. α-galactosidase BOARDS;
TOPNOTCH
MD FROM
EAC)
138 A 10-month old infant was admitted due to vomiting and Fructose intolerance is an autosomal recessive LYNN MIDTERM 1
decreased sensorium after her mother gave him an disorder resulting from deficiency in Aldolase B. DARYL EXAM - AUG
orange juice. On history, he was born full term after an Patient present wirh hypoglycemia, jaundice, cirrhosis FELICIANO 2015
uncomplicated pregnancy, solely breastfed for the first 6 and vvomiting due to accumulation of fructose 1-P VILLAMAT
months. Urine was noted to contain non-reducing phosphate. (Decrease glycogenolysis, and decrease ER, MD
sugars. What is the patient's most likely condition? gluconeogenesis). In Essential fructosuria, there is a (TOP 5 -
A. Essential fructosuria defect in fructokinase. However, it is more benign and FEB 2015
B. Classic Galactosemia asymptomatic. Galactokinase deficiency presents with MED
C. Aldolase B deficiency galactosuria and cataracts in early childhood. Classic BOARDS;
D. Galactokinase deficiency galactosemia may present with vomiting, diarrhea TOPNOTCH
E. Aldose reductase deficiency after milk ingestion (not after ingestion of fruit juice). MD FROM
EAC)

139 Which of the following vitamins has a protective effect SIMILAR TO PREVIOUS BOARD EXAM CONCEPT. LYNN MIDTERM 1
by combating lipid peroxidation? DARYL EXAM - AUG
A. Vitamin A FELICIANO 2015
B. Pyridoxine VILLAMAT
C. Ascorbic Acid ER, MD
D. Vitamin D (TOP 5 -
E. Vitamin E FEB 2015
MED
BOARDS;
TOPNOTCH
MD FROM
EAC)
140 Which of the following antibiotics prevents binding of Chloramphenicol - inhibits peptidyltransferase; LYNN MIDTERM 1
aminoacyl-tRNA to the A site? Streptomycin - interferes with initiation of protein DARYL EXAM - AUG
A. Chloramphenicol synthesis, Rifampicin - binds to bacterial RNA FELICIANO 2015
B. Streptomycin polymerase, inhibiting RNA synthesis; Clindamycin - VILLAMAT
C. Tetracycline inhibit translocation ER, MD
D. Rifampicin (TOP 5 -
E. Clindamycin FEB 2015
MED
BOARDS;
TOPNOTCH
MD FROM
EAC)
141 Which of the following is not a participant in creatinine SIMILAR TO PREVIOUS BOARD EXAM EDWARD MIDTERM 2
formation? CONCEPT/PRINCIPLE. The AA lysine is not involved in HARRY EXAM - AUG
A. Lysine creatinine formation VALLAJER 2015
B. Arginine A, MD (TOP
C. Methionine 8 - FEB
D. Glycine 2015 MED
E. None of the above BOARDS;
TOPNOTCH
MD FROM
PERPETUA
L BINAN)

TOPNOTCH MEDICAL BOARD PREP BIOCHEMISTRY SUPEREXAM Page 19 of 92


For inquiries visit www.topnotchboardprep.com.ph or email us at topnotchmedicalboardprep@gmail.com
TOPNOTCH MEDICAL BOARD PREP BIOCHEMISTRY SUPEREXAM
For inquiries visit www.topnotchboardprep.com.ph or email us at topnotchmedicalboardprep@gmail.com
Item QUESTION EXPLANATION AUTHOR TOPNOTCH
# EXAM
142 Which of the following enzymes is elevated in Gout? The activity of xanthine oxidase is elevated in gout EDWARD MIDTERM 2
A. HGPRT because elevated activity of this enzyme causes HARRY EXAM - AUG
B. Adenosine deaminase increased formation of uric acid thus precipitating VALLAJER 2015
C. Xanthine Oxidase attacks of gout. The rest are enzymes which shunt A, MD (TOP
D. A & B purine derivatives back into the available nucleic acid 8 - FEB
E. None of the above pool and thus deplete xanthine oxidase of its 2015 MED
substrate. BOARDS;
TOPNOTCH
MD FROM
PERPETUA
L BINAN)
143 The sequencing of a genetic make up of an individual is Genetics is the study of genes, heredity, and genetic EDWARD MIDTERM 2
known as? variation in living organisms. Genomics is the study of HARRY EXAM - AUG
A. Genetics the genetic make-up of an individual VALLAJER 2015
B. Genomics A, MD (TOP
C. Gene Therapy 8 - FEB
D. Genetic Engineering 2015 MED
E. None of the above BOARDS;
TOPNOTCH
MD FROM
PERPETUA
L BINAN)
144 What is the principal cholesterol excreted in human Cholic acid and chenodeoxycholic acid are primary EDWARD MIDTERM 2
feces? bile acids secreted in bile, lithocholic acid is a HARRY EXAM - AUG
A. Cholic Acid secondary bile acid. Coprostanol is the primary VALLAJER 2015
B. Chenodeoxycholic acid cholesterol in the stool synthesized from cholesterol A, MD (TOP
C. Coprostanol from the GIT by the gut bacteria. 8 - FEB
D. Lithocholic acid 2015 MED
E. None of the above BOARDS;
TOPNOTCH
MD FROM
PERPETUA
L BINAN)
145 How many base pairs are there in the human genome? Reference: Harper's biochem EDWARD MIDTERM 2
A. 3 x 1010 HARRY EXAM - AUG
B. 3 x 1011 VALLAJER 2015
C. 3 x 1012 A, MD (TOP
D. 3 x 109 8 - FEB
E. 3 x 108 2015 MED
BOARDS;
TOPNOTCH
MD FROM
PERPETUA
L BINAN)
146 What is the enzyme is defective in Smith-Lemli-Opitz SIMILAR TO PREVIOUS BOARD EXAM EDWARD MIDTERM 2
syndrome? CONCEPT/PRINCIPLE. The enzyme 7- HARRY EXAM - AUG
A. HMG-CoA reductase dehydrocholesterol reductase is deficient in patients VALLAJER 2015
B. 25-Hydroxyvitamin D3 1-alpha-hydroxylase with Smith-Lemli-Opitz syndrome A, MD (TOP
C. HMG CoA synthase 8 - FEB
D. Acetyl CoA carboxylase 2015 MED
E. 7-dehydrocholesterol reductase BOARDS;
TOPNOTCH
MD FROM
PERPETUA
L BINAN)
147 Which of the following cells do not synthesize heme RBCs when they reach maturity do not synthesize EDWARD MIDTERM 2
despite being mature? heme anymore as they rely on the preformed heme HARRY EXAM - AUG
A. Erythroid Bone Marrow inside the cytoplasm, the other options are the VALLAJER 2015
B. RBC progenitor sources of the RBCs and therefore A, MD (TOP
C. Yolk Sac synthesize heme. 8 - FEB
D. Liver 2015 MED
E. Spleen BOARDS;
TOPNOTCH
MD FROM
PERPETUA
L BINAN)
148 RM, a 30 year old political activist was imprisoned, he At this point, ketogenesis predominates which EDWARD MIDTERM 2
was brought to the infirmary due to weakness, upon produces ketone bodies to serve as the body's main HARRY EXAM - AUG
questioning, you learned that he was deliberately energy source. VALLAJER 2015
withheld food for 2 weeks, thus when the brain is A, MD (TOP
deprived of glucose at this point in time, which of the 8 - FEB
following is expected to happen? 2015 MED
A. Glycolysis BOARDS;
B. Fatty acid synthesis TOPNOTCH
C. Cholesterol synthesis MD FROM
D. Ketogenesis PERPETUA
E. A & B L BINAN)

TOPNOTCH MEDICAL BOARD PREP BIOCHEMISTRY SUPEREXAM Page 20 of 92


For inquiries visit www.topnotchboardprep.com.ph or email us at topnotchmedicalboardprep@gmail.com
TOPNOTCH MEDICAL BOARD PREP BIOCHEMISTRY SUPEREXAM
For inquiries visit www.topnotchboardprep.com.ph or email us at topnotchmedicalboardprep@gmail.com
Item QUESTION EXPLANATION AUTHOR TOPNOTCH
# EXAM
149 RVD, a 60 year old man with hypertension and diabetes Anaerobic glycolysis yields only 2 ATP per molecule of EDWARD MIDTERM 2
mellitus came in due to chest pain, you know that the glucose HARRY EXAM - AUG
cause of his chest pain is lactic acid accumulation in the VALLAJER 2015
heart muscle. How many ATPs are generated by the A, MD (TOP
cardiac muscle in anaerobic respiration? 8 - FEB
A. 4 2015 MED
B. 2 BOARDS;
C. 16 TOPNOTCH
D. 8 MD FROM
E. 10 PERPETUA
L BINAN)
150 Pyruvate is oxidized to which substrate to enter the Acetyl CoA is formed when Pyruvate is oxidized by EDWARD MIDTERM 2
Kreb's cycle? pyruvate dehydrogenase in aerobic respiration, in the HARRY EXAM - AUG
A. Acetyl CoA absence of aerobic respiration as in hypoxia or lack of VALLAJER 2015
B. Lactic Acid mitochondria (e.g. RBC) the pyruvate is not oxidized A, MD (TOP
C. Phosphoenolpyruvate but is reduced by LDH to lactate. 8 - FEB
D. Glyceraldhyde 3-Phosphate 2015 MED
E. Oxaloacetate BOARDS;
TOPNOTCH
MD FROM
PERPETUA
L BINAN)
151 What is the complex amino acid alcohol found in Sphingosine is the complex amino acid alcohol found EDWARD MIDTERM 2
sphingomyelin? in sphingomyelin, ceramide is a combination of HARRY EXAM - AUG
A. Sphingosine sphingosine and a fatty acid. VALLAJER 2015
B. Ceramide A, MD (TOP
C. Phosphatidylserine 8 - FEB
D. Phosphatidylinositol 2015 MED
E. Lysophosphatidylcholine BOARDS;
TOPNOTCH
MD FROM
PERPETUA
L BINAN)
152 Sickle cell is an example of point mutation in the gene Valine is the amino acid that substituted glutamic acid EDWARD MIDTERM 2
coding for the globin molecule. What is the amino acid in the normal Hgb molecule. HARRY EXAM - AUG
implicated in the polymerization of hemoglobin in sickle VALLAJER 2015
cell disease A, MD (TOP
A. Aspartic acid 8 - FEB
B. Glutamic acid 2015 MED
C. Isoleucine BOARDS;
D. Valine TOPNOTCH
E. Phenylalanine MD FROM
PERPETUA
L BINAN)
153 An infant was brought to your clinic because of yellowish Crigler-Najjar and Gilbert's syndrome are defects in EDWARD MIDTERM 2
discoloration of the skin, upon PE, you noted jaundice. the conjugation of bilirubin while Rotor syndrome HARRY EXAM - AUG
Labs indicated elevated levels of direct bilirubin and also causes elevated direct bilirubin but the liver VALLAJER 2015
liver biopsy showed darkly stained hepatocytes, what is biopsy shows that the hepatocytes are not darkly A, MD (TOP
the most likely cause? stained unlike in Dubin-Johnson syndrome. 8 - FEB
A. Crigler-Najjar syndrome 2015 MED
B. Dubin-Johnson syndrome BOARDS;
C. Rotor syndrome TOPNOTCH
D. Gilbert's syndrome MD FROM
E. None of the above PERPETUA
L BINAN)
154 A patient came in to the ER suspected of suffering from a Clopidogrel inhibits the ADP receptor on platelet cell EDWARD MIDTERM 2
MI, you know that the mechanism of action of which of membranes HARRY EXAM - AUG
the following involves the inhibition of ADP binding to VALLAJER 2015
its receptor in the platelet cell membrane? A, MD (TOP
A. Aspirin 8 - FEB
B. Tirofiban 2015 MED
C. Abciximab BOARDS;
D. Clopidogrel TOPNOTCH
E. Dipyridamole MD FROM
PERPETUA
L BINAN)
155 These are enzymes whose main function is to catalyze EDWARD MIDTERM 2
the union of 2 molecules with hydrolysis of ATP HARRY EXAM - AUG
A. Ligases VALLAJER 2015
B. Lyases A, MD (TOP
C. Hydrolases 8 - FEB
D. Oxidoreductases 2015 MED
E. None of the above BOARDS;
TOPNOTCH
MD FROM
PERPETUA
L BINAN)
156 All of the following require the co-factor biotin except: It is catalyzed by pyruvate dehydrogenase and does EDWARD MIDTERM 2
A. pyruvate ---> oxaloacetate not require biotin as co-factor. HARRY EXAM - AUG
B. pyruvate ---> acetyl CoA VALLAJER 2015
C. acetyl CoA ---> malonyl CoA A, MD (TOP
D. propionyl CoA ---> succinyl CoA 8 - FEB
E. None of the above 2015 MED
BOARDS;
TOPNOTCH
TOPNOTCH MEDICAL BOARD PREP BIOCHEMISTRY SUPEREXAM Page 21 of 92
For inquiries visit www.topnotchboardprep.com.ph or email us at topnotchmedicalboardprep@gmail.com
TOPNOTCH MEDICAL BOARD PREP BIOCHEMISTRY SUPEREXAM
For inquiries visit www.topnotchboardprep.com.ph or email us at topnotchmedicalboardprep@gmail.com
Item QUESTION EXPLANATION AUTHOR TOPNOTCH
# EXAM
MD FROM
PERPETUA
L BINAN)

157 Our understanding today of DNA was made possible James Watson and Francis Crick were the first to EDWARD MIDTERM 2
through the contribution of? describe the double helical structure of DNA. HARRY EXAM - AUG
A. Oliver and Wilbur Wright VALLAJER 2015
B. Jane Goodall and Marie Leakey A, MD (TOP
C. Gregor Mendel and James Watson 8 - FEB
D. James Watson and Francis Crick 2015 MED
E. None of the above BOARDS;
TOPNOTCH
MD FROM
PERPETUA
L BINAN)
158 2,3-Bisphosphoglycerate formation is catalyzed by Bisphosphoglycerate mutase catalyzes the conversion EDWARD MIDTERM 2
which enzyme? of 1,3-BPG to 2,3-BPG HARRY EXAM - AUG
A. Phosphoglycerate kinase VALLAJER 2015
B. Phosphoglycerate mutase A, MD (TOP
C. Phosphotriose isomerase 8 - FEB
D. Bisphosphoglycerate mutase 2015 MED
E. Glyceraldehyde 3-phosphate dehydrogenase BOARDS;
TOPNOTCH
MD FROM
PERPETUA
L BINAN)
159 RDL, a 17 year old female was brought to you due to CO2 and H2O are the substrates of carbonic anhydrase EDWARD MIDTERM 2
stunted growth and recurrent fractures, a diagnosis of to form carbonic acid which in an aqueous solution HARRY EXAM - AUG
osteopetrosis was made, biopsy showed osteoclasts dissociates into H ions and bicarbonate ions VALLAJER 2015
lacking carbonic anhydrase activity, what are the A, MD (TOP
substrates of this enzyme? 8 - FEB
A. H+ and sodium 2015 MED
B. Potassium and HCO3- BOARDS;
C. CO2 and HCl TOPNOTCH
D. CO2 and H2O MD FROM
E. None of the above PERPETUA
L BINAN)

160 Acetyl Co-A combines with which substrate to enter the The first step in Kreb's cycle is when Acetyl Co-A EDWARD MIDTERM 2
Kreb's cycle? combines with oxaloacetate to form citrate catalyzed HARRY EXAM - AUG
A. Oxaloacetate by citrate synthase VALLAJER 2015
B. Malonate A, MD (TOP
C. Citrate 8 - FEB
D. Succinate 2015 MED
E. Alpha keto-glutarate BOARDS;
TOPNOTCH
MD FROM
PERPETUA
L BINAN)
161 Oxidative phosphorylation which involves several oligomycin inhibits complex 5 or atp synthase. HAROLD MIDTERM 3
compounds and complexes that transfer electron Barbiturates inhibit complex 1. cyanide and hydrogen JAY S. EXAM - AUG
between mitochondrial membranes is an efficient sulfate inhibits complex 4. malonate inhibits complex BAYTEC, 2015
process of ATP production in aerobic species. Therefore 3. MD (TOP
when one of its components is blocked, ATP production 10 - FEB
may cause pathologic disturbance. Which of the 2015 MED
following compounds inhibits ATP synthase? BOARDS;
A.Barbiturates TOPNOTCH
B.Oligomycin MD FROM
C.Cyanide FEU)
D.Malonate
E.Hydrogen Sulfate

162 How many net ATPs will be produced in a single glucose 4 will be produced 2 ATPs will be used (SIMILAR TO HAROLD MIDTERM 3
molecule when a cell undergoes anaerobic glycolysis? PREVIOUS BOARD EXAM CONCEPT/PRINCIPLE) JAY S. EXAM - AUG
A. 2 BAYTEC, 2015
B. 3 MD (TOP
C. 4 10 - FEB
D. 5 2015 MED
E. 0 BOARDS;
TOPNOTCH
MD FROM
FEU)
163 3. NADH is an important compound for ATP production 2 NADH will be produced but they will be used in HAROLD MIDTERM 3
in both aerobic and anaerobic glycolysis. What is the net converting pyruvate to lactate (SIMILAR TO JAY S. EXAM - AUG
NADH produced in a cell undergoing anaerobic PREVIOUS BOARD EXAM CONCEPT/PRINCIPLE) BAYTEC, 2015
glycolysis? MD (TOP
A. 0 10 - FEB
B. 1 2015 MED
C. 2 BOARDS;
D. 3 TOPNOTCH
E. 4 MD FROM
FEU)

TOPNOTCH MEDICAL BOARD PREP BIOCHEMISTRY SUPEREXAM Page 22 of 92


For inquiries visit www.topnotchboardprep.com.ph or email us at topnotchmedicalboardprep@gmail.com
TOPNOTCH MEDICAL BOARD PREP BIOCHEMISTRY SUPEREXAM
For inquiries visit www.topnotchboardprep.com.ph or email us at topnotchmedicalboardprep@gmail.com
Item QUESTION EXPLANATION AUTHOR TOPNOTCH
# EXAM
164 4. Which of the following enzyme is both involved in phosphoglycerate kinase can convert 1,3- HAROLD MIDTERM 3
gluconeogenesis and glycolysis? bisphosphoglycerate to 3-phosphoglycerate in JAY S. EXAM - AUG
A. PFK 1 glycolysis and vice versa for gluconeogenesis BAYTEC, 2015
B. Hexokinase (SIMILAR TO PREVIOUS BOARD EXAM MD (TOP
C. Pyruvate dehydrogenase CONCEPT/PRINCIPLE) 10 - FEB
D. Phosphoglycerate kinase 2015 MED
E. None of the above BOARDS;
TOPNOTCH
MD FROM
FEU)
165 5. Smith-Lemil-Opitz syndrome (SLOS) is an autosomal Previous board exam question concept. This is an HAROLD MIDTERM 3
recessive inborn error of cholesterol synthesis which autosomal recessive disorder characterized by JAY S. EXAM - AUG
causes broad spectrum of effects ranging from mild multiple malformation syndrome caused by mutation BAYTEC, 2015
intellectual disability and behavioral problems to lethal of 7-dehydrocholesterol reductase MD (TOP
malformations. Which of the following enzymes is 10 - FEB
defective in SLOS? 2015 MED
A. HMG Co A reductase BOARDS;
B. HMG Co A synthase TOPNOTCH
C. 7-dehydrocholesterol reductase MD FROM
D. 11-dehydrocholesterol synthase FEU)
E. carnitine transport
166 6. What is the Body Mass Index of a 34 year old male BMI= wt in kg/ height in m2.. Factor for pounds to kg HAROLD MIDTERM 3
patient who has a weight of190 lbs and a height of 5 foot is 2.2 while inches to meter is 0.0254 JAY S. EXAM - AUG
10 inches? BAYTEC, 2015
A. 27.3 MD (TOP
B. 32.2 10 - FEB
C. 26.6 2015 MED
D. 31.4 BOARDS;
E. 25.5 TOPNOTCH
MD FROM
FEU)
167 7. Which of the following sugar molecules is an epimer of mannose and glucose only differ structurally at carbon HAROLD MIDTERM 3
D-glucose at carbon number 2? number 2 JAY S. EXAM - AUG
A. fructose BAYTEC, 2015
B. galactose MD (TOP
C. mannose 10 - FEB
D. ribose 2015 MED
E. maltose BOARDS;
TOPNOTCH
MD FROM
FEU)
168 8. Which among the following amino acids is not Essential amino acids are Phenylalanine, valine, HAROLD MIDTERM 3
essential for human survival for as long as all the tryptophan, threonine, isoleucine, methionine, JAY S. EXAM - AUG
essential amino acids are present in a normal person’s histidine, arginine, leucine, lysine ( PVT TIM HALL BAYTEC, 2015
diet? always Argues never Tyres) MD (TOP
A. threonine 10 - FEB
B. tryptophan 2015 MED
C. tyrosine BOARDS;
D. lysine TOPNOTCH
E. Leucine MD FROM
FEU)
169 9. Which of the following vitamins is/are needed by pyruvate dehydrogenase needs Thiamine (B1), FAD HAROLD MIDTERM 3
pyruvate dehydrogenase to convert Pyruvate to Acetyl (B2), NAD (B3), Coenzyme A (B5), and lipoic acid JAY S. EXAM - AUG
CoA before it enters the Kreb's cycle? BAYTEC, 2015
A. B1 MD (TOP
B. B2 10 - FEB
C. B3 2015 MED
D. B5 BOARDS;
E. All of the above TOPNOTCH
MD FROM
FEU)
170 10. Which among the lipoproteins has the greatest the higher the density, the more its protein HAROLD MIDTERM 3
concentration of proteins? component JAY S. EXAM - AUG
A. chylomicrons BAYTEC, 2015
B. HDL MD (TOP
C. LDL 10 - FEB
D. VLDL 2015 MED
E. IDL BOARDS;
TOPNOTCH
MD FROM
FEU)
171 Sphingomyelin is an important constituent of myelin SIMILAR TO PREVIOUS BOARD EXAM HAROLD MIDTERM 3
sheath of nerves. What is the backbone structure of CONCEPT/PRINCIPLE JAY S. EXAM - AUG
sphingomyelin? BAYTEC, 2015
A. Sphingosine MD (TOP
B. Fatty Acid 10 - FEB
C. Palmitic acid 2015 MED
D. oligosaccharide BOARDS;
E. N-acetyl neuraminic acid TOPNOTCH
MD FROM
FEU)

TOPNOTCH MEDICAL BOARD PREP BIOCHEMISTRY SUPEREXAM Page 23 of 92


For inquiries visit www.topnotchboardprep.com.ph or email us at topnotchmedicalboardprep@gmail.com
TOPNOTCH MEDICAL BOARD PREP BIOCHEMISTRY SUPEREXAM
For inquiries visit www.topnotchboardprep.com.ph or email us at topnotchmedicalboardprep@gmail.com
Item QUESTION EXPLANATION AUTHOR TOPNOTCH
# EXAM
172 Tay-Sach's disease is a lipid storage disease which can (alpha galactosidase: Fabry's disease); (Ceraminidase: HAROLD MIDTERM 3
manifest as mental retardation, hypotonia and cherry Farbers); (beta-glucosidase: gauchers); (beta- JAY S. EXAM - AUG
red macula. Which of th following enzymes is deficient in galactosidase:Krabbe's disease) BAYTEC, 2015
the disease? MD (TOP
A. Alpha-galactosidase 10 - FEB
B. ceramidase 2015 MED
C. Beta-glucosidase BOARDS;
D. Hexosaminidase A TOPNOTCH
E. Beta-galactosidase MD FROM
FEU)
173 LT, a 24 year old male, went to a blood donation center (type A: N-acetyl galactosamine) (Type B: galactose) HAROLD MIDTERM 3
to donate blood. Upon examination, he found out that he (Type AB: both) (Type O:none) JAY S. EXAM - AUG
is blood type B. Which of the following BAYTEC, 2015
glycosphingolipids is found is his RBC? MD (TOP
A.N-acetyl-galactosamine 10 - FEB
B. galactose 2015 MED
C. N-acetyl-glucosamine BOARDS;
D. B and C TOPNOTCH
E. None MD FROM
FEU)
174 Which of the following techniques involves the use of Southern blot is for DNA; Northern blot is for RNA; HAROLD MIDTERM 3
radioactive DNA probe binding to sample RNA and this is Western blot is for proteins JAY S. EXAM - AUG
used to measure sizes and amounts of specific mRNA BAYTEC, 2015
molecules? MD (TOP
A. Southern Blot 10 - FEB
B. Western Blot 2015 MED
C. Northern Blot BOARDS;
D. Eastern Blot TOPNOTCH
E. ELISA MD FROM
FEU)
175 During transciption, a mutation happened which UGG was changed to UGA which is one of the stop HAROLD MIDTERM 3
changed the codon UGG-GGU-UGG-UGG-UGA to UGG- codons JAY S. EXAM - AUG
GGU-UGG-UGA-UGA. What kind of mutation happened? BAYTEC, 2015
A. Silent mutation MD (TOP
B. Missense mutation 10 - FEB
C. Frameshift mutation 2015 MED
D. Nonsense mutation BOARDS;
E. None of the above TOPNOTCH
MD FROM
FEU)
176 Lead poisoning causes several manifestations like Lead inhibits FALAD (ferrocheletase and ALA HAROLD MIDTERM 3
basophilic stipling of RBC, headache, memory loss, dehydratase) JAY S. EXAM - AUG
nausea, abdominal pain, neuropathy, and lead deposits BAYTEC, 2015
in some organ systems. Which of the following enzymes MD (TOP
is/are inactivated by lead? 10 - FEB
A. ALA dehydratase 2015 MED
B. Ferrochelatase BOARDS;
C. ALA synthase TOPNOTCH
D. Uroporphyrinogen MD FROM
E. A and B FEU)

177 Which of the following nucleotides are pyrimidines pyrimidines are cytosine, uracil, and thymine. Uracil HAROLD MIDTERM 3
found in an RNA molecule? and cytosine are for RNA while cytosine and thymine JAY S. EXAM - AUG
A. Uracil and Cytosine are for DNA BAYTEC, 2015
B. Thymine and Cytosine MD (TOP
C. Uracil ang Guanine 10 - FEB
D. Adenine and Guanin 2015 MED
E. Thymine and Guanine BOARDS;
TOPNOTCH
MD FROM
FEU)
178 Which of the following nucleotides is a purine found in purines are guanine and adenine and both are found HAROLD MIDTERM 3
DNA but not in RNA? in DNA and RNA. SIMILAR TO PREVIOUS BOARD JAY S. EXAM - AUG
A. thymine EXAM CONCEPT/PRINCIPLE BAYTEC, 2015
B. Uracil MD (TOP
C. Adenine 10 - FEB
D. Guanine 2015 MED
E. None of the above BOARDS;
TOPNOTCH
MD FROM
FEU)
179 Which of the following amino acids is purely ketogenic? purely ketogeneic amino acids are the two Ls (lysine HAROLD MIDTERM 3
A. Tyrosine and leucine), purely glucogenic are WIFY(tryptophan, JAY S. EXAM - AUG
B. Arginine isoleucine, phenyalanine and tyrosine) SIMILAR TO BAYTEC, 2015
C. Histidine PREVIOUS BOARD EXAM CONCEPT/PRINCIPLE MD (TOP
D. Methionie 10 - FEB
E. Lysine 2015 MED
BOARDS;
TOPNOTCH
MD FROM
FEU)

TOPNOTCH MEDICAL BOARD PREP BIOCHEMISTRY SUPEREXAM Page 24 of 92


For inquiries visit www.topnotchboardprep.com.ph or email us at topnotchmedicalboardprep@gmail.com
TOPNOTCH MEDICAL BOARD PREP BIOCHEMISTRY SUPEREXAM
For inquiries visit www.topnotchboardprep.com.ph or email us at topnotchmedicalboardprep@gmail.com
Item QUESTION EXPLANATION AUTHOR TOPNOTCH
# EXAM
180 Sickle cell anemia involves a mutation substituting glutamate is a polar amino acid while valine is HAROLD MIDTERM 3
thymine for adenine which causes the coding of Valine nonpolar. Valine will tend to fold the hemoglobin so JAY S. EXAM - AUG
instead of glutamate at position 6 of the hemoglobin beta that it will position itself to the internal parts of the BAYTEC, 2015
chain. Which of the following best describes the amino hgb MD (TOP
acid change in the disease? 10 - FEB
A. from non-polar amino acid to polar amino acid 2015 MED
B. from polar amino acid to non-polar amino acid BOARDS;
C. from acidic amino acid to basic amino acid TOPNOTCH
D. from basic amino acid to acidic amino acid MD FROM
E. from essential amino acid to non-essential amino FEU)
acid
181 Creatine is derived from the following amino acids (SIMILAR TO PREVIOUS BOARD EXAM JEAN FINAL EXAM -
except? CONCEPT/PRINCIPLE.). Creatine can be derived from PAOLO M. AUG 2015
A. methionine methionine, arginine and glysine. Choline is derived DELFINO,
B. lysine from methionine. Carnitine is from methionine and MD (TOP
C. arginine lysine. Carnosine and anserine are from alanine. 10 - FEB
D. glycine Spermine and spermidine are from methionine. 2015 MED
E. None of the above BOARDS;
TOPNOTCH
MD FROM
FATIMA)
182 Which of the following is found in the basement SIMILAR TO PREVIOUS BOARD EXAM JEAN FINAL EXAM -
membrane? CONCEPT/PRINCIPLE.. Heparan sulfate is found in the PAOLO M. AUG 2015
A. Chondroitin sulfate basement membrane of the kidney where it plays a DELFINO,
B. Hyaluronic acid major role in determining the charge selectiveness of MD (TOP
C. Dermatan sulfate glomerular filtration. Memorize this! Hyaluronic acid- 10 - FEB
D. Heparan sulfate found in synovial fluid, vitreous humor, loose 2015 MED
E. Heparin connective tissue. Chondroitin sulfate- cartilage, bone, BOARDS;
cornea. Keratan sulfate- cornea. Heparan sulfate- skin TOPNOTCH
fibroblast, aortic wall. Dermatan sulfate- wide MD FROM
distribution. FATIMA)
183 True statement about fructose intolerance. SIMILAR TO PREVIOUS BOARD EXAM JEAN FINAL EXAM -
A. It is autosomal recessive CONCEPT/PRINCIPLE.. topnotch biochem handout PAOLO M. AUG 2015
B. It is caused by deficiency of aldolase B page 17. DELFINO,
C. Symptoms include hypoglycemia, jaundice, vomiting MD (TOP
D. There is accumulation of fructose-1-phosphate 10 - FEB
E. All of the above 2015 MED
BOARDS;
TOPNOTCH
MD FROM
FATIMA)
184 The last step in the synthesis of cortisol is catalyzed by The order of enzymes used in cortisol synthesis: 17 -> JEAN FINAL EXAM -
what enzyme? 21 -> 11. Please refer to steroid hormone synthesis PAOLO M. AUG 2015
A. 17-α hydroxylase pathway diagram for better comprehension. You can DELFINO,
B. 21 hydroxylase easily eliminate 1-hydroxylase and 25-hydroxylase MD (TOP
C. 11-β hydroxylase because they are involved in Vit.D activation, not in 10 - FEB
D. 1- hydroxylase cortisol synthesis. 2015 MED
E. 25- hydroxylase BOARDS;
TOPNOTCH
MD FROM
FATIMA)
185 What is the only amino acid that undergoes oxidative Glutamate undergoes oxidative deamination to yield JEAN FINAL EXAM -
deamination in order to yield free ammonia? free ammonia which will be used in the urea cycle. PAOLO M. AUG 2015
A. arginine DELFINO,
B. glutamine MD (TOP
C. asparagine 10 - FEB
D. aspartate 2015 MED
E. glutamate BOARDS;
TOPNOTCH
MD FROM
FATIMA)
186 In the liver, how many ATPs can be produced from 1 mol 2 from substrate level phosphorylation and 6 from 2 JEAN FINAL EXAM -
of glucose via aerobic glycolysis? NADH that undergo oxidative phosphorylation. Take PAOLO M. AUG 2015
A. 6 note the transport shuttle used here is malate- DELFINO,
B. 10 aspartate shuttle (because it is in the liver), so 1 NADH MD (TOP
C. 2 yields 3 ATP. Whereas if glycerol phosphate shuttle is 10 - FEB
D. 8 used, 1 NADH yields only 2 ATP. 2015 MED
E. 38 BOARDS;
TOPNOTCH
MD FROM
FATIMA)
187 Which of the following statements about DNA is false? In DNA, adenine pairs with thymine while guanine JEAN FINAL EXAM -
A. DNA serves as storage of genetic information. pairs with cytosine. PAOLO M. AUG 2015
B. The most common form is the B-DNA DELFINO,
C. The 2 strands of the double helix are held in register MD (TOP
by H-bond 10 - FEB
D. Watson, Crick and Wilkins proposed the double- 2015 MED
stranded DNA molecule model in the 1950s BOARDS;
E. Adenine pairs with guanine while cytosine pairs with TOPNOTCH
thymine MD FROM
FATIMA)

TOPNOTCH MEDICAL BOARD PREP BIOCHEMISTRY SUPEREXAM Page 25 of 92


For inquiries visit www.topnotchboardprep.com.ph or email us at topnotchmedicalboardprep@gmail.com
TOPNOTCH MEDICAL BOARD PREP BIOCHEMISTRY SUPEREXAM
For inquiries visit www.topnotchboardprep.com.ph or email us at topnotchmedicalboardprep@gmail.com
Item QUESTION EXPLANATION AUTHOR TOPNOTCH
# EXAM
188 Which of these is true during fasting state? In fasting state, glycogen synthase is inactive and JEAN FINAL EXAM -
A. Glycogen synthase is dephosphorylated phosphorylated while glycogen phosphorylase is PAOLO M. AUG 2015
B. Glycogen synthase is active active and dephosphorylated. DELFINO,
C. Glycogen phosphorylase is phosphorylated MD (TOP
D. Glycogen phosphorylase is inactive 10 - FEB
E. None of the above 2015 MED
BOARDS;
TOPNOTCH
MD FROM
FATIMA)
189 What enzyme is defective in Richner-Hanhart syndrome? Tyrosinemia has 2 types: Type 1 is caused by a defect JEAN FINAL EXAM -
A. Tyrosine aminotransferase on the enzyme fumarylacetoacetate and affects PAOLO M. AUG 2015
B. Fumarylacetoacetate hydrolase hepatorenal. Type 2 (aka Richner-Hanhart Syndrome) DELFINO,
C. Phenylalanine hydroxylase is caused by defect in tyrosine aminotransferase and MD (TOP
D. Homogentisate oxidase has oculocutaneous effects. Note: expect at least 1 10 - FEB
E. Arginosuccinate synthase question about never-before-heard eponyms. 2015 MED
BOARDS;
TOPNOTCH
MD FROM
FATIMA)
190 Which of the following functions as the storage of genetic giveaway question last boards JEAN FINAL EXAM -
material? PAOLO M. AUG 2015
A. protein DELFINO,
B. lipid MD (TOP
C. nucleic acid 10 - FEB
D. carbohydrate 2015 MED
E. Both A and C BOARDS;
TOPNOTCH
MD FROM
FATIMA)
191 Which of the following is the main glucose transporter in Glut 1 and Glut 3 are both found primarily in the brain. JEAN FINAL EXAM -
the brain? Glut 1 is the most abundant and is expressed in high PAOLO M. AUG 2015
A. Glut 1 levels in erythrocytes and endothelial cells of the DELFINO,
B. Glut 2 blood brain barrier, whereas Glut3 is expressed MD (TOP
C. Glut 3 mostly in neurons. So yes, I think Glut 3 is the better 10 - FEB
D. Glut 4 answer between the two since it is particularly 2015 MED
E. Glut 5 present in the parenchyma of the brain. BOARDS;
TOPNOTCH
MD FROM
FATIMA)
192 Which of the following relies on anaerobic glycolysis as Glycolysis in erythrocytes always terminates in JEAN FINAL EXAM -
source of energy? lactate, because the subsequent reactions of pyruvate PAOLO M. AUG 2015
A. RBC oxidation are mitochondrial, and erythrocytes lack DELFINO,
B. Renal medulla mitochondria. Other tissues that normally derive MD (TOP
C. Retina much of their energy from glycolysis and produce 10 - FEB
D. All of the above lactate include brain, gastrointestinal tract, renal 2015 MED
E. A and C medulla, retina, and skin. BOARDS;
TOPNOTCH
MD FROM
FATIMA)
193 Which of these enzymes is used in both glycolysis and phosphoglycerate kinase catalyzes a reversible JEAN FINAL EXAM -
gluconeogenesis? reaction, thus utilized in both glycolysis and PAOLO M. AUG 2015
A. Hexokinase gluconeogenesis. DELFINO,
B. Phosphoglycerate kinase MD (TOP
C. Phosphofructokinase I 10 - FEB
D. Pyruvate kinase 2015 MED
E. Glucokinase BOARDS;
TOPNOTCH
MD FROM
FATIMA)
194 True statement about Kreb's Cycle except? 1 mol of GTP (not ATP) is produced via substrate level JEAN FINAL EXAM -
A. 1 mol of ATP is produced via substrate level phosphorylation in the Kreb's cycle. PAOLO M. AUG 2015
phosphorylation DELFINO,
B. 3 NADH and 1 FADH2 are produced per turn MD (TOP
C. The rate limiting step is catalyzed by isocitrate 10 - FEB
dehydrogenase 2015 MED
D. The enzyme that catalyzes substrate level BOARDS;
phosphorylation is succinate thiokinase TOPNOTCH
E. α-ketoglutarate dehydrogenase is an enzyme MD FROM
complex that requires vitamins B1, B2, B3, B5 and lipoic FATIMA)
acid
195 Amyloid seen in Alzheimers is an example of what level SIMILAR TO PREVIOUS BOARD EXAM JEAN FINAL EXAM -
of protein structure? CONCEPT/PRINCIPLE. PAOLO M. AUG 2015
A. Primary DELFINO,
B. Secondary MD (TOP
C. Tertiary 10 - FEB
D. Quaternary 2015 MED
E. All of the above BOARDS;
TOPNOTCH
MD FROM
FATIMA)

TOPNOTCH MEDICAL BOARD PREP BIOCHEMISTRY SUPEREXAM Page 26 of 92


For inquiries visit www.topnotchboardprep.com.ph or email us at topnotchmedicalboardprep@gmail.com
TOPNOTCH MEDICAL BOARD PREP BIOCHEMISTRY SUPEREXAM
For inquiries visit www.topnotchboardprep.com.ph or email us at topnotchmedicalboardprep@gmail.com
Item QUESTION EXPLANATION AUTHOR TOPNOTCH
# EXAM
196 A patient was diagnosed with a hereditary condition SIMILAR TO PREVIOUS BOARD EXAM JEAN FINAL EXAM -
affecting the mitochondria. What would be the most CONCEPT/PRINCIPLE. PAOLO M. AUG 2015
likely manifestation? DELFINO,
A. myopathy MD (TOP
B. mental retardation 10 - FEB
C. stroke 2015 MED
D. seizure BOARDS;
E. MI TOPNOTCH
MD FROM
FATIMA)
197 Which among these amino acids is polar and uncharged? serine, threonine and tyrosine contain hydroxyl JEAN FINAL EXAM -
A. serine groups making it polar but uncharged. PAOLO M. AUG 2015
B. threonine DELFINO,
C. tyrosine MD (TOP
D. A and C 10 - FEB
E. All of the above 2015 MED
BOARDS;
TOPNOTCH
MD FROM
FATIMA)
198 Hormones that bind to cytoplasmic receptor Among all the hormones that bind to intracellular JEAN FINAL EXAM -
A. cortisol receptors, only aldosterone and cortisol have PAOLO M. AUG 2015
B. aldosterone receptors located in the cytoplasm. Others are in the DELFINO,
C. testosterone nucleus. MD (TOP
D. A and B 10 - FEB
E. None of the above 2015 MED
BOARDS;
TOPNOTCH
MD FROM
FATIMA)
199 What is the mechanism of action of tetracycline? A- rifampicin. B- chloramphenicol. D- Aminoglycoside. JEAN FINAL EXAM -
A. Inhibits DNA-dependent RNA polymerase E. Clindamycin and macrolides PAOLO M. AUG 2015
B. Inhibits prokaryotic peptidyltransferase and peptide DELFINO,
bond formation MD (TOP
C. prevents binding of aminoacyl-tRNAs to the A site 10 - FEB
D. Binds to 30s subunit and blocks the formation of 2015 MED
initiation complex BOARDS;
E. binds to 50s subunit and inhibit translocation TOPNOTCH
MD FROM
FATIMA)
200 What do adipose tissues use for the synthesis of TAG? Glycerokinase is only present in liver. DHAP is an JEAN FINAL EXAM -
A. diacylglycerol intermediate of glycolysis which undergoes reduction PAOLO M. AUG 2015
B. Phosphatidic acid to become glycerol phosphate. DELFINO,
C. Through a reaction catalyzed by glycerokinase MD (TOP
D. DHAP 10 - FEB
E. Acetyl coa 2015 MED
BOARDS;
TOPNOTCH
MD FROM
FATIMA)
201 Zero-order kinetics of enzyme reactions occur when: In zero-order kinetics, the rate of reaction depends on GRACE DIAGNOSTIC
A. Substrate concentration is above Km the availability of enzyme and not in the substrate ARVIOLA, EXAM - FEB
B. Velocity of enzyme reaction is below Vmax concentration. This occurs when all enzymes are MD (TOP 3 2015
C. Allosteric effectors are present active; that is, when the substrate concentration is - AUG 2014
D. The Lineweaver-Burke plot assumes a sigmoidal above Km. MED
curve BOARDS;
E. Enzyme inhibitors are not present TOPNOTCH
MD)
202 In oxidative phosphorylation, energy is generated when The electron transport chain occurs in the GRACE DIAGNOSTIC
hydrogen accumulates in the ________. mitochondrial matrix. However, the proton gradient ARVIOLA, EXAM - FEB
A. Cytosol created by the accumulation of hydrogen atoms MD (TOP 3 2015
B. Mitochondrial matrix occurs in the intermembranous space. - AUG 2014
C. Intermembranous space MED
D. Inner mitochondrial membrane BOARDS;
E. Outer mitochondrial membrane TOPNOTCH
MD)

203 Which of the following is NOT true of anabolic reactions? Anabolic reactions are divergent. GRACE DIAGNOSTIC
A. Endergonic ARVIOLA, EXAM - FEB
B. Convergent MD (TOP 3 2015
C. ΔG > 0 - AUG 2014
D. Non-spontaneous MED
E. Synthesis of compounds occurs. BOARDS;
TOPNOTCH
MD)
204 Which among the following is amphibolic? Amphibolic means a link between anabolic and GRACE DIAGNOSTIC
A. Glycolysis catabolic systems. The TCA is an amphibolic cycle. ARVIOLA, EXAM - FEB
B. TCA MD (TOP 3 2015
C. ETC - AUG 2014
D. PPP MED
E. Urea cycle BOARDS;
TOPNOTCH
MD)

TOPNOTCH MEDICAL BOARD PREP BIOCHEMISTRY SUPEREXAM Page 27 of 92


For inquiries visit www.topnotchboardprep.com.ph or email us at topnotchmedicalboardprep@gmail.com
TOPNOTCH MEDICAL BOARD PREP BIOCHEMISTRY SUPEREXAM
For inquiries visit www.topnotchboardprep.com.ph or email us at topnotchmedicalboardprep@gmail.com
Item QUESTION EXPLANATION AUTHOR TOPNOTCH
# EXAM
205 Which enzyme catalyzes a reversible reaction in Choices B, C, and D catalyze irreversible reactions in GRACE DIAGNOSTIC
glycolysis? glycolysis. ARVIOLA, EXAM - FEB
A. phosphoglycerate kinase MD (TOP 3 2015
B. phosphofructokinase-1 - AUG 2014
C. hexokinase MED
D. pyruvate kinase BOARDS;
E. glycogen synthase TOPNOTCH
MD)
206 What is an important role of phosphofructokinase-2 in In the well-fed state, fructose-2,6-bisphosphate is the GRACE DIAGNOSTIC
the well-fed state? most potent activator of PFK-1, the rate-limiting ARVIOLA, EXAM - FEB
A. It promotes glycogenesis enzyme in glycolysis, so that the glycolytic cycle will MD (TOP 3 2015
B. It inactivates insulin continue. - AUG 2014
C. It forms fructose-2,6-bosphosphate MED
D. It inhibits glycolysis BOARDS;
E. It metabolizes sucrose into fructose and glucose. TOPNOTCH
MD)
207 Which form of arsenic inhibits dehydrogenases by its Arsenite (the trivalent form of arsenic) forms a stable GRACE DIAGNOSTIC
affinity to lipoic acid? complex with the thiol (-SH) groups of lipoic acid, ARVIOLA, EXAM - FEB
A. Univalent make this compound unavailable to serve as a MD (TOP 3 2015
B. Divalent coenzyme. - AUG 2014
C. Trivalent MED
D. Tetravalent BOARDS;
E. Pentavalent TOPNOTCH
MD)
208 Which TCA enzyme is located in the inner mitochondrial Succinate dehydrogenase is the only enzyme of the GRACE DIAGNOSTIC
membrane? TCA that is not found in the mitochondrial matrix. ARVIOLA, EXAM - FEB
A. Aconitase Instead, it functions in the inner mitochondrial MD (TOP 3 2015
B. Alpha-ketoglutarate dehydrogenase membrane. - AUG 2014
C. Succinate thiokinase MED
D. Succinate dehydrogenase BOARDS;
E. Fumarase TOPNOTCH
MD)
209 In glycogenesis, bonds between glucose residues are Elongation of a glycogen chain involves the transfer of GRACE DIAGNOSTIC
formed at the non-reducing end of the growing chain. glucose from UDP-glucose to the non-reducing end of ARVIOLA, EXAM - FEB
This is carbon ____. the growing chain, forming a new glycosidic bond MD (TOP 3 2015
A. 1 between the anomeric hydroxyl of carbon 1 of the - AUG 2014
B. 2 activated glucose and carbon 4 of the accepting MED
C. 3 glucosyl residue. Hence, the alpha(1→4) linkages. BOARDS;
D. 4 TOPNOTCH
E. 5 MD)

210 This is the most important product of the pentose Ribose-5-phosphate is a precursor of the nucleotides GRACE DIAGNOSTIC
phosphate pathway. DNA and RNA. ARVIOLA, EXAM - FEB
A. ATP MD (TOP 3 2015
B. NADPH - AUG 2014
C. Ribose-5-phosphate MED
D. GTP BOARDS;
E. NADH TOPNOTCH
MD)
211 What form of hemoglobin is produced when 2,3-BPG When 2,3-BPG combines with hemoglobin, it forms GRACE DIAGNOSTIC
forms salt bridges with hemoglobin thus promoting salt bridges leading to the formation of the T ('taut") ARVIOLA, EXAM - FEB
oxygen release? form of hemoglobin. This form has lesser affinity to MD (TOP 3 2015
A. T form oxygen. Oxygen is, thus, released for use by the - AUG 2014
B. R form tissues. MED
C. S form BOARDS;
D. C form TOPNOTCH
E. A form MD)

212 What functional group is absent in DNA rendering it In contrast to RNA, DNA has no 2-hydroxyl group. GRACE DIAGNOSTIC
more stable in alkali solutions? Hence, its name 2-deoxyribonucleic acid. ARVIOLA, EXAM - FEB
A. 1-carbonyl group MD (TOP 3 2015
B. 2-hydroxyl group - AUG 2014
C. 1-phosphate group MED
D. 2-sulfate group BOARDS;
E. 1-alkyl group TOPNOTCH
MD)

213 What amino acid plays a crucial part in hemoglobin's Histidine located at F8 and E7 positions of hemoglobin GRACE DIAGNOSTIC
affinity to oxygen? participate in oxygen binding. ARVIOLA, EXAM - FEB
A. Glycine MD (TOP 3 2015
B. Tyrosine - AUG 2014
C. Histidine MED
D. Leucine BOARDS;
E. Proline TOPNOTCH
MD)
214 Sucrose is a non-reducing sugar because __________. The reducing property of sugars is determined by its GRACE DIAGNOSTIC
A. It is not a pentose free anomeric carbon. ARVIOLA, EXAM - FEB
B. It has no hydroxyl group MD (TOP 3 2015
C. It is a disaccharide - AUG 2014
D. It's anomeric carbon is not free MED
E. Its enantiomers do not exist BOARDS;
TOPNOTCH
MD)

TOPNOTCH MEDICAL BOARD PREP BIOCHEMISTRY SUPEREXAM Page 28 of 92


For inquiries visit www.topnotchboardprep.com.ph or email us at topnotchmedicalboardprep@gmail.com
TOPNOTCH MEDICAL BOARD PREP BIOCHEMISTRY SUPEREXAM
For inquiries visit www.topnotchboardprep.com.ph or email us at topnotchmedicalboardprep@gmail.com
Item QUESTION EXPLANATION AUTHOR TOPNOTCH
# EXAM
215 What is the important role of pyruvate carboxylase in Pyruvate carboxylase catalyzes the conversion of GRACE DIAGNOSTIC
the Krebs Cycle? pyruvate to oxaloacetate so that enough oxaloacetate ARVIOLA, EXAM - FEB
A. It depletes pyruvate thereby allowing glycolysis to is present to combine with pyruvate in the Krebs MD (TOP 3 2015
continue cycle. - AUG 2014
B. It prevents flooding of intermediates in the TCA MED
C. It conserves carbon atoms by generating two BOARDS;
molecules of carbon dioxide TOPNOTCH
D. It inhibits lactate dehydrogenase MD)
E. It ensures that enough oxaloacetate is present
216 This glycosaminoglycan is implicated in atherosclerosis Dermatan sulfate appears to be the major GAG GRACE DIAGNOSTIC
as it is known to bind LDL in arteries. synthesized by arterial smooth muscle cells, cells ARVIOLA, EXAM - FEB
A. Heparan sulfate which are known to proliferate in atherosclerotic MD (TOP 3 2015
B. Keratan sulfate lesions. - AUG 2014
C. Dermatan sulfate MED
D. Chondroitin sulfate BOARDS;
E. Hyaluronic acid TOPNOTCH
MD)

217 The Van Den Bergh reaction measures: More specifically, it measures the amount of GRACE DIAGNOSTIC
A. Glycosylated hemoglobin conjugated bilirubin in the blood. ARVIOLA, EXAM - FEB
B. Bilirubin MD (TOP 3 2015
C. Creatinine - AUG 2014
D. Amylase MED
E. Ck-MB BOARDS;
TOPNOTCH
MD)
218 Which B vitamin is part of fatty acid synthesis? Pantothenate or vitamin B5 is a component of the GRACE DIAGNOSTIC
A. B1 fatty acyl coA synthase complex of fatty acid synthesis. ARVIOLA, EXAM - FEB
B. B2 MD (TOP 3 2015
C. B3 - AUG 2014
D. B5 MED
E. B6 BOARDS;
TOPNOTCH
MD)
219 Which of the following is TRUE regarding uncouplers of Uncouplers increase the permeability of the inner GRACE DIAGNOSTIC
the ETC? mitochondrial membrane to hydrogen ions. As a ARVIOLA, EXAM - FEB
A. A proton gradient is formed; however, no ATP is result, no protein gradient is formed. Complex V is MD (TOP 3 2015
generated. affected. Aspirin is an uncoupler. - AUG 2014
B. Uncouplers decrease the permeability of the MED
intermembranous space to hydrogen atoms. BOARDS;
C. There is increased oxygen consumption. TOPNOTCH
D. Uncouplers affect complexes I to IV of the ETC. MD)
E. Acetylsalicylic acid is not an uncoupler.
220 In DNA transcription, the template strand is also known The template strand is the one that is GRACE DIAGNOSTIC
as the ________. read/copied/transcribed during DNA transcription to ARVIOLA, EXAM - FEB
A. Coding strand form the mRNA transcript. It is also known as the non- MD (TOP 3 2015
B. Non-coding strand coding strand and is complementary to the mRNA - AUG 2014
C. Intron transcript. The other strand is called the coding strand MED
D. Exon because this strand bears the exact same code as the BOARDS;
E. None of the above mRNA transcript (with the exception of T for U TOPNOTCH
changes). MD)
221 Which of the following is true about myoglobin except? SIMILAR TO PREVIOUS BOARD EXAM LEAN MIDTERM
A.it contains a tetrapyrrole ring connected to its CONCEPT/PRINCIPLE. Bohr effect is dependent ANGELO EXAM 1 - FEB
individual components through methylene bridges cooperative interaction between hemes of more than SILVERIO, 2015
B. it it similar to the secondary-tertiary structure of the one domain.this is only represented in hemoglobin MD (TOP 4
hemoglobin which has a tetrameric structure. Letter A is correct - AUG 2014
C. it exhibits bohr effect during extreme oxygen since myoglobin contains a heme protein. B- is also MED
deprivation correct, since the only major difference between BOARDS;
D. oxidation of Fe2 to Fe3 completely destroys its hemoglobin and myoglobin is that the latter has only TOPNOTCH
biologic activity domain while the former exhibits 4 domains. their MD), MD
E. it has only one domain therefore it is monomeric. alpha helix and tertiary configuration is very similar.
both myoglobin and hemoglobin needs a reduced
state of iron (ferrous form) to allow binding of oxygen
for storage or for transport.
222 which of the following is true about a competitive Competitive inhibition is best extrapolated using line LEAN MIDTERM
enzyme inhibitor? weaver burk plot since it can easily demonstrate the ANGELO EXAM 1 - FEB
A. It can be best extrapolated using Michaelis Menten values of Km and Vmax. It is also called as substrate SILVERIO, 2015
equation analog they bind to the catalytic site decreasing the MD (TOP 4
B. It increases the value of the Km number of free enzyme molecules availabe for - AUG 2014
C. It affects the Vmax by one half during high levels of substrate binding. kinetics will show that affinity will MED
inhibitor compared to original substrate be decreased as manifested by increase in Km ( note BOARDS;
D. it acts by decreasing the number of free substrate that Km and affinity has inverse relationship) and TOPNOTCH
molecules availabole to bind to the enzyme with no effect to the Vmax regardless of the amount of MD), MD
E. all of the above are correct the inhibitor
223 which of the following sites of respiratory chain does barbiturates exert its inhibitory effect by binding to LEAN MIDTERM
Barbiturates act? complex I. Malonate is an inhibitor of complex II, ANGELO EXAM 1 - FEB
A. Complex I Antimycin A and Dimercaprol is for Complex III, SILVERIO, 2015
B. Complex II Hydrogen sulfide, carbon monoxide and cyanide MD (TOP 4
C. Complex III inhibits complex IV and lastly oligomycin inhibits ATP - AUG 2014
D. Complex IV synthase. MED
E. ATP synthase BOARDS;
TOPNOTCH
MD), MD

TOPNOTCH MEDICAL BOARD PREP BIOCHEMISTRY SUPEREXAM Page 29 of 92


For inquiries visit www.topnotchboardprep.com.ph or email us at topnotchmedicalboardprep@gmail.com
TOPNOTCH MEDICAL BOARD PREP BIOCHEMISTRY SUPEREXAM
For inquiries visit www.topnotchboardprep.com.ph or email us at topnotchmedicalboardprep@gmail.com
Item QUESTION EXPLANATION AUTHOR TOPNOTCH
# EXAM
224 which of the following correctly describes Glucokinase? Glucokinase is predominantly found in the liver and LEAN MIDTERM
A. Found in hepatocytes beta cells of the pancreas. It has a higher Km ANGELO EXAM 1 - FEB
B. High affinity to glucose compared to Hexokinase, therefore it only SILVERIO, 2015
C. Low Vmax phosphorylate glucose during high glucose load ( after MD (TOP 4
D. Can be inhibited by glucose 6 phosphate meal consumption). It has a high Vmax such that it - AUG 2014
E. None of the above effectively remove glucose from portal blood. MED
furthermore, it is not allosterically inhibited by its BOARDS;
product Glucose 6 phosphate. all the other choices TOPNOTCH
describes Hexokinase which is most tissues of the MD), MD
body.
225 What is the most common glycolytic enzyme deficiency? Pyruvate kinase deficiency is responsible for about LEAN MIDTERM
A. Glucokinase 96% of patient suffering from glycolytic enzyme ANGELO EXAM 1 - FEB
B. Pyruvate kinase defect. SILVERIO, 2015
C. Glucose phosphate isomerase MD (TOP 4
D. Phosphoglycerate kinase - AUG 2014
E. Phosphoglycerate mutase MED
BOARDS;
TOPNOTCH
MD), MD
226 which of the following is an effect of high glucagon high glucagon/insulin ratio will promote LEAN MIDTERM
states? gluconeogenesis through a cascade of events: increase ANGELO EXAM 1 - FEB
A. Phosphorylation of PFK2 leading to decrease activity cAMP leads to increase Protein Kinase A which will SILVERIO, 2015
B. Phosphorylation of Fructose 1,6 biphosphatase leads inactivate PFK2 and activate FBP both via MD (TOP 4
to decrease activity phosphorylation. - AUG 2014
C. Decrease levels of Protein kinase A MED
D. all of the above BOARDS;
E. none of the above TOPNOTCH
MD), MD
227 which of the following enzyme catalyze the formation of Succinyl CoA synthetase or succinate thiokinase LEAN MIDTERM
GTP via substrate level phosphorylation in Kreb cycle? converts succinyl CoA to succinate in mammalian ANGELO EXAM 1 - FEB
A. Succinate dehydrogenase tissues resulting to phosphorylation of GDP to GTP. SILVERIO, 2015
B. Succinyl CoA synthetase MD (TOP 4
C. Aconitase - AUG 2014
D. Isocitrate dehydrogenase MED
E. Fumarase BOARDS;
TOPNOTCH
MD), MD
228 In Kreb cycle, which of the following dehydrogenase in kreb cycle, it is necessary to reduce citrate ( a C6 LEAN MIDTERM
catalyze the last step for CO2 production? atom) to oxaloacetate ( C4 atom). There are only two ANGELO EXAM 1 - FEB
A. Isocitrate dehydrogenase sites for CO2 production. The first step is catalyzed by SILVERIO, 2015
B. Malate dehydrogenase isocitrate dehydrogenase and the last is by the alpha MD (TOP 4
C. Succinate dehydrogenase ketoglutarate dehydrogenase. All the other proceeding - AUG 2014
D. Alpha ketoglutarate dehydrogenase dehydrogenase will only reduce FAD or NAD without MED
E. None of the above forming CO2. BOARDS;
TOPNOTCH
MD), MD
229 A 1 month old male presents with severe hypotonia and Glycogen storage disease type II aka Pompe disease is LEAN MIDTERM
cardiomegaly, which enzyme deficiency most likely a fatal condition most commonly manifested during ANGELO EXAM 1 - FEB
represent this type of glycogen storage disease. the first month of life. It primarily affects the heart SILVERIO, 2015
A. Glucose 6 phosphatase muscle causing cardiomyopathy. This is secondary to MD (TOP 4
B. Acid maltase deficiency of acid maltase of alpha 1-4 glucosidase. - AUG 2014
C. Glycogen debranching enzyme MED
D. muscle phosphorylase BOARDS;
E. none of the above TOPNOTCH
MD), MD
230 which of the following is a correctly paired enzyme Hunter syndrome is the only X linked type of LEAN MIDTERM
deficiency among mucopolysaccharidoses except? mucopolysaccharidosis characterized by deficiency in ANGELO EXAM 1 - FEB
A. Hurler syndrome : a-L iduronidase iduronate sulfatase. It presents with mental SILVERIO, 2015
B. Hunter syndrome: N - acetyltransferase retardation, variable physical deformities however, MD (TOP 4
C. Sly syndrome: B- Glucuronidase minimal to no corneal clouding. It is secondary to - AUG 2014
D. Sanfilippo syndrome: Heparan sulfamidase accumulation of dermatan sulfate and heparan sulfate. MED
E. none of the above BOARDS;
TOPNOTCH
MD), MD
231 which of the following is a false statement regarding acetyl CoA carboxylase is activated by citrate and LEAN MIDTERM
acetyl CoA carboxylase? inhibited by its product palmitate. ANGELO EXAM 1 - FEB
A. It is activated by palmitate SILVERIO, 2015
B. It is inhibited by phosphorylation during high MD (TOP 4
glucagon/insulin ratio - AUG 2014
C. It requires biotin MED
D. It is the rate limiting step in fatty acid synthesis BOARDS;
E. only A and C are incorrect statement TOPNOTCH
MD), MD
232 this lack of enzyme is responsible why the liver cannot acetoacetate: succinylCoa transferase or thiophorase LEAN MIDTERM
utilize its own ketone bodies? is responsible for conversion of acetoacetate to its CoA ANGELO EXAM 1 - FEB
A. Acetoacetate: succinyl CoA transferase derivative. This is present in almost all tissues but SILVERIO, 2015
B. B- ketothiolase absent in the liver. MD (TOP 4
C. HMG CoA lyase - AUG 2014
D. B hydroxybutyrate dehydrogenase MED
E. None of the above BOARDS;
TOPNOTCH
MD), MD

TOPNOTCH MEDICAL BOARD PREP BIOCHEMISTRY SUPEREXAM Page 30 of 92


For inquiries visit www.topnotchboardprep.com.ph or email us at topnotchmedicalboardprep@gmail.com
TOPNOTCH MEDICAL BOARD PREP BIOCHEMISTRY SUPEREXAM
For inquiries visit www.topnotchboardprep.com.ph or email us at topnotchmedicalboardprep@gmail.com
Item QUESTION EXPLANATION AUTHOR TOPNOTCH
# EXAM
233 A 27 y/o female presents with hypertriglyceridemia with A- is a pathologic basis of familial LEAN MIDTERM
a normal or low cholesterol count. Which of the familial hyperbetalipoproteinemia. In this disease, the ANGELO EXAM 1 - FEB
syndrome will explain the abnormal lipid profile of the cholesterol rises (inc LDL) while the clearance of SILVERIO, 2015
patient? chylomicrons and VLDL is normal ( Normal TAG). B- MD (TOP 4
A. LDL receptor defect impaired clearance of VLDL and chylomicrons by an - AUG 2014
B. Lipoprotein lipase deficiency absent or abnormal lipoprotein lipase will lead to MED
C. overproduction of VLDL increase TAG. since less is transformed into LDL, BOARDS;
D. apolipoprotein E deficiency cholesterol will be normal or low. overproduction of TOPNOTCH
E. none of the above VLDL alone will lead to increase in both Cholesterol MD), MD
and TAG. Apolipoprotein E deficiency decreasing the
clearance of VLDL remnant, will result to increase
VLDL,LDL and TAG.
234 what type of cell is affected in terms of glucose transport Among the GLUT transporters, it is only GLUT 4 that is LEAN MIDTERM
if there is an insulin deficiency states? insulin dependent. It is primarily distributed in two ANGELO EXAM 1 - FEB
A. Neurons major cell population: the adipocytes and myocytes. SILVERIO, 2015
B. Pancreatic acinar cells Pancreatic acinar cells particularly beta cells and MD (TOP 4
C. Adipocytes hepatocytes uses Glut 2 receptors, Neuorns and RBC - AUG 2014
D. Erythrocytes uses GLut 1 receptors. MED
E. Hepatocytes BOARDS;
TOPNOTCH
MD), MD
235 Which of the following intermediate products of heme Lead intoxication affects two enzyme in the heme LEAN MIDTERM
synthesis will accumulate if the patient suffers from lead synthesis. It is ALA dehydratase and ferrochelatase. ANGELO EXAM 1 - FEB
intoxication? The former combines two molecules of d-ALA to form SILVERIO, 2015
A. Succinyl CoA Porphobilinogen, while ferrochelatase catalyzes the MD (TOP 4
B. Uroporpholinogen incorporation of ferrous iron to the Protoporphyrin IX - AUG 2014
C. Aminolevulinic acid to form heme. therefore both B and D will be MED
D. porphobilinogen deficient.succinyl CoA along with glycine will not BOARDS;
E. All of the above accumulate since its conversion to d-ALA is TOPNOTCH
irreversible by the enzyme ALA synthase ( rate MD), MD
limiting step)
236 which of the following is true regarding Phenylketonuria Hyperphenylalaninemia (PKU) can arise from either LEAN MIDTERM
? absence of phenylalanine hydroxylase(PH) which ANGELO EXAM 1 - FEB
A. It can be secondary to dihydrobiopterin reductase convert phenylalanine to tyrosine or absence of its SILVERIO, 2015
deficiency enzyme cofactor tetrahydrobiopterin. Blood MD (TOP 4
B. Blood phenylalanine levels rises significantly at Day 2 phenylalanine levels accumulates only after Day 3 or - AUG 2014
postnatal life. Day 4 of life. absence of PH will make tyrosine MED
C. There would be normal levels of catecholamines in essential AA, without complete supplementation, BOARDS;
the blood there would be a depletion of catecholamine levels TOPNOTCH
D. tyrosine need not be supplemented since tyrosine is deficient. MD), MD
E. none of the above
237 which of the following is an essential amino acid? Essential amino acid: phenylalanine, valine, LEAN MIDTERM
A. asparagine tryptophan, threonine, isoleucine, methionine, ANGELO EXAM 1 - FEB
B. aspartate histidine, arginine, leucine, lysine. SILVERIO, 2015
C. arginine MD (TOP 4
D. Cysteine - AUG 2014
E. Alalnine MED
BOARDS;
TOPNOTCH
MD), MD
238 what bacterialDNA polymerase is responsible for gap there are only three types of bacterial DNA LEAN MIDTERM
filling and synthesis of lagging strand during replication? polymerase. Pol I is responsible ofr gap filling and ANGELO EXAM 1 - FEB
A. DNA pol I lagging strand synthesis. Pol II is for DNA SILVERIO, 2015
B. DNA pol II proofreading and repair. Pol III is for leading strand MD (TOP 4
C. DNA pol III synthesis - AUG 2014
D. DNA pol IV MED
E. DNA pol V BOARDS;
TOPNOTCH
MD), MD
239 which of the following is false about the Genetic code? one amino acid can be encoded by more than one LEAN MIDTERM
A. One amino acid is encoded only by one codon codon.this is the degenerate feature of a genetic code. ANGELO EXAM 1 - FEB
B. There are no punctation marks in the reading of the SILVERIO, 2015
code MD (TOP 4
C. A specific codon indicates only one specific amino - AUG 2014
acid MED
D. It is universal BOARDS;
E. none of the above TOPNOTCH
MD), MD
240 which of the following is true about type 1 muscle fiber? type 1 are slow twitch muscle fiber widely distributed LEAN MIDTERM
A. Fast contraction rate along the antigravity muscles. It is important in ANGELO EXAM 1 - FEB
B. Predominant in soleus and erector spinae muscle maintaining posture. All the other choices describes SILVERIO, 2015
C. High energy utilization Type II fast twitch or glycolytic fibers. MD (TOP 4
D. High myosin ATPase activity - AUG 2014
E. Short duration of action MED
BOARDS;
TOPNOTCH
MD), MD
241 Sphingomyelin is a substance participating in nerve SIMILAR TO PREVIOUS BOARD EXAM KEVIN MIDTERM 2
conduction and insulation, what is its predominant fatty CONCEPT/PRINCIPLE, very difficult to answer BRYAN LO, EXAM - FEB
acid composition? because there are a lot to consider especially type of MD (TOP 7 2015
A. palmitic acid tissue involved, but assuming human neural tissue, - AUG 2014
B. stearic acid based on journals from pubmed the most MED
C. lignoceric acid predominant fatty acid is stearic acid C18 with BOARDS;
D. nervonic acid lignocericC24 and nervonic C24:1 coming next TOPNOTCH
E. sphanganine MD)

TOPNOTCH MEDICAL BOARD PREP BIOCHEMISTRY SUPEREXAM Page 31 of 92


For inquiries visit www.topnotchboardprep.com.ph or email us at topnotchmedicalboardprep@gmail.com
TOPNOTCH MEDICAL BOARD PREP BIOCHEMISTRY SUPEREXAM
For inquiries visit www.topnotchboardprep.com.ph or email us at topnotchmedicalboardprep@gmail.com
Item QUESTION EXPLANATION AUTHOR TOPNOTCH
# EXAM
242 Which of the following precursors is the one converted SIMILAR TO PREVIOUS BOARD EXAM KEVIN MIDTERM 2
into vitamin D3 in the skin by activation with the help of CONCEPT/PRINCIPLE, The presence of this compound BRYAN LO, EXAM - FEB
sunlight? in human skin enables humans to manufacture MD (TOP 7 2015
A. 7-dehydrocholesterol vitamin D3 from ultraviolet rays in the sun light, via - AUG 2014
B. 3-hydroxycholesterol an intermediate isomer pre-vitamin D3. MED
C. 25-dehydrocholesterol BOARDS;
D. 1,25 - dihydroxycholecalciferol TOPNOTCH
E. pregnenolone MD)

243 Which of the following is a common enzyme in both SIMILAR TO PREVIOUS BOARD EXAM KEVIN MIDTERM 2
gluconeogenesis and glycolysis CONCEPT/PRINCIPLE. Of all the enzymes given only BRYAN LO, EXAM - FEB
A. pyruvate carboxylase phosphoglycerate kinase is common to both MD (TOP 7 2015
B. pyruvate kinase pathways, pyruvate kinase is glycolytic, the others are - AUG 2014
C. phosphoenolpyruvate carboxykinase involved in gluconeogenesis MED
D. phosphoglycerate kinase BOARDS;
E. fructose 1,6 bisphosphatase TOPNOTCH
MD)

244 The committed step in the glycolytic pathway is SIMILAR TO PREVIOUS BOARD EXAM KEVIN MIDTERM 2
catalyzed by the enzyme phosphofructokinase 1, the CONCEPT/PRINCIPLE. All are positive regulators BRYAN LO, EXAM - FEB
following substrates and or substances positively except citrate and ATP which represent high energy MD (TOP 7 2015
regulates or allosterically activates this enzyme EXCEPT states therefore exerting negative feedback regulation - AUG 2014
A. citrate on the glycolytic pathway. Fructose 2,6 bisphosphate MED
B. 5'AMP is the strongest positive allosteric activator BOARDS;
C. fructose 2,6 bisphosphate TOPNOTCH
D. fructose 6 phosphate MD)
E. none of the above (all are positive regulators)

245 Which of the following substances is known to inhibit SIMILAR TO PREVIOUS BOARD EXAM KEVIN MIDTERM 2
complex 2 succinate -q-reductase of the electron CONCEPT/PRINCIPLE. At that time we were asked BRYAN LO, EXAM - FEB
transport chain? about complex 3 and 4 inhibitors Antimycin and MD (TOP 7 2015
A. amobarbital cyanide respectively. Dimercaprol also inhibits - AUG 2014
B. malonate complex 3. Amobarbital inhibits complex 1 MED
C. antimycin A BOARDS;
D. dimercaprol TOPNOTCH
E. cyanide MD)

246 which of the following storage forms of energy in SIMILAR TO PREVIOUS BOARD EXAM KEVIN MIDTERM 2
skeletal muscle provides energy for the first few seconds CONCEPT/PRINCIPLE. The most immediate source of BRYAN LO, EXAM - FEB
of muscle activity or exertion? energy in the first few seconds of muscular activity is MD (TOP 7 2015
A. muscle glycogen phosphocreatine, muscle glycogen is then used next - AUG 2014
B. free glucose and over long periods of activity, fatty acids start to be MED
C. phosphocreatine used as the main source. BOARDS;
D. fatty acids TOPNOTCH
E. ketone bodies MD)

247 The disease characterized as having a deficiency of SIMILAR TO PREVIOUS BOARD EXAM KEVIN MIDTERM 2
homogentisic acid oxidase manifesting with dark CONCEPT/PRINCIPLE. Homocystinemia and BRYAN LO, EXAM - FEB
discoloration of the urine when allowed to stand and can homocystinuria are manifestations of cystathionine MD (TOP 7 2015
cause damage to cartilage (ochronosis), heart valves and synthase deficiency and other enzymes involved in - AUG 2014
kidney stones. methionine metabolism, may also be seen with B9 B12 MED
A. homocystinemia deficiencies. Alkaptonuria is a basement membrane BOARDS;
B. homocystinuria disease, maple syrup urine disease is deficiency of TOPNOTCH
C. maple syrup urine disease branched chain amino acid dehydrogenase MD)
D. alkaptonuria
E. alport's disease
248 Which of the following genes act as a tumor suppressor SIMILAR TO PREVIOUS BOARD EXAM KEVIN MIDTERM 2
gene which regulates cell apoptosis and the dysfunction CONCEPT/PRINCIPLE. P53 also known as the BRYAN LO, EXAM - FEB
of which is implicated in the pathogenesis of a lot of guardian of the genome regulates promotes cell MD (TOP 7 2015
human cancers? apoptosis when there is irreversible DNA damage. All - AUG 2014
A. BCR-ABL the other genes are oncogenes which function in MED
B. RAS promoting growth when mutated promotes BOARDS;
C. RET uncontrolled cellular growth. TOPNOTCH
D. C-kit MD)
E. p-53

249 Juan was trapped inside a collapsed building after an SIMILAR TO PREVIOUS BOARD EXAM KEVIN MIDTERM 2
earthquake. Having been trapped with no access to food CONCEPT/PRINCIPLE. A lot of questions on starvation BRYAN LO, EXAM - FEB
and water, his main source of energy for the first 24 and metabolic integration. First 24 hours glycogen is MD (TOP 7 2015
hours following the event would be derived from which the main source. After that gluconeogenesis to - AUG 2014
of the following sources? maintain blood glucose, then fats and ketones and MED
A. muscle glycogen then breakdown of proteins after prolonged BOARDS;
B. muslce protein starvation. Muscle glycogen cannot be utilized as free TOPNOTCH
C. liver glycogen glucose by other organs because of lack of glucose 6 MD)
D. ketones phosphatase.
E. gluconeogenesis

TOPNOTCH MEDICAL BOARD PREP BIOCHEMISTRY SUPEREXAM Page 32 of 92


For inquiries visit www.topnotchboardprep.com.ph or email us at topnotchmedicalboardprep@gmail.com
TOPNOTCH MEDICAL BOARD PREP BIOCHEMISTRY SUPEREXAM
For inquiries visit www.topnotchboardprep.com.ph or email us at topnotchmedicalboardprep@gmail.com
Item QUESTION EXPLANATION AUTHOR TOPNOTCH
# EXAM
250 Juan was trapped inside a collapsed building after an SIMILAR TO PREVIOUS BOARD EXAM KEVIN MIDTERM 2
earthquake. Having been trapped with no access to food CONCEPT/PRINCIPLE. The brain can obtain up to BRYAN LO, EXAM - FEB
and water, and experienced prolonged starvation, other 70% of its energy from ketone bodies with prolonged MD (TOP 7 2015
than glucose what predominant source of energy will his starvation. Fatty acids cannot be utilized by the brain. - AUG 2014
brain be utilizing? MED
A. protein BOARDS;
B. glycosphingolipids TOPNOTCH
C. free fatty acids MD)
D. ketone bodies
E. glycogen

251 The following substances are known to be antioxidants SIMILAR TO PREVIOUS BOARD EXAM KEVIN MIDTERM 2
preventing lipid peroxidation and buffers against free CONCEPT/PRINCIPLE. Vitamin B1 is the least BRYAN LO, EXAM - FEB
radical production EXCEPT implicated vitamin as an antixoidant among the MD (TOP 7 2015
A. glutathione choices - AUG 2014
B. vitamin D MED
C. vitamin C BOARDS;
D. vitamin E TOPNOTCH
E. vitamin B1 MD)

252 The following hormones act via intracellular nuclear SIMILAR TO PREVIOUS BOARD EXAM KEVIN MIDTERM 2
receptors binding to DNA response elements EXCEPT CONCEPT/PRINCIPLE. All steroid based hormones BRYAN LO, EXAM - FEB
A. Estrogen have intracellular nuclear receptors. Insulin binds to a MD (TOP 7 2015
B. insulin cell membrane receptor associated with tyrosine - AUG 2014
C. thyroid hormone kinase MED
D. vitamin D BOARDS;
E. cortisol TOPNOTCH
MD)

253 Which of the following cell profiles are usually SIMILAR TO PREVIOUS BOARD EXAM KEVIN MIDTERM 2
associated with chronic autoimmune diseases and CONCEPT/PRINCIPLE, difficult one, from research I BRYAN LO, EXAM - FEB
cancers like hodgkin's lymphoma? found the answer to be double positive, meaning cells MD (TOP 7 2015
A. CD4(+) CD8(-) have both CD4 CD8 characteristics evading thymic - AUG 2014
B. CD4(-) CD8(-) double negative immune regulation causing diseases like autoimmune MED
C. CD4(-) CD8(+) and malignancies. BOARDS;
D. CD4(+) CD8(+) double positive TOPNOTCH
E. all of the above MD)

254 What is the most active potent form of androgen SIMILAR TO PREVIOUS BOARD EXAM KEVIN MIDTERM 2
involved in the development of male external genitalia CONCEPT/PRINCIPLE, dihydrotestosterone or DHT is BRYAN LO, EXAM - FEB
and also found predominant in the prostate and hair the active form of testosterone and the most potent. MD (TOP 7 2015
follicles? Androstenedione is a precursor with androgen - AUG 2014
A. testosterone properties, Dehydroepiandrosterone or DHEA is MED
B. dihydrotestosterone secreted in the adrenal and gonads where it also acts BOARDS;
C. dehydroepiandrosterone as an intermediate for estrogen and androgen TOPNOTCH
D. androstenedione synthesis MD)
E. none of the above

255 From which metabolic intermediate does the synthesis SIMILAR TO PREVIOUS BOARD EXAM KEVIN MIDTERM 2
of cholesterol begin with? CONCEPT/PRINCIPLE. Cholesterol synthesis starts BRYAN LO, EXAM - FEB
A. acetylCoA with joining of 2 molecules of Acetyl CoA to form MD (TOP 7 2015
B. HMG-CoA AcetoacetylCoA with the enzyme acetoacetylCoA - AUG 2014
C. mevalonate thiolase eventually forming HMG-CoA and so on. All MED
D. squalene the others are intermediates further down the BOARDS;
E. lanosterol pathway. TOPNOTCH
MD)

256 Increasing insulin levels has which of the following SIMILAR TO PREVIOUS BOARD EXAM KEVIN MIDTERM 2
effects on the different enzyme systems? CONCEPT/PRINCIPLE, know the functions of the BRYAN LO, EXAM - FEB
A. glycogen phosphorylase will be dephosphorylated hormones by heart, a lot of metabolic integration. MD (TOP 7 2015
B. AcetylCoA carboxylase will be inhibited Insulin predominantly dephosphorylates enzymes to - AUG 2014
C. pyruvate dehydrogenase is phosphorylated activate them or inhibit them. Glucagon Epinephrine MED
D. glycogen synthase is phosphorylated predominantly phosphorylates enzymes to activate BOARDS;
E. pyruvate carboxylase is activated them via cAMP Protein Kinase A pathway. Increase TOPNOTCH
Insulin like in the well fed state will cause storage of MD)
energy as glycogen therefore glycogen phosphorylase
the one responsible for breakdown is inhibited and
dephosphorylated form, opposite is true with
glycogen synthase where it is activated and
dephosphorylated, acetylcoa carboxylase is activated
because insulin promotes energy storage by
lipogenesis, pyruvate dehydrogenase is
dephosphorylated and activated because insulin
promotes glucose entry and utilization, insulin
promotes glycolysis and inhibits gluconeogenesis
therefore pyruvate carboxylase should be inhibited no
new glucose needs to be made in a well fed state.

TOPNOTCH MEDICAL BOARD PREP BIOCHEMISTRY SUPEREXAM Page 33 of 92


For inquiries visit www.topnotchboardprep.com.ph or email us at topnotchmedicalboardprep@gmail.com
TOPNOTCH MEDICAL BOARD PREP BIOCHEMISTRY SUPEREXAM
For inquiries visit www.topnotchboardprep.com.ph or email us at topnotchmedicalboardprep@gmail.com
Item QUESTION EXPLANATION AUTHOR TOPNOTCH
# EXAM
257 Which of the following statements are true regarding SIMILAR TO PREVIOUS BOARD EXAM KEVIN MIDTERM 2
skeletal muscle? CONCEPT/PRINCIPLE, suprisingly there was a lot of BRYAN LO, EXAM - FEB
A. skeletal muscle is unable to contract in the absence of physiology question in the biochemistry exam, muscle MD (TOP 7 2015
extracellular calcium influx stimulating the release of physiology had 3-5 questions alone. Skeletal muscle - AUG 2014
calcium from the sarcoplasmic reticulum does not need extracellular calcium, dihydropyridine MED
B. action potential travels down the T-tubules closely receptors sense the action potential causing the BOARDS;
related to the 2 terminal cisternae of the sarcoplasmic Ryanodine receptors in the SR to open and release TOPNOTCH
reticulum releasing calcium into the cytoplasm calcium, calcium binds to troponin C MD)
C. the dihydropyridine receptor opens and releases the
calcium from the sarcoplasmic reticulum into the
cytoplasm to stimulate excitation contraction coupling
D. calcium released from the sarcoplasmic reticulum
binds to troponin T allowing actin myosin interaction to
occur
E. all of the above statments are correct
258 Which of the following biochemical processes SIMILAR TO PREVIOUS BOARD EXAM KEVIN MIDTERM 2
exclusively occur in the mitochondria of the cell? CONCEPT/PRINCIPLE, glycolysis and fatty acid BRYAN LO, EXAM - FEB
A. glycolysis synthesis occurs in the cytoplasm, urea cycle has MD (TOP 7 2015
B. pentose phosphate pathway steps in both the cytoplasm and mitochondria, - AUG 2014
C. tricarboxylic acid cycle pentose phosphate pathway in the cytoplasm MED
D. urea cycle BOARDS;
E. fatty acid synthesis TOPNOTCH
MD)

259 What is the predominant pathway utilized by red blood SIMILAR TO PREVIOUS BOARD EXAM KEVIN MIDTERM 2
cells for energy? CONCEPT/PRINCIPLE, specifically answer should be BRYAN LO, EXAM - FEB
A. beta oxidation anaerobic glycolysis since RBCs don’t have MD (TOP 7 2015
B. embden meyerhoff pathway mitochondria. Lueberin rapoport pathway - pathway - AUG 2014
C. luebering rapoport pathway involved in the formation of 2,3 BPG. MED
D. hexose monophosphate shunt BOARDS;
E. Citric acid cycle TOPNOTCH
MD)
260 This important coenzyme A involved in various SIMILAR TO PREVIOUS BOARD EXAM KEVIN MIDTERM 2
biochemical reactions is derived from which of the CONCEPT/PRINCIPLE, what was asked was FAD B2 BRYAN LO, EXAM - FEB
following substances? and NAD B3. memorize the coenzymes and vitamin MD (TOP 7 2015
A. vitamin B1 derivatives, this is after all both biochemistry and - AUG 2014
B. vitamin B2 nutrition MED
C. vitamin B3 BOARDS;
D. vitamin B5 TOPNOTCH
E. vitamin B6 MD)

261 If an amino acid with a nonpolar side chain has a pk1 of At this pH, the COOH group of the amino acid will have RAYMUND MIDTERM 3
2.3 and a pk2 of 9.1, what will be its expected net charge already given up its H+ making the net charge 0. MARTIN LI, EXAM - FEB
in an alkalinized urine with a pH of 8.5? MD (TOP 1 2015
A. +2 - AUG 2014
B. 0 MED
C. -1 BOARDS;
D. +1 TOPNOTCH
E. MD)
262 A patient with long-standing Diabetes mellitus type 2 is Sorbitol dehydrogenase converts sorbitol to fructose. RAYMUND MIDTERM 3
diagnosed with peripheral neuropathy. The absence of Absence of this enzyme in the presence of MARTIN LI, EXAM - FEB
this enzyme in the presence of hyperglycemia is said to hyperglycemia will result in increased sorbitol in the MD (TOP 1 2015
be responsible: tissue. - AUG 2014
A. sorbitol dehydrogenase MED
B. aldose reductase BOARDS;
C. fructokinase TOPNOTCH
D. aldolase B MD)

263 Which of the following characterizes the lagging strand A B and D are required by both strands RAYMUND MIDTERM 3
but not the leading strand in DNA replication? MARTIN LI, EXAM - FEB
A. Requires an RNA primer MD (TOP 1 2015
B. Synthesized in a 5’ to 3’ direction - AUG 2014
C. Requires DNA ligase activity MED
D. Bound by single-stranded DNA binding proteins BOARDS;
TOPNOTCH
MD)
264 A diet rich in which of the following dietary fats is most Monounsaturated fatty acids and omega 6 PUFA RAYMUND MIDTERM 3
responsible for a decrease in LDL levels: lowers LDL levels. MARTIN LI, EXAM - FEB
A. polyunsaturated omega 3 fatty acids MD (TOP 1 2015
B. monounsaturated fatty acids - AUG 2014
C. saturated fat MED
D. transfat BOARDS;
TOPNOTCH
MD)
265 Which of the following is true about enzyme kinetics? Review enzyme kinetics. Michaelis-menten shows a RAYMUND MIDTERM 3
A. Allosteric enzymes show a sigmoidal curve hyperbolic curve. Irreversible inhibitor lowers Vmax MARTIN LI, EXAM - FEB
B. A numerically small Km indicates low affinity for MD (TOP 1 2015
substrate - AUG 2014
C. Enzymes that follow Michaelis-Menten kinetics show a MED
sigmoidal curve BOARDS;
D. An irreversible competitive antagonist leads to an TOPNOTCH
increase in Km MD)
E. None of the above

TOPNOTCH MEDICAL BOARD PREP BIOCHEMISTRY SUPEREXAM Page 34 of 92


For inquiries visit www.topnotchboardprep.com.ph or email us at topnotchmedicalboardprep@gmail.com
TOPNOTCH MEDICAL BOARD PREP BIOCHEMISTRY SUPEREXAM
For inquiries visit www.topnotchboardprep.com.ph or email us at topnotchmedicalboardprep@gmail.com
Item QUESTION EXPLANATION AUTHOR TOPNOTCH
# EXAM
266 Which of the following functions as a reservoir of HDL is a reservoir of apoCII and apo E RAYMUND MIDTERM 3
apolipoproteins? MARTIN LI, EXAM - FEB
A. VLDL MD (TOP 1 2015
B. HDL - AUG 2014
C. LDL MED
D. Chylomicrons BOARDS;
E. IDL TOPNOTCH
MD)
267 A patient is diagnosed with megaloblastic anemia and All the others may also be found in folic acid RAYMUND MIDTERM 3
undergoes several laboratory tests. Which of the deficiency MARTIN LI, EXAM - FEB
following differentiates cobalamin from folic acid MD (TOP 1 2015
deficiency? - AUG 2014
A. Homocystinuria MED
B. Increased MCV BOARDS;
C. Decreased Hgb TOPNOTCH
D. Increased methylmalonic acid levels MD)
E. Decreased activity of methionine synthase

268 Interconversion of sugars between fructose 6-P, xylulose HMP shunt or Pentose phosphate pathway RAYMUND MIDTERM 3
5-P, Ribose 5-P, and Glyceraldehyde 3-P is made possible MARTIN LI, EXAM - FEB
by which pathway? MD (TOP 1 2015
A. Hexose monophosphate shunt - AUG 2014
B. Embden Meyerhoff pathway MED
C. Pentose Phosphate pathway BOARDS;
D. A and B TOPNOTCH
E. A and C MD)

269 The reaction catalyzed by the enzyme phosphoglucose A + standard free energy mostly proceed from right to RAYMUND MIDTERM 3
isomerase (glucose 6-P →fructose 6-P) in the glycolytic left and vice versa for a (-) standard free energy MARTIN LI, EXAM - FEB
pathway has a standard free energy +400 cal/mol. If MD (TOP 1 2015
isolated from the other enzymatic reactions from the - AUG 2014
pathway under standard conditions, which of the MED
following conclusions may be derived: BOARDS;
A. The reaction is in equilibrium TOPNOTCH
B. The reaction mostly proceeds from right to left MD)
C. More fructose 6-P is produced
D. A predominantly forward reaction takes place
E. No reaction takes place
270 Arsenic poisoning inhibits the conversion of pyruvate to Arsenic inhibits lipoic acid an essential component of RAYMUND MIDTERM 3
acetyl-coA. It will also inhibit the following: the enzymes pyruvate dehydrogenase, alpha- MARTIN LI, EXAM - FEB
A. reaction catalyzed by isocitrate dehydrogenase ketoglutarate dehydrogenase, and branched chain MD (TOP 1 2015
B. conversion of phosphoenolpuruvate to pyruvate dehydrogenase - AUG 2014
C. synthesis of ribose 5-phosphate MED
D. catabolism of branched-chain amino acids BOARDS;
E. None of the above TOPNOTCH
MD)

271 A patient presenting with fasting hypoglycemia and Cori disease or deficiency in debranching enzyme RAYMUND MIDTERM 3
accumulation of intracellular glycogen with abnormal MARTIN LI, EXAM - FEB
structure at branch points has which disorder: MD (TOP 1 2015
A. Glycogen phoshorylase deficiency - AUG 2014
B. Pompe disease MED
C. Glucose 6-Phosphatase deficiency BOARDS;
D. Cori disease TOPNOTCH
E. McArdle disease MD)

272 Myoglobin is a globular, flexible molecule found All the others bonds that stabilize tertiary and RAYMUND MIDTERM 3
predominantly in slow-twitch muscle fibers. It is quaternary structures MARTIN LI, EXAM - FEB
composed of alpha-helical secondary structure that is MD (TOP 1 2015
stabilized by: - AUG 2014
A. Disulfide bonds MED
B. Hydrophobic interactions BOARDS;
C. Ionic interactions TOPNOTCH
D. Hydrogen bonds MD)
E. All of the above

273 An infant presenting with increased orotic acid levels, The presence of hyperammonemia differentiates RAYMUND MIDTERM 3
hyperammonemia, and neurologic manifestations will ornithine transcarbamylase deficiency from deficiency MARTIN LI, EXAM - FEB
most likely have which deficiency? of pyrimidine synthesis enzyme deficiency that cause MD (TOP 1 2015
A. Ornithine transcarbamylase orotic aciduria - AUG 2014
B. N-acetylglutamate MED
C. Carbamoyl phosphate synthase I BOARDS;
D. Carbamoyl phosphate synthase II TOPNOTCH
E. Bifunctional pyrimidine synthesis enzyme MD)

274 What is the rate-limiting enzyme in the TCA cycle? RAYMUND MIDTERM 3
A. Alpha-ketoglutarate dehydrogenase MARTIN LI, EXAM - FEB
B. Malate dehydrogenase MD (TOP 1 2015
C. Succinate dehydrogenase - AUG 2014
D. Isocitrate dehydrogenase MED
E. None of these BOARDS;
TOPNOTCH
MD)

TOPNOTCH MEDICAL BOARD PREP BIOCHEMISTRY SUPEREXAM Page 35 of 92


For inquiries visit www.topnotchboardprep.com.ph or email us at topnotchmedicalboardprep@gmail.com
TOPNOTCH MEDICAL BOARD PREP BIOCHEMISTRY SUPEREXAM
For inquiries visit www.topnotchboardprep.com.ph or email us at topnotchmedicalboardprep@gmail.com
Item QUESTION EXPLANATION AUTHOR TOPNOTCH
# EXAM
275 A person is in his second day of religious fasting, which Glycogen stores good for 12-18 hours only. After that, RAYMUND MIDTERM 3
of the following metabolic pathway is primarily gluconeogenesis is responsible for maintaining MARTIN LI, EXAM - FEB
responsible for maintaining glucose concentration in the glucose levels MD (TOP 1 2015
circulation: - AUG 2014
A. breakdown of glycogen stores MED
B. ketones synthesis BOARDS;
C. uptake and phosphorylation of glycerol by the liver TOPNOTCH
D. Embden Meyerhoff pathway MD)
E. Pentose phosphate pathway

276 Collagen undergoes extensive posttranslational All others occur inside the cell RAYMUND MIDTERM 3
modification. Which of the following occurs outside the MARTIN LI, EXAM - FEB
cell? MD (TOP 1 2015
A. Triple helix formation - AUG 2014
B. Cleavage of C-terminal sequence MED
C. Hydroxylation of lysine and proline BOARDS;
D. Removal of signal sequence TOPNOTCH
E. Glycosylation of lysine MD)

277 Glycogenesis is enhanced by: All others enhance gluconeogenesis RAYMUND MIDTERM 3
A. enzyme phosphorylation MARTIN LI, EXAM - FEB
B. activation of protein phosphatase MD (TOP 1 2015
C. increased cAMP concentration - AUG 2014
D. activation of protein kinase A MED
E. All of the above BOARDS;
TOPNOTCH
MD)
278 Which of the following amino acids is a direct donor of Aspartate directly donates NH3 in urea synthesis RAYMUND MIDTERM 3
NH3 in urea synthesis? MARTIN LI, EXAM - FEB
A. Aspartate MD (TOP 1 2015
B. Glutamate - AUG 2014
C. Glutamine MED
D. Arginine BOARDS;
E. Alanine TOPNOTCH
MD)
279 In gluconeogenesis, reversal of the glycolytic enzyme Succinate thiokinase is responsible for substrate level RAYMUND MIDTERM 3
pyruvate kinase involves two reactions, the second of phosphorylation that produces GTP, which is MARTIN LI, EXAM - FEB
which is catalyzed by phosphoenolpyruvate preferentially used by phosphoenolpyruvate MD (TOP 1 2015
carboxykinase which utilizes a source of energy from a carboxykinase - AUG 2014
substrate level phosphorylation such as the reaction MED
catalyzed by: BOARDS;
A. alpha-ketoglutarate dehydrogenase TOPNOTCH
B. malate dehydrogenase MD)
C. phosphofructokinase 1
D. succinate thiokinase
E. A and B
280 Hypogonadotrophic dwarfism and acrodermatitis RAYMUND MIDTERM 3
enteropathica is caused by: MARTIN LI, EXAM - FEB
A. Vitamin E deficiency MD (TOP 1 2015
B. Vitamin A excess - AUG 2014
C. Zinc deficiency MED
D. Ascorbic acid deficiency BOARDS;
E. Vitamin A deficiency TOPNOTCH
MD)
281 Which of the following characterizes the enzyme Characteristics of Glucokinase: 1. Present only in liver ERIC ROYD FINAL EXAM -
glucokinase? parenchymal cells and islet cells of pancreas, 2. TALAVERA, FEB 2015
A. Low Vmax inhibited by fructose-6-phosphate, 3. High km (low MD (TOP 1
B. High Km affinity), 4. high vmax, . - AUG 2014
C. Inhibited by glucose-6-phosphate MED
D. Present in most tissues BOARDS;
E. All of the above TOPNOTCH
MD)
282 Which of the following amino acids does not undergo All amino acids with the exception of LYSINE and ERIC ROYD FINAL EXAM -
transamination at some point in its catabolism? THREONINE participate in transamination TALAVERA, FEB 2015
A. Histidine MD (TOP 1
B. Valine - AUG 2014
C. Tryptophan MED
D. Aspartate BOARDS;
E. Lysine TOPNOTCH
MD)

283 Which of the following is true regarding Vitamin K? Vitamin K, a fat soluble vitamin, is an essential ERIC ROYD FINAL EXAM -
A. It is a water soluble vitamin cofactor for the synthesis of factors II, VII, IX and X. It TALAVERA, FEB 2015
B. It is not synthesized in the human body DECREASES/SHORTENS the coagulation time MD (TOP 1
C. Plays an essential role in coagulation (particularly the PT). It is synthesized by the normal - AUG 2014
D. Present at high concentration in cow's or breast milk gut flora and is present at low concentration in MED
E. Important in management of hemorrhagic disorders cow's/breast milk. BOARDS;
by prolonging the coagulation time. TOPNOTCH
MD)

TOPNOTCH MEDICAL BOARD PREP BIOCHEMISTRY SUPEREXAM Page 36 of 92


For inquiries visit www.topnotchboardprep.com.ph or email us at topnotchmedicalboardprep@gmail.com
TOPNOTCH MEDICAL BOARD PREP BIOCHEMISTRY SUPEREXAM
For inquiries visit www.topnotchboardprep.com.ph or email us at topnotchmedicalboardprep@gmail.com
Item QUESTION EXPLANATION AUTHOR TOPNOTCH
# EXAM
284 Which of the following pathways produces carbon Heme catabolism is first carried out by the enzyme ERIC ROYD FINAL EXAM -
monoxide as a by-product? heme oxygenase which produces an equimolar TALAVERA, FEB 2015
A. Heme Catabolism amount of biliverdin, with ferric iron and carbon MD (TOP 1
B. Glycogenolysis monoxide being released - AUG 2014
C. Fatty Acid Synthesis MED
D. Urea Cycle BOARDS;
E. Hexose Monophosphate Shunt TOPNOTCH
MD)
285 How does azithromycin inhibit bacterial protein A - aminoglycoside, B - Rifampicin, D - ERIC ROYD FINAL EXAM -
synthesis in order to extert its antibacterial effect? Chloramphenicol, E- Tetracycline TALAVERA, FEB 2015
A. Interferes with the initiation of protein synthesis MD (TOP 1
B. Inhibits the DNA depended RNA polymerase - AUG 2014
C. Inhibits translocation by irreversibly binding to a MED
site on the 50s ribosome BOARDS;
D. Inhibits the enzyme peptidyltransferase TOPNOTCH
E. Blocks the binding of aminoacyl TRNA to the MD)
acceptor site
286 Chronic Alcoholism often leads to fatty liver by As part of alcohol metabolism, ethanol is oxidized to ERIC ROYD FINAL EXAM -
consuming what coenzyme which is needed for beta acetaldehyde by the enzyme alcohol dehydrogenase to TALAVERA, FEB 2015
oxidation of fatty acids? produce acetaldehyde, this is coupled with NAD being MD (TOP 1
A. NAD+ reduced to NADH2. NAD is also essential in one of the - AUG 2014
B. NADH oxidative steps in the beta oxidation of fatty acids. If MED
C. NADPH NAD is consumed by means of chronic alcohol BOARDS;
D. FAD consumption, the beta oxidation will not proceed TOPNOTCH
E. FADH2 hence accumulation of fat within the hepatocytes MD)
(steatosis)
287 What is the biochemical mechanism behind the Hepatic encephalopathy results in the shunting of ERIC ROYD FINAL EXAM -
development of seizures in individuals with hepatic nitrogenous material INSIDE the neurons. The TALAVERA, FEB 2015
encephalopathy? increased ammonia levels shift the transamination MD (TOP 1
A. Elevated levels of alpha ketoglutrate hence reaction to the synthesis of glutamate rather than - AUG 2014
producing increased ATP production alpha ketoglutarate, hence the TCA will not proceed MED
B. An increase in the level of the neurotransmitter leading to DEPLETION of ATP. Glutamate is an BOARDS;
glycine inside the brain EXCITATORY neurotransmitter and its elevated levels TOPNOTCH
C. Nitrogenous substances shunted out of the brain can stimulate the cortical neurons producing seizure MD)
resulting in depletion of ammonia levels episodes. GABA is also increased in individuals with
D. Increased synaptic release of glutamate hepatic encephalopathy as it is produced through
E. None of the above decarboxylation of glutamate. However it is an
inhibitory neurotransmitter and is primarily
responsibe for COMA in advanced stages. Glycine is an
inhibitory neurotransmitter in the spinal cord
288 Cataracts are common in poorly controlled diabetic Poorly controlled diabtetics have elevated levels of ERIC ROYD FINAL EXAM -
individuals due to what biochemical mechanism? glucose in the blood. The glucose accumulates inside TALAVERA, FEB 2015
A. Decreased sorbitol levels which produce cellular the lens (the lens does not require insulin for glucose MD (TOP 1
degradation of the proteins in the lens uptake), which is eventually metabolized to sorbitol - AUG 2014
B. Insulin deficiency inhibits synthesis of proteins in through the enzyme ALDOSE REDUCTASE. The lens MED
the lens lack the enzyme SORBITOL DEHYDROGENASE which BOARDS;
C. Osmotic shift due to the increased levels of sorbitol metabolizes sorbitol further to fructose TOPNOTCH
inside the lens MD)
D. Absence of the enzyme aldose reductase which is
responsible for sorbitol metabolism
E. Both C and D are correct
289 Increased formation of ketone bodies during starvation Free fatty acids are increased due to the increased ERIC ROYD FINAL EXAM -
is due to? activity of hormone sensitive lipase (stimulated by TALAVERA, FEB 2015
A. Decreased levels of circulating glucagon Glucagon, in the fasting state). Acetyl-CoA is the end MD (TOP 1
B. Increased levels of free fatty acids in the blood product of fatty acid oxidation which is a substrate - AUG 2014
C. Decreased levels of acetyl-CoA in the blood needed for ketone body synthesis MED
D. Decreased activity of hormone sensitive lipase BOARDS;
E. Inhibition of beta oxidation of fatty acid TOPNOTCH
MD)
290 Which of the following chemotherapeutic drugs acts by 5FU inhibits thymidylate synthase leading to a ERIC ROYD FINAL EXAM -
inhibiting the enzyme thymidylate synthase? "thymineless" cell death, TALAVERA, FEB 2015
A. Gemcitabine MD (TOP 1
B. Paclitaxel - AUG 2014
C. Bleomycin MED
D. Vincristine BOARDS;
E. 5-FU TOPNOTCH
MD)

291 Which of the following is true about the enzyme acetyl- ACC requires BIOTIN as a coenzyme. It is found in the ERIC ROYD FINAL EXAM -
CoA carboxylase? CYTOPLASM. It utilizes ACETYL-COA as a substrate. It TALAVERA, FEB 2015
A. It requires thiamine for the carboxylation of acetyl- is the rate limiting step of FATTY ACID SYNTHESIS MD (TOP 1
CoA - AUG 2014
B. It is located mainly in the matrix of the hepatic MED
mitochondria BOARDS;
C. It utilizes citrate as a substrate TOPNOTCH
D. It produces malonyl-CoA MD)
E. It catalyzes the rate limitting step in the beta
oxidation of fatty acids

TOPNOTCH MEDICAL BOARD PREP BIOCHEMISTRY SUPEREXAM Page 37 of 92


For inquiries visit www.topnotchboardprep.com.ph or email us at topnotchmedicalboardprep@gmail.com
TOPNOTCH MEDICAL BOARD PREP BIOCHEMISTRY SUPEREXAM
For inquiries visit www.topnotchboardprep.com.ph or email us at topnotchmedicalboardprep@gmail.com
Item QUESTION EXPLANATION AUTHOR TOPNOTCH
# EXAM
292 An infant diagnosed with Von Gierke's disease has the Von Gierke's disease: deficiency in glucose-6- ERIC ROYD FINAL EXAM -
following characteristics EXCEPT? phosphatase which is an enzyme utilized in TALAVERA, FEB 2015
A. Cannot utilize hepatic gluconeogenesis to maintain gluconeogenesis and glycogenolysis. Affected infants MD (TOP 1
blood glucose levels develop hypoglycemia and if severe enough could - AUG 2014
B. Characterized by a deficiency of the enzyme glucose- progress to lactic acidosis. Muscle glycogen cannot be MED
6-phosphatase used (even in normal individuals) to produce free BOARDS;
C. Can utilize the muscle glycogen to maintain blood blood glucose since muscle lacks glucose-6- TOPNOTCH
glucose levels phosphatase MD)
D. Develops lactic acidosis due to the hypoglycemia
E. None of the above
293 Which of the following substances inhibit Complex III of SIMILAR TO PREVIOUS BOARD EXAM ERIC ROYD FINAL EXAM -
the Electron Transport Chain? CONCEPT/PRINCIPLE: Complex I inhibitors TALAVERA, FEB 2015
A. Barbiturates (Barbiturate, Piericidin A, Amytal, Rotenone), MD (TOP 1
B. Dimercaprol Complex II inhibitors (Malonate, Carboxin, TTFA), - AUG 2014
C. Cyanide Complex III inhibitors (Dimercaprol, Antimycin A) MED
D. Hydrogen Sulfide Complex IV inhibitors (Cyanide, Carbon monoxide, BOARDS;
E. Malonate Sodium azide, Hydrogen sulfide) TOPNOTCH
MD)

294 An individual with lipoprotein lipase deficiency would If the activity of lipoprotein lipase is decreased, both ERIC ROYD FINAL EXAM -
have which of the following changes in blood lipid plasma chylomicrons and VLDL would become TALAVERA, FEB 2015
levels? elevated MD (TOP 1
A. Elevation of plasma HDL level - AUG 2014
B. Elevation of chylomicron level MED
C. Decreased triglyceride levels BOARDS;
D. Elevated LDL levels TOPNOTCH
E. Decreased VLDL levels MD)

295 An infant who was initially normal at birth began to Arginosuccinate synthase catalyzes the condensation ERIC ROYD FINAL EXAM -
develop lethargy, hypothermia and apnea. He was later of citrulline with L-aspartate to form arginine. Any TALAVERA, FEB 2015
found out to have a deficiency of the enzyme intermediates before this step would accumulate MD (TOP 1
arginosuccinate synthase. Which of the following would while all those that occur afterwards would have - AUG 2014
be expected? decreased levels MED
A. High blood levels of arginine BOARDS;
B. Low blood levels of ammonia TOPNOTCH
C. Low blood levels of aspartate MD)
D. High blood levels of ornithine
E. High blood levels of citrulline
296 Patients with xeroderma pigmentosum suffer DNA UV light causes formation of pyrimidine dimers in ERIC ROYD FINAL EXAM -
damage upon exposure to UV light through formation of DNA TALAVERA, FEB 2015
which of the following? MD (TOP 1
A. Pyrimidine dimers - AUG 2014
B. Purine dimers MED
C. Deoxyribose dimers BOARDS;
D. Anhydrous bonds in the DNA TOPNOTCH
E. All of the above MD)

297 The carbon skeleton of triacylglycerol can be used for Glycerol is released during TAG hydroysis, Glycerol is ERIC ROYD FINAL EXAM -
gluconeogenesis by entering the pathway as what phosphorylated to glycerol phosphate which is then TALAVERA, FEB 2015
substrate? oxidized to form dihydroxyacetone phosphate. MD (TOP 1
A. 3-phosphoglycerate - AUG 2014
B. Glucose-6-phosphate MED
C. Fructose 1,6-bisphosphate BOARDS;
D. Dihydroxyacetone phosphate TOPNOTCH
E. Phosphoenolpyruvate MD)

298 The neurologic manifestations seen in Niemann Pick Niemann Pick disease - deficiency of ERIC ROYD FINAL EXAM -
disease are associated with the accumulation of which of sphingomyelinase enzyme TALAVERA, FEB 2015
the following? MD (TOP 1
A. Gangliosides - AUG 2014
B. Cerebroside MED
C. Prostaglandin BOARDS;
D. Leukotriene TOPNOTCH
E. Sphingomyelin MD)

299 A 24 year old male medical student complains of Prolactinoma - most common functioning pituitary ERIC ROYD FINAL EXAM -
bilateral white discharge from his breast. Which of the tumor, located in the anterior pituitary gland. Most TALAVERA, FEB 2015
following is true about his probable condition ? are microadenoma and is the only pituitary tumor that MD (TOP 1
A. There is excessive production of ADH by his can be treated medically with dopamine analogs (such - AUG 2014
posterior pituitary as Bromocriptine) MED
B. His testosterone levels are markedly decreased BOARDS;
C. His disease condition would respond by TOPNOTCH
administering Bromocriptine MD)
D. Oxytocin levels are markedly elevated
E. There is most likely a tumor present that involves
the posterior pituitary
300 Which of the following minerals is essential for the In the form of selenocysteine, selenium is essential for ERIC ROYD FINAL EXAM -
activity of the enzyme glutathione peroxidase? the activity of the anti-oxidanrt glutathione TALAVERA, FEB 2015
A. Molybdenum peroxidase MD (TOP 1
B. Manganese - AUG 2014
C. Iron MED
D. Selenium BOARDS;
E. Copper TOPNOTCH
MD)

TOPNOTCH MEDICAL BOARD PREP BIOCHEMISTRY SUPEREXAM Page 38 of 92


For inquiries visit www.topnotchboardprep.com.ph or email us at topnotchmedicalboardprep@gmail.com
TOPNOTCH MEDICAL BOARD PREP BIOCHEMISTRY SUPEREXAM
For inquiries visit www.topnotchboardprep.com.ph or email us at topnotchmedicalboardprep@gmail.com
Item QUESTION EXPLANATION AUTHOR TOPNOTCH
# EXAM
301 Which of the following is true about mammalian pyruvate dehydrogenase complex is one of the most LEAN BACK-UP
pyruvate dehydrogenase complex under physiologic highly regulated enzyme in terms of carbohydrate ANGELO MIDTERM
condition ? metabolism. It is a mitochondrial enzyme complex SILVERIO, EXAM - FEB
A. It is composed of 3 types of catalytic enzyme subunits composed of 3 catalytic subunits namely: pyruvate MD (TOP 4 2015
B. Phosphorylation of this enzyme will increase the dehydrogenase, dihydrolipoyl transacetylase, and - AUG 2014
formation of acetyl CoA dihydrolipoyl dehydrogenase. cofactors essential for MED
C. the presence of epinephrine will activate the PDH in the complex include the ff: FAD, thiamine BOARDS;
adipose tissue pyrophosphate, lipoamide,Coenzyme A (pantothenic TOPNOTCH
D. high NADH/NAD ratio will promote active form of acid), and NAD. the process involves the conversion of MD), MD
the complex pyruvate to acetyl CoA along with release of CO2 and
E. pyridoxal phosphate is an esential cofactor of the NADH as byproduct. accumulation of its product (
complex acetylCoA,NADH) will inactivate the complex via
competitive feedback. furthermore, phosphorylation
of the enzyme by protein kinase will also inactivate
the complex. the reverse is true about
dephosphorylation. catecholamines particularly
epinephrine will stimulate PDH in the cardiac tissue
while insulin will stimulate this adipose tissue PDH.
Devlin biochemistry pp 540-542 6th ed.
302 Which of the following intermediate substances in the the administration of fluoroacetate will inhibit the LEAN BACK-UP
kreb cycle will accumulate when fluoroacetate enters the enzyme aconitase found in the mitochondrial matrix. ANGELO MIDTERM
mitochondrial matrix? This enzyme is part of the kreb cycel responsible for SILVERIO, EXAM - FEB
A. Alpha ketoglutarate converting citrate into isocitrate. Therefore, citrate MD (TOP 4 2015
B. citrate will accumulate as a result. On the otherhand, arsenic - AUG 2014
C. Isocitrate inhibit alpha ketoglutarate dehydrogenase complex MED
D. Succinyl CoA resulting to accumulation of alpha ketoglutarate and BOARDS;
E. None of the abvoe deficiency in succinyl CoA. harpers 27th ed pp 145- TOPNOTCH
146. MD), MD
303 Which of the following is an effect of salicylate overdose supradoses of aspirin (salicylates) will uncouple the LEAN BACK-UP
in the oxidative phosphorylation process? electron transport chain by promoting permeabilty of ANGELO MIDTERM
A. It binds to the ATP synthetase preventing the H the inner mitochondrial membrane to hydrogen. This SILVERIO, EXAM - FEB
reentry into the mitochondrial matrix will result to loss of hydrogen gradient despite MD (TOP 4 2015
B. It inhibits the electron transport via complex I by continous process of ETC. in effect, much of the energy - AUG 2014
blocking the transfer from Fe-S to Q produced in the electron transfer will dissipate as heat MED
C. competitively inhibits the succinate Q reductase( and consequently leads to fever on the part of the BOARDS;
Complex II) patient. the same mechanism is involved in ingestion TOPNOTCH
D. it promotes increase permeability of the inner of 2,4 dinitrophenol and during brown fat MD), MD
mitochondrial membrane thermogenesis via thermogenin protein in
E. inhibits the cytochrome c oxidase (complex IV) neonates.barbiturates, ptericidin A, rotenone are
resulting to inability to transfer electrons to oxygen inhibitors of complex I, carboxin, TTFA, malonate are
atom to Complex II; antimycin A and dimercaprol (BAL) are
to Complex III while hydrogen sulfide, CO, and
Cyanide are to complex IV. oligomycin on the
otherhand inhibits the H reentry in the ATP
syntethase. Harpers 27th ed pp 107-108
304 which of the following enzymes is highly active in the (SIMILAR TO PREVIOUS BOARD EXAM LEAN BACK-UP
seminal vesicles? CONCEPT/PRINCIPLE). The major energy source of ANGELO MIDTERM
A. Sorbitol dehydrogenase spermatozoa is fructose. This is produced by the cells SILVERIO, EXAM - FEB
B. Aldolase B of the seminal vesicles from glucose by two step MD (TOP 4 2015
C. Aldose reductase process. First is by NADPH dependent reduction of - AUG 2014
D. Glycerol 3 phosphate dehydrogenase glucose to sorbitol( aldose reductase) followed by MED
E. None of the above NAD dependent oxidation of sorbitol to fructose BOARDS;
(sorbitol dehydrogenase). of the two enzymes, the TOPNOTCH
activity of the former is comparable to all other cells MD), MD
while the latter enzymes is specifically active in
seminal vesicles. Devlin 6th p 615
305 Which of the following is the end product of propionyl CoA is the end product of the ff metabolism: LEAN BACK-UP
isoleucine,valine and methionine metabolism? isoleucine, valine, methionine, odd chain fatty acid ANGELO MIDTERM
A. Acetyl CoA oxidation and side chain of cholesterol degradation. SILVERIO, EXAM - FEB
B. Succinyl CoA Conversion of proprionyl CoA to methylmalonyl CoA MD (TOP 4 2015
C. Propionyl CoA is by the biotin requiring enzyme propionyl CoA - AUG 2014
D. Methylmalonyl CoA carboxylase. further conversion of methylmalonyl CoA MED
E. None of the above to succinyl CoA is by Vit B12 requiring methylmalonyl BOARDS;
CoA mutase. Devlin 6th ed p777 TOPNOTCH
MD), MD
306 how many ATP/s is/are required to produce urea in Kreb Henseleit cycle or the urea cycle requires 4 ATPs LEAN BACK-UP
Kreb Henseleit Cycle? to produce and excrete urea. Two ATPs are required ANGELO MIDTERM
A. 1 ATP in the formation of carbamoyl phosphate and 2 ATPs SILVERIO, EXAM - FEB
B. 2 ATP for the formation of arginosuccinate. Refer to your MD (TOP 4 2015
C. 3 ATP handouts for the diagram. Devlin 6th ed pp 752 - AUG 2014
D. 4 ATP MED
E. No ATP needed. BOARDS;
TOPNOTCH
MD), MD
307 which of the following amino acid is an essential phosphatidylcholine is an essential component of cell LEAN BACK-UP
component in the de novo synthesis of membranes, myelin sheath and mature pulmonary ANGELO MIDTERM
phosphatidylcholine? surfactant. It is formed by phosphorylation of choline SILVERIO, EXAM - FEB
A. arginine obtained from the diet followed by addition of 3 MD (TOP 4 2015
B. threonine methyl groups. The activated form of methionin (S- - AUG 2014
C. serine adenosyl methionine) is the only amino acid that can MED
D. Methionine donate methyl groups. BOARDS;
E. None of the above TOPNOTCH
MD), MD

TOPNOTCH MEDICAL BOARD PREP BIOCHEMISTRY SUPEREXAM Page 39 of 92


For inquiries visit www.topnotchboardprep.com.ph or email us at topnotchmedicalboardprep@gmail.com
TOPNOTCH MEDICAL BOARD PREP BIOCHEMISTRY SUPEREXAM
For inquiries visit www.topnotchboardprep.com.ph or email us at topnotchmedicalboardprep@gmail.com
Item QUESTION EXPLANATION AUTHOR TOPNOTCH
# EXAM
308 true about sphingolipidoses except? sphingolipidosis is a group of inherited disorder LEAN BACK-UP
A. There is upregulation in the synthesis of stored characterized by accumulation of complex lipids. This ANGELO MIDTERM
lipids is primarily due to defective lyososomal degradation SILVERIO, EXAM - FEB
B. Most are autosomal recessive pathway. It particularly affect the CNS because of its MD (TOP 4 2015
C. Accumulation of GM2 ganglioside is caused by reliance to complex lipid myelination processes. The - AUG 2014
hexosaminidase A deficiency synthesis of stored lipids is intact and not MED
D. Deficiency of alpha galactosidase is the cause of upregulated. Most are autosomal recessive except for BOARDS;
fabry's disease fabry disease ( deficiency in a galactosidase) which is TOPNOTCH
E. none of the above x linked recessive.Tay sach disease presenting mental MD), MD
retardation, cherry red macula and muscular
weakness, is due to deficiency in hexosaminidase A
leading accumulation of GM2 ganglioside.
309 Which of the following processes does not occur during adipocytes does not have glycerol kinase instead LEAN BACK-UP
mobilization of stored fats and fatty acid oxidation? glycerol is transported to the liver wherein it is ANGELO MIDTERM
A. cAMP mediated activation of hormone sensitive phosphorylated to produce dihydroacetone phosphate SILVERIO, EXAM - FEB
lipase which then can be converted to metabolic precursors MD (TOP 4 2015
B. Upregulation of glycerol kinase in the adipocytes for glycolysis or gluconeogenesis. Lippincott 2nd ed - AUG 2014
C. Transfer of acyl group from coenzyme A to carnitine pp 181-182 MED
for transfer to the mitochondrial matrix BOARDS;
D. shortening of the fatty acid chain via sequential TOPNOTCH
oxidation,hydration,oxidation, thiolysis reaction MD), MD
E. none of the above

310 failure in the addition of this signal sequence to I cell disease is characterized by accumulation of LEAN BACK-UP
lysosomal proteins leads to characteristic mental lysosomal proteins in the serum due to failure of the ANGELO MIDTERM
retardation, coarse facial features, and multiple joint latter to be compartmentalized in the lysosomes. This SILVERIO, EXAM - FEB
restriction of I cell disease. is due to absent addition of mannose 6 phosphate MD (TOP 4 2015
A. Glyceraldehyde 3 phosphate that is responsible for internalization of these - AUG 2014
B. apolipoproteins A2 enzymes. MED
C. Galactoacylceramide BOARDS;
D. Mannose 6 phosphate TOPNOTCH
E. none of the above MD), MD

311 During eukaryotic DNA replication, what type of important eukaryotic polymerase enzymes in DNA LEAN BACK-UP
polymerase is responsible for the synthesis of RNA replication are as follows: alpha - formation of RNA ANGELO MIDTERM
primer in both leading and lagging strand? primers for both leading and lagging strands, delta- SILVERIO, EXAM - FEB
A. Pol alpha elongation of leading strand, epsilon - elongation of MD (TOP 4 2015
B. Pol beta lagging strand, gamma - replication of mitochondrial - AUG 2014
C. Pol gamma DNA. MED
D. Pol delta BOARDS;
E. Pol epsilon TOPNOTCH
MD), MD
312 which of the following enzyme- product pair is inhibited alpha amanitin inhibits eukaryotic RNA polymerase II LEAN BACK-UP
even by low concentration amanitin (amanita phalloides enzyme that is responsible for the formation of mRNA. ANGELO MIDTERM
mushroom) poisoning ? RNA pol I forms the rRNA while RNA pol III is for SILVERIO, EXAM - FEB
A. RNA pol I/ tRNA tRNA,snRNA and 5S ribosomal RNA. Lippincott 2nd ed MD (TOP 4 2015
B. RNA pol II/mRNA p 384. - AUG 2014
C. RNA pol III/rRNA MED
D. RNA pol I/rRNA BOARDS;
E. RNA pol II/ snRNA TOPNOTCH
MD), MD
313 During bacterial RNA synthesis, what is the promoter for prokaryotic organisms, the promoter regions in LEAN BACK-UP
region for RNA polymerase to bind that is located 10 the DNA are the pribnow box ( 8-10 nucleotide ANGELO MIDTERM
bases upstream from transcription start site? upstream) and -35 sequence while for the eukaryotics SILVERIO, EXAM - FEB
A. Sigma factor it is the CAAT box (80 nucleotide upstream) and MD (TOP 4 2015
B. CAAT box TATA/hogness (25 nucleotide upstream) box. On the - AUG 2014
C. Pribnow box otherhand, sigma factor is a component of the RNA MED
D. TATA box polymerase responsible for recognizing the specific BOARDS;
E. Rho factor promoter region in the DNA strand while rho factor TOPNOTCH
promotes termination of RNA synthesis. MD), MD

314 The atoms of the purine ring are contributed by the purine ring atoms are contributed by the following: LEAN BACK-UP
following except? aspartate, glycine, glutamine, N10 ANGELO MIDTERM
A. Aspartic acid formyltetrahydrofolate and carbon dioxide. For SILVERIO, EXAM - FEB
B. glutamine pyrimidine ring it is contributed by glutamine, MD (TOP 4 2015
C. CO2 aspartic acid and CO2 lippincott 2nd ed p 345 - AUG 2014
D. Thiamine pyrophosphate MED
E. Tetrahydrofolate BOARDS;
TOPNOTCH
MD), MD
315 what is the specific mechanism on how clindamycin (SIMILAR TO PREVIOUS BOARD EXAM LEAN BACK-UP
exerts its effect in protein synthesis? CONCEPT/PRINCIPLE) A- streptomycin,B- ANGELO MIDTERM
A. Binds to 30s subunit inhibiting initiation of protein tetracycline, D- chlorampenicol. Lippincott 2nd ed p SILVERIO, EXAM - FEB
synthesis 396 MD (TOP 4 2015
B. Blocks the interaction between aminoacyl tRNA to the - AUG 2014
mRNA ribosome complex MED
C. irreversible binding to 50s subunit preventing BOARDS;
translocation TOPNOTCH
D. inhibits 50s peptidyl transferase preventing MD), MD
translocation
E. none of the above

TOPNOTCH MEDICAL BOARD PREP BIOCHEMISTRY SUPEREXAM Page 40 of 92


For inquiries visit www.topnotchboardprep.com.ph or email us at topnotchmedicalboardprep@gmail.com
TOPNOTCH MEDICAL BOARD PREP BIOCHEMISTRY SUPEREXAM
For inquiries visit www.topnotchboardprep.com.ph or email us at topnotchmedicalboardprep@gmail.com
Item QUESTION EXPLANATION AUTHOR TOPNOTCH
# EXAM
316 the compromise in the energy production seen in wet wet beriberi is a type of severe thiamine deficiency LEAN BACK-UP
beriberi is primarily due to what altered process? characterized by high output heart failure, ANGELO MIDTERM
A. Absence of cofactor in the transketolase activity neuromuscular symptoms and weakness. The lack of SILVERIO, EXAM - FEB
B. Inhibition of PDH and alpha ketoglutarate complex energy generation by the cells is due to inhibition of MD (TOP 4 2015
C. Lack of cofactor supply during oxidation and PDH and a ketogluterate dehydrogenase complex - AUG 2014
reduction reactions leading to decrease in the levels of ATP and reducing MED
D. inhibition of transamination reaction leading to equivalents. transketolase reaction is also catalyzed BOARDS;
underutilization of amino acids during gluconeogenesis by thiamine however it is not involved in energy TOPNOTCH
E. all of the above generating pathway but rather it is required in HMP. MD), MD
transamination reaction requires pyridoxine as a
cofactor. oxidation reduction reactions uses riboflavin
(FAD) and niacin (NAD and NADP) Devlin 6th ed pp
1102-1103
317 Supplementation of which trace mineral appears to chromium is the component of low molecular weight LEAN BACK-UP
improve the glycemic control of patients with type II chromodulin which potentiates the effect of insulin by ANGELO MIDTERM
diabetes mellitus? facilitating insulin binding to its receptor and receptor SILVERIO, EXAM - FEB
A. Chromium tyrosine kinase signalling. Devlin pp 1114 MD (TOP 4 2015
B. Copper - AUG 2014
C. Zinc MED
D. Selenium BOARDS;
E. Molybdenum TOPNOTCH
MD), MD
318 Formation of Nitric oxide requires which of the following Nitric oxide formerly known as EDRF is formed by the LEAN BACK-UP
substances? enzyme NO synthase. It uses arginine as a substrate ANGELO MIDTERM
A. Asparagine for oxidation to produce citrulline and NO. Harpers pp SILVERIO, EXAM - FEB
B. Acetylcholine 580 MD (TOP 4 2015
C. Citrulline - AUG 2014
D. Arginine MED
E. Glycine BOARDS;
TOPNOTCH
MD), MD
319 The following are the characteristics of smooth muscle Caldesmon is a regulatory protein ubiquitous in the LEAN BACK-UP
except? smooth muscle.at low calcium concentration, it binds ANGELO MIDTERM
A. Caldesmon promotes interaction of actin and myosin to tropomyosin and actin thereby preventing the SILVERIO, EXAM - FEB
B. Contains rudimentary sarcoplasmic reticulum interaction of actin and myosin. Harpers 27th ed p MD (TOP 4 2015
C. Extracellular fluid Ca is important for contraction 579-580 - AUG 2014
D. Exhibits latch state MED
E. none of the above BOARDS;
TOPNOTCH
MD), MD
320 A 4 weeks old neonate presents to your clinic with this is a classic case of Pompe disease caused by LEAN BACK-UP
generalized hypotonia, respiratory distress, dilated deficiency of a 1,4 glucosidase aka acid maltase. It is a ANGELO MIDTERM
jugular veins, and displaced apex beat. Laboratory shows form of enzyme defect wherein there is accumulation SILVERIO, EXAM - FEB
normal glucose levels, uric acid and blood lactate level. of glycogen granules inside the lysosomes. It doesn’t MD (TOP 4 2015
which of the following enzyme is deficient in this present with hypoglycemia and lactic acidosis since - AUG 2014
patient? enzymes for glycogen metabolism are intact. the most MED
A. glycogen debranching enzyme common presentation is cardiomyopathy with heart BOARDS;
B. muscle glycogen phosphorylase failure. no treatment is available and death usually TOPNOTCH
C. alpha 1,4 glucosidase occurs at 1 mo of age. Devlin p 621. MD), MD
D. glucose 6 phosphatase
E. none of the above
321 Which glycolytic enzyme catalyzes a reversible reaction? SIMILAR TO PREVIOUS BOARD EXAM DEBBIE BACK-UP
A. glyceraldehyde phosphate dehydrogenase CONCEPT/PRINCIPLE. Glyceraldehyde phosphate ROSE MIDTERM
B. hexokinase dehydrogenase yields 1,3-bisphosphoglycerate using TANENGSY EXAM - FEB
C. phosphofructokinase-1 glyceraldehyde-3-phosphate as its substrate. , MD (TOP 2015
D. pyruvate kinase 5 - AUG
E. none of the above 2014 MED
BOARDS;
TOPNOTCH
MD)
322 Deficiency of this fatty acid leads to decreased vision and SIMILAR TO PREVIOUS BOARD EXAM DEBBIE BACK-UP
altered learning behavior. CONCEPT/PRINCIPLE. Linolenic acid / omega 3 fatty ROSE MIDTERM
A. linoleic acid acid deficiency results in decreased vision and altered TANENGSY EXAM - FEB
B. omega 6 fatty acid learning behavior. Linoleic acid / omega 6 fatty acid is , MD (TOP 2015
C. arachidonic acid a precursor to arachidonic acid is thus essential in 5 - AUG
D. linolenic acid prostaglandin synthesis. 2014 MED
E. none of the above BOARDS;
TOPNOTCH
MD)
323 The following are causes of conjugated In Dubin Johnson syndrome, the gene encoding MRP- DEBBIE BACK-UP
hyperbilirubinemia, except: 2, involved in the secretion of already conjugated ROSE MIDTERM
A. Dubin Johnson syndrome bilirubin into bile, is mutated. Rotor syndrome is TANENGSY EXAM - FEB
B. Rotor syndrome characterized by chronic conjugated , MD (TOP 2015
C. biliary atresia hyperbilirubinemia and a normal liver histology. 5 - AUG
D. Gilbert syndrome Biliary atresia is a form of biliary tree obstruction and 2014 MED
E. no exception a cause of conjugated hyperbilirubinemia which BOARDS;
results in the regurgiation of bilirubin glucuronide TOPNOTCH
into hepatic veins and lymphatics. Gilbert syndrome is MD)
caused by mutations in the gene encoding bilirubin-
UGT. Harper Biochemistry 28th edition p.282

TOPNOTCH MEDICAL BOARD PREP BIOCHEMISTRY SUPEREXAM Page 41 of 92


For inquiries visit www.topnotchboardprep.com.ph or email us at topnotchmedicalboardprep@gmail.com
TOPNOTCH MEDICAL BOARD PREP BIOCHEMISTRY SUPEREXAM
For inquiries visit www.topnotchboardprep.com.ph or email us at topnotchmedicalboardprep@gmail.com
Item QUESTION EXPLANATION AUTHOR TOPNOTCH
# EXAM
324 True regarding the biochemical events occurring during Harper Biochemistry 28th edition p.550 DEBBIE BACK-UP
muscle contraction: ROSE MIDTERM
A. The S-1 head of myosin hydrolyzes ATP to ADP and TANENGSY EXAM - FEB
P1, which remain bound. , MD (TOP 2015
B. Formation of the actin-myosin-ADP-P1 complex 5 - AUG
promotes the release of P1, inititating the power stroke. 2014 MED
C. Myosin-ATP has a low affinity for actin, which is thus BOARDS;
released. TOPNOTCH
D. all of the above MD)
E. none of the above
325 What is the most potent positive allosteric activator of Harper Biochemistry 28th edition p.168 DEBBIE BACK-UP
phosphofructokinase-1, and an inhibitor of fructose-1,6- ROSE MIDTERM
bisphosphatase? TANENGSY EXAM - FEB
A. fructose-2,6-bisphosphate , MD (TOP 2015
B. fructose-6-phosphate 5 - AUG
C. glucose 2014 MED
D. acetyl CoA BOARDS;
E. none of the above TOPNOTCH
MD)
326 In the fed state, the following enzymes' activities are In the fasting state, glucagon stimulates the DEBBIE BACK-UP
expected to increase, except: production of cAMP, activating cAMP-dependent ROSE MIDTERM
A. phosphofructokinase-1 protein kinase, which in turn inactivates PFK-1 and TANENGSY EXAM - FEB
B. pyruvate dehydrogenase activates fructose-2,6-bisphosphatase by , MD (TOP 2015
C. glucokinase phosphorylation. Harper Biochemistry 28th edition 5 - AUG
D. glycogen synthase p.169 2014 MED
E. cAMP dependent protein kinase BOARDS;
TOPNOTCH
MD)
327 Bile acid conjugation with glycine or taurine takes place Harper Biochemistry 28th edition p.229 DEBBIE BACK-UP
in which organelle? ROSE MIDTERM
A. golgi apparatus TANENGSY EXAM - FEB
B. endoplasmic reticulum , MD (TOP 2015
C. cytosol 5 - AUG
D. peroxisomes 2014 MED
E. none of the above BOARDS;
TOPNOTCH
MD)
328 What is true regarding the pentose phosphate pathway? The enzymes involved in the pentose phosphate DEBBIE BACK-UP
A. The enzymes involved are mitochondrial. pathway are cytosolic. NADP+ is the electron acceptor, ROSE MIDTERM
B. Oxidation is achieved by dehydrogenation using NAD+ and this cycle does not yield ATP. Harper TANENGSY EXAM - FEB
as an electron acceptor. Biochemistry 28th edition p.175-1 , MD (TOP 2015
C. Like the TCA cycle, this pathway yields ATP. 5 - AUG
D. This pathway can account for the complete oxidation 2014 MED
of glucose. BOARDS;
E. Glutathione reductase contains selenocysteine at its TOPNOTCH
active site. MD)
329 Which is a true statement? Neonatal physiologic jaundice is the most common DEBBIE BACK-UP
A. Gilbert syndrome is the most common cause of cause of unconjugated hyperbilirubinemia. ROSE MIDTERM
unconjugated hyperbilirubinemia. Phenobarbital has no effect in the treatment of Crigler TANENGSY EXAM - FEB
B. Phenobarbital is effective in the treatment of Crigler Najar type I. An obstructive condition is suspected , MD (TOP 2015
Najar type I. when urinalysis reveals increased amount of 5 - AUG
C. Crigler Najar type II is the benign form. conjugated bilirubin but no urobilinogen. 2014 MED
D. A hemolytic condition is suspected when urinalysis BOARDS;
reveals increased amount of conjugated bilirubin but no TOPNOTCH
urobilinogen. MD)
E. none of the above
330 The following are glucogenic, except: Ketogenic only: Leu & Lys. Propionyl resude from an DEBBIE BACK-UP
A. leucine odd chain fatty acid is the only part of a fatty acid ROSE MIDTERM
B. propionyl CoA that's glucogenic. TANENGSY EXAM - FEB
C. lysine , MD (TOP 2015
D. A & B 5 - AUG
E. A & C 2014 MED
BOARDS;
TOPNOTCH
MD)
331 The following are true regarding non-alpha-amino acids, All statements are true. Harper Biochemistry 28th DEBBIE BACK-UP
except? edition pp.266-268 ROSE MIDTERM
A. GABA functions as an inhibitory neurotransmitter in TANENGSY EXAM - FEB
brain tissue , MD (TOP 2015
B. Catabolism of uracil yields B-alanine. 5 - AUG
C. Catabolism of thymine yields B-aminoisobutyrate. 2014 MED
D. none of the above BOARDS;
TOPNOTCH
MD)
332 Which is a true statement? In the reduction of pyruvate to lactate, NADH+ acts as DEBBIE BACK-UP
A. Pyruvate is reduced to form lactate. the electron donor; whereas oxidation of lactate uses ROSE MIDTERM
B. Lactate is oxidized to form pyruvate. NAD+ as the electron acceptor. TANENGSY EXAM - FEB
C. all of the above , MD (TOP 2015
D. none of the above 5 - AUG
2014 MED
BOARDS;
TOPNOTCH
MD)

TOPNOTCH MEDICAL BOARD PREP BIOCHEMISTRY SUPEREXAM Page 42 of 92


For inquiries visit www.topnotchboardprep.com.ph or email us at topnotchmedicalboardprep@gmail.com
TOPNOTCH MEDICAL BOARD PREP BIOCHEMISTRY SUPEREXAM
For inquiries visit www.topnotchboardprep.com.ph or email us at topnotchmedicalboardprep@gmail.com
Item QUESTION EXPLANATION AUTHOR TOPNOTCH
# EXAM
333 The following amino acids are catabolized via a The catabolism of leucine, isoleucine, and valine DEBBIE BACK-UP
multimeric enzyme complex closely resembling pyruvate involves reactions analogous to catabolism of fatty ROSE MIDTERM
dehydrogenase, except: acids. There is an involvement of the enzyme TANENGSY EXAM - FEB
A. leucine branched chain alpha keto acid dehydrogenase, a , MD (TOP 2015
B. isoleucine multimeric enzyme complex of a decarboxylase, 5 - AUG
C. lysine transacetylase, and a dihydrolipoyl dehydrogenase 2014 MED
D. valine closely resembling pyruvate dehydrogenase. Harper BOARDS;
Biochemistry 28th edition pp.257-258 TOPNOTCH
MD)
334 Of the following steps in the processing of the fibrillar Harper Biochemistry 28th edition p.529 table 48-3 DEBBIE BACK-UP
collagen precursor, which one takes place ROSE MIDTERM
extracellularly? TANENGSY EXAM - FEB
A. cleavage of the signal peptide , MD (TOP 2015
B. formation of the triple helix 5 - AUG
C. quarter staggered alignment assembly 2014 MED
D. formation of intrachain and interchain disulfide bonds BOARDS;
in extension peptides TOPNOTCH
MD)
335 What is true about cardiac muscle but not of smooth Both cardiac and smooth muscle are syncytial. Nerve DEBBIE BACK-UP
muscle? impulse initiates contraction in skeletal muscle. ROSE MIDTERM
A. syncytial Troponin system is present in skeletal and cardiac TANENGSY EXAM - FEB
B. nerve impulse initiates contraction muscle. Caldesmon is important in smooth muscle , MD (TOP 2015
C. troponin system present only. Harper Biochemistry 28th edition p.555 table 5 - AUG
D. caldesmon an important regulatory protein 49-3 2014 MED
BOARDS;
TOPNOTCH
MD)
336 True regarding skeletal muscle, except: Harper Biochemistry 28th edition p.562 table 49-12 DEBBIE BACK-UP
A. Skeletal muscle can utilize ketone bodies during ROSE MIDTERM
starvation. TANENGSY EXAM - FEB
B. Skeletal muscle can contribute to blood glucose. , MD (TOP 2015
C. Its proteolysis during starvation supplies amino acids 5 - AUG
for gluconeogenesis. 2014 MED
D. Glucagon is unable to stimulate glycogenolysis in BOARDS;
skeletal muscle. TOPNOTCH
MD)
337 The following clotting factors are properly paired with There is no factor VI. Proconvertin / cothromboplastin DEBBIE BACK-UP
their common name, except: / serum prothrombin convertion accelerator (SPCA) is ROSE MIDTERM
A. III - tissue factor factor VII. Harper Biochemistry 28th edition p. 585 TANENGSY EXAM - FEB
B. IV - calcium table 51-1 , MD (TOP 2015
C. V - proaccelerin 5 - AUG
D. VI - proconvertin 2014 MED
BOARDS;
TOPNOTCH
MD)
338 RBCs make use of this glucose transporter: Harper Biochemistry 28th edition p. 596 DEBBIE BACK-UP
A. GLUT 1 ROSE MIDTERM
B. GLUT 2 TANENGSY EXAM - FEB
C. GLUT 3 , MD (TOP 2015
D. GLUT 4 5 - AUG
2014 MED
BOARDS;
TOPNOTCH
MD)
339 Not true regarding anti-oxidants and free radicals: Beta carotene is an autocatalytic pro-oxidant at higher DEBBIE BACK-UP
A. Anti-oxidants can also be pro-oxidants. concentrations of oxygen and is only an antioxidant at ROSE MIDTERM
B. Radical damage is a known cause of atherosclerosis. low oxygen concentrations. Harper Biochemistry 28th TANENGSY EXAM - FEB
C. Oxygen radicals are the most damaging radicals in edition pp. 482-485 , MD (TOP 2015
biological systems. 5 - AUG
D. Beta carotene is an antioxidant at high concentrations 2014 MED
of oxygen. BOARDS;
TOPNOTCH
MD)
340 With regards to isomerism, glucose and galactose are: Glucose and galactose are epimers, differing only at DEBBIE BACK-UP
A. epimers, differing only at carbon 2 carbon 4. Choice A refers to glucose and mannose. ROSE MIDTERM
B. isomers TANENGSY EXAM - FEB
C. enantiomers , MD (TOP 2015
D. anomers 5 - AUG
2014 MED
BOARDS;
TOPNOTCH
MD)
341 True of the peptide bond, except: Characteristics of the peptide bond: JESSICA BACK-UP
A. Formed by a reaction of the α-COOH and α-NH2 of two - attaches the α-COOH to the α-NH2 of another MAE MIDTERM
amino acids - very stable, can only be disrupted by hydrolysis SANCHEZ, EXAM - FEB
B. Has a partial double bond character through prolonged exposure to a strong acid or base MD (TOP 4 2015
C.Generally in the cis configuration at elevated temperatures - AUG 2014
D. Rigid and planar - polar and can form hydrogen bonds MED
- partial double bond character, which makes the bond BOARDS;
rigid and planar TOPNOTCH
- generally in the trans configuration MD)

Reference: Topnotch Biochem Handouts

TOPNOTCH MEDICAL BOARD PREP BIOCHEMISTRY SUPEREXAM Page 43 of 92


For inquiries visit www.topnotchboardprep.com.ph or email us at topnotchmedicalboardprep@gmail.com
TOPNOTCH MEDICAL BOARD PREP BIOCHEMISTRY SUPEREXAM
For inquiries visit www.topnotchboardprep.com.ph or email us at topnotchmedicalboardprep@gmail.com
Item QUESTION EXPLANATION AUTHOR TOPNOTCH
# EXAM
342 Which of the following is NOT a tyrosine derived Tyrosine is a precursor of L-dopa, dopamine (also JESSICA BACK-UP
hormone? known as Prolactin Inhibiting Hormone or Prolactin MAE MIDTERM
A. Vasopressin Inhibiting Factor), norepinephrine, epinephrine, SANCHEZ, EXAM - FEB
B. Prolactin inhibiting hormone thyroxine and melanin. MD (TOP 4 2015
C. Thyroxine - AUG 2014
D. Epinephrine Vasopressin, or anti-diuretic hormone, is a peptide MED
hormone composed of 9 amino acids, and is not BOARDS;
directly derived from tyrosine. TOPNOTCH
MD)
343 Enzymes as biologic catalysts accelerate biochemical Enzymes act by lowering the free energy of activation, JESSICA BACK-UP
reactions by: but do not change the energy of the reactants and MAE MIDTERM
A. Lowering the net free energy change (ΔG0) of the products, nor the equilibrium of the reaction. SANCHEZ, EXAM - FEB
reaction MD (TOP 4 2015
B. Lowering the free energy of activation Reference: Topnotch Biochem Handouts - AUG 2014
C. Altering the thermodynamic properties of the reaction MED
D. Decreasing the Km of the enzyme for its substrate BOARDS;
TOPNOTCH
MD)
344 All of the following complexes in the Electron Transport Protons are pumped to the intermembranous space to JESSICA BACK-UP
Chain are sites of ATP synthesis, except: create a gradient (therefore, produce ATP) in 3 MAE MIDTERM
A. NADH dehydrogenase complexes: SANCHEZ, EXAM - FEB
B. Cytochrome b-c1 complex - Complex I or NADH dehydrogenase MD (TOP 4 2015
C. Succinate-CoQ dehydrogenase - Complex III or Cytochrome b/c1 complex - AUG 2014
D. Cytochrome aa3 complex - Complex IV or Cytochrome a/a3 or cytochrome MED
oxidase BOARDS;
TOPNOTCH
Complex II (Succinate dehydrogenase) does not MD)
produce ATP.
345 Which of the following compounds inhibit biological Uncouplers act by dissociating respiration from ATP JESSICA BACK-UP
oxidation by dissociating respiration from ATP synthesis. MAE MIDTERM
synthesis? Examples of uncouplers are 2,4 dinitrophenol, aspirin, SANCHEZ, EXAM - FEB
A. Antimycin A and thermogenin (brown fat). MD (TOP 4 2015
B. 2,4 dinitrophenol - AUG 2014
C. Carbon monoxide Antimycin A - inhibits Complex III MED
D. Malonate Carbon monoxide - inhibits Complex IV BOARDS;
Malonate - inhibits Complex II TOPNOTCH
MD)

346 Which of the following is true of the amino acids valine, Valine, leucine and isoleucine are branched chain JESSICA BACK-UP
leucine, and isoleucine? amino acids, with nonpolar, uncharged side chains; MAE MIDTERM
A. Characterized with a polar but uncharged side chain they are non ionizable and are considered SANCHEZ, EXAM - FEB
B. Classified under branched chain amino acids nutritionally essential. MD (TOP 4 2015
C. All are nutritionally non essential - AUG 2014
D. Possess ionizable functional groups MED
BOARDS;
TOPNOTCH
MD)
347 Hemoglobin and proteins composed of several subunits Primary, secondary, and tertiary levels of protein JESSICA BACK-UP
assume this level of protein structure: structure are composed of only one polypeptide chain. MAE MIDTERM
A. Primary SANCHEZ, EXAM - FEB
B. Secondary MD (TOP 4 2015
C. Tertiary - AUG 2014
D. Quaternary MED
BOARDS;
TOPNOTCH
MD)
348 Which of the following is true of Km? Km, or the Michaelis constant, is the substrate JESSICA BACK-UP
A. Corresponds to the substrate concentration when concentration at which the initial velocity is half the MAE MIDTERM
initial velocity is Vmax/2 maximal velocity (Vmax/2). SANCHEZ, EXAM - FEB
B. Its value increases when the affinity of the substrate to Km increases when the affinity to the substrate is low, MD (TOP 4 2015
the enzyme increases. and decreases when the affinity is high. - AUG 2014
C. Corresponds to the reciprocal value of the y-axis in the In the Lineweaver Burk plot, the Km is the reciprocal MED
Lineweaver Burk plot. value of the x-axis. BOARDS;
D. is equal to the value of Vmax when the enzyme is fully Km is always half of Vmax, and is never equal to its TOPNOTCH
saturated with the substrate value. MD)
349 This protein digestive enzyme has the capacity to Trypsin is capable of activating trypsinogen (its JESSICA BACK-UP
activate its own zymogen: zymogen) into more trypsin, which in turn activates MAE MIDTERM
A. Pepsinogen other protein digestive enzymes. SANCHEZ, EXAM - FEB
B. Elastase MD (TOP 4 2015
C. Trypsin - AUG 2014
D. Chymotrypsin MED
BOARDS;
TOPNOTCH
MD)
350 Classic galactosemia can cause cataract formation due to Galactitol, once formed from galactose by aldose JESSICA BACK-UP
the following, except: reductase, accumulates within the lens and causes MAE MIDTERM
A. UDP-Glc:galactose-1PO4 uridyltransferase deficiency early cataract formation found in classic galactosemia. SANCHEZ, EXAM - FEB
B. Accumulation of galactitol inside the lens cells MD (TOP 4 2015
C. Action of aldose reductase forming galactitol - AUG 2014
D. Ability of galactitol to diffuse out of the lens cell MED
BOARDS;
TOPNOTCH
MD)

TOPNOTCH MEDICAL BOARD PREP BIOCHEMISTRY SUPEREXAM Page 44 of 92


For inquiries visit www.topnotchboardprep.com.ph or email us at topnotchmedicalboardprep@gmail.com
TOPNOTCH MEDICAL BOARD PREP BIOCHEMISTRY SUPEREXAM
For inquiries visit www.topnotchboardprep.com.ph or email us at topnotchmedicalboardprep@gmail.com
Item QUESTION EXPLANATION AUTHOR TOPNOTCH
# EXAM
351 Carbamoyl phosphate synthase I differs from Carbamoyl All the other choices except D describe CPS II, the rate JESSICA BACK-UP
phosphate synthase II in that the former: limiting enzyme in de novo pyrimidine synthesis. MAE MIDTERM
A. Is involved in pyrimidine de novo synthesis SANCHEZ, EXAM - FEB
B. Is stimulated by PRPP MD (TOP 4 2015
C. Is found in the cytosol - AUG 2014
D. Is a mitochondrial urea cycle enzyme MED
BOARDS;
TOPNOTCH
MD)
352 Which apolipoprotein is vital in the clearance of TAGs Apo C-II activates lipoprotein lipase for the clearance JESSICA BACK-UP
from chylomicrons and VLDL by lipoprotein lipase? of triacylglycerols from chylomicrons and VLDS. MAE MIDTERM
A. Apo E Apo E - mediates uptake of the chylomicron remnant SANCHEZ, EXAM - FEB
B. Apo C-II Apo A-1 - found in HDL; activates LCT/PCAT to MD (TOP 4 2015
C. Apo A-1 produce cholesterol esters - AUG 2014
D. Apo B-48 Apo B-48 - functions in chylomicron assembly and MED
secretion BOARDS;
TOPNOTCH
Reference: Topnotch Biochem Handouts MD)
353 A 30 year old patient was seen by his physician because Familial Hypercholesterolemia, which manifests as JESSICA BACK-UP
of hypertension. Aside from the high blood pressure, lab high LDL levels in laboratory determinations, is due to MAE MIDTERM
tests revealed elevated blood cholesterol but normal a deficiency in the LDL receptor. This confers an SANCHEZ, EXAM - FEB
triglycerides. He was diagnosed as a case of Familial increased risk of atherosclerosis and coronary heart MD (TOP 4 2015
Hypercholesterolemia. The elevated cholesterol in this disease. - AUG 2014
condition is mainly due to: MED
A. Absence of feedback inhibition on the committed step BOARDS;
of cholesterol synthesis TOPNOTCH
B. Defect in LDL receptor and receptor-mediated MD)
endocytosis of LDL cholesterol
C. Decreased lipoprotein lipase activity
D. Absence of apoprotein B-48, C-II and E
354 Which condition promotes a rightward shift of the An increase in CO2, hydrogen ions (acidosis), 2,3-BPG, JESSICA BACK-UP
hemoglobin saturation curve? temperature, and exercise, promotes a shift to the MAE MIDTERM
A. Decrease in intracellular pH of 7.2 right of oxygen-hemoglobin dissociation curve. SANCHEZ, EXAM - FEB
B. Core body temperature of 32°C MD (TOP 4 2015
C. pCO2 of 25 - AUG 2014
D. Decreased 2,3 BPG concentration MED
BOARDS;
TOPNOTCH
MD)
355 The following biochemical events take place during High CO2 within the muscles during exercise should JESSICA BACK-UP
exercise, except: promote diffusion of CO2 from the tissues to the RBC. MAE MIDTERM
A. High CO2 and low O2 concentration in the muscle SANCHEZ, EXAM - FEB
inhibit diffusion of CO2 from the tissues to the RBC MD (TOP 4 2015
B. Increased [H+] from lactic acid formation will promote - AUG 2014
O2 release by oxyhemoglobin MED
C. Increase in CO2 will favor deoxyhemoglobin BOARDS;
formation, promoting O2 unloading to the tissues TOPNOTCH
D. Increased heat generated in the muscles shifts the MD)
oxyhemoglobin binding curve to the right resulting to O2
unloading at the tissue level.
356 The structural feature of myoglobin that distinguishes it All the other choices except B describe hemoglobin. JESSICA BACK-UP
from hemoglobin: MAE MIDTERM
A. Quaternary protein composed of alpha and beta SANCHEZ, EXAM - FEB
globin chains stabilized by non-covalent interactions MD (TOP 4 2015
B. A single polypeptide chains folded to form a globular - AUG 2014
protein MED
C. A central cavity occupied by 2,3 bisphosphoglycerate BOARDS;
D. Salt bridges on the 4 terminal carboxyl groups of the TOPNOTCH
tetramer MD)
357 Which type of mutation causes premature termination of JESSICA BACK-UP
protein synthesis because of the creation of a stop codon MAE MIDTERM
during the genetic change? SANCHEZ, EXAM - FEB
A. Missense MD (TOP 4 2015
B. Neutral - AUG 2014
C. Nonsense MED
D. Silent BOARDS;
TOPNOTCH
MD)

358 Muscle relaxation that takes place after actin-myosin JESSICA BACK-UP
dissociation is due to: MAE MIDTERM
A. Binding of calcium to troponin C SANCHEZ, EXAM - FEB
B. Depletion of ATP in the muscle MD (TOP 4 2015
C. Increased formation of creatine phosphate - AUG 2014
D. Binding of ATP to the S-1 head of myosin to form a MED
myosin-ATP complex BOARDS;
TOPNOTCH
MD)

TOPNOTCH MEDICAL BOARD PREP BIOCHEMISTRY SUPEREXAM Page 45 of 92


For inquiries visit www.topnotchboardprep.com.ph or email us at topnotchmedicalboardprep@gmail.com
TOPNOTCH MEDICAL BOARD PREP BIOCHEMISTRY SUPEREXAM
For inquiries visit www.topnotchboardprep.com.ph or email us at topnotchmedicalboardprep@gmail.com
Item QUESTION EXPLANATION AUTHOR TOPNOTCH
# EXAM
359 Because of the amphipathic nature of bile acids, their Bile acids are not involved in the packaging of TAGs JESSICA BACK-UP
role in lipid digestion includes the following, except: into chylomicrons, which takes place inside the MAE MIDTERM
A. Serve as detergents binding to fat globules as they are intestinal epithelial cell. SANCHEZ, EXAM - FEB
broken down by peristaltic action of the GI tract MD (TOP 4 2015
B. Suspends dietary fat in smaller particles in the - AUG 2014
aqueous environment in the small intestines MED
C. involved in the packaging of TAGs into chylomicrons BOARDS;
D. solubilize the TAGs into micelles and transport them TOPNOTCH
to the intestinal cell surface MD)
360 This component of total energy expenditure plays the 60% of energy is used to maintain Resting or Basal JESSICA BACK-UP
most significant role in the total energy requirement of metabolic rate. MAE MIDTERM
an individual: 30% for physical activity, and 10% for diet-induced SANCHEZ, EXAM - FEB
A. Physical activity thermogenesis or thermic effect of food. MD (TOP 4 2015
B. Thermic effect of food - AUG 2014
C. Diet induced thermogenesis Reference: Topnotch Biochem Handouts MED
D. Basal energy expenditure BOARDS;
TOPNOTCH
MD)
361 Reduced glutathione (GSH) is composed of which of the Reduced glutathione is a tripeptide-thiol (γ- MAIRRE BACK-UP
following amino acids EXCEPT? glutamylcysteinylglycine) present in most cells which JAMES MIDTERM
A. Glycine can chemically detoxify hydrogen peroxide Lippincott GADDI, MD EXAM - FEB
B. Cysteine 5th pg 148 (TOP 4 - 2015
C. Glutamate AUG 2013
D. Serine MED
E. None of the above BOARDS;
TOPNOTCH
MD)
362 The following are TRUE of the urea cycle EXCEPT CPS I absolutely requires N-acetyl glutamate as an MAIRRE BACK-UP
A. The rate limiting step of the cycle is catalyzed by allosteric activator. In the urea cycle, 3 molecules of JAMES MIDTERM
carbamoyl phosphate synthetase I ATP are required but ultimately 4 ATP equivalents are GADDI, MD EXAM - FEB
B. CPS I absolutely requires N-acetyl glutamate consumed, (2 ATP for CPS I and 1 ATP for (TOP 4 - 2015
C. There are two ATP requiring enzymes and one of them argininosuccinate synthetase [converted to AMP AUG 2013
is argininosuccinate synthetase hence 2 equivalents used]). The cycle can also be used MED
D. It can be used to synthesize citrulline, ornithine and to synthesize ornithine, citrulline, arginine, fumarate BOARDS;
aspartate and urea. Lippincott 5th pg 254 TOPNOTCH
E. Carbon dioxide and free ammonia provides for the MD)
atoms of urea
363 HMG-CoA is involved in the synthesis of: The first two reactions in the cholesterol synthetic MAIRRE BACK-UP
A. Cholesterol pathway are similar to those in the pathway that JAMES MIDTERM
B. Ketone bodies produces ketone bodies. They result in the production GADDI, MD EXAM - FEB
C. Palmitate of HMG CoA through HMG CoA synthase. HMG CoA (TOP 4 - 2015
D. A and B synthase is the limiting enzyme of ketone body AUG 2013
E. All of the above synthesis while HMG CoA reductase is the limiting MED
enzyme of cholesterol synthesis. Lippincott 5th pg 220 BOARDS;
TOPNOTCH
MD)
364 The following are TRUE regarding prokaryotic RNA polymerase does not need a primer and has no MAIRRE BACK-UP
transcription EXCEPT proofreading activity in contrast to DNA polymerase. JAMES MIDTERM
A. Transcription starts at the promoter which contain Lippincott 5th 419-421 GADDI, MD EXAM - FEB
consensus sequences such as the -35 sequence and the (TOP 4 - 2015
TATAAT box AUG 2013
B. Termination of transcription can be spontaneous MED
through formation of a hairpin loop BOARDS;
C. RNA polymerase needs a primer to initiate TOPNOTCH
transcription MD)
D. Transcription proceeds from a 5’ to 3’ direction and is
identical to the sense strand with U replacing T
E. All are correct
365 A 14/F recently broke up with her boyfriend. Due to this Silver jewelry cleaner usually contains cyanide and MAIRRE BACK-UP
she ingested silver jewelry cleaner causing her to lose this causes inhibition of Cytochrome c oxidase JAMES MIDTERM
consciousness after a few minutes. She was immediately (Complex IV). Other substances which inhibit complex GADDI, MD EXAM - FEB
taken to the ER. On PE, she was still unconscious and had IV include sodium azide, hydrogen sulfide and carbon (TOP 4 - 2015
cherry red cold clammy skin. The patient most likely had monoxide. NADH dehydrogenase (Complex I), AUG 2013
inhibition of: Succinate dehydrogenase (Complex II), Cytochrome MED
A. NADH dehydrogenase b/c1 (Complex III). Lippincott 5th pg 75 BOARDS;
B. Succinate dehydrogenase TOPNOTCH
C. Cytochrome b/c1 MD)
D. Cytochrome c oxidase
E. Coenzyme Q
366 True of lipid metabolism EXCEPT Short and medium chain length fatty acids do not MAIRRE BACK-UP
A. Short and medium chain fatty acids are absorbed into require the assistance of mixed micelles for JAMES MIDTERM
the enterocyte through micelles absorption by the intestinal mucosa. GADDI, MD EXAM - FEB
B. Digestion of lipids begins in the stomach through acid Lippincott 5th pgs 173-177 (TOP 4 - 2015
stable lingual lipase and gastric lipase AUG 2013
C. Short and medium chain fatty acids are released MED
directly into the portal circulation bypassing the lacteals BOARDS;
D. Pancreatic lipase preferentially cleaves the fatty acids TOPNOTCH
at carbons 1 and 3 forming 2-monoacylglycerol plus free MD)
fatty acids

TOPNOTCH MEDICAL BOARD PREP BIOCHEMISTRY SUPEREXAM Page 46 of 92


For inquiries visit www.topnotchboardprep.com.ph or email us at topnotchmedicalboardprep@gmail.com
TOPNOTCH MEDICAL BOARD PREP BIOCHEMISTRY SUPEREXAM
For inquiries visit www.topnotchboardprep.com.ph or email us at topnotchmedicalboardprep@gmail.com
Item QUESTION EXPLANATION AUTHOR TOPNOTCH
# EXAM
367 Simvastatin is a reversible competitive inhibitor of the Competitive inhibition would cause an apparent MAIRRE BACK-UP
rate-limiting step in cholesterol biosynthesis. What increase in Km causing -1/Km to move closer to zero. JAMES MIDTERM
change would you expect for Lineweaver-Burke plot? Choice D would be expected for non-competitive GADDI, MD EXAM - FEB
A. The x-intercept would be closer to the zero due to an inhibition. Lippincott 5th pgs 60-61 (TOP 4 - 2015
apparent increase in Km AUG 2013
B. The x-intercept would be farther from zero due to an MED
apparent increase in Km BOARDS;
C. The y-intercept would be closer to zero due to an TOPNOTCH
apparent decrease in Vmax MD)
D. The y-intercept would be farther from zero due to an
apparent decrease in Vmax
368 The electron transport chain is embedded in the inner Mobile electron carriers: coenzyme Q and cytochrome MAIRRE BACK-UP
mitochondrial membrane. Which of the following is/are c. Lippincott 5th pg 74 JAMES MIDTERM
the mobile electron carrier/s? GADDI, MD EXAM - FEB
A. NADH dehydrogenase (TOP 4 - 2015
B. Coenzyme Q AUG 2013
C. Cytochrome b/c1 MED
D. Cytochrome c oxidase BOARDS;
E. B and D TOPNOTCH
MD)
369 The following are TRUE regarding the HMP shunt Phase 1 reactions produce NADPH and ribulose 5-P; MAIRRE BACK-UP
EXCEPT? Phase 2 reactions produce ribose 5-P not ribulose 5-P JAMES MIDTERM
A. Phase 1 reactions are irreversible Lippincott 5th pg 146 GADDI, MD EXAM - FEB
B. The main products of phase 2 reactions are ribulose 5- (TOP 4 - 2015
P, glyceraldehyde 3-P and fructose 6-P AUG 2013
C. The only product of phase 1 reactions is NADPH MED
D. B and C BOARDS;
E. No exception TOPNOTCH
MD)
370 Which essential amino acid is both ketogenic and Leucine and lysine are ketogenic and essential; MAIRRE BACK-UP
glucogenic? Threonine glucogenic and essential; Tyrosine both JAMES MIDTERM
A. Threonine glucogenic and ketogenic but nonessential Lippincott GADDI, MD EXAM - FEB
B. Tyrosine 5th pg 262 (TOP 4 - 2015
C. Tryptophan AUG 2013
D. Leucine MED
E. Lysine BOARDS;
TOPNOTCH
MD)
371 Which of the following is/are TRUE regarding RNA? introns are removed while exons are joined together MAIRRE BACK-UP
A. Postranslational modification of mRNA includes the Lippincott 5th pg 422 JAMES MIDTERM
addition of a polyadenylate tail and a methylguanosine GADDI, MD EXAM - FEB
cap (TOP 4 - 2015
B. snRPs are responsible for the removal of exons and AUG 2013
the joining of introns MED
C. RNA polymerase II synthesizes mRNA and miRNA BOARDS;
D. A and C TOPNOTCH
E. All of the above MD)
372 What percentage of total caloric intake of fat has been AMDR for adults is 45–65% of their total calories from MAIRRE BACK-UP
associated with reduced risk of chronic disease while carbohydrates, 20–35% from fat, and 10–35% from JAMES MIDTERM
still providing adequate amounts of the nutrient? protein Lippincott 5th pg 360 GADDI, MD EXAM - FEB
A. 45-65% (TOP 4 - 2015
B. 10-20% AUG 2013
C. 10-35% MED
D. 20-35% BOARDS;
E. 5-10% TOPNOTCH
MD)
373 Cephalin is made up of: Cephalin/phosphatidylethanolamine; A - cardiolipin; MAIRRE BACK-UP
A. two phosphatidic acid residues and a glycerol C - phosphatidylglycerol; D - JAMES MIDTERM
backbone lecithin/phosphatidylcholine Lippincott 5th pg 202 GADDI, MD EXAM - FEB
B. phosphatidic acid and ethanolamine (TOP 4 - 2015
C. phosphatidic acid and glycerol AUG 2013
D. phosphatidic acid and choline MED
E. None of the above BOARDS;
TOPNOTCH
MD)
374 Which of the following is/are TRUE of Vitamin A? Rhodopsin consists of 11-cis retinal and opsin; MAIRRE BACK-UP
A. Rhodopsin consists of 11-trans retinal and opsin Pseudotumor cerebri is caused by excessive intake of JAMES MIDTERM
B. Vitamin A deficiency can produce nightblindness, vitamin A Lippincott 5th pg 384-386 GADDI, MD EXAM - FEB
xerophthalmia and pseudotumor cerebri (TOP 4 - 2015
C. Retinol and retinal can readily be interconverted and AUG 2013
can be derived from retinoic acid MED
D. A and C BOARDS;
E. All of the above TOPNOTCH
MD)

375 NADPH required for fatty acid synthesis comes from the: Hexose monophosphate pathway is the major supplier MAIRRE BACK-UP
A. Hexose monophosphate pathway of NADPH for fatty acid synthesis but cytosolic malic JAMES MIDTERM
B. Cytosolic malic enzyme enzyme or NADP+ dependent malate dehydrogenase GADDI, MD EXAM - FEB
C. Succinyl dehydrogenase also produces cytosolic NADPH Lippincott 5th pg 186 (TOP 4 - 2015
D. A and B AUG 2013
E. All of the above MED
BOARDS;
TOPNOTCH
MD)

TOPNOTCH MEDICAL BOARD PREP BIOCHEMISTRY SUPEREXAM Page 47 of 92


For inquiries visit www.topnotchboardprep.com.ph or email us at topnotchmedicalboardprep@gmail.com
TOPNOTCH MEDICAL BOARD PREP BIOCHEMISTRY SUPEREXAM
For inquiries visit www.topnotchboardprep.com.ph or email us at topnotchmedicalboardprep@gmail.com
Item QUESTION EXPLANATION AUTHOR TOPNOTCH
# EXAM
376 Triacylglycerols comprise the majority of which CM 90%; VLDL 60%; LDL 8%; HDL 5% MAIRRE BACK-UP
lipoprotein? triacylglycerols Lippincott 5th pg 232 JAMES MIDTERM
A. Chylomicrons GADDI, MD EXAM - FEB
B. VLDL (TOP 4 - 2015
C. IDL AUG 2013
D. LDL MED
E. HDL BOARDS;
TOPNOTCH
MD)
377 Which of the following is/are the needed substrate/s in Sources of the atoms in the purine ring include MAIRRE BACK-UP
de novo purine synthesis? aspartate, carbon dioxide, glutamine, glycine and N10- JAMES MIDTERM
A. Aspartate THF. Pyrimidine ring: aspartate, carbon dioxide, GADDI, MD EXAM - FEB
B. Carbon dioxide glutamine Lippincott 5th pg 293 and 302 (TOP 4 - 2015
C. Glutamate AUG 2013
D. A and B MED
E. All of the above BOARDS;
TOPNOTCH
MD)
378 Which of the following is/are INCORRECTLY matched? Southwestern blot for DNA-binding proteins MAIRRE BACK-UP
A. Northern blot: RNA JAMES MIDTERM
B. Southern blot: DNA-binding proteins GADDI, MD EXAM - FEB
C. Western blot: protein (TOP 4 - 2015
D. A and B AUG 2013
E. None of the above MED
BOARDS;
TOPNOTCH
MD)
379 Which of the following amino acids is/are non-polar? glycine, alanine, valine, leucine, isoleucine, MAIRRE BACK-UP
A. Phenylalanine phenylalanine, tryptophan, methionine, proline: non JAMES MIDTERM
B. Proline polar Lippincott 5th pg 2 GADDI, MD EXAM - FEB
C. Tryptophan (TOP 4 - 2015
D. A and B AUG 2013
E. All of the above MED
BOARDS;
TOPNOTCH
MD)
380 Which of the following is/are TRUE regarding deoxy/taut (T) form - low affinity; oxy/relaxed (R) MAIRRE BACK-UP
hemoglobin? form - high affinity for oxygen Lippincott 5th pg 28 JAMES MIDTERM
A. One gram of hemoglobin is capable of binding 1.34 mL GADDI, MD EXAM - FEB
of O2 (TOP 4 - 2015
B. Binding of an oxygen molecule to one heme group AUG 2013
increases the oxygen affinity of the other groups MED
C. The deoxy form of hemoglobin is constrained and has BOARDS;
low affinity for oxygen TOPNOTCH
D. A and B MD)
E. All of the above
381 Which of the following are non-polar amino acids? SIMILAR TO PREVIOUS BOARD EXAM SCOTT BACK-UP
A. Leucine CONCEPT/PRINCIPLE. Non-polar amino acids: Gly, RILEY ONG, MIDTERM
B. Phenylalanine Ala, Phe, Trp, Met, Pro. Uncharged, polar amino acids: MD (TOP 5 EXAM - FEB
C. Methionine Cys, Tyr, Ser, Thr, Asn, Gln. Basic amino acids: His, Arg, - AUG 2014 2015
D. All of the above Lys. Acidic amino acids: Glu, Asp. MED
E. None of the above BOARDS;
TOPNOTCH
MD)
382 Which of the following enzymes participate in both The other choices participate in non-reversible steps SCOTT BACK-UP
glycolysis and gluconeogenesis? of glycolysis only. RILEY ONG, MIDTERM
A. Phosphofructokinase-1 MD (TOP 5 EXAM - FEB
B. Hexokinase - AUG 2014 2015
C. Phosphoglycerate kinase MED
D. Pyruvate kinase BOARDS;
E. Glucokinase TOPNOTCH
MD)

383 Which of the following substances is a known inhibitor Complex I inhibitors: barbiturate, piericidin A, amytal, SCOTT BACK-UP
of complex III of the electron transport chain? rotenone. Complex II inhibitors: malonate, carboxin, RILEY ONG, MIDTERM
A. Amytal TTFA. Complex III inhibitors: antimycin A, MD (TOP 5 EXAM - FEB
B. Carboxin dimercaprol. Complex IV inhibitors: cyanide, carbon - AUG 2014 2015
C. Malonate monoxide, sodium azide, hydrogen sulfide. MED
D. Actinomycin A BOARDS;
E. Sodium azide TOPNOTCH
MD)

384 What is the rate-limiting enzyme in glycolysis? SCOTT BACK-UP


A. Hexokinase RILEY ONG, MIDTERM
B. Phosphofructokinase MD (TOP 5 EXAM - FEB
C. Phosphoglycerate kinase - AUG 2014 2015
D. Pyruvate kinase MED
E. Both A and B BOARDS;
TOPNOTCH
MD)

TOPNOTCH MEDICAL BOARD PREP BIOCHEMISTRY SUPEREXAM Page 48 of 92


For inquiries visit www.topnotchboardprep.com.ph or email us at topnotchmedicalboardprep@gmail.com
TOPNOTCH MEDICAL BOARD PREP BIOCHEMISTRY SUPEREXAM
For inquiries visit www.topnotchboardprep.com.ph or email us at topnotchmedicalboardprep@gmail.com
Item QUESTION EXPLANATION AUTHOR TOPNOTCH
# EXAM
385 Which of the following is a cofactor of xanthine oxidase? SCOTT BACK-UP
A. Molybdenum RILEY ONG, MIDTERM
B. Pyridoxine MD (TOP 5 EXAM - FEB
C. Selenium - AUG 2014 2015
D. Copper MED
E. Vitamin C BOARDS;
TOPNOTCH
MD)
386 Which of the following vitamins is required for proper SCOTT BACK-UP
hydroxylation of amino acid residues in collagen? RILEY ONG, MIDTERM
A. Vitamin A MD (TOP 5 EXAM - FEB
B. Vitamin B6 - AUG 2014 2015
C. Vitamin C MED
D. Vitamin D BOARDS;
E. Vitamin E TOPNOTCH
MD)

387 Apo-CII can be found in the following lipoproteins Apo-CII is an activator of lipoprotein lipase and is SCOTT BACK-UP
except: naturally found in chylomicrons and VLDL. HDL is RILEY ONG, MIDTERM
A. Chylomicron used to shuttle Apo-CII. MD (TOP 5 EXAM - FEB
B. VLDL - AUG 2014 2015
C. LDL MED
D. HDL BOARDS;
E. None of the above TOPNOTCH
MD)
388 In diabetes mellitus, excess glucose is converted to Sorbitol is metabolized by sorbital dehydrogenase SCOTT BACK-UP
sorbitol, causing some of its complications including into fructose. Remember that fructose is one of the RILEY ONG, MIDTERM
cataract and neuropathy. Sorbital accumulates in the secretions of the seminal vesicles for the seminal fluid. MD (TOP 5 EXAM - FEB
body because its metabolism is very slow, occurring only - AUG 2014 2015
in the liver and which other body organ? MED
A. Seminal vesicle BOARDS;
B. Brain TOPNOTCH
C. Pancreas MD)
D. Choroid body
E. Kidney

389 Which of the following enzymes can be seen in cancer SIMILAR TO PREVIOUS BOARD EXAM SCOTT BACK-UP
cells and is partly responsible for their unlimited CONCEPT/PRINCIPLE. RILEY ONG, MIDTERM
replicative potential by preventing the shortening of the MD (TOP 5 EXAM - FEB
highly repetitive DNA sequences at the ends of their - AUG 2014 2015
chromosomes? MED
A. DNA polymerase III BOARDS;
B. Telomerase TOPNOTCH
C. Primase MD)
D. DNA A protein
E. Replicase

390 Which of the following fatty acids is essential and is Linolenic acid = omega-3 fatty acid SCOTT BACK-UP
needed for proper development of vision and learning RILEY ONG, MIDTERM
behavior? MD (TOP 5 EXAM - FEB
A. Linolenic acid - AUG 2014 2015
B. Linoleic acid MED
C. Oleic acid BOARDS;
D. Arachidonic acid TOPNOTCH
E. Stearic acid MD)

391 Which of the following is the precursor of the active form Cholecalciferol, also known as 7-dehydrocholesterol, SCOTT BACK-UP
of vitamin D that is found naturally occurring in the skin? is the naturally occurring form of vitamin D in animals RILEY ONG, MIDTERM
A. Cholecalciferol and human skin. It serves as the precursor for MD (TOP 5 EXAM - FEB
B. Ergocalciferol calcitriol, the active form of vitamin D. Ergocalciferol - AUG 2014 2015
C. Sitocalciferol is the form of vitamin D obtained from vegetables. MED
D. Calcitriol BOARDS;
E. Dihydroergocalciferol TOPNOTCH
MD)

392 How much oxygen does 1 g of hemoglobin carry? SIMILAR TO PREVIOUS BOARD EXAM SCOTT BACK-UP
A. 1.00 mL CONCEPT/PRINCIPLE. RILEY ONG, MIDTERM
B. 1.25 mL MD (TOP 5 EXAM - FEB
C. 1.34 mL - AUG 2014 2015
D. 1.55 mL MED
E. 1.67 mL BOARDS;
TOPNOTCH
MD)
393 Which of the following amino acids is the precursor of SIMILAR TO PREVIOUS BOARD EXAM SCOTT BACK-UP
catecholamines such as dopamine, norepinephrine and CONCEPT/PRINCIPLE. RILEY ONG, MIDTERM
epinephrine? MD (TOP 5 EXAM - FEB
A. Asparagine - AUG 2014 2015
B. Glutamate MED
C. Methionine BOARDS;
D. Serine TOPNOTCH
E. Tyrosine MD)

TOPNOTCH MEDICAL BOARD PREP BIOCHEMISTRY SUPEREXAM Page 49 of 92


For inquiries visit www.topnotchboardprep.com.ph or email us at topnotchmedicalboardprep@gmail.com
TOPNOTCH MEDICAL BOARD PREP BIOCHEMISTRY SUPEREXAM
For inquiries visit www.topnotchboardprep.com.ph or email us at topnotchmedicalboardprep@gmail.com
Item QUESTION EXPLANATION AUTHOR TOPNOTCH
# EXAM
394 Carbamoyl phosphate synthetase I is the rate-limiting SCOTT BACK-UP
enzyme in the urea cycle. This enzyme is found in which RILEY ONG, MIDTERM
part of the cell? MD (TOP 5 EXAM - FEB
A. Cytosol - AUG 2014 2015
B. Mitochondria MED
C. Golgi body BOARDS;
D. Smooth ER TOPNOTCH
E. Peroxisome MD)

395 Fructose enters glycolysis as what intermediate Fructose is phosphorylated by fructosekinase into SCOTT BACK-UP
substance in this pathway? fructose-1-phosphate. Fructose-1-phosphate is then RILEY ONG, MIDTERM
A. Dihydroxyacetone phosphate cleaved by aldolase B to form dihydroxyacetone MD (TOP 5 EXAM - FEB
B. Fructose-6-phosphate phosphate (which can now enter glycolysis) and - AUG 2014 2015
C. Fructose-1,6-bisphosphate glyceraldehyde. Glyceraldehyde can be MED
D. Glucose phosphorylated by triokinase into glyceraldehyde-3- BOARDS;
E. 2-phosphoglycerate phosphate (which can also now enter glycolysis). TOPNOTCH
MD)
396 Which of the following substances would you expect to Homocysteine and N5-methyltetrahydrofolate SCOTT BACK-UP
accumulate when there is a deficiency of homocysteine accumulate when there is a deficiency of RILEY ONG, MIDTERM
methyltransferase, such as in cases of homocyteinuria? homocysteine methyltransferase. This enzyme is MD (TOP 5 EXAM - FEB
A. N5-methyltetrahydrofolate required to transfer a methyl group from N5- - AUG 2014 2015
B. Methionine methylTHF to homocysteine to form methionine and MED
C. N10-formyltetrahydrofolate tetrahydrofolate. BOARDS;
D. Cysteine TOPNOTCH
E. S-adenosylmethionine MD)

397 Which of the following amino acids is purely ketogenic? Purely ketogenic: Lys and Leu. Both ketogenic and SCOTT BACK-UP
A. Phenylalanine glucogenic: Phe, Tyr, Ile, Trp (mnemonic: F-Y-I-W). RILEY ONG, MIDTERM
B. Tryptophan MD (TOP 5 EXAM - FEB
C. Tyrosine - AUG 2014 2015
D. Isoleucine MED
E. Leucine BOARDS;
TOPNOTCH
MD)
398 Which of the following is not a common precursor in Precursors of purine: Asp, Gln, Gly, CO2, N10- SCOTT BACK-UP
both de novo purine and pyrimidine synthesis? formyltetrahydrofolate or N5,N10- RILEY ONG, MIDTERM
A. Aspartate methenyltetrahydrofolate. Precursors of pyrimidineL MD (TOP 5 EXAM - FEB
B. Glutamine Asp, Gln, CO2 - AUG 2014 2015
C. Glycine MED
D. Carbon dioxide BOARDS;
E. None of the above TOPNOTCH
MD)

399 Which of the following vitamins is most likely to be Intrinsic factor, which is required for vitamin B12 SCOTT BACK-UP
deficient in patients who underwent gastrectomy? absorption in the ileum, is produced by parietal cells RILEY ONG, MIDTERM
A. Vitamin C in the gastric fundus. MD (TOP 5 EXAM - FEB
B. Vitamin A - AUG 2014 2015
C. Vitamin B6 MED
D. Vitamin B12 BOARDS;
E. Vitamin E TOPNOTCH
MD)

400 Which of the following is a co-factor of glutathione SCOTT BACK-UP


peroxidase? RILEY ONG, MIDTERM
A. Molybdenum MD (TOP 5 EXAM - FEB
B. Pyridoxine - AUG 2014 2015
C. Selenium MED
D. Copper BOARDS;
E. Vitamin C TOPNOTCH
MD)
401 Which of the following substances inhibits the Electron Dimercaprol (and Antimycin A) inhibits the Ubiquinol: JOSE DIAGNOSTIC
Transport chain via the Ubiquinol: Ferricytochrome Ferricytochrome Oxidoreductase Complex simply CARLO EXAM - AUG
Oxidoreductase Complex? known as Complex III. Babiturate: Complex I; MASANGK 2014
A. Barbiturate Malonate: Complex II, Cyanide Complex IV; AY III, MD
B. Malonate Oligomycin: ATP synthase (Previous Board Ques) (TOP 8 -
C. Dimercaprol FEB 2014
D. Cyanide MED
E. Oligomycin BOARDS;
TOPNOTCH
MD)
402 Which of the following is true regarding Enzymes? JOSE DIAGNOSTIC
A. First-order kinetics is followed when the substrate CARLO EXAM - AUG
concentration is above Km MASANGK 2014
B. Temperature does not affect the reaction rate AY III, MD
C. The Km is directly proportional to the substrate (TOP 8 -
affinity FEB 2014
D. Vi is the substrate concentration at which Km is half MED
the maximal Velocity BOARDS;
E. Enzymes that follow the Michaelis-Menten kinetics TOPNOTCH
presents with a hyperbolic curve? MD)

TOPNOTCH MEDICAL BOARD PREP BIOCHEMISTRY SUPEREXAM Page 50 of 92


For inquiries visit www.topnotchboardprep.com.ph or email us at topnotchmedicalboardprep@gmail.com
TOPNOTCH MEDICAL BOARD PREP BIOCHEMISTRY SUPEREXAM
For inquiries visit www.topnotchboardprep.com.ph or email us at topnotchmedicalboardprep@gmail.com
Item QUESTION EXPLANATION AUTHOR TOPNOTCH
# EXAM
403 What Glucose transporter is present in Erythrocytes? GLUT1: Erythrocytes, brain, kidney, colon, placenta; JOSE DIAGNOSTIC
A. GLUT-1 GLUT2 Liver, pancreas, SI, kidney; GLUT-3 Brain, CARLO EXAM - AUG
B. GLUT-2 kidney, placenta; GLUT-4 Heart and skeletal muscle, MASANGK 2014
C. GLUT-3 adipose tissue; GLUT5: SI AY III, MD
D. GLUT-4 (TOP 8 -
E. GLUT-5 FEB 2014
MED
BOARDS;
TOPNOTCH
MD)
404 Utilization of Galactose by the body enters the Embden- Galactose is converted to Galactose-1-phosphate by JOSE DIAGNOSTIC
Meyerhof Pathway thru which Intermediate? galactokinase/hexokinase, Gal-1-Phosphate is then CARLO EXAM - AUG
A. Citrate converted to Glucose-1-phosphate by Hexose-1- MASANGK 2014
B. Glucose-6-Phosphate phosphate uridyltransferase and is then converted AY III, MD
C. Fructose-6-phosphate again to Glucose-6-phosphate by (TOP 8 -
D. Succinyl CoA Phosphoglucomutase, Embden Meyerhof pathway is FEB 2014
E. Fumarate the other name for Glycolysis, A,D and E are MED
intermediates of the Kreb's Cycle. BOARDS;
TOPNOTCH
MD)
405 A Gold olympic medalist for boxing has suffered an Energy intake=Energy expenditure. Sedentary:30; JOSE DIAGNOSTIC
injury and was eventually admitted, you were his Moderately active:35; Very Active:40. CARLO EXAM - AUG
admitting physician and upon ordering his diet MASANGK 2014
requirements you computed it according to his average AY III, MD
dietary energy intake which is: (TOP 8 -
A. 25 kcal/kg/day FEB 2014
B. 30 kcal/kg/day MED
C. 35 kcal/kg/day BOARDS;
D. 40 kcal/kg/day TOPNOTCH
E. 45 kcal/kg/day MD)

406 Fat provides how many percentage of the energy Carbohydrates: 45-65%; Protein 10-35%(SIMILAR TO JOSE DIAGNOSTIC
requirements in humans? PREVIOUS BOARD EXAM CONCEPT/PRINCIPLE) CARLO EXAM - AUG
A. 10-35% MASANGK 2014
B. 10-20% AY III, MD
C. 20-35% (TOP 8 -
D. 20-50% FEB 2014
E. 45-65% MED
BOARDS;
TOPNOTCH
MD)
407 Which of the following amino acids will be most likely Leucine, Isoleucine, Valine, Alanine, Glycine, JOSE DIAGNOSTIC
found in the interior of a protein? Phenylalanine, Tyrosine, Methionine and Proline are CARLO EXAM - AUG
A. Cysteine all Non-polar Amino acids, hence can be found in the MASANGK 2014
B. Leucine interior of a protein. AY III, MD
C. Isoleucine (TOP 8 -
D. B and C FEB 2014
E. All of the above MED
BOARDS;
TOPNOTCH
MD)
408 The following statements are true regarding the All are correct except B which is supposed to be JOSE DIAGNOSTIC
utilization of Pyridoxine by the human body, EXCEPT: "Coenzyme for ALA synthase" CARLO EXAM - AUG
A. Coenzyme for Glycogen phosphorylase MASANGK 2014
B. Coenzyme for ALA Reductase AY III, MD
C. Synthesis of Niacin from Tryptophan (TOP 8 -
D. Synthesis of GABA from Glutamate FEB 2014
E. Coenzyme of Cystathione synthase MED
BOARDS;
TOPNOTCH
MD)
409 A pediatric patient presented to you with noted This is a classic case of Lead poisoning. ALA JOSE DIAGNOSTIC
Basophilic stippling on RBC, Memory loss, Peripheral dehydratase together with ferrochelatase are both CARLO EXAM - AUG
Neuropathy, and lines in the gums, due to your diligent inhibited by lead. (similar to a SIMILAR TO PREVIOUS MASANGK 2014
history, you extracted from the patient's mother that the BOARD EXAM CONCEPT/PRINCIPLEion) AY III, MD
walls of their house had several "chapped" paints, which (TOP 8 -
the patient may have ingested. Which of the following FEB 2014
enzymes in heme synthesis is inhibited by the said MED
suspected ingested substance? BOARDS;
A. ALA Dehydratase TOPNOTCH
B. ALA reductase MD)
C. ALA synthase
D. Uroporphyrinogen decarboxylase
E. Uroporphyrinogen synthase

410 An 8 month-old patient presented to you with noted This is a classic case of Alkaptonuria, an Inborn error JOSE DIAGNOSTIC
black conjunctival pigment, upon examination you noted of metabolism. CARLO EXAM - AUG
that the patients diaper had black stains. What enzyme is MASANGK 2014
most likely deficient in this patient? AY III, MD
A. Dihydropteroate synthase (TOP 8 -
B. Cystathione synthase FEB 2014
C. Phenylalanine hydroxylase MED
D. Homogentisic Acid Oxidase BOARDS;
E. Tyrosine hydroxylase TOPNOTCH
MD)

TOPNOTCH MEDICAL BOARD PREP BIOCHEMISTRY SUPEREXAM Page 51 of 92


For inquiries visit www.topnotchboardprep.com.ph or email us at topnotchmedicalboardprep@gmail.com
TOPNOTCH MEDICAL BOARD PREP BIOCHEMISTRY SUPEREXAM
For inquiries visit www.topnotchboardprep.com.ph or email us at topnotchmedicalboardprep@gmail.com
Item QUESTION EXPLANATION AUTHOR TOPNOTCH
# EXAM
411 Which of the following Apolipoproteins is a cofactor for JOSE DIAGNOSTIC
Lipoprotein Lipase? CARLO EXAM - AUG
A. A-1 MASANGK 2014
B. B-100 AY III, MD
C. C-II (TOP 8 -
D. B-48 FEB 2014
E. E MED
BOARDS;
TOPNOTCH
MD)
412 A female african-american patient presented to you with This is a classic case of Sickle Cell Anemia, caused by a JOSE DIAGNOSTIC
anemia, tissue anoxia, and painful crises, Upon doing a point mutation in both genes coding for the B chain CARLO EXAM - AUG
PBS, the pathologist noted several misshapen RBCs, that results in the replacemnt of Glutamate by Valine MASANGK 2014
forming crescent shapes. Which of the following is true in the 6th position. AY III, MD
regarding the pathophysiology of this disease? (TOP 8 -
A. Lysine is substituted for Glutamate in the 6th FEB 2014
position MED
B. Lysine is substituted for Glutamate in the 8th BOARDS;
position TOPNOTCH
C. Valine is substituted for Glutamate in the 6th MD)
position
D. Valine is substituted for Glutamate in the 8th
position
E. There is a mutation in the development of
Ankyrin/Spectrin
413 When old RBCs are degraded by the spleen, Free Haptoglobin prevents loss of free hemoglobin thru the JOSE DIAGNOSTIC
hemoglobin is produced. One of the following binds kidneys, thus conserving the Iron present in CARLO EXAM - AUG
extracorpuscular hemoglobin, preventing free hemoglobin which would otherwise be lost in the MASANGK 2014
hemoglobin from entering the kidneys: body.(SIMILAR TO PREVIOUS BOARD EXAM AY III, MD
A. Ferritin CONCEPT/PRINCIPLEion) (TOP 8 -
B. Transferrin FEB 2014
C. Albumin MED
D. Haptoglobin BOARDS;
E. Hemopexin TOPNOTCH
MD)
414 Jaundice is a clinical sign when there is systemic B-Glucuronidase is an enzyme present in intestinal JOSE DIAGNOSTIC
yellowish discoloration. This is prevented by its bacteria, which converts conjugated bilirubin to a CARLO EXAM - AUG
catabolic pathway, which of the following enzymes colorless substance called Urobilinogen and is then MASANGK 2014
convert bilirubin into a colorless compound? converted to Stercobilin (Feces) or Urobilin (Urine) AY III, MD
A. Urobilinogen reductase (SIMILAR TO PREVIOUS BOARD EXAM (TOP 8 -
B. B-Glucuronidase CONCEPT/PRINCIPLEion) FEB 2014
C. UDP-Glucose Dehydrogenase MED
D. Biliverdin reductase BOARDS;
E. Cytochrome P450 TOPNOTCH
MD)
415 This is a branch of medical science wherein it involves SIMILAR TO PREVIOUS BOARD EXAM JOSE DIAGNOSTIC
the design of computer algorithms and construction of CONCEPT/PRINCIPLE CARLO EXAM - AUG
databases that enable biomedical scientists to access and MASANGK 2014
analyze the growing avalanche of biomedical data AY III, MD
A. Genomics (TOP 8 -
B. Genetics FEB 2014
C. Bioinformatics MED
D. Computational Biology BOARDS;
E. Human Genome Project TOPNOTCH
MD)
416 Eicosanoids are important groups of physiologically and Linoleic Acid is an essential fatty acid which is the JOSE DIAGNOSTIC
pharmacologically active compounds, including precursor of a membrane phospholipid, Arachidonic CARLO EXAM - AUG
Prostaglandins, thromboxanes, leukotrienes and Acid which is the main component for the synthesis of MASANGK 2014
lipoxins. Which of the following Fatty acids can Eicosanoids. AY III, MD
Eicosanoids be derived from? (TOP 8 -
A. Linoleic Acid FEB 2014
B. Palmitic Acid MED
C. Linolenic Acid BOARDS;
D. Oleic Acid TOPNOTCH
E. Stearic Acid MD)
417 Sickle Cell Anemia is a mutation were an amino acid in a Silent-New codon, same amino acid; Nonsense-Stop JOSE DIAGNOSTIC
sequence has been replaced by a different amino acid, codon; Transition-Purine-Purine change; CARLO EXAM - AUG
What type of mutation is indicated in this case? Transversion-Purine-Pyrimidine Change(PSIMILAR MASANGK 2014
A. Silent TO PREVIOUS BOARD EXAM CONCEPT/PRINCIPLE) AY III, MD
B. Missense (TOP 8 -
C. Nonsense FEB 2014
D. Transition MED
E. Transversion BOARDS;
TOPNOTCH
MD)
418 This rule indicates that the total amount of purines In any sample of dsDNA, the amount of Adenine JOSE DIAGNOSTIC
equals the total amount of pyrimidines in a sequence: equals to the amount of Thymine, the amount of CARLO EXAM - AUG
A. Shine-Dalgarno's Rule Guanine equals to the amount of Cytosine. MASANGK 2014
B. Chargaff's Rule AY III, MD
C. Pribnow's Rule (TOP 8 -
D. Hogness' Rule FEB 2014
E. Sanger's Rule MED
BOARDS;
TOPNOTCH
MD)
TOPNOTCH MEDICAL BOARD PREP BIOCHEMISTRY SUPEREXAM Page 52 of 92
For inquiries visit www.topnotchboardprep.com.ph or email us at topnotchmedicalboardprep@gmail.com
TOPNOTCH MEDICAL BOARD PREP BIOCHEMISTRY SUPEREXAM
For inquiries visit www.topnotchboardprep.com.ph or email us at topnotchmedicalboardprep@gmail.com
Item QUESTION EXPLANATION AUTHOR TOPNOTCH
# EXAM
419 Tetracycline is an antibiotic which inhibits protein JOSE DIAGNOSTIC
synthesis in prokaryotes, specifically, on what aspect of CARLO EXAM - AUG
protein synthesis does it act? MASANGK 2014
A. Inhibits prokaryotic peptidyltransferase AY III, MD
B. Binds to the 50s subunit and distorts its structure (TOP 8 -
C. Binds to the B-subunit of bacterial DNA-dependent FEB 2014
RNA polymerase MED
D. Prevents binding of aminoacyl tRNAs to the A site BOARDS;
E. Inactivates Elongation Factor-2 TOPNOTCH
MD)
420 You and your friends went to Vikings for an eat-all-you- Increased Epinephrine levels are seen in the fasting JOSE DIAGNOSTIC
can buffet, during your fed state the following state, which promotes Glycogenolysis to maintain CARLO EXAM - AUG
biochemical reactions are currently occuring in your normal blood glucose levels MASANGK 2014
body except: AY III, MD
A. Increased Protein synthesis by the muscles (TOP 8 -
B. Increased Triglyceride synthesis and storage FEB 2014
C. Replenished its glycogen stores MED
D. Decreased Glucagon:Insulin ratio BOARDS;
E. Increased Epinephrine levels TOPNOTCH
MD)
421 To maintain normal protein and energy balance in the Leucine and lysine are the only ketogenic amino acids. WEBSTER MIDTERM 1
body, the carbon skeletons of amino acids are They can be degraded directly to become acetyl CoA ALINDOG, EXAM - AUG
catabolized yielding different intermediates including by ketogenesis (acetoacetate). They cannot be MD (TOP 3 2014
ketone bodies. Which of the following amino acids is converted to glucose since the carbon atoms in the - FEB 2014
exclusively ketogenic? ketones are ultimately degraded to carbon dioxide. MED
A. Phenylalanine Phenylalanine and isoleucine, as well as tyrosine and BOARDS;
B. Leucine tryptophan are both ketogenic and glucogenic; while TOPNOTCH
C. Isoleucine methionine and the rest of the amino acids are MD)
D. Methionine glucogenic.
E. All of the above

422 Glucose-6-phosphate dehydrogenase deficiency is WEBSTER MIDTERM 1


characterized by hemolytic episodes secondary to poor ALINDOG, EXAM - AUG
defense against oxidative stress. The most common MD (TOP 3 2014
precipitating factor to hemolysis among these patients - FEB 2014
is: MED
A. Drugs BOARDS;
B. Diet (fava beans) TOPNOTCH
C. Infection MD)
D. Spontaneous, undetermined

423 A 9-month old female was observed to have excessive The first clue here is the involvement of a REDUCING WEBSTER MIDTERM 1
REDUCING sugars in the urine. She was also noted to be sugar, and fructose should be our main consideration. ALINDOG, EXAM - AUG
jaundiced, hypoglycemic, acidotic, and with elevated There are only 2 metabolic disorders involving MD (TOP 3 2014
AST. There was significant improvement upon removal fructose that we should know - essential fructosuria - FEB 2014
of the sugar from her diet. The patient is most likely (fructokinase deficiency) which is essentially benign MED
suffering from deficiency of: and asymptomatic, and hereditary fructose BOARDS;
A. Aldolase B intolerance in which aldolase B is deficient. Absence of TOPNOTCH
B. Fructokinase aldolase B leads to intracellular trapping of fructose 1- MD)
C. Glucokinase phosphate which results in vomiting, severe
D. Galactose-1-phosphate uridyltransferase A hypoglycemia, jaundice, hemorrhage and finally,
hepatic failure. Removal of dietary fructose on the
other hand serves as the main therapy.
424 Peptide bonds covalently join amino acids to form The partial double-bond feature of peptide bond WEBSTER MIDTERM 1
proteins. They are amide linkages between the alpha- makes the structure of protein rigid and planar at the ALINDOG, EXAM - AUG
carboxyl group of 1 amino acid and the alpha-amino same time, just enough to allow assumption of certain MD (TOP 3 2014
group of another. Which of the following statements configurations. Prolonged exposure to strong acid or - FEB 2014
about peptide bonds is true? base at elevated temperatures (at 110oC for 24 hours) MED
A. Rapid exposure to strong acids is required to is required to hydrolyze these bonds non- BOARDS;
hydrolyze peptide bonds non-enzymatically. enzymatically. Urea cannot break down these bonds TOPNOTCH
B. They can be broken by exposure to high whereas trypsin can cleave them only at the carbonyl MD)
concentrations of urea. side of either arginine or lysine.
C. Trypsin can cleave peptide bonds on the amino end
of proteins.
D. They possess a partial double-bond character.
E. All of these are true.

425 Deficiency of which of the following vitamins Vitamin B6 (pyridoxine phosphate) is a co-factor in WEBSTER MIDTERM 1
predisposes a patient to anemia? the formation of delta-aminolevulinic acid, the rate- ALINDOG, EXAM - AUG
A. Iron limiting step in heme synthesis; and is catalyzed by MD (TOP 3 2014
B. Vitamin B2 ALA synthase. This is also the reason why isoniazid - FEB 2014
C. Vitamin B6 therapy in which pyridoxine deficiency may be MED
D. Vitamin D common, can result in anemia. Iron, although essential BOARDS;
in heme synthesis, is not considered a vitamin but a TOPNOTCH
mineral. (Always remember the rule, CHOOSE THE MD)
BEST ANSWER). Vitamin is organic, mineral is not.

TOPNOTCH MEDICAL BOARD PREP BIOCHEMISTRY SUPEREXAM Page 53 of 92


For inquiries visit www.topnotchboardprep.com.ph or email us at topnotchmedicalboardprep@gmail.com
TOPNOTCH MEDICAL BOARD PREP BIOCHEMISTRY SUPEREXAM
For inquiries visit www.topnotchboardprep.com.ph or email us at topnotchmedicalboardprep@gmail.com
Item QUESTION EXPLANATION AUTHOR TOPNOTCH
# EXAM
426 A 24-year old asymptomatic patient was noted to have It may not be your typical biochem question but you WEBSTER MIDTERM 1
decreased hemoglobin and hematocrit. Her RBC indices should know that anything can be asked in any ALINDOG, EXAM - AUG
were MCHC 26%, MCV 74 fL - hypochromic, microcytic. subject, and biochem is no exception. Our main hint MD (TOP 3 2014
At this point, your most likely consideration for the here is the RBC indices indicating a hypochromatic, - FEB 2014
cause of anemia would be: microcytic type of anemia. You should only remember MED
A. Iron deficiency 5 anemias exhibiting this picture and the mnemonic is BOARDS;
B. Folate deficiency CLITS - anemia of Chronic disease, Lead poisoning, TOPNOTCH
C. Spherocytosis Iron deficiency anemia, Thalassemia and Sideroblastic MD)
D. Acute blood loss from menstruation anemia. For megaloblastic (macrocytic) anemias, just
remember folate and vitamin B12 deficiency, and
pernicious anemia. Acute blood loss and spherocytosis
is often times normocytic and normochromic. Keeping
this in mind, you'll get a very good chance in
answering questions not only in biochem, but more
likely in physio, patho, IM, and pedia.
427 Bilirubin is conjugated in the liver mainly for the Keep yourself from choosing "all of the above" that WEBSTER MIDTERM 1
purpose of: easily. You have to really read through and analyze. ALINDOG, EXAM - AUG
A. Increasing the levels of delta bilirubin which is the The goal of bilirubin conjugation is to make it more MD (TOP 3 2014
less toxic form. soluble in water (plasma) so that it can be easily - FEB 2014
B. Making it more water-soluble for easier excretion. excreted (direct bili/B2). Delta bilirubin is bilirubin MED
C. Inhibiting its binding with albumin to reduce risk of covalently bound to albumin, which is also non-toxic BOARDS;
jaundice. but cannot be excreted; it happens in prolonged TOPNOTCH
D. Keeping its value less than 0.1 mg/dL in the blood. periods of hyperbilirubinemia. Indirect bilirubin MD)
E. All of these. naturally binds with albumin for transport to the liver
for conjugation. Bilirubin levels must be below 2.5
mg/dl to prevent development of jaundice.
428 What is the default product of fatty acid synthesis? Be thrilled with quick and simple questions like this WEBSTER MIDTERM 1
A. Linoleic acid =). ALINDOG, EXAM - AUG
B. Arachidonic acid MD (TOP 3 2014
C. Formic acid - FEB 2014
D. Palmitic acid MED
BOARDS;
TOPNOTCH
MD)
429 In the intestine, primary bile acids are dehydroxylated to Just remember, cholic acid and chenodeoxycholic acid WEBSTER MIDTERM 1
form secondary bile acids. Which of the following is a are primary bile acids, whereas deoxycholic and ALINDOG, EXAM - AUG
secondary bile acid? lithocholic acids are secondary bile acids. When MD (TOP 3 2014
A. Cholic acid primary bile acids are conjugated with either taurine - FEB 2014
B. Cholic acid + taurine or glycine, they become bile salts. Secondary bile acids MED
C. Chenodeoxycholic acid are formed when bacteria act upon (deconjugate and BOARDS;
D. Lithocholic acid dehydroxylate) the bile salts. TOPNOTCH
MD)

430 Which secondary messenger system is utilized by the Must remember at least 2 examples for each WEBSTER MIDTERM 1
hormone glucagon? secondary messenger system: glucagon and ALINDOG, EXAM - AUG
A. Tyrosine kinase epinephrine (β and α2) for cAMP pathway; ANP and MD (TOP 3 2014
B. cGMP-guanylate cyclase NO for cGMP; insulin and growth factors for tyrosine - FEB 2014
C. None, it binds intracellularly kinase; epinephrine α1 for IP3/DAG; and MED
D. cAMP-adenylate cyclase glucocorticoids and thyroid hormone for the BOARDS;
intracellular pathway. TOPNOTCH
MD)
431 The rate-limiting step in the Krebs cycle involves the More often than not, you'll get clues from the question. WEBSTER MIDTERM 1
conversion of isocitrate to alpha-ketoglutarate. The Even without studying Krebs, the answer is already ALINDOG, EXAM - AUG
enzyme responsible for this step is: obvious from the question itself. MEMORIZE all the MD (TOP 3 2014
A. Isocitrate dehydrogenase rate-limiting enzymes and steps. - FEB 2014
B. Alpha-ketoglutarate dehydrogenase MED
C. Aconitase BOARDS;
D. Succinate thiokinase TOPNOTCH
MD)

432 The last step in the enterohepatic circulation involves: The answer lies in the main purpose of the WEBSTER MIDTERM 1
A. Removal of taurine/glycine from the bile salt enterohepatic circuit and that is to reabsorb majority ALINDOG, EXAM - AUG
compound. of bile acids and recycle them in the liver while also MD (TOP 3 2014
B. Reabsorption of about 95-99% of primary and serving as the mean for excreting body cholesterol. - FEB 2014
secondary bile acids in the ileum with excretion of some MED
unesterified cholesterol. BOARDS;
C. Dehydroxylating action of intestinal bacteria on TOPNOTCH
primary bile acids with their conversion to secondary MD)
bile acids in equal molar amounts.
D. The 7-alpha-hydroxylation of cholesterol as
catalyzed by cholesterol 7-alpha hydroxylase.
E. None of these.
433 What is the amino acid precursor of glutathione and Must know the other important amino acid WEBSTER MIDTERM 1
GABA? precursors: PHENYLALANINE --- tyrosine (another ALINDOG, EXAM - AUG
A. Glutamate amino acid), L-dopa, dopamine, norepinephrine, MD (TOP 3 2014
B. Glutamine epinephrine; TYROSINE --- thyroxine, melanin; - FEB 2014
C. Methionine TRYPTOPHAN --- niacin, serotonin, melatonin; MED
D. Glycine METHIONINE --- homocysteine. (high yield) BOARDS;
TOPNOTCH
MD)

TOPNOTCH MEDICAL BOARD PREP BIOCHEMISTRY SUPEREXAM Page 54 of 92


For inquiries visit www.topnotchboardprep.com.ph or email us at topnotchmedicalboardprep@gmail.com
TOPNOTCH MEDICAL BOARD PREP BIOCHEMISTRY SUPEREXAM
For inquiries visit www.topnotchboardprep.com.ph or email us at topnotchmedicalboardprep@gmail.com
Item QUESTION EXPLANATION AUTHOR TOPNOTCH
# EXAM
434 The enzyme responsible to replace the stretches of WEBSTER MIDTERM 1
highly repititive DNA found at the ends of linear ALINDOG, EXAM - AUG
chromosomes in cells in order to avoid aging as seen in MD (TOP 3 2014
cancer and germline cells is: - FEB 2014
A. Elongase MED
B. Transcriptase BOARDS;
C. Telomerase TOPNOTCH
D. Polymerase MD)

435 This is a disease condition resulting from formation of WEBSTER MIDTERM 1


thymine dimers secondary to exposure to UV without its ALINDOG, EXAM - AUG
normal removal by UV-specific endonuclease: MD (TOP 3 2014
A. Epidermolysis bullosa - FEB 2014
B. HNPCC MED
C. Xeroderma pigmentosum BOARDS;
D. FAP TOPNOTCH
MD)

436 What constitute the central dogma? WEBSTER MIDTERM 1


A. Replication, DNA synthesis, proofreading ALINDOG, EXAM - AUG
B. Replication, transcription, translation MD (TOP 3 2014
C. Transcription, translation, modification - FEB 2014
D. Translation, modification, labeling and secretion MED
E. All of these may constitute the central dogma. BOARDS;
TOPNOTCH
MD)
437 The type of mutation which results in a new codon Frameshift means there is deletion or addition of a WEBSTER MIDTERM 1
specifying a different amino acid which may or may not base which results in protein products that are ALINDOG, EXAM - AUG
cause variation in protein function is known as: shorter than normal and hence usually non-functional. MD (TOP 3 2014
A. Frameshift Non-sense, on the other hand, refers to formation of a - FEB 2014
B. Nonsense new stop codon making the protein shorter and also MED
C. Missense non-functional. A silent mutation involves a new BOARDS;
D. Silent codon specifying the same amino acid and therefore TOPNOTCH
has no effect in the protein function. MD)

438 Cystinuria is an inherited condition characterized by Cystinuria is the most common genetic error of amino WEBSTER MIDTERM 1
formation of kidney stones brought about by defects in acid transport. It has defective kidney tubule ALINDOG, EXAM - AUG
the renal tubular amino acid transporter for: reabsorption of the amino acids cysteine, ornithine, MD (TOP 3 2014
A. Ornithine lysine and arginine (mnemonic: COLA). There is - FEB 2014
B. Arginine precipitation of cystine to form kidney stones which MED
C. Cystine can cause urinary tract obstruction. Under the BOARDS;
D. All of these microscope, you can appreciate cystine as hexagonal- TOPNOTCH
E. None of these shaped crystals in the urine, looking much like your MD)
favorite "Piattos" chips.
439 In the regular human diet, the estimated energy This topic is must know. For carbohydrate, it is 45- WEBSTER MIDTERM 1
requirement coming from fat is set at: 65% and for protein 10-35%. The expected total ALINDOG, EXAM - AUG
A. 10-25% energy required per day for a sedentary person is set MD (TOP 3 2014
B. 20-35% at 30 kcal/kg/day; for the moderately active and in- - FEB 2014
C. 45-65% patients, 35 kcal/kg/day; and for the very active, 40 MED
D. >60% kcal/kg/day. BOARDS;
TOPNOTCH
MD)

440 Hemoglobin is different from myoglobin by: Myoglobin on the other hand, has only 1 polypetide, it WEBSTER MIDTERM 1
A. Having oxygen dissociation curve that is sigmoidal in has a hyperbolic-shaped oxygen dissociation curve ALINDOG, EXAM - AUG
shape and functions mainly as storage for oxygen both in MD (TOP 3 2014
B. Possessing 4 polypeptides in the form of globins heart and skeletal muscles. Myoglobin is the - FEB 2014
C. Exhibiting allosteric effects EARLIEST detectable cardiac element found in plasma MED
D. Being utilized for oxygen transport rather than during MI. BOARDS;
storage TOPNOTCH
E. All of these MD)
441 Electrons are passed down the transport chain to create Electron Transport Chain Inhibitors: Complex I- JULIET MIDTERM 2
proton gradient which results to ATP synthesis. Several Barbiturate, Piericidin A, Amytal, Rotenone, Complex KRISTINE EXAM - AUG
inhibitors stop electron flow from substrate to oxygen. II-Malonate, Carboxin, TTFA, Complex III-Antimycin A, EVANGELIS 2014
Inhibitor/s of Cytochrome oxidase include/s: Dimercaprol, Complex IV-Cyanide, Carbon monoxide, TA, MD
A. Cyanide sodium azide, hydrogen sulfide (TOP 9 -
B. Malonate FEB 2014
C. Hydrogen sulfide MED
D. A and B BOARDS;
E. A and C TOPNOTCH
MD)
442 A newborn was brought to a clinic by her mother due to Classic Galactosemia occurs in the absence of JULIET MIDTERM 2
vomiting every after feeding. PE revealed that the galactose 1-phosphate uridyltransferase resulting to KRISTINE EXAM - AUG
newborn had severe jaundice and enlarged liver. NB accumulation of galactitol. The manifestations are EVANGELIS 2014
screening result was positive for galactosemia and vomiting and diarrhea after milk ingestion, TA, MD
lactose-containing substances are removed from the hypoglycemia, liver disease and cirrhosis, lethargy (TOP 9 -
diet. In this case, lactose is toxic to the NB because: and hypotonia and mental retardation. FEB 2014
A. Excess galactose shuts glucose metabolism MED
B. Galactose is toxic even in little amounts BOARDS;
C. Galactose is converted to galactitol TOPNOTCH
D. Excess glucose accumulated in the blood MD)
E. Excess galactose is converted to sorbitol

TOPNOTCH MEDICAL BOARD PREP BIOCHEMISTRY SUPEREXAM Page 55 of 92


For inquiries visit www.topnotchboardprep.com.ph or email us at topnotchmedicalboardprep@gmail.com
TOPNOTCH MEDICAL BOARD PREP BIOCHEMISTRY SUPEREXAM
For inquiries visit www.topnotchboardprep.com.ph or email us at topnotchmedicalboardprep@gmail.com
Item QUESTION EXPLANATION AUTHOR TOPNOTCH
# EXAM
443 All are true of the pentose phosphate pathway, EXCEPT:
Pentose phosphate pathway occurs in the cytoplasm JULIET MIDTERM 2
A. It provides the route for metabolism of 5-carbon of RBCs, liver, adipose tissue, adrenals, thyroid, testes, KRISTINE EXAM - AUG
sugars lactating mammaries and tissues that are producing EVANGELIS 2014
B. It is the source of ribose 5-phosphate for nucleotide lipids. TA, MD
synthesis (TOP 9 -
C. It is only functional in red blood cells FEB 2014
D. It is the source of reduced NADP MED
E. It is active in tissues that produce lipids BOARDS;
TOPNOTCH
MD)
444 A non-competitive inhibitor of an enzyme:
Non-competitive inhibitor results to decreased Vmax JULIET MIDTERM 2
A. Increases Km and decreases affinity but it does not affect KM and affinity. KRISTINE EXAM - AUG
B. Decreases Km and increases affinity EVANGELIS 2014
C. Increases Vmax TA, MD
D. Decreases Vmax (TOP 9 -
E. None of the above FEB 2014
MED
BOARDS;
TOPNOTCH
MD)
445 In prokaryotic RNA synthesis, the function of the "rho" rho factor is required for termination of transcription JULIET MIDTERM 2
factor: A. of genes. KRISTINE EXAM - AUG
It initiates the binding of RNA polymerase EVANGELIS 2014
B. It participates in proper termination of transciption TA, MD
C. It binds repressor to the promoter region (TOP 9 -
D. It increases the rate of RNA synthesis FEB 2014
E. It eliminates binding of RNA polymerase to promoter MED
region BOARDS;
TOPNOTCH
MD)
446 In contrast to RNA polymerase, DNA polymerase:
DNA polymerase synthesizes DNA in both strands, the JULIET MIDTERM 2
A. Synthesizes RNA primer to initiate DNA synthesis leading and lagging strands, catlyzes chain elongation, KRISTINE EXAM - AUG
B. It fills in the gap between Okazaki fragments proofreads the newly synthesized DNA, read template EVANGELIS 2014
C. Synthesizes in a 3' to 5' direction in 3' to 5' direction and synthesize in 5' to 3' direction. TA, MD
D. Does not edit as it synthesizes Primase not DNA polymerase synthesizes short (TOP 9 -
E. Does not require a primer stretches of RNA called primers. FEB 2014
MED
BOARDS;
TOPNOTCH
MD)
447 A 1 year-old boy was seen at the ER with severe Pompe Disease or Type II Gylcogen storage disease is JULIET MIDTERM 2
difficulty of breathing. His blood sugar was normal. caused by deficiency in acid maltase or lysosomal α1- KRISTINE EXAM - AUG
Glycogen structure was normal although excessive 4 and α1:6 glucosidase. Manifestations include muscle EVANGELIS 2014
concentrations in abnormal vacuoles in the lysosomes is hypotonia, death from heart failure by age 2 TA, MD
found. He had also massive cardiomegaly and he died (TOP 9 -
from heart failure. The most probable enzyme deficient FEB 2014
in the patient leading to diagnosis is: MED
A. glucose-6-phosphatase BOARDS;
B. debranching enzyme TOPNOTCH
C. branching enzyme MD)
D. acid maltase
E. glycogen phosphorylase

448 The oxidation of pyruvate to acetyl coA by pyruvate Conversion of pyruvate to acetyl CoA is through the JULIET MIDTERM 2
dehydrogenase is dependent on this vitamin co-factor: pyruvate dehydrogenase complex which requires the KRISTINE EXAM - AUG
A. Biotin presence of 5 co-enzymes including: thiamine, FAD, EVANGELIS 2014
B. Niacin NAD, Pantothenic acid and lipoic acid. TA, MD
C. Thiamine (TOP 9 -
D. Vitamin C FEB 2014
E. Vitamin A MED
BOARDS;
TOPNOTCH
MD)
449 Which RNA viral genome encodes a precursor protein gag gene encodes a precursor protein that is cleaved JULIET MIDTERM 2
that is cleaved to form a protein core: to form a protein core, tat gene is for activation of KRISTINE EXAM - AUG
A. tat transcription of viral genes, pol encodes for enzymes EVANGELIS 2014
B. gag reverse transcriptase, protease intregrase and TA, MD
C. pol ribonuclease, env codes for envelope glycoproteins, (TOP 9 -
D. rev rev is for transport of late mRNAs to cytoplasm FEB 2014
E. env MED
BOARDS;
TOPNOTCH
MD)
450 Key enzyme in amino acid metabolism which is Through glutamate dehydrogenase, glutamate is JULIET MIDTERM 2
responsible for release of nitrogen as free ammonia, for oxidatively deaminated to liberate free ammonia. KRISTINE EXAM - AUG
subsequent detoxification in the urea cycle: EVANGELIS 2014
A. Glutamate dehydrogenase TA, MD
B. Pyruvate dehydrogenase (TOP 9 -
C. Alpha-ketoglutarate dehydrogenase FEB 2014
D. Succinate dehydrogenase MED
E. Glutaminase BOARDS;
TOPNOTCH
MD)

TOPNOTCH MEDICAL BOARD PREP BIOCHEMISTRY SUPEREXAM Page 56 of 92


For inquiries visit www.topnotchboardprep.com.ph or email us at topnotchmedicalboardprep@gmail.com
TOPNOTCH MEDICAL BOARD PREP BIOCHEMISTRY SUPEREXAM
For inquiries visit www.topnotchboardprep.com.ph or email us at topnotchmedicalboardprep@gmail.com
Item QUESTION EXPLANATION AUTHOR TOPNOTCH
# EXAM
451 All of these enzymes act on protein substrates in the polypeptides and amino acids are digested further in JULIET MIDTERM 2
small intestines the small intestines by elastase, carboxypeptidase, KRISTINE EXAM - AUG
A. Trypsin, chymotrypsin, elastase trypsin, chymotrypsin; pepsin is in the stomach, EVANGELIS 2014
B. Pepsin, trypsin, chymotrypsin amylase is for digestion of carbohydrates TA, MD
C. Amylase, pepsin, trypsin (TOP 9 -
D. Elastase, pepsin, amylase FEB 2014
E. Pepsin, chymotrypsin, elastase MED
BOARDS;
TOPNOTCH
MD)
452 A defect in peroxisomal activation of very long-chain X-linked adrenaleukodystrophy occurs with defect in JULIET MIDTERM 2
fatty acid which leads to its accumulation in the blood peroxisomal activation of VLCFA leading to its KRISTINE EXAM - AUG
and tissues: accumulation in the blood and tissues. Initial EVANGELIS 2014
A. Zellweger syndrome abnormalities are apathy and behavioral change. Late TA, MD
B. X-linked adrenaleukodystrophy findings include visual loss, spasticity, ataxia and (TOP 9 -
C. Refsum's Disease death a few years after onset of neurologic symptoms. FEB 2014
D. Jamaican vomiting sickness Zellweger syndrome is a rare inheroted absence of MED
E. Fatty liver peroxisomes in tissues. Refsum's disease is a defect BOARDS;
which results in accumulation of phytanic aid. TOPNOTCH
Jamaican vomiting sickness is caused by eating unripe MD)
fruit of akee tree which contains hypoglycin. Fatty
liver is due to alcoholism which leads to fat
accumulation in the liver.
453 It is the average daily dietary intake level that is Recommended Daily Allowance (RDA) is the average JULIET MIDTERM 2
sufficient to meet the requirements of >95% of all daily dietary intake level that is sufficient to meet the KRISTINE EXAM - AUG
individuals in a life stage and gender group: requirements of >95% of all individuals in a life stage EVANGELIS 2014
A. Dietary requirement and gender group. TA, MD
B. Total caloric requirement (TOP 9 -
C. Recommended daily allowance FEB 2014
D. Total energy requirement MED
E. Estimated average requirement BOARDS;
TOPNOTCH
MD)
454 In an adult, the energy requirement for protein is: Energy requirements in Humans: Fat=20 to 35%, JULIET MIDTERM 2
A. 10 to 35% Carbohydrate=45 to 65%, Protein=10 to 35% KRISTINE EXAM - AUG
B. 15 to 30% EVANGELIS 2014
C. 25 to 30% TA, MD
D. 20 to 35% (TOP 9 -
E. 45 to 65% FEB 2014
MED
BOARDS;
TOPNOTCH
MD)
455 In order to oxidize fatty acids through B-oxidation in the Carnithine shuttle transports fatty acids from JULIET MIDTERM 2
mitochondria, fatty acids must be ferried through the cytoplasm to mitochondria which uses 2 ATPs in Beta KRISTINE EXAM - AUG
mitochondrial membrane by: exidation. EVANGELIS 2014
A. Acetyl CoA TA, MD
B. Thiokinase (TOP 9 -
C. Thiolases FEB 2014
D. Carnithine MED
E. Citrate BOARDS;
TOPNOTCH
MD)
456 The common biochemical pathology seen in Creutzfeld- Creutzfeld-Jakob disease is a fatal JULIET MIDTERM 2
Jakob disease and Alzheimer's Disease: neurodegenerative disease caused by an altered KRISTINE EXAM - AUG
A. Improper folding of affected proteins due to absence version of normal protein to a pathologic EVANGELIS 2014
of chaperones conformation which is rich in B-sheets. Alzheimer TA, MD
B. Mutated apolipoprotein E is the primary cause disease is progressive cognitive and behavioral (TOP 9 -
C. Aggregation of infected proteins forming impairment due to accumulation of amyloid plaques FEB 2014
neurofibrillary tangles and neurofibrillary tangles. Similar to CJB, alzheimer's MED
D. Secondary helical structure of affected proteins are disease also is rich in B-sheet prone to self- BOARDS;
transformed to B sheet aggregation. Harper's, 27th Ed. p38. TOPNOTCH
E. Fatal neurodegenerative disease due to MD)
accumulation of amyloid plaques
457 This inborn error of metabolism is manifested by Homocystinuria is an inborn error of metabolism JULIET MIDTERM 2
convulsive seizures, lens subluxation, damage of the which is manifested by convulsive seizures, lens KRISTINE EXAM - AUG
arterial walls, myocardial infacrtion and skeletal subluxation, damage of the arterial walls, myocardial EVANGELIS 2014
deformities: infacrtion and skeletal deformities; I-cell Disease is TA, MD
A. I-cell Disease due to accumulation of partially degraded (TOP 9 -
B. Marfan syndrome glycoproteins in lysosomes manifested as severe FEB 2014
C. Homocystinuria mental retardation, coarse facial features, Marfan MED
D. Cystinuria syndrome is due to defect in the gene encoding for BOARDS;
E. Porphyrias fibrillin resulting to lens subluxation, aortic dilatation, TOPNOTCH
archnodactyly, Cystinuria is due to inherited defect or MD)
renal tubular amino acid transporter leading to
formation of staghorn calculi, porphyrias is due to
defect in heme synthesis resulting to photosensitivity
and overt blistering after sun exposure.
458 The rate-limiting step of fatty acid oxidation is: Transport of fatty acyl CoA to mitochondria is the JULIET MIDTERM 2
A. Activation of fatty acid rate-limiting step in B-oxidation which is KRISTINE EXAM - AUG
B. Transport of fatty acyl CoA to mitochondria catalyzed by carnitine acyltransferase. Activation EVANGELIS 2014
C. Formation of malonyl CoA of fatty acids is the only step in the complete TA, MD
D. Provision of fatty acid degradation of a fatty acid that requires energy from (TOP 9 -
E. None of the above ATP. Harper's, 27th Ed. p187. FEB 2014
MED
BOARDS;
TOPNOTCH MEDICAL BOARD PREP BIOCHEMISTRY SUPEREXAM Page 57 of 92
For inquiries visit www.topnotchboardprep.com.ph or email us at topnotchmedicalboardprep@gmail.com
TOPNOTCH MEDICAL BOARD PREP BIOCHEMISTRY SUPEREXAM
For inquiries visit www.topnotchboardprep.com.ph or email us at topnotchmedicalboardprep@gmail.com
Item QUESTION EXPLANATION AUTHOR TOPNOTCH
# EXAM
TOPNOTCH
MD)

459 Deficiency of the activities of DNA repair system leading Nucleotide excision repair is used to replace regions JULIET MIDTERM 2
to sunlight sensitivity and susceptibility in persons at of damaged DNA caused by UV light inducing the KRISTINE EXAM - AUG
risk for Xeroderma pigmentosum is in the: formation of thymine dimers. A special excision EVANGELIS 2014
A. Base excision repair system nuclease cuts the DNA upstream and downstream of TA, MD
B. Nucleotide excision repair system the defective region. The gap is then filled by a (TOP 9 -
C. Double-strand break system polymerase. Harper's p. 344-345 FEB 2014
D. Mismatch repair system MED
E. Multifunctional repair system BOARDS;
TOPNOTCH
MD)
460 A 6 year-old girl was brought to the OPD due to skin Epidermolysis bullosa is characterized by skin breaks JULIET MIDTERM 2
breaks and blisters which noted to occur during minor and blisters as a result of minor trauma with KRISTINE EXAM - AUG
scratching. This is a dystrophic form which is due to mutations affecting the structure of Type VII collagen. EVANGELIS 2014
mutations affecting the structure of collagen type: Type I collagen is affected in osteogenesis imperfecta, TA, MD
A. Type I Type III collagen in Ehlers-Danlos syndrome and Type (TOP 9 -
B. Type III IV collagen structure is affected in Alport syndrome. FEB 2014
C. Type IV MED
D. Type V BOARDS;
E. Type VII TOPNOTCH
MD)
461 Which of the following is a known inhibitor of complex I Amytal is a complex I inhibitor, while others are LUISA BACK-UP
of the electron transport chain? inhibitors of the following: malonate - complex II; SARANILL MIDTERM
A. amytal Antimycin A - complex III; cyanide - complex IV O, MD (TOP EXAM AUG
B. malonate 6 - FEB 2014 - FOR
C. Antimycin A 2014 MED INCLUSION IN
D. cyanide BOARDS; THE SAMPLEX
E. None of the choices TOPNOTCH
MD)

462 The Krebs or Citric acid cycle is the final common krebs or citric acid cycle is the final common pathway LUISA BACK-UP
pathway of the following except: of carbohydrate, protein, and lipid SARANILL MIDTERM
A. carbohydrate O, MD (TOP EXAM AUG
B. protein 6 - FEB 2014 - FOR
C. lipid 2014 MED INCLUSION IN
D. A and B BOARDS; THE SAMPLEX
E. None of the choices TOPNOTCH
MD)

463 What is the rate-limiting enzyme in cholesterol The following are the rate limiting enzymes for: HMG LUISA BACK-UP
synthesis? CoA synthase = ketogenesis; HMG CoA reductase = SARANILL MIDTERM
A. HMG CoA synthase cholesterol synthesis; acetyl CoA carboxylase = fatty O, MD (TOP EXAM AUG
B. HMG CoA reductase acid synthesis; carnitine acyltransferase = fatty acid 6 - FEB 2014 - FOR
C. Acetyl CoA carboxylase oxidation; cholesterol - 7 - α - hydoxylase = bile acid 2014 MED INCLUSION IN
D. Carnitine acyltransferase synthesis BOARDS; THE SAMPLEX
E. cholesterol - 7 - α - hydroxylase TOPNOTCH
MD)
464 Bile acids are synthesized from cholesterol. Which of the chenodeoxycholic acid and cholic acid are the primary LUISA BACK-UP
following is an example of primary bile acids? bile acids, while lithocholic and deoxycholic acid are SARANILL MIDTERM
A. Lithocholic acid the secondary bile acids O, MD (TOP EXAM AUG
B. Chenodeoxycholic acid 6 - FEB 2014 - FOR
C. Deoxycholic acid 2014 MED INCLUSION IN
D. All of the choices BOARDS; THE SAMPLEX
E. None of the choices TOPNOTCH
MD)

465 What is the "mother hormone" of all steroid hormones? cholesterol is the precursor of all steroid hormones LUISA BACK-UP
A. pregnenolone while pregnenolone is the "mother hormone" of all SARANILL MIDTERM
B. cholesterol steroid hormones. Other choices are derivatives of O, MD (TOP EXAM AUG
C. hydroxypregnenolone pregnenolone. 6 - FEB 2014 - FOR
D. progesterone 2014 MED INCLUSION IN
E. Aldosterone BOARDS; THE SAMPLEX
TOPNOTCH
MD)
466 Cardiolipin is an important phospholipid characterized Cardiolipin is a phospholipid that is antigenic which LUISA BACK-UP
as follows except: reacts with antibodies against Treponema pallidum. It SARANILL MIDTERM
A. antigenic phospholipid is found only in mitochondria which is essential for O, MD (TOP EXAM AUG
B. Found in mitochondria and cytoplasm mitochondrial function. 6 - FEB 2014 - FOR
C. Found in mitochondria only 2014 MED INCLUSION IN
D. Reacts with antibodies produced against Treponema BOARDS; THE SAMPLEX
pallidum TOPNOTCH
E. none of the choices MD)

TOPNOTCH MEDICAL BOARD PREP BIOCHEMISTRY SUPEREXAM Page 58 of 92


For inquiries visit www.topnotchboardprep.com.ph or email us at topnotchmedicalboardprep@gmail.com
TOPNOTCH MEDICAL BOARD PREP BIOCHEMISTRY SUPEREXAM
For inquiries visit www.topnotchboardprep.com.ph or email us at topnotchmedicalboardprep@gmail.com
Item QUESTION EXPLANATION AUTHOR TOPNOTCH
# EXAM
467 Which of the following is an essential fatty acids? linoleic, linolenic and arachidonic acid are the LUISA BACK-UP
A. Linoleic acid essential fatty acids. Other choices are essential amino SARANILL MIDTERM
B. isoleucine acids. O, MD (TOP EXAM AUG
C. phenylalanine 6 - FEB 2014 - FOR
D. valine 2014 MED INCLUSION IN
E. All of the above BOARDS; THE SAMPLEX
TOPNOTCH
MD)
468 A single DNA base substitution resulting to formation of frameshift mutation involves deletion or addition of a LUISA BACK-UP
a different amino acid. base resulting to a shorter nonfunctional protein. SARANILL MIDTERM
A. Frame shift mutation Nonsense mutation results to a stop codon. Missense O, MD (TOP EXAM AUG
B. Nonsense mutation mutation forms a different amino acid. Silent mutation 6 - FEB 2014 - FOR
C. Missense mutation forms same amino acid. 2014 MED INCLUSION IN
D. Silent mutation BOARDS; THE SAMPLEX
E. None of the choices TOPNOTCH
MD)
469 The following are essential amino acids that are both all of the choices are both ketogenic and glucogenic, LUISA BACK-UP
ketogenic and glucogenic except: and all are essential amino acids except tyrosine. SARANILL MIDTERM
A. phenylalanine O, MD (TOP EXAM AUG
B. tyrosine 6 - FEB 2014 - FOR
C. tryptophan 2014 MED INCLUSION IN
D. isoleucine BOARDS; THE SAMPLEX
E. None of the choices TOPNOTCH
MD)

470 In the Landsteiner ABO blood system, what determines type A= N-acetyl-galactosamine; type B = galactose; LUISA BACK-UP
specificity of blood type A? type AB = N-acetyl-galactosamine and galactose SARANILL MIDTERM
A. N-acetyl-galactosamine O, MD (TOP EXAM AUG
B. galactose 6 - FEB 2014 - FOR
C. N-acetyl-glucosamine 2014 MED INCLUSION IN
D. A and B BOARDS; THE SAMPLEX
E. All of the choices TOPNOTCH
MD)

471 A 60 year old female diabetic patient was given 20U of The counter-regulatory hormones of insulin are LUISA BACK-UP
regular insulin at bedtime by her daughter. Few hours glucagon, cortisol, epinephrine, and growth hormone. SARANILL MIDTERM
later, the blood sugar level was at 39mg/dl. In order to O, MD (TOP EXAM AUG
increased her blood sugar level, her body should 6 - FEB 2014 - FOR
produce what hormone/s? 2014 MED INCLUSION IN
A. glucagon BOARDS; THE SAMPLEX
B. epinephrine TOPNOTCH
C. cortisol MD)
D. all of the above
E. A and C only
472 Approximately, proteins comprised what percentage of 10-35% = protein; 20-35% = fat; 45-65% = LUISA BACK-UP
energy requirements in human? carbohydrates SARANILL MIDTERM
A. 10-25% O, MD (TOP EXAM AUG
B. 45-60% 6 - FEB 2014 - FOR
C. 20-35% 2014 MED INCLUSION IN
D. 40-45% BOARDS; THE SAMPLEX
E. 50-55% TOPNOTCH
MD)

473 What is the fat soluble vitamin that is considered the Vit. D is the most toxic vitamin while Vitamin E is the LUISA BACK-UP
most toxic vitamin? least toxic vitamin. SARANILL MIDTERM
A. Vitamin A O, MD (TOP EXAM AUG
B. Vitamin D 6 - FEB 2014 - FOR
C. Vitamin E 2014 MED INCLUSION IN
D. Vitamin K BOARDS; THE SAMPLEX
E. Vitamin B TOPNOTCH
MD)
474 A 6 year old child presents with pallor. As her attending This child is most likely having an iron deficiency LUISA BACK-UP
physician, you request for CBC which revealed a anemia. Iron should be given in this patient along with SARANILL MIDTERM
hemoglobin of 9g/dl. What medication/s should you give Vitamin C which increases iron absorption. O, MD (TOP EXAM AUG
to this patient. 6 - FEB 2014 - FOR
A. Iron 2014 MED INCLUSION IN
B. Vitamin C BOARDS; THE SAMPLEX
C. Zinc TOPNOTCH
D. Iron and Vitamin C MD)
E. Iron and zinc

475 Among the following pyrimidine bases, which one is Cytosine, uracil, and thymine are pyrimidine bases, LUISA BACK-UP
present only in the RNA? but uracil is present only in RNA while thymine is SARANILL MIDTERM
A. cytosine present only in DNA. Adenine and guanine are purine O, MD (TOP EXAM AUG
B. thymine bases. 6 - FEB 2014 - FOR
C. uracil 2014 MED INCLUSION IN
D. guanine BOARDS; THE SAMPLEX
E. Adenine TOPNOTCH
MD)

TOPNOTCH MEDICAL BOARD PREP BIOCHEMISTRY SUPEREXAM Page 59 of 92


For inquiries visit www.topnotchboardprep.com.ph or email us at topnotchmedicalboardprep@gmail.com
TOPNOTCH MEDICAL BOARD PREP BIOCHEMISTRY SUPEREXAM
For inquiries visit www.topnotchboardprep.com.ph or email us at topnotchmedicalboardprep@gmail.com
Item QUESTION EXPLANATION AUTHOR TOPNOTCH
# EXAM
476 This disease is caused by low activities of orotidine LUISA BACK-UP
phosphate decarboxylase and orotate SARANILL MIDTERM
phosphoribosyltransferase which causes abnormal O, MD (TOP EXAM AUG
growth and anemia. What type of anemia is present in 6 - FEB 2014 - FOR
this disease? 2014 MED INCLUSION IN
A. hypochromic anemia BOARDS; THE SAMPLEX
B. microcytic anemia TOPNOTCH
C. megaloblastic anemia MD)
D. sideroblastic anemia
E. none of the choices
477 What is the smallest type of RNA? tRNA is the smallest RNA. mRNA is the largest RNA. LUISA BACK-UP
A. tRNA rRNA is the most common type of RNA. snRNA is a SARANILL MIDTERM
B. mRNA subset of RNA for gene regulation. O, MD (TOP EXAM AUG
C. rRNA 6 - FEB 2014 - FOR
D. snRNA 2014 MED INCLUSION IN
E. None of the choices BOARDS; THE SAMPLEX
TOPNOTCH
MD)
478 In metabolic pathways, catabolic reaction is an example Catabolic reaction is an exergonic reaction. Anabolic LUISA BACK-UP
of: reaction is an endergonic reaction. Amphibolic is the SARANILL MIDTERM
A. Endergonic reaction link between anabolic and catabolic reactions. O, MD (TOP EXAM AUG
B. Exergonic reaction 6 - FEB 2014 - FOR
C. Amphibolic reaction 2014 MED INCLUSION IN
D. A and B only BOARDS; THE SAMPLEX
E. B and C only TOPNOTCH
MD)
479 NADH produced from aerobic glycolysis proceeds to the NADH needs a shuttle in order for it to enter the LUISA BACK-UP
electron transport chain. If one molecule of NADH enters mitochondrial membrane to enter the ETC. There are SARANILL MIDTERM
a neuron, how many ATP are produced? 2 shuttles used - the malate aspartate shuttle present O, MD (TOP EXAM AUG
A. 1 in liver, kidney, and heart which produces 3 ATP in 6 - FEB 2014 - FOR
B. 2 one NADH; and the glycerol phosphate shuttle present 2014 MED INCLUSION IN
C. 3 in skeletal muscle and brain which produces 2 ATP in BOARDS; THE SAMPLEX
D. 4 one NADH. TOPNOTCH
E. 5 MD)

480 A 3 day old infant presents with vomiting and diarrhea Gal-1-P uridyltransferase deficiency causes classic LUISA BACK-UP
after milk ingestion. Blood sugar level was 40 mg/dl. The galactosemia which presents with vomiting, diarrhea, SARANILL MIDTERM
infant develops a cataract few days after. What is hypoglycemia, lethargy, hypotonia, liver disease, and O, MD (TOP EXAM AUG
responsible for the disease. cataracts within a few days of birth. While in 6 - FEB 2014 - FOR
A. Galactokinase deficiency galactokinase deficiency, cataracts develop later in 2014 MED INCLUSION IN
B. galactokinase excess early childhood. In aldolase B deficiency, cataract is BOARDS; THE SAMPLEX
C. Gal-1-P uridyltransferase deficiency not characteristic. TOPNOTCH
D. hexokinase deficiency MD)
E. aldolase B deficiency

481 An excess of this vitamin causes sensory neuropathy: Ironically, intake of pyridoxine (B6) which prevents ANGELIS FINAL EXAM -
A. Vitamin A peripheral neuropathy can result in sensory ANDREA AUG 2014
B. Vitamin B neuropathy with altered sensation of touch and pain if COCOS, MD
C. Vitamin C taken in excess. (TOP 1 -
D. Vitamin D FEB 2014
MED
BOARDS;
TOPNOTCH
MD)
482 Which is FALSE regarding beta bends? Proline and glycine are frequently found in beta ANGELIS FINAL EXAM -
A. They are also known as beta turns. bends; proline because its cyclic structure is ideally ANDREA AUG 2014
B. They fall under secondary structure. suited for the beta turn, and glycine because with the COCOS, MD
C. They are usually composed of proline and alanine. smallest side chain of all the amino acids, it is the most (TOP 1 -
D. Beta bends often promote the formation of stericallly flexible. FEB 2014
antiparallel beta sheets. MED
BOARDS;
TOPNOTCH
MD)
483 The most appropriate and prompt treatment for Galactosemia is due to an enzyme deficiency ANGELIS FINAL EXAM -
galactosemia is: (galactose-1-phosphate uridyl transferase) which ANDREA AUG 2014
A. Enzyme replacement leads to accumulation of galactose-1-phosphate in the COCOS, MD
B. Hormone therapy tissues. Treatment consists of prompt elimination of (TOP 1 -
C. Special infant formula lactose-containing milk from the diet in infancy. FEB 2014
D. Vitamin therapy MED
BOARDS;
TOPNOTCH
MD)
484 A 13-year-old healthy Jewish girl, Anne Frank, is found to This is a case of Gaucher disease. The diagnosis is ANGELIS FINAL EXAM -
have mild anemia, leukopenia, and thrombocytopenia. confirmed by the absence of glucocerebrosidase ANDREA AUG 2014
Physical exam reveals an enlarged spleen. Bone marrow activity in leukocytes, in cultured skin fibroblasts and COCOS, MD
exam shows abnormal cells. The diagnosis can be liver cells. Spingomyelinase deficiency causes type A (TOP 1 -
confirmed by measurement of the activity of which of Niemann-Pick disease, hexosaminidase A deficiency FEB 2014
the following enzymes? causes Sandhoff disease, sulfatase A deficiency causes MED
A. glucocerebrosidase metachromatic leukodystrophy. SIMILAR TO BOARDS;
B. sphingomyelinase PREVIOUS BOARD EXAM CONCEPT/PRINCIPLE TOPNOTCH
C. hexosamidase A MD)
D. sulfatase A

TOPNOTCH MEDICAL BOARD PREP BIOCHEMISTRY SUPEREXAM Page 60 of 92


For inquiries visit www.topnotchboardprep.com.ph or email us at topnotchmedicalboardprep@gmail.com
TOPNOTCH MEDICAL BOARD PREP BIOCHEMISTRY SUPEREXAM
For inquiries visit www.topnotchboardprep.com.ph or email us at topnotchmedicalboardprep@gmail.com
Item QUESTION EXPLANATION AUTHOR TOPNOTCH
# EXAM
485 A 60-kg medical student rides the jeepney daily, spends Calorie intake for weight maintenance is as follows ANGELIS FINAL EXAM -
8 hours in class sitting down, goes home and studies for according to Schwartz: for sedentary lifestyle: 25 ANDREA AUG 2014
another 3 hours or so. What should be his total caloric kcal/kg, moderately active lifestyle: 30 kcal/kg, heavy COCOS, MD
intake daily in order for him to maintain his current activity lifestyle: 35 kcal/kg. Minus 5kcal each for (TOP 1 -
weight? weightloss and add 5kcal each for weight gain. There FEB 2014
A. 1,200 kcal were some computations like this during our exam. MED
B. 1,000 kcal Biochem lec values as follows: 30 kcal/kg sedentary, BOARDS;
C. 1,800 kcal 35 kcal/kg moderate, 40 kcal/kg heavy. TOPNOTCH
D. 2,400 kcal MD)
486 A 6-month old child was noted to have slow weight gain Strokes in the young should increase the suspicion of ANGELIS FINAL EXAM -
and mild delay in achieving developmental milestones. homocystinuria. It can cause thromboembolic ANDREA AUG 2014
One morning the parents noted that their child seems phenomena in the pulmonary and systemic arteries, COCOS, MD
unable to move the right side of his body. Which and particulary the cerebral vasculature. (TOP 1 -
condition could explain this child's presentation? FEB 2014
A. phenylketonuria MED
B. cystathioninuria BOARDS;
C. homocystinuria TOPNOTCH
D. maple syrup disease MD)

487 Which of the following is a ketogenic amino acid? SIMILAR TO PREVIOUS BOARD EXAM ANGELIS FINAL EXAM -
A. isoleucine CONCEPT/PRINCIPLE. Only leucine and lysine are ANDREA AUG 2014
B. valine purely ketogenic. Isoleucine gives rise to both COCOS, MD
C. lysine ketogenic and glucogenic fragments. (TOP 1 -
D. alanine FEB 2014
MED
BOARDS;
TOPNOTCH
MD)
488 Which of the following is FALSE regarding gene therapy? Several questions regarding gene therapy were asked ANGELIS FINAL EXAM -
A. It is a safe and effective therapy. during our exam. Although gene therapy is a ANDREA AUG 2014
B. It can be used to treat not only inherited disorders promising treatment option for a number of diseases, COCOS, MD
but certain types of cancer and viral infections as well. the technique remains risky and is still under study to (TOP 1 -
C. It is the use of DNA encoding for a functional, ensure that it is safe and effective. FEB 2014
therapeutic gene to replace a mutated gene. MED
D. Viruses have been used as a vector for gene therapy. BOARDS;
TOPNOTCH
MD)
489 Which of the following statements about the urea cycle is Urea is formed during the pathway wherein arginine ANGELIS FINAL EXAM -
FALSE? is converted to ornithine. ANDREA AUG 2014
A. Urinary urea is increased by a diet rich in protein. COCOS, MD
B. The two nitrogen atoms that are incorporated into (TOP 1 -
urea enter the cycle as ammonia and aspartate. FEB 2014
C. ATP is not required for the reaction in which MED
arginosuccinate is converted to arginine. BOARDS;
D. Urea is produced directly by the hydrolysis of TOPNOTCH
ornithine. MD)
490 Which apolipoprotein activates lecithin-cholesterol Apolipoprotein A-1 is found in HDL alone. ANGELIS FINAL EXAM -
acyltransferase (LCAT)? ANDREA AUG 2014
A. E COCOS, MD
B. A-1 (TOP 1 -
C. C-II FEB 2014
D. B-48 MED
BOARDS;
TOPNOTCH
MD)
491 Paulo ordered two-piece chicken and large fries for Pancreatic lipase is used for the degradation of dietary ANGELIS FINAL EXAM -
lunch. Which among the enzymes would increase in TG in the small intestine. LPL is for TG degradation ANDREA AUG 2014
activity? circulating in chylomicrons and VLDLs. HL is for the COCOS, MD
A. Pancreatic lipase degradation of TG remaining in IDL. Hormone- (TOP 1 -
B. Lipoprotein lipase sensitive lipase (sensitive to catecholamines, glucagon FEB 2014
C. Hepatic TG lipase etc.) is for the degradation of TG stored in adipocytes. MED
D. Hormone-sensitive lipase BOARDS;
TOPNOTCH
MD)
492 Urine test for ketones detect: This is the reason why serum ketones are more ANGELIS FINAL EXAM -
A. acetoacetate accurate than urine ketones. ANDREA AUG 2014
B. beta-hydroxybutyrate COCOS, MD
C. acetone (TOP 1 -
D. all of the above FEB 2014
MED
BOARDS;
TOPNOTCH
MD)
493 The liver is NOT able to utilize ketone bodies because of SIMILAR TO PREVIOUS BOARD EXAM ANGELIS FINAL EXAM -
the lack of the enzyme: CONCEPT/PRINCIPLE. Thiophorase is an enzyme that ANDREA AUG 2014
A. thiolase converts ketone bodies to acetyl CoA. COCOS, MD
B. thiosuccinase (TOP 1 -
C. thiophorase FEB 2014
D. Thionase MED
BOARDS;
TOPNOTCH
MD)

TOPNOTCH MEDICAL BOARD PREP BIOCHEMISTRY SUPEREXAM Page 61 of 92


For inquiries visit www.topnotchboardprep.com.ph or email us at topnotchmedicalboardprep@gmail.com
TOPNOTCH MEDICAL BOARD PREP BIOCHEMISTRY SUPEREXAM
For inquiries visit www.topnotchboardprep.com.ph or email us at topnotchmedicalboardprep@gmail.com
Item QUESTION EXPLANATION AUTHOR TOPNOTCH
# EXAM
494 Carbon monoxide poisoning inhibits which complex in Carbon monoxide, together with cyanide, hydrogen ANGELIS FINAL EXAM -
the mitochondrial electron transport chain? sulfide and azides, inhibits cytochrome c oxidase or ANDREA AUG 2014
A. Complex II complex IV. SIMILAR TO PREVIOUS BOARD EXAM COCOS, MD
B. Complex III CONCEPT/PRINCIPLE. Know the other complex (TOP 1 -
C. Complex I inhibitors. FEB 2014
D. None of the above MED
BOARDS;
TOPNOTCH
MD)
495 Biotin is used as a cofactor for the following enzymes, The rest of the choices use biotin which adds a 1- ANGELIS FINAL EXAM -
EXCEPT: carbon group to the substrate. ANDREA AUG 2014
A. Pyruvate carboxylase COCOS, MD
B. Acetyl-CoA carboxylase (TOP 1 -
C. Propionyl-CoA carboxylase FEB 2014
D. Succinyl-Coa carboxylase MED
BOARDS;
TOPNOTCH
MD)
496 A middle aged woman presents with depression, CGG is for Fragile X, GAA is for Friedrich's ataxia, CTG ANGELIS FINAL EXAM -
progressive dementia, and choreiform movements. You is for myotonic dystrophy. ANDREA AUG 2014
suspect that she is suffering from a trinucleotide repeat COCOS, MD
expansion disease which is multiple repeats of: (TOP 1 -
A. CGG FEB 2014
B. GAA MED
C. CAG BOARDS;
D. CTG TOPNOTCH
MD)
497 Which of the following post-transcription modifications All these steps happen to produce the mature ANGELIS FINAL EXAM -
take place to produce eukaryotic mRNA? eukaryotic mRNA. ANDREA AUG 2014
A. Addition of 7-methylguanosine cap on the 5' end COCOS, MD
B. Polyadenation on 3' end (TOP 1 -
C. Splicing out of introns FEB 2014
D. All of the above MED
BOARDS;
TOPNOTCH
MD)
498 Which is NOT a feature of the genetic code? The genetic code is unambiguous, degenerate (except ANGELIS FINAL EXAM -
A. Each codon specifies only 1 amino acid. methionine and tryptophan), ANDREA AUG 2014
B. Tryptophan and methionine is encoded by only 1 nonoverlapping/commaless, and universal. COCOS, MD
codon. (TOP 1 -
C. The genetic code is conserved throughout evolution. FEB 2014
D. It is overlapping. MED
BOARDS;
TOPNOTCH
MD)
499 A person at risk for all types of skin cancer would have a Nucleotide excision repair is the primary defect in ANGELIS FINAL EXAM -
defect in: xeroderma pigmentosum, which prevents repair of ANDREA AUG 2014
A. Base excision repair pyrimidine dimers because of ultraviolet light COCOS, MD
B. Mismatch repair exposure. (TOP 1 -
C. Nucleotide excision repair FEB 2014
D. Any of the above MED
BOARDS;
TOPNOTCH
MD)
500 A deficiency in this micronutrient results in Zinc deficiency results in delayed wound healing, ANGELIS FINAL EXAM -
hypogonadism and decrease in adult hair whether hypogonadism, decrease in axillary, facial and pubic ANDREA AUG 2014
axillary, facial, or pubic. hair, dysgeusia and anosmia. COCOS, MD
A. copper (TOP 1 -
B. zinc FEB 2014
C. manganese MED
D. selenium BOARDS;
TOPNOTCH
MD)
501 1. A sphingolipid which is present in high amounts in Harper's 27th ed., 126. SIMILAR TO PREVIOUS BOARD JAN BACK-UP
myelin sheath: EXAM CONCEPT/PRINCIPLE (board question, feb CHARMAIN MIDTERM
A. Sphingomyelin 2014) E EXAM AUG
B. Glycerol PALOMAR, 2014
C. Lysophosphatidylcholine MD (TOP 9
D. Cholesterol - FEB 2014
E. Phosphatidylinositol MED
BOARDS;
TOPNOTCH
MD)

TOPNOTCH MEDICAL BOARD PREP BIOCHEMISTRY SUPEREXAM Page 62 of 92


For inquiries visit www.topnotchboardprep.com.ph or email us at topnotchmedicalboardprep@gmail.com
TOPNOTCH MEDICAL BOARD PREP BIOCHEMISTRY SUPEREXAM
For inquiries visit www.topnotchboardprep.com.ph or email us at topnotchmedicalboardprep@gmail.com
Item QUESTION EXPLANATION AUTHOR TOPNOTCH
# EXAM
502 2. These vitamins are toxic in excess: There is only a limited capacity to metabolize vitamin JAN BACK-UP
A. Vitamin A and B A, and excessive intakes lead to accumulation beyond CHARMAIN MIDTERM
B. Vitamin B and C the capacity of binding proteins, so that unbound E EXAM AUG
C. Vitamin C and D vitamin A causes tissue damage. Symptoms of toxicity PALOMAR, 2014
D. Vitamin A and D affect the central nervous system (headache, nausea, MD (TOP 9
E. Vitamin D and E ataxia, and anorexia, all associated with increased - FEB 2014
cerebrospinal fluid pressure); the liver (hepatomegaly MED
with histologic changes and hyperlipidemia); calcium BOARDS;
homeostasis (thickening of the long bones, TOPNOTCH
hypercalcemia, and calcification of soft tissues); and MD)
the skin (excessive dryness, desquamation, and
alopecia).
Harper's 27th ed., 491-92.
Some infants are sensitive to intakes of vitamin D as
low as 50 g/day, resulting in an elevated plasma
concentration of calcium. This can lead to contraction
of blood vessels, high blood pressure, and calcinosis—
the calcification of soft tissues.
Harper's 27th ed., 93
503 3. The enzyme responsible for the yellow color of a SIMILAR TO PREVIOUS BOARD EXAM JAN BACK-UP
maturing hematoma: CONCEPT/PRINCIPLE. Heme oxygenase - enzyme CHARMAIN MIDTERM
A. Heme oxygenase system responsible for catabolism of heme from all of E EXAM AUG
B. Heme reductase the heme proteins PALOMAR, 2014
C. Biliverdin reductase Heme reductase MD (TOP 9
D. Glucoronosyl ttransferase Biliverdin reductase - reduces the methyne bridge - FEB 2014
E. a-aminolevulinate synthase b/w pyrrole III and pyrole IV of biliverdin to a MED
methylene group to produce bilirubin a yellow BOARDS;
pigment Katzung 27th ed., 287 TOPNOTCH
Glucoronosyl ttransferase - catalyzes conjugation of MD)
bilirubin
a-aminolevulinate synthase - rate-controlling enzyme
in porphyrin biosynthesis in mammalian liver
504 4. Major end product of nitrogen catabolism in humans: Harper's 27th ed., 249 JAN BACK-UP
A. glutamate CHARMAIN MIDTERM
B. urea E EXAM AUG
C. uric acid PALOMAR, 2014
D. ammonia MD (TOP 9
E. Creatinine - FEB 2014
MED
BOARDS;
TOPNOTCH
MD)
505 5. Most important intracellular buffer: Buffers are solutions of weak acids or bases and their JAN BACK-UP
A. bicarbonate conjugates that has the ability to resist a change in pH CHARMAIN MIDTERM
B. carbon dioxide following addition of strong acid or base. Bicarbonate E EXAM AUG
C. water is the most important extracellular buffer. Proteins PALOMAR, 2014
D. proteins are the most important intracellular buffer. MD (TOP 9
E. Phosphate - FEB 2014
MED
BOARDS;
TOPNOTCH
MD)
506 6. Which of the following statements regarding enzymes Enzymes lower the free energy of activation. JAN BACK-UP
is not true? CHARMAIN MIDTERM
A. Enzymes are protein catalysts that increase the E EXAM AUG
velocity of a chemical reaction but are not consumed in PALOMAR, 2014
the process. MD (TOP 9
B. Isoenzymes like hexokinase and glucokinase are - FEB 2014
physically distinct versions of a given enzyme that MED
catalyze the same reaction. BOARDS;
C. Enzymes do not change the energy of the reactants TOPNOTCH
and products and the equilibrium of the reaction. MD)
D. Enzymes are highly specific in the reactions they
catalyze.
E. Enzymes increase the free energy of activation.
507 7. Inhibitor of Complex III of the electron transport SIMILAR TO PREVIOUS BOARD EXAM JAN BACK-UP
chain: CONCEPT/PRINCIPLE. 3-4 questions of this type came CHARMAIN MIDTERM
A. Barbiturate and Amytal out in my boards Complex I inhibitors: E EXAM AUG
B. Dimercaprol and antimycin Barbiturate, Piercidin A, Amytal, Rotenone PALOMAR, 2014
C. Cyanide and hydrogen sulfide Complex II inhibitors - Malonate, Carboxin, TTFA
MD (TOP 9
D. Malonate and carboxin Complex III inhibitors - Antimycin A, Dimercaprol
- FEB 2014
E. Cyanide and carbon monoxide Complex IV inhibitors - Cyanide, Carbon monoxide, MED
sodium azide, hydrogen sulfide BOARDS;
TOPNOTCH
MD)
508 8. Which of the following substances increase the SIMILAR TO PREVIOUS BOARD EXAM JAN BACK-UP
permeability of the inner mitochondrial membrane to CONCEPT/PRINCIPLE. All of them are uncouplers. CHARMAIN MIDTERM
protons causing a decrease in proton gradient and an First Aid 2013, p. 102. E EXAM AUG
increase in oxygen consumption with resultant PALOMAR, 2014
production of heat but cessation of ATP synthesis? MD (TOP 9
A. 2,4-dinitrophenol - FEB 2014
B. brown fat MED
C. aspirin BOARDS;
D. all of the above TOPNOTCH
E. none of the above MD)

TOPNOTCH MEDICAL BOARD PREP BIOCHEMISTRY SUPEREXAM Page 63 of 92


For inquiries visit www.topnotchboardprep.com.ph or email us at topnotchmedicalboardprep@gmail.com
TOPNOTCH MEDICAL BOARD PREP BIOCHEMISTRY SUPEREXAM
For inquiries visit www.topnotchboardprep.com.ph or email us at topnotchmedicalboardprep@gmail.com
Item QUESTION EXPLANATION AUTHOR TOPNOTCH
# EXAM
509 9. Metabolic pathway that provides a source of NADPH HMP of Pentose phosphate pathway is an important JAN BACK-UP
required for reductive reactions and also yields ribose source of NADPH and ribose. Gluconeogenesis is CHARMAIN MIDTERM
for nucleotide synthesis and glycolytic intermediates. responsible for generation of glucose from non- E EXAM AUG
A. Hexose monophosphate shunt carbohydrate sources as glycerol and amino acid. PALOMAR, 2014
B. Gluconeogenesis Glycolysis is breakdown of carbohydrates into glucose MD (TOP 9
C. Aerobic Glycolysis molecules that can be absorbed by the body. - FEB 2014
D. Anaerobic Glycolysis Tricarboxylic acid cycle or Kreb's cycle produces 3 MED
E. Tricarboxylic acid cycle NADH, 1 FADH2, 2 CO2, and 1 GTP per acetyl-CoA. BOARDS;
First Aid 2013, p.103 TOPNOTCH
MD)
510 10. Deficiency of this enzyme results in intracellular First Aid 201, p. 105
JAN BACK-UP
accumulation of sorbitol, causing osmotic damage in the Aldose reductase sorbitol dehydrogenase
CHARMAIN MIDTERM
form of cataracts, retinopathy and peripheral Glucose → Sorbitol → Fructose
E EXAM AUG
neuropathy seen in uncontrolled diabetics : NADPH NAD+ PALOMAR, 2014
A. Aldose reductase MD (TOP 9
B. Sorbitol dehydrogenase - FEB 2014
C. Lactase MED
D. Galactokinase BOARDS;
E. Fructokinase TOPNOTCH
MD)
511 11. A newborn presents with dark ears and brown- SIMILAR TO PREVIOUS BOARD EXAM JAN BACK-UP
pigmented sclera. Mother also complains that his urine CONCEPT/PRINCIPLE. This is a case of alkaptonuria, CHARMAIN MIDTERM
turns black on exposure to air. Which enzyme is AKA ochronosis wherein there is a deficiency of E EXAM AUG
deficient? homogentisic oxidase. First Aid 2013, p.108 PALOMAR, 2014
A. Melanin synthase tyrosonase is deficient in albinism MD (TOP 9
B. Phenylalanine hydroxylase - FEB 2014
C. Homocysteine methyl transferase MED
D. Tyrosinase BOARDS;
E. Homogentisic oxidase TOPNOTCH
MD)
512 12. These are compounds that are mirror images of each SIMILAR TO PREVIOUS BOARD EXAM JAN BACK-UP
other. CONCEPT/PRINCIPLE.( 2 questions of this type) CHARMAIN MIDTERM
A. isomers Isomers - compounds that have the same chemical E EXAM AUG
B. anomers formula but different structures. Epimers are PALOMAR, 2014
C. epimers compounds that differ in the 3-d configuration around MD (TOP 9
D. enantiomers only one specific carbon atom with the exception of - FEB 2014
E. All of the above the carbonyl carbon. Enantiomers/optical MED
isomers/stereoisomers- are pairs of structures that BOARDS;
are mirror images of each other and are designated as TOPNOTCH
D- and L-sugars. Anomers are convertible between a MD)
linear and a ring form.
513 13. Which of the following is not true regarding HDL picks up cholesterol accumulating in blood JAN BACK-UP
lipoproteins? vessels (reverse cholesterol transport) to the liver and CHARMAIN MIDTERM
A. Chylomicrons transport triglyceride and cholesterol streroidogenic tissues via scavenger receptor (SR-B1). E EXAM AUG
from intestine to tissues following a meal. PALOMAR, 2014
B. LDL transport cholesterol into the cells. MD (TOP 9
C. VLDL transport endogenous triglyceride from liver - FEB 2014
to tissues. MED
D. HDL is responsible delivering cholesterol back to the BOARDS;
intestines for excretion. TOPNOTCH
E. None of the above MD)
514 14. This is the most common lysosomal storage disease SIMILAR TO PREVIOUS BOARD EXAM JAN BACK-UP
wherein there is a deficiency of glucocerebrosidase CONCEPT/PRINCIPLE. Niemann-Pick- deficient in CHARMAIN MIDTERM
enzyme. Patient may present with hepatosplenomegaly, sphingomyelinase enzyme; Taysach's- deficient in E EXAM AUG
aseptic necrosis of femur and macrophages look like hexosaminidase A; Hurler's- deficient in a-L- PALOMAR, 2014
crumpled tissue paper microscopically. iduronidase, a mucopolysaccharidoses; Hunter's - MD (TOP 9
A. Niemann-Pick disease Idoronate sulfatase deficiency - FEB 2014
B. Gaucher's disease MED
C. Taysachs disease BOARDS;
D. Hurler's syndrome TOPNOTCH
E. Hunter's syndrome MD)

515 15. Adipose tissue and muscles lacks this enzyme which Harper's 27th ed 161. In liver (and kidney) but not in JAN BACK-UP
explains why it cannot export glucose into the blood muscle, glucose 6 phosphatase hydrolyzes glucose 6- CHARMAIN MIDTERM
stream: phosphate, yielding glucose that is exported, leading E EXAM AUG
A. HMG CoA reductase to an increase in the blood glucose concentration. PALOMAR, 2014
B. Fatty acid synthetase MD (TOP 9
C. Glycogen phosphorylase - FEB 2014
D. Hexokinase MED
E. Glucose 6-phosphatase BOARDS;
TOPNOTCH
MD)
516 16. Which of the following regarding glycogen is not Harper's 27th ed., 161. In the liver the role of glycogen JAN BACK-UP
true? is to provide free glucose for export to maintain the CHARMAIN MIDTERM
A. In the muscle, the role of glycogen is to provide free blood concentration of glucose; in muscle, the role of E EXAM AUG
glucose for export to maintain the blood concetration of glycogen is to provide a source of glucose 6-phosphate PALOMAR, 2014
glucose. for glycolysis in response to the need for ATP for MD (TOP 9
B. Glycogen is the major storage carbohydrate in muscle contraction. - FEB 2014
animals. MED
C. Glycogenolysis is not the reverse of glycogenesis, but BOARDS;
is a separate pathway. TOPNOTCH
D. Insulin inhibits glycogenolysis and stimulates MD)
glycogenesis.
E. McArdle's syndrome is a glycogen storage disease
presenting with poor exercise tolerance and decreased
TOPNOTCH MEDICAL BOARD PREP BIOCHEMISTRY SUPEREXAM Page 64 of 92
For inquiries visit www.topnotchboardprep.com.ph or email us at topnotchmedicalboardprep@gmail.com
TOPNOTCH MEDICAL BOARD PREP BIOCHEMISTRY SUPEREXAM
For inquiries visit www.topnotchboardprep.com.ph or email us at topnotchmedicalboardprep@gmail.com
Item QUESTION EXPLANATION AUTHOR TOPNOTCH
# EXAM
blood lactate after exercise.

517 17. The following amino acids can be converted to SIMILAR TO PREVIOUS BOARD EXAM JAN BACK-UP
glucose except: CONCEPT/PRINCIPLE: Glycogenic Amino acids: CHARMAIN MIDTERM
A. Lysine and Isoleucine Alanine, Arginine, Asparagine, Cysteine, Glutamate, E EXAM AUG
B. Tryptophan and Tyrosine Histidine, Methionine, Proline, Serine, Threonine, PALOMAR, 2014
C. Isoleucine and Alanine Valine MD (TOP 9
D. Leucine and Lysine Ketogenic: Leucine and Lysine
- FEB 2014
E. Alanine and Arginine Glycogenic and Ketogenic: Phenylalanine, Tyrosine, MED
Isoleucine, Tryptophan (FYIW) BOARDS;
TOPNOTCH
MD)
518 18. Which of the following statements show degeneracy SIMILAR TO PREVIOUS BOARD EXAM JAN BACK-UP
in the genetic code? CONCEPT/PRINCIPLE. Degenerate/Redundant - Most CHARMAIN MIDTERM
A. Six different codons specify serine. amino acids are coded by multiple codos except E EXAM AUG
B. For any specific codon, only a single amino acid is methionine and tryptophan which are encoded by PALOMAR, 2014
indicated. only 1 codon (AUG and UGG, respectively). MD (TOP 9
C. The reading of the genetic code during the process of Unambiguous- Each codon specifies only 1 amino acid. - FEB 2014
protein synthesis does not involve any overlap of Commaless/non-overlapping- read from a fixed MED
codons. starting point as a continuous sequence of bases. BOARDS;
D. Once the reading of the genetic code is commenced Universal -genetic code is conserved. First aid 2013, TOPNOTCH
at a specific codon, the message is read in a continuous p.66. Choice E depicts wobble or the MD)
sequence of nucleotide triplets until translation stop ability of certain anti-codons to pair with codons that
codon is reached. differ at the 3rd base.
E. Codons differing only in the 3rd base positions may
code for the same tRNA/amino acid.
519 19. If the cytosine content of a double-helical DNA is Chargaff's rule: In DNA molecules the concentration of JAN BACK-UP
30% of the total bases, the thymidine content would be: deoxyadenine nucleotide equals that of thymidine CHARMAIN MIDTERM
A. 60% nucleotides (A=T), while the concentration of deoxy E EXAM AUG
B. 50% guanosine nucleotides equals that of deoxycytidine PALOMAR, 2014
C. 40% nucleotides (G=C). Harper's 27th ed., 311. MD (TOP 9
D. 30% - FEB 2014
E. 20% MED
BOARDS;
TOPNOTCH
MD)
520 20. This enzyme is responsible for the release of free SIMILAR TO PREVIOUS BOARD EXAM JAN BACK-UP
fatty acids from adipose in fasting states and is inhibitted CONCEPT/PRINCIPLE. Pancreatic lipase acts on CHARMAIN MIDTERM
by insulin. dietary TAG, Lipoprotein lipase acts on TAG E EXAM AUG
A. Pancreatic lipase chylomicrons and VLDL. Hormone sensitive TAG acts PALOMAR, 2014
B. Lipoprotein lipase on TAG stored in adipose and is activated by glucagon. MD (TOP 9
C. Hormone-sensitive lipase Lingual lipase originates from the back of the tongue. - - FEB 2014
D. Gastric lipase Biochem Topnotch handouts MED
E. Lingual lipase BOARDS;
TOPNOTCH
MD)
521 A 23-year-old male develops steatorrhea, weight loss, Celiac disease MIGUEL MIDTERM 1
and bloody diarrhea. He notes that his diarrhea is worse RAFAEL EXAM - FEB
when he eats breads or cereals. A gastroenterologist RAMOS, 2013
performs a biopsy during a colonoscopy, which reveals MD (TOP 3
celiac disease. This disorder is most directly due to - FEB 2012
which of the following? MED
A) excess lipids in the feces BOARDS;
B) deficiency of enterokinase TOPNOTCH
C) defective transport of amino acid cysteine MD)
D) hypersensitivity to the protein gluten

TOPNOTCH MEDICAL BOARD PREP BIOCHEMISTRY SUPEREXAM Page 65 of 92


For inquiries visit www.topnotchboardprep.com.ph or email us at topnotchmedicalboardprep@gmail.com
TOPNOTCH MEDICAL BOARD PREP BIOCHEMISTRY SUPEREXAM
For inquiries visit www.topnotchboardprep.com.ph or email us at topnotchmedicalboardprep@gmail.com
Item QUESTION EXPLANATION AUTHOR TOPNOTCH
# EXAM
522 A 32-year-old man with alcoholism is brought to the MIGUEL MIDTERM 1
emergency department by friends because he has been RAFAEL EXAM - FEB
unable to stand without support and has had "funny eye RAMOS, 2013
movements"; they report that he has been drinking MD (TOP 3
approximately 18 beers daily over the past month and - FEB 2012
has been increasingly confused over the past 5 days. He MED
is awake and confused but is noncombative. His speech BOARDS;
is slurred, and his breath smells of alcohol. His TOPNOTCH
temperature is 37.2 C (99 F), blood pressure is 180/60 MD)
mm Hg, pulse is 110/min, and respirations are 18/min.
Physical examination shows sixth cranial nerve palsy,
horizontal diplopia, strabismus, and an asymmetric
horizontal-gaze evoked nystagmus. Neurologic
examination shows no focal weakness or numbness.
When helped up and told to walk, he has a broad-based,
uncertain gait. When asked how he arrived at the
emergency department, he relates that "I drove to this
place to visit some friends." The most likely cause of
these findings is a deficiency of which of the following?
A) Folic acid
B) Magnesium
C) Vitamin B1 (thiamine)
D) Vitamin B12 (cyanocobalamin)
523 A 3-month-old boy is brought for a well-child MIGUEL MIDTERM 1
examination. He has poor head control. Examination RAFAEL EXAM - FEB
shows generalized hypotonia. The point of maximal RAMOS, 2013
impulse is at the left anterior axillary line. The liver edge MD (TOP 3
is palpated 4 cm below the right costal margin. The - FEB 2012
spleen is not palpable. Which of the following is the most MED
likely diagnosis? BOARDS;
A) Congenital muscular dystrophy TOPNOTCH
B) Glycogen storage disease, type II (Pompe's disease) MD)
C) GM1 gangliosidosis
D) Infant botulism

524 A 1-week-old newborn has had poor feeding, vomiting, MIGUEL MIDTERM 1
and progressive lethargy over the past 4 days. She was RAFAEL EXAM - FEB
born at term; pregnancy, labor, and delivery were RAMOS, 2013
uncomplicated, and she had no congenital anomalies. MD (TOP 3
She is being breast-fed. She has a healthy 2-year-old - FEB 2012
brother; a sister died at 10 days of age after a full-term MED
birth. Examination BOARDS;
shows decreased muscle tone and poor responsiveness; TOPNOTCH
reflexes are normal. Serum bicarbonate level is 8 MD)
mEq/L, pH is 7.15, and plasma ammonia level is 10 times
the upper limit of normal. Which of the following is the
most likely cause?
A) Mitochondrial disorder
B) Mucopolysaccharidoses disorder
C) Organic acid metabolism disorder
D) Renal tubular acidosis
525 A 1-year-old infant came to the physician due to fever. Trisomy 21 >> most common is nondisjunction in 95 MIGUEL MIDTERM 1
On physical examination, the patient has epicanthal % of cases. RAFAEL EXAM - FEB
folds, flat facial profile, macroglossia and presence of RAMOS, 2013
simian crease. Further history revealed that the patient MD (TOP 3
was born from a 48-year-old mother. What is the most - FEB 2012
common cause of this chromosal disorder? MED
A) Nondisjunction BOARDS;
B) Robertsonian translocation TOPNOTCH
C) Mosaicism MD)
D) Chromosomal deletion
526 What enzyme converts Norepinephrine to Epinephrine? MIGUEL MIDTERM 1
A) Dopamine B-hydroxylase RAFAEL EXAM - FEB
B) Dopa decarboxylase RAMOS, 2013
C) Norepinephrine hydroxylase MD (TOP 3
D) Phenylethanolamine -N-Methyl transferase - FEB 2012
MED
BOARDS;
TOPNOTCH
MD)
527 A molecule of palmitate will yield how many ATP? MIGUEL MIDTERM 1
A)12 RAFAEL EXAM - FEB
B) 38 RAMOS, 2013
C) 39 MD (TOP 3
D) 129 - FEB 2012
MED
BOARDS;
TOPNOTCH
MD)

TOPNOTCH MEDICAL BOARD PREP BIOCHEMISTRY SUPEREXAM Page 66 of 92


For inquiries visit www.topnotchboardprep.com.ph or email us at topnotchmedicalboardprep@gmail.com
TOPNOTCH MEDICAL BOARD PREP BIOCHEMISTRY SUPEREXAM
For inquiries visit www.topnotchboardprep.com.ph or email us at topnotchmedicalboardprep@gmail.com
Item QUESTION EXPLANATION AUTHOR TOPNOTCH
# EXAM
528 Typtophan is an amino acid with the largest side chain MIGUEL MIDTERM 1
containing an indole ring. It is a precursor of what RAFAEL EXAM - FEB
substances? RAMOS, 2013
A) Niacin MD (TOP 3
B) Melatonin - FEB 2012
C) Serotonin MED
D) All of the above BOARDS;
TOPNOTCH
MD)
529 Desmolase is the rate limiting enzyme in androgen and desmolase >> cholesterol to pregnenolone MIGUEL MIDTERM 1
steroid synthesis. It is responsible for what reaction? RAFAEL EXAM - FEB
A) Conversion of progesterone to 11 RAMOS, 2013
deoxycorticosterone MD (TOP 3
B) Conversion of pregnenolone to 17 - FEB 2012
hydroxypregnenolone MED
C) Conversion of cholesterol to pregnenolone BOARDS;
D) Conversion of DHEA to Androstenedione TOPNOTCH
MD)
530 A 45-year-old male is hospitalized for treatment of Familial hypercholesterolemia MIGUEL MIDTERM 1
myocardial infarction. His father and a paternal uncle RAFAEL EXAM - FEB
also had heart attacks at an early age. His cholesterol is RAMOS, 2013
elevated, and lipoprotein electrophoresis demonstrates MD (TOP 3
an abnormally high ratio of low- to high-density - FEB 2012
lipoproteins (LDL to HDL). Which of the following is the MED
most likely explanation for this problem? BOARDS;
A) Mutant LDL receptors are deficient in cholesterol TOPNOTCH
uptake MD)
B) Mutant LDL is not responding to high cholesterol
levels
C) Mutant caveolae proteins are not responding to high
cholesterol levels
D) Mutant HDL is not responding to high cholesterol
levels
531 A healthy 28-year-old woman comes to physician for liver will burn lipids to provide energy for MIGUEL MIDTERM 1
advice on losing weight. She is 150 cm ( 4ft 11in) tall and gluconeogenesis RAFAEL EXAM - FEB
56 kg (124 lb): BMI 25kg/m2. Physical examination RAMOS, 2013
reveals no other abnormalities. Physician recommends a MD (TOP 3
diet that will restrict her daily intake by 500 kilocalories. - FEB 2012
Which of following processes is most likely to increase in MED
this patient as a reuslt of following this diet? BOARDS;
A) Adipocyte glucose uptake TOPNOTCH
B) Cerebral keotne utilization MD)
C) Hepatic lipid oxidation
D) Muscle glucose uptake
532 A 4-year-old girl with a history of mental retardation is Marfan-like habitus >> homocystinuria >> MIGUEL MIDTERM 1
brought to the physician because of poor vision. Physical cystathionine synthase deficiency >> accumulated RAFAEL EXAM - FEB
examination shows long limbs, tall stature, and kyphosis. homocysteine is alternatively metabolized to RAMOS, 2013
Funduscopic examination shows bilateral lens methionine by homocysteine methyl transferase and MD (TOP 3
dislocations. Laboratory studies are most likely to show cofactor B12 - FEB 2012
a selective elevation of which of the following amino MED
acids in serum? BOARDS;
A) Cystathionine TOPNOTCH
B) Methionine MD)
C) Cysteine
D) Phenylalanine
533 A 25-year-old man eats 20 raw eggs a day to increase Raw eggs >> biotin deficiency >> biotin cofactor for MIGUEL MIDTERM 1
protein for his bodybuilding routine. This unusual diet carboxylation reactions RAFAEL EXAM - FEB
affects the activity of which of the following enzymes? RAMOS, 2013
A) Acetyl-CoA carboxylase MD (TOP 3
B) Acyl-CoA dehydrogenase - FEB 2012
C) Fatty acid synthase MED
D) Fatty acid thiokinase BOARDS;
TOPNOTCH
MD)
534 A 44 year old alcoholic was brought to the emergency Glucokinase >> glucose storage >> fbs is 73 so you MIGUEL MIDTERM 1
department by his friends. During their usual gathering don’t need storage RAFAEL EXAM - FEB
at the local bar, he passed out and his friends were RAMOS, 2013
unable to revive him. The physician ordered an injection MD (TOP 3
of Thiamine , followed by overnight parenteral glucose. - FEB 2012
The next morning the patient was alert and coherent, MED
serum thiamine was normal, and blood glucose was 73 BOARDS;
mg/dl (4mM). The patient was then sent home. At the TOPNOTCH
time of discharge from the hospital, which of the the MD)
following proteins would have no significant physiologic
activity in this patient?
A) Malate Dehydrogenase
B) Glucokinase
C) alpha-ketoglutarage dehydrogenase
D) GLUT 1 transporter

TOPNOTCH MEDICAL BOARD PREP BIOCHEMISTRY SUPEREXAM Page 67 of 92


For inquiries visit www.topnotchboardprep.com.ph or email us at topnotchmedicalboardprep@gmail.com
TOPNOTCH MEDICAL BOARD PREP BIOCHEMISTRY SUPEREXAM
For inquiries visit www.topnotchboardprep.com.ph or email us at topnotchmedicalboardprep@gmail.com
Item QUESTION EXPLANATION AUTHOR TOPNOTCH
# EXAM
535 Some humans are unable to generate NADPH from MIGUEL MIDTERM 1
glucose metabolism but are able to synthesize ribose RAFAEL EXAM - FEB
from fructose-6-phosphate. Which of the following RAMOS, 2013
enzymes are essential? MD (TOP 3
A) G6PD - FEB 2012
B) Glutathione reductase MED
C) Enolase BOARDS;
D) Transketolase TOPNOTCH
MD)
536 Okazaki fragments are formed on the ______ strands and MIGUEL MIDTERM 1
in the ______ direction. RAFAEL EXAM - FEB
A) Leading; 3’-5’ RAMOS, 2013
B) Lagging; 3’-5’ MD (TOP 3
C) Leading; 5’-3’ - FEB 2012
D) Lagging; 5’-3’ MED
BOARDS;
TOPNOTCH
MD)
537 Two couples present to the ER with severe nausea, a-amanitin from Amanita phylloides >> inhibits RNA MIGUEL MIDTERM 1
vomiting, and diarrhea. One of the patients admits that polymerase II RAFAEL EXAM - FEB
she served salad at the dinner party to which she had RAMOS, 2013
added a few mushrooms that she picked outside. With MD (TOP 3
such information, it is likely that their symptoms are a - FEB 2012
result of inhibition of what molecular event? MED
A) RNA polymerase II BOARDS;
B) RNA polymerase I TOPNOTCH
C) RNA splicing MD)
D) RNA polymerase III
538 Which of the following is not a characteristic of the one amino acid is translated by a codon = MIGUEL MIDTERM 1
genetic code? unambiguous; degenerate = one amino acid may be RAFAEL EXAM - FEB
A) It is a triplet code generated by more than one codon RAMOS, 2013
B) It is nonoverlapping and without punctuation MD (TOP 3
C) It is degenerate in that one codon may code for more - FEB 2012
than one amino acid MED
D) It is universal BOARDS;
TOPNOTCH
MD)
539 A 4-month-old boy is being evaluated for seizures, pyruvate carboxylase >> converts pyruvate to OAA >> MIGUEL MIDTERM 1
psychomotor retardation, and hypotonia. Work-up enters TCA or gluconeogenesis RAFAEL EXAM - FEB
reveals elevated serum levels of lactate alanine, RAMOS, 2013
pyruvate, and ketoacids. Based on the clinical MD (TOP 3
presentation, pyruvate carboxylase activity is measured - FEB 2012
using fibroblasts from a skin biopsy and is found to be MED
markedly decreased. This enzyme is normally used to BOARDS;
directly synthesize which of the following molecules? TOPNOTCH
A) Pyruvate MD)
B) Oxaloacetate
C) Malate
D) Acetyl CoA

540 In glycogen, glucose residues form a straight chain via MIGUEL MIDTERM 1
which of the following? RAFAEL EXAM - FEB
A) a-1,4 linkages RAMOS, 2013
B) a-1.6 linkages MD (TOP 3
C) a-1,4 linkages with glycogenin at the non-reducing - FEB 2012
end MED
D) a-1,6 linkages with UDP-glucose at any end BOARDS;
TOPNOTCH
MD)
541 A 65/M, heavy smoker, presented with a 5 month history In complete obstruction of the bile duct, no ABDELSIM FINAL EXAM -
of vague epigastric pain and nausea; associated with urobilinogen is found in the urine, since bilirubin has AR OMAR FEB 2014
weight loss and pruritus. Lately, patient has also started no access to the intestine, where it can be converted to II, MD (TOP
to experience back pain and appeared jaundiced. On PE, urobilinogen. In this case, the presence of bilirubin 2 - AUG
you note icteric conjunctivae and epigastric tenderness. (conjugated) in the urine without urobilinogen 2013 MED
An abdominal ultrasound reveals a pancreatic head suggests obstructive jaundice, either intrahepatic or BOARDS;
mass. Expected results of biochemical testing include the posthepatic. The commonest cause of obstructive TOPNOTCH
following except: (posthepatic) jaundice are cancer of the head of the MD - 200
A. Increased direct bilirubin pancreas and a gallstone lodged in the common bile QUESTION
B. Present urine bilirubin duct. (Harpers 26e p.284) S) AND
C. Trace fecal urobilinogen MARC
D. Increased total bilirubin DENVER
E. Increased urine urobilinogen TIONGSON,
MD (40
QUESTION
S)

TOPNOTCH MEDICAL BOARD PREP BIOCHEMISTRY SUPEREXAM Page 68 of 92


For inquiries visit www.topnotchboardprep.com.ph or email us at topnotchmedicalboardprep@gmail.com
TOPNOTCH MEDICAL BOARD PREP BIOCHEMISTRY SUPEREXAM
For inquiries visit www.topnotchboardprep.com.ph or email us at topnotchmedicalboardprep@gmail.com
Item QUESTION EXPLANATION AUTHOR TOPNOTCH
# EXAM
542 The above patient was noted to lose 10 kg in 5 months. A - C describe the three mechanisms by which cancer ABDELSIM FINAL EXAM -
On PE, you note obvious wasting. Which of the following causes cachexia. In cachexia, protein catabolism is AR OMAR FEB 2014
is NOT true regarding the biochemical and metabolic INCREASED; this differentiates it from marasmus in II, MD (TOP
changes in cancer cachexia? which protein synthesis is reduced but catabolism is 2 - AUG
A. The tumor releases lactate which is recycled into unaffected (Harpers p.479). 2013 MED
glucose consuming 6 ATPs / glucose recycles. BOARDS;
B. There is upregulation of uncoupling proteins, leading TOPNOTCH
to thermogenesis and oxidation of fuels. MD - 200
C. There is futile cycling of lipids. Hormone sensitive QUESTION
lipase is activated which breaks down triglycerides to S) AND
FFA, which ae then re-esterified at ATP cost. MARC
D. In cachexia, protein synthesis is reduced but DENVER
catabolism is unaffected. TIONGSON,
E. None of the above MD (40
QUESTION
S)
543 Which of the following immunoglobulins opsonizes Main antibody in the secondary response. Also fixes ABDELSIM FINAL EXAM -
bacteria making them easier to phagocytose? complement, which enhances bacterial killing, and AR OMAR FEB 2014
A. IgE neutralizes bacterial toxins and virus (Harper's p.594) II, MD (TOP
B. IgM 2 - AUG
C. IgG 2013 MED
D. IgA BOARDS;
E. IgD TOPNOTCH
MD - 200
QUESTION
S) AND
MARC
DENVER
TIONGSON,
MD (40
QUESTION
S)
544 A 24/M came in for pre-employment examination. On Patient has Marfan's syndrome. ABDELSIM FINAL EXAM -
PE, you note that the patient is over the 95th percentile AR OMAR FEB 2014
for height; and he has a slight scoliosis, skin striae and II, MD (TOP
pectus excavatum. Ausculation reveals a heart murmur. 2 - AUG
You then order a 2D echo which shows an enlarged 2013 MED
aortic root, aortic valve regurgitation and mitral valve BOARDS;
prolapse. You suspect a genetic condition in which there TOPNOTCH
is abnormal production of a glycoprotein, leading to MD - 200
abnormalities in the mechanical stability and elastic QUESTION
properties of connective tissue. Production of which of S) AND
the following glycoproteins is defective? MARC
A. Elastin DENVER
B. Collagen TIONGSON,
C. Fibronectin MD (40
D. Fibrillin QUESTION
E. Laminin S)
545 A neonate is admited to the intensive neonatal care and Haptoglobin was asked during the last board exam. ABDELSIM FINAL EXAM -
incubated due to jaundice. He was born to a 30-year old This protein prevents loss of free hemoglobin into the AR OMAR FEB 2014
G3P2 who is known to be Rh-negative with an Rh- kidney; conserving Fe present in Hg. Levels are II, MD (TOP
positive sexual partner. The mother has not received decreased in hemolytic anemia because of increased 2 - AUG
anti-D prophylaxis during the current pregnancy. You free Hg levels (Hg-Hp complex has a short half life of 2013 MED
are suspecting Rh incompatibility. Which of the 90 mins vs 5 days for normal halptoglobin). Increased BOARDS;
following serum proteins is expected to be decreased in levels are seen in chronic inflammatory states TOPNOTCH
the neonate? (haptoglobin is an acute phase reactant) MD - 200
A. a1-antitrypsin QUESTION
B. C reactive protein S) AND
C. Haptoglobin MARC
D. Ceruloplasmin DENVER
E. a2-macroglobulin TIONGSON,
MD (40
QUESTION
S)
546 A 78/M, car mechanic, presented with a 24 hour history MNEMONIC: CO (carbon monoxide) inhibits CO ABDELSIM FINAL EXAM -
of progressive headache, vomiting and dizziness. On (cytochrome oxidase / complex IV). AR OMAR FEB 2014
examination, he is awake, but appears confused and has II, MD (TOP
generalized weakness. His carboxyhemoglobin level is 2 - AUG
18%. You diagnose the patient as having carbon 2013 MED
monoxide poisoning. Carbon monoxide is dangerous BOARDS;
because it is a potent inhibitor of which complex in the TOPNOTCH
electron transport chain. MD - 200
A. NADH dehydrogenase QUESTION
B. Oxygen dehydrogenase S) AND
C. Succinate dehydrogenase MARC
D. Ubiquinol:ferricytochrome oxidoreductase DENVER
E. Cytochrome oxidase TIONGSON,
MD (40
QUESTION
S)

TOPNOTCH MEDICAL BOARD PREP BIOCHEMISTRY SUPEREXAM Page 69 of 92


For inquiries visit www.topnotchboardprep.com.ph or email us at topnotchmedicalboardprep@gmail.com
TOPNOTCH MEDICAL BOARD PREP BIOCHEMISTRY SUPEREXAM
For inquiries visit www.topnotchboardprep.com.ph or email us at topnotchmedicalboardprep@gmail.com
Item QUESTION EXPLANATION AUTHOR TOPNOTCH
# EXAM
547 A 24/F who came in for dysuria and frequency was Patient has G6P deficiency. G6PD is the key enzyme in ABDELSIM FINAL EXAM -
prescribed cotrimoxazole. Two days later, she develops the oxidative phase of PPP. AR OMAR FEB 2014
nausea and exhaustion and comes back to the ER after II, MD (TOP
passing "tea-colored" urine. She was also noted to be 2 - AUG
jaundiced. Work-ups reveal that the patient has anemia, 2013 MED
azotemia, deranged liver function and unconjugated BOARDS;
hyperbilirubinemia. She is diagnosed as having drug- TOPNOTCH
induced hemolytic anemia. You suspect that the patient MD - 200
has an inherited condition that makes her unable to QUESTION
tolerate oxidative stress leading to red cell hemolysis. S) AND
This condition is due to a deficiency in the key enzyme in MARC
which of the following biochemical reactions: DENVER
A. Glycolysis TIONGSON,
B. Tricarboxylic acid cycle MD (40
C. Gluconeogenesis QUESTION
D. Serial oxidative phosphorylation in the electron S)
transport chain
E. Pentose phosphate pathway
548 Sorbitol dehydrogenase is an enzyme which converts Remember that fructose is used by semen as fuel. ABDELSIM FINAL EXAM -
sorbitol to fructose. Aside from the liver, this enzyme is AR OMAR FEB 2014
also found in significant quantities in the: II, MD (TOP
A. Ovaries 2 - AUG
B. Retina 2013 MED
C. Schwann cells BOARDS;
D. Pancreas TOPNOTCH
E. Seminal vesicles MD - 200
QUESTION
S) AND
MARC
DENVER
TIONGSON,
MD (40
QUESTION
S)
549 Which of the following dyslipoproteinemia is a condition Remember that ApoA is the major lipoprotein of HDL ABDELSIM FINAL EXAM -
apparently beneficial to health and longevity? aka good cholesterol. In hyperalphalipoproteinemia, AR OMAR FEB 2014
A. Familial dysbetalipoproteinemia (ApoA = alpha) there is increased concentration of II, MD (TOP
B. Familial hypertriacylglycerolemia HDL, conferring longevity to the lucky patient. 2 - AUG
C. Familial hyperalphalipoproteinemia 2013 MED
D. Familial type III hyperlipoproteinemia BOARDS;
E. All increase risk of atherosclerosis and coronary TOPNOTCH
disease. MD - 200
QUESTION
S) AND
MARC
DENVER
TIONGSON,
MD (40
QUESTION
S)
550 Collagen is rich in which of the following amino acids: Proline facilitates kinking. ABDELSIM FINAL EXAM -
A. Lysine AR OMAR FEB 2014
B. Serine II, MD (TOP
C. Threonine 2 - AUG
D. Leucine 2013 MED
E. Proline BOARDS;
TOPNOTCH
MD - 200
QUESTION
S) AND
MARC
DENVER
TIONGSON,
MD (40
QUESTION
S)
551 A newborn presenting with vomiting and hypotonia has This is the most common biochemical cause of ABDELSIM FINAL EXAM -
been found to have severe lactic acidosis. Serum analysis congenital lactic acidosis. Because enzyme is deficient, AR OMAR FEB 2014
reveals elevated lactate and alanine. You are considering pyruvate is shunted towards lactate production. II, MD (TOP
a biochemical condition characterized by a deficiency in: Patients present with lactic acidosis and neurologic 2 - AUG
A. Muscle phosphofructokinase disturbance. Alanine increases from conversion of 2013 MED
B. Pyruvate kinase pyruvate to alanine by alanine aminotransferase. BOARDS;
C. Pyruvate carboxylase TOPNOTCH
D. Pyruvate dehydrogenase MD - 200
E. Lactate dehydrogenase QUESTION
S) AND
MARC
DENVER
TIONGSON,
MD (40
QUESTION
S)

TOPNOTCH MEDICAL BOARD PREP BIOCHEMISTRY SUPEREXAM Page 70 of 92


For inquiries visit www.topnotchboardprep.com.ph or email us at topnotchmedicalboardprep@gmail.com
TOPNOTCH MEDICAL BOARD PREP BIOCHEMISTRY SUPEREXAM
For inquiries visit www.topnotchboardprep.com.ph or email us at topnotchmedicalboardprep@gmail.com
Item QUESTION EXPLANATION AUTHOR TOPNOTCH
# EXAM
552 An 8 month old male of Ashkenazi Jewish heritage was Patient has Tay-Sach's disease (hexosaminidase A ABDELSIM FINAL EXAM -
referred to you for developmental delay. Parents noted deficiency) characterized by mental retardation, AR OMAR FEB 2014
that by 7 months, he had poor head control and blindness and muscular weakness. Buzz phrase: II, MD (TOP
remained inattentive to surroundings. Parents also cherry red spot in the macula. 2 - AUG
noted unusual eye movements and staring episodes. The 2013 MED
patient was already referred to an ophthalmologist who BOARDS;
observed a "cherry-red spot" within a pale macula. You TOPNOTCH
suspect that the patient has an inherited deificency in MD - 200
which of the following enzymes: QUESTION
A. Sphingomyelinase S) AND
B. Arylsulfatase A MARC
C. Hexosaminidase A DENVER
D. B-galactosidase TIONGSON,
E. B-glucosidase MD (40
QUESTION
S)
553 Signs and symptoms similar to pellagra are seen in Pellagra is due to niacin deficiency. Remember that ABDELSIM FINAL EXAM -
Hartnup disease due to impaired transport of this amino tryptophan is required for niacin synthesis. AR OMAR FEB 2014
acid: II, MD (TOP
A. Tyrosine 2 - AUG
B. Threonine 2013 MED
C. Tryptophan BOARDS;
D. Valine TOPNOTCH
E. Cystine MD - 200
QUESTION
S) AND
MARC
DENVER
TIONGSON,
MD (40
QUESTION
S)
554 A newborn is seen with blisters and erosions covering Patient has epidermolysis bullosa. ABDELSIM FINAL EXAM -
the entire body. It was noted that handling of the baby AR OMAR FEB 2014
easily causes tearing of the skin. You suspect an II, MD (TOP
inherited condition characterized by mutations affecting 2 - AUG
a protein which forms delicate fibrils that anchor the 2013 MED
basal lamina to the dermis. Which protein is abnormally BOARDS;
produced in this condition? TOPNOTCH
A. Type I collagen MD - 200
B. Type II collagen QUESTION
C. Type III collagen S) AND
D. Type IV collagen MARC
E. Type VII collagen DENVER
TIONGSON,
MD (40
QUESTION
S)
555 A 28/F, whose partner was recently diagnosed with HIV- ABDELSIM FINAL EXAM -
AIDS, comes in and requests to be tested for the AR OMAR FEB 2014
presence of HIV. To confirm an HIV infection, you would II, MD (TOP
need to order a test which employs: 2 - AUG
A. Enzyme linked immunoassay 2013 MED
B. Genomic outhern blot BOARDS;
C. Northern blot TOPNOTCH
D. Western blot MD - 200
E. VNTR analysis QUESTION
S) AND
MARC
DENVER
TIONGSON,
MD (40
QUESTION
S)
556 A 20/M comes in for knee pain and swelling. On PE, you Patient has alkaptonuria due to deficiency in ABDELSIM FINAL EXAM -
noted dark spots in his sclera. On probing, the patient homogentisate oxidase. Homogentisate is a product of AR OMAR FEB 2014
reports that when he leaves his urine standing, it turns tyrosine catabolism. Oxidation of homogentisate leads II, MD (TOP
black. You suspect an inherited condition characterized to darkening of the urine. Oxidation of homogentisate 2 - AUG
by a deficiency in an enzyme necessary for the to benzoquinone acetate poiymerizes and binds to 2013 MED
catabolism of this amino acid. connective tissue, leading to arthritis and BOARDS;
A. Tyrosine pigmentation (ochronosis). TOPNOTCH
B. Threonine MD - 200
C. Leucine QUESTION
D. Serine S) AND
E. Glycine MARC
DENVER
TIONGSON,
MD (40
QUESTION
S)

TOPNOTCH MEDICAL BOARD PREP BIOCHEMISTRY SUPEREXAM Page 71 of 92


For inquiries visit www.topnotchboardprep.com.ph or email us at topnotchmedicalboardprep@gmail.com
TOPNOTCH MEDICAL BOARD PREP BIOCHEMISTRY SUPEREXAM
For inquiries visit www.topnotchboardprep.com.ph or email us at topnotchmedicalboardprep@gmail.com
Item QUESTION EXPLANATION AUTHOR TOPNOTCH
# EXAM
557 A 28/M, with a history of recurrent nephrolithiasis, Cystinuria ABDELSIM FINAL EXAM -
comes in due to severe intermittent right flank pain and No transporter for COAL AR OMAR FEB 2014
hematuria. On examination, you note CVA tenderness on Cystine, Ornithine, Arginine, Lysine II, MD (TOP
the right. Urinalysis reveals hexagonal crystals upon 2 - AUG
cooling of acidified urine sediment. The patient tells you 2013 MED
that a physician who saw him for a previous bout of BOARDS;
nephrolithiasis was considering an inherited condition. TOPNOTCH
You suspect that the patient may have a condition MD - 200
characterized by impaired renal tubular absorption of QUESTION
which of the following amino acids: S) AND
A. Ornithine MARC
B. Arginine DENVER
C. Lysine TIONGSON,
D. Cystine MD (40
E. All of the above QUESTION
S)
558 Too much of a good thing can be bad for you. Excessive ABDELSIM FINAL EXAM -
ingestion of this vitamin for example can cause headache AR OMAR FEB 2014
and nasuea related to increased CSF, hepatomegaly, II, MD (TOP
thickening of long bones and hypercalcemia, and 2 - AUG
excessive dryness and desquamation of skin. 2013 MED
A. Vitamin A BOARDS;
B. Vitamin D TOPNOTCH
C. Vitamin E MD - 200
D. Vitamin K QUESTION
E. Niacin S) AND
MARC
DENVER
TIONGSON,
MD (40
QUESTION
S)
559 Activation of erythrocyte transaminases is an accepted The question could be rephrased as such: 'which of ABDELSIM FINAL EXAM -
index of nutritionals status of this vitamin: the following vitamins is a co-factor in transaminase AR OMAR FEB 2014
A. Thiamine reactions?' Answer would be pyridoxine. II, MD (TOP
B. Pyridoxine 2 - AUG
C. Riboflavin 2013 MED
D. Biotin BOARDS;
E. Folic acid TOPNOTCH
MD - 200
QUESTION
S) AND
MARC
DENVER
TIONGSON,
MD (40
QUESTION
S)
560 It has been common practice to prescribe multivitamins ABDELSIM FINAL EXAM -
for patients with peripheral neuropathy, complaining of AR OMAR FEB 2014
"pangangalay." However, the use of multivitamins for II, MD (TOP
this indication is not evidence-based. Moreover, 2 - AUG
excessive intake of this vitamin is commonly associated 2013 MED
with sensory neuropathy. BOARDS;
A. Thiamine TOPNOTCH
B. Pyridoxine MD - 200
C. Riboflavin QUESTION
D. Biotin S) AND
E. Folic acid MARC
DENVER
TIONGSON,
MD (40
QUESTION
S)
561 1. The main source of Estrogen in a 27 year old Estriol is only produced in significant amounts during BLAKE FINAL EXAM -
primigravid patient is from: pregnancy as it is made by the placenta from 16-OH WARREN FEB 2014
A. placenta DHEAS, an androgen steroid made in the fetal liver ANG, MD
B. Maternal adrenals and adrenal glands (TOP 1 -
C. fetus AUG 2013
D. Peripheral aromatization of DHEA MED
BOARDS;
TOPNOTCH
MD)
562 Hemoglobin is a functional protein belonging to what Quaternary structures are composed of at least 2 BLAKE FINAL EXAM -
structural order? polypeptide chains. Hemoglobin is composed of 2 WARREN FEB 2014
A. primary alpha and 2 beta chains and is therefore quaternary in ANG, MD
B. secondary nature. (TOP 1 -
C. tertiary AUG 2013
D. quaternary MED
BOARDS;
TOPNOTCH
MD)

TOPNOTCH MEDICAL BOARD PREP BIOCHEMISTRY SUPEREXAM Page 72 of 92


For inquiries visit www.topnotchboardprep.com.ph or email us at topnotchmedicalboardprep@gmail.com
TOPNOTCH MEDICAL BOARD PREP BIOCHEMISTRY SUPEREXAM
For inquiries visit www.topnotchboardprep.com.ph or email us at topnotchmedicalboardprep@gmail.com
Item QUESTION EXPLANATION AUTHOR TOPNOTCH
# EXAM
563 This amino acid is considered the most basic among the BLAKE FINAL EXAM -
group: WARREN FEB 2014
A. Histidine ANG, MD
B. Lysine (TOP 1 -
C. Arginine AUG 2013
D. Glycine MED
BOARDS;
TOPNOTCH
MD)
564 The major inhibitory neurotransmitter of the Brain is Glutamate, the major stimulant in the CNS is BLAKE FINAL EXAM -
derived from a decarboxylation reaction requiring consequently carboxylated by L-glutamic WARREN FEB 2014
pyridoxal phosphate. The amino acid decarboxylase into GABA, the major inhibitory ANG, MD
A. Glutamine transmitter of the CNS (TOP 1 -
B. Glycine AUG 2013
C. Glutamate MED
D. Lysine. BOARDS;
TOPNOTCH
MD)
565 In alpha thalassemia (-/-, -/-) the hemoglobin formed In Alpha thalassemia, accumulation of 4 beta chains is BLAKE FINAL EXAM -
composed of B4 (beta4) is referred to as : referred to as HbH, whereas 4 gamma chains is WARREN FEB 2014
A. HbH referred to as Hb Bart’s. Both can exist in this ANG, MD
B. HbS condition. (TOP 1 -
C. Hb Barts AUG 2013
D. Hb C MED
BOARDS;
TOPNOTCH
MD)
566 True of Noncompetitive inhibition In noncompetitive inhibition, formation of the product BLAKE FINAL EXAM -
a. Formation of product continues continues. Allosteric inhibition causes the Vmax to WARREN FEB 2014
b. The Km of the enzyme targeted decreases decrease. However, Km remains unchanged. ANG, MD
c. The Km of the enzyme targeted increases (TOP 1 -
d. The Vmax of the enzyme targeted remains unchanged AUG 2013
MED
BOARDS;
TOPNOTCH
MD)
567 Gibbs free energy predicts the spontaneity of a reaction The formula of Gibbs free energy is enthalpy minus BLAKE FINAL EXAM -
that is about to take place. The factors to be considered temperature x entropy WARREN FEB 2014
are the following except: ANG, MD
a. enthalpy (TOP 1 -
b. temperature AUG 2013
c. entropy MED
d. none of the above BOARDS;
TOPNOTCH
MD)
568 A Gibbs free energy change of 0 means that the reaction BLAKE FINAL EXAM -
is: WARREN FEB 2014
a. Spontaneously occurring Zero Gibbs free energy change means that the reaction ANG, MD
b. Non-spontaneous that requires coupling with an is in equilibrium (TOP 1 -
exothermic reaction AUG 2013
c. In equilibrium MED
d. No significance BOARDS;
TOPNOTCH
MD)
569 beta oxidation of palmitic acid into acetyl coA residues BLAKE FINAL EXAM -
occur in which site of the cell? WARREN FEB 2014
a. cytosol ANG, MD
b. peroxisome (TOP 1 -
c. mitochondrial matrix AUG 2013
d. both a and c MED
BOARDS;
TOPNOTCH
MD)

570 terminal digestion of carbohydrates into Terminal digestion occurs in the brush borders due to BLAKE FINAL EXAM -
monosaccharides occur in which site of the the presence of disaccharidases WARREN FEB 2014
gastrointestinal tract? ANG, MD
a. brush border of the intestines (TOP 1 -
b. duodenal lumen when acted upon by pancreatic AUG 2013
enzymes MED
c. enterocytes BOARDS;
d. in the mouth by the enzyme ptyalin TOPNOTCH
MD)
571 which of the following is true of phosphatidylinositol? PI is synthesized from free inositol and CDP-DAG. It is BLAKE FINAL EXAM -
a. synthesized from free inositol and CDP-diacylglycerol an unusual phospholipid in that it contains stearic WARREN FEB 2014
b. it is an unusual phospholipid in that it contains acid on C-1 and Arachidonic acid on C-2. It serves as a ANG, MD
arachidonic acid in Carbon 1 and stearic acid on carbon 2 reservoir for prostaglandin synthesis. (TOP 1 -
c. the arachidonic acid present is bound and unaccessible AUG 2013
for prostaglandin synthesis MED
d. all of the above BOARDS;
TOPNOTCH
MD)

TOPNOTCH MEDICAL BOARD PREP BIOCHEMISTRY SUPEREXAM Page 73 of 92


For inquiries visit www.topnotchboardprep.com.ph or email us at topnotchmedicalboardprep@gmail.com
TOPNOTCH MEDICAL BOARD PREP BIOCHEMISTRY SUPEREXAM
For inquiries visit www.topnotchboardprep.com.ph or email us at topnotchmedicalboardprep@gmail.com
Item QUESTION EXPLANATION AUTHOR TOPNOTCH
# EXAM
572 which component of phosphatidylinositol degradation of The products IP3 and DAG mediate the mobilization of BLAKE FINAL EXAM -
the secondary messenger complex is responsible for intracellular Ca and the activation of Protein kinase C, WARREN FEB 2014
Protein kinase C activation? respectively. ANG, MD
a. Diacylglycerol (TOP 1 -
b. IP3 AUG 2013
c. PHosphatidylinositol 4,5 diphosphate MED
d. calcium BOARDS;
TOPNOTCH
MD)
573 the irreversible inhibition of cyclooxygenase enzyme Acetyl salicylate causes acetylation of the COX enzyme BLAKE FINAL EXAM -
system in platelets by aspirin is by virtue of: rendering it ineffective permanently. WARREN FEB 2014
a. acetylation ANG, MD
b. hydroxylation (TOP 1 -
c. sulfation AUG 2013
d. conjugation MED
BOARDS;
TOPNOTCH
MD)
574 which among the following is considered a primary bile Two primary bile acids are: chenodeoxycholic and BLAKE FINAL EXAM -
acid cholic acid. WARREN FEB 2014
a. lithocolic acid ANG, MD
b. cholic acid (TOP 1 -
c. deoxycholic acid AUG 2013
d. ursodeoxycholic acid MED
BOARDS;
TOPNOTCH
MD)
575 the most common enzyme deficiency in Congenital 21-hydroxylase deficiency BLAKE FINAL EXAM -
Adrenal hyperplasia is: WARREN FEB 2014
a. 11-hydroxylase ANG, MD
b. 17- hydroxylase (TOP 1 -
c. 21-hydroxylase AUG 2013
d. Aromatase MED
BOARDS;
TOPNOTCH
MD)
576 Deficiency of this vitamin causes hemolytic anemia: Vitamin E deficiency causes hemolytic anemia; on the BLAKE FINAL EXAM -
a. Vitamin A other hand, excess Vitamin K may also cause WARREN FEB 2014
b. vitamin D hemolysis. ANG, MD
c. vitamin E (TOP 1 -
d. vitamin K AUG 2013
MED
BOARDS;
TOPNOTCH
MD)
577 Warfarin inhibits vitamin K dependent clotting factors Vitamin K Epoxide reductase inhibition is the main BLAKE FINAL EXAM -
by direct inhibition of: mechanism of Warfarin WARREN FEB 2014
a. vitamin K epoxide reductase ANG, MD
b. glutamyl carboxylase (TOP 1 -
c. Lipid absorption of vitamin K in the GI tract and the AUG 2013
liver MED
d. protein synthesis of factors 2,7,9,10 BOARDS;
TOPNOTCH
MD)
578 a disorder of purine metabolism characterized by Lysch Nyhan Syndrome is due to the deficiency of the BLAKE FINAL EXAM -
hyperuricemia, self-mutilation, and neurologic features Purine Salvage Mechanism caused by HGPRT WARREN FEB 2014
with involuntary movements is mainly due to a defect of deficiency. ANG, MD
what enzyme? (TOP 1 -
a. glucose 6-phosphatase AUG 2013
b. HGPRT MED
c. Adenine phosphoribosyl transferase BOARDS;
d. ribonucleotide reductase TOPNOTCH
MD)

579 Adenosine deaminase deficiency causes this disorder BLAKE FINAL EXAM -
among children characterized by both Tcell and Bcell WARREN FEB 2014
dysfunction: ANG, MD
a. Severe combined immunodeficiency (TOP 1 -
b. combined variable immunodeficiency AUG 2013
c. DiGeorge syndrome MED
d. X-linked agammaglobulinemia BOARDS;
TOPNOTCH
MD)
580 which among the following amino acids does not Valine does not contribute any carbon to the purine BLAKE FINAL EXAM -
contribute directly to the synthesis of a purine ring? ring. WARREN FEB 2014
a. glycine ANG, MD
b. glutamine (TOP 1 -
c. aspartate AUG 2013
d. valine MED
BOARDS;
TOPNOTCH
MD)

TOPNOTCH MEDICAL BOARD PREP BIOCHEMISTRY SUPEREXAM Page 74 of 92


For inquiries visit www.topnotchboardprep.com.ph or email us at topnotchmedicalboardprep@gmail.com
TOPNOTCH MEDICAL BOARD PREP BIOCHEMISTRY SUPEREXAM
For inquiries visit www.topnotchboardprep.com.ph or email us at topnotchmedicalboardprep@gmail.com
Item QUESTION EXPLANATION AUTHOR TOPNOTCH
# EXAM
581 Which compounds act as inhibitors for Cytochrome All compounds are inhibitors of cytochrome oxidase. TIMOTHY MIDTERM 1
Oxidase (Complex IV) of the Electron Transport Chain? TANG LEE EXAM - FEB
A. CO SAY, MD 2014
B. CN (TOP 4 -
C. H2S AUG 2013
D. All of the Above MED
E. None of the Above BOARDS;
TOPNOTCH
MD)
582 Which irreversible enzymes in glycolysis is coupled to Although all 3 enzymes catalyzed irreversible steps in TIMOTHY MIDTERM 1
the release of energy and requires different enzymes to glycolysis, only pyruvate kinase is exergonic or is TANG LEE EXAM - FEB
catalyzed the reverse reactions in the pathway of coupled to the release of energy. Hexokinase and PFK SAY, MD 2014
gluconeogenesis? are in the energy investment stage of the Glycolysis. (TOP 4 -
A. Hexokinase AUG 2013
B. Phosphofructokinase MED
C. Pyruvate Kinase BOARDS;
D. All of the above TOPNOTCH
E. None of the above MD)

583 The reaction rates of adding substance A in a solution Remember the equation y=mx+b where m is the slope TIMOTHY MIDTERM 1
with known enzyme kinetics can be exemplified using of the line and b is the y-intercept (ordinate). TANG LEE EXAM - FEB
the Lineweaver-Burk Plot (1/Vi=(Km/Vmax)(1/S) + Increasing the ordinate will result in a decrease in SAY, MD 2014
1/Vmax). When adding substance A, the ordinate of the Vmax (since we get the reciprocal of Vmax as the (TOP 4 -
Lineweaver-Burk plot increases while the abscissa ordinate). A decrease in Vmax while Km (-1/Km is the AUG 2013
remains the same. Substance A can then be classified as abscissa or x-intercept) remains the same is MED
a? characteristic of Non-competitive inhibition. BOARDS;
A. Complete Agonist TOPNOTCH
B. Partial Agonist MD)
C. Competitive Antagonist
D. Non-competitive Antagonist
E. Uncompetitive Antagonist

584 In the intestinal brush border which monosaccaharide All monosaccharides can use the GLUT-5 transporter TIMOTHY MIDTERM 1
can only used the GLUT-5 (facilitated diffusion) and not while only aldoses can use the SGLT transporter. TANG LEE EXAM - FEB
the SGLT transporter (active co-transport)? Fructose is the only ketose sugar in the group. SAY, MD 2014
A. Glucose Glucose=Dextrose. Glucose, galactose and mannose (TOP 4 -
B. Mannose are aldoses AUG 2013
C. Galactose MED
D. Dextrose BOARDS;
E. Fructose TOPNOTCH
MD)
585 Aerobic glycolysis in the heart produces a net total of Aerobic glycolysis produces either 36 or 38 ATP TIMOTHY MIDTERM 1
how many ATP molecules for each molecule of glucose? molecules for each molecule of glucose depending on TANG LEE EXAM - FEB
A. 30 the shuttle used. The liver, kidney, and the heart uses SAY, MD 2014
B. 32 the malate aspartate shuttle so each NADH produce in (TOP 4 -
C. 34 glycolysis in the cytoplasm nets 3 ATP. The skeletal AUG 2013
D. 36 muscle and the brain uses the gycerol phosphate MED
E. 38 shuttle so each NADH only produces 2 ATP. BOARDS;
TOPNOTCH
MD)
586 A patient always experienced pain and cramps after The signs and symptoms are typical of McArdle's TIMOTHY MIDTERM 1
exercise. Initial work-up shows normal ECG disease (Cramps+Myoglobinuria (brownish TANG LEE EXAM - FEB
tracing,electrolytes were within normal limits and discoloration) with no lactate acidosis) SAY, MD 2014
normal lactate levels. The patient's urine had a brownish (TOP 4 -
color. The patient likely has which glycogen storage AUG 2013
disease? MED
A. Von Gierke's BOARDS;
B. Pompe's TOPNOTCH
C. Cori's MD)
D. McArdle's
E. Andersen's
587 Which of the following statements are TRUE for lipids? The melting temperature is inversely proportional to TIMOTHY MIDTERM 1
A. All lipids are substances that are hydrophobic. the number of double bonds. Cholesterol is not used to TANG LEE EXAM - FEB
B. The melting temperature of fatty acids is directly obtain energy, only fatty acids and glycerol. Proteins, SAY, MD 2014
proportional to the number of double bonds. not lipids are the main components of enzymes. Fatty (TOP 4 -
C. Lipids such as cholesterol can be used by the body to acid synthesis occur in the cytoplasm but β-oxidation AUG 2013
obtain energy through β-oxidation. occurs in the mitochondria. MED
D. Lipids are the main component of cellular enzymes BOARDS;
needed for metabolism. TOPNOTCH
E. Fatty acid metabolism occurs in the cytoplasm. MD)

588 In the formation of oleic acid, aside from enzymes The end of fatty acid synthesis in the cytoplasm is the TIMOTHY MIDTERM 1
located in the cytoplasm which organelle is needed for formation of the 14-C palmitic acid. Further TANG LEE EXAM - FEB
its synthesis? elongation can be done in both the SER and SAY, MD 2014
A. Peroxisomes mitochondria but insertion of double bonds, such as in (TOP 4 -
B. Lysosomes Oleic acid can only be done in the ER. Peroxisomes are AUG 2013
C. Endoplasmic Reticulum needed to oxidize fatty acids with double bonds. MED
D. Golgi Bodies BOARDS;
E. Mitochondria TOPNOTCH
MD)

TOPNOTCH MEDICAL BOARD PREP BIOCHEMISTRY SUPEREXAM Page 75 of 92


For inquiries visit www.topnotchboardprep.com.ph or email us at topnotchmedicalboardprep@gmail.com
TOPNOTCH MEDICAL BOARD PREP BIOCHEMISTRY SUPEREXAM
For inquiries visit www.topnotchboardprep.com.ph or email us at topnotchmedicalboardprep@gmail.com
Item QUESTION EXPLANATION AUTHOR TOPNOTCH
# EXAM
589 Which of the following is not an essential amino acid? PVT TIM HALL, always ARGues, never TYRes. TIMOTHY MIDTERM 1
A. Histidine Asparagine can be synthesized from aspartate. TANG LEE EXAM - FEB
B. Threonine SAY, MD 2014
C. Leucine (TOP 4 -
D. Asparagine AUG 2013
E. Methionine MED
BOARDS;
TOPNOTCH
MD)
590 Denaturation is the disruption of a protein's structure Altering pH whether increasing or decreasing it TIMOTHY MIDTERM 1
(secondary structure and above). This could be disrupts the protein structure. However only heating TANG LEE EXAM - FEB
accomplished by any of these mechanisms EXCEPT? will result in denaturation, cooling would decrease the SAY, MD 2014
A. Decreasing pH velocity of the catalyzed reactions but the structure of (TOP 4 -
B. Increasing pH the protein is unchanged. AUG 2013
C. Cooling MED
D. Addition of heavy metals BOARDS;
E. Addition of detergents TOPNOTCH
MD)
591 What amino acid is the precursor of the main inhibitory The main inhibitory neurotransmitter of the spinal TIMOTHY MIDTERM 1
neurotransmitter of the spinal cord? cord is glycine. Serine is necessary for its synthesis. TANG LEE EXAM - FEB
A. Serine SAY, MD 2014
B. Glycine (TOP 4 -
C. Glutamate AUG 2013
D. Glutamine MED
E. Tryptophan BOARDS;
TOPNOTCH
MD)
592 Maple syrup disease is a disorder characterized by Lysine is not a branched chain amino acid. TIMOTHY MIDTERM 1
blocked degradation of branched chain amino acids. It is TANG LEE EXAM - FEB
therefore important to limit the following amino acids SAY, MD 2014
EXCEPT? (TOP 4 -
A. Lysine AUG 2013
B. Leucine MED
C. Valine BOARDS;
D. Isoleucine TOPNOTCH
E. None, all needs to be limited MD)

593 A 5-yr old boy has distinctive coarseness in his facial Enzyme defects: TIMOTHY MIDTERM 1
features, including a prominent forehead, a nose with a Hunter's Syndrome: Iduronate Sulfatase TANG LEE EXAM - FEB
flattened bridge, and an enlarged tongue. There are Hurler's Syndrome and Scheie's Syndrome: α- SAY, MD 2014
developmental delays and nervous system problems. iduronidase (TOP 4 -
Biochemical testing reveals there is a defect in the Morquio's Syndrome: Galactose-6-sulfatase AUG 2013
degradation of dermatan sulfate and keratan sulfate. Sly Syndrome: β-Glucuronidase MED
Genetic testing shows there is a polymorphism in the BOARDS;
gene encoding for iduronate-2-sulfatase. The patient TOPNOTCH
most likely has? MD)
A. Hunter's Syndrome
B. Hurler's Syndrome
C. Morquio's Syndrome
D. Scheie's Syndrome
E. Sly Syndrome
594 A 25 yr old obese person had just ate one cup of fried The person is in the well-fed state so we know that the TIMOTHY MIDTERM 1
rice, 2 pieces of fried chicken and a mixed berry shake. metabolic pathways active are glycolysis and TANG LEE EXAM - FEB
Which enzymes are dephosphorylated and active? glycogenesis and not gluconeogenesis and SAY, MD 2014
A. Hexokinase glycogenolysis which are active in the fasting state. (TOP 4 -
B. Phosphofructokinase-1 Hexokinase is not the rate-limiting step of glycolysis AUG 2013
C. Glucose-6-phosphatase and is not subject to phosphorylation and MED
D. Fructose 1,6-bisphophatase dephosphorylation so the answer is PFK-1, the rate BOARDS;
E. Glycogen phosphorylase limiting step of glycolysis. TOPNOTCH
MD)

595 A group of four cavers were rescued today after Glucose is the main energy fuel in the well-fed state. TIMOTHY MIDTERM 1
spending five days lost underground with no food. Glycogen stores are only sufficient for 18-24 hrs. TANG LEE EXAM - FEB
Which of the following metabolites were used for fuel by Prolonged fasting will make the body used ketones as SAY, MD 2014
their bodies just before they were rescued? its main energy fuel (TOP 4 -
A. Glucose AUG 2013
B. Glycogen MED
C. Fatty Acids BOARDS;
D. Ketone bodies TOPNOTCH
E. Glycerol MD)

TOPNOTCH MEDICAL BOARD PREP BIOCHEMISTRY SUPEREXAM Page 76 of 92


For inquiries visit www.topnotchboardprep.com.ph or email us at topnotchmedicalboardprep@gmail.com
TOPNOTCH MEDICAL BOARD PREP BIOCHEMISTRY SUPEREXAM
For inquiries visit www.topnotchboardprep.com.ph or email us at topnotchmedicalboardprep@gmail.com
Item QUESTION EXPLANATION AUTHOR TOPNOTCH
# EXAM
596 An athlete who has extensive training of 8 hours per day No matter the physical activity, the basal metabolic TIMOTHY MIDTERM 1
has a higher energy requirement than people who have rate accounts for at least 50% of energy requirements. TANG LEE EXAM - FEB
sedentary lifestyles. Most of the energy requirement is Most of the basal metabolic rate is used for the SAY, MD 2014
used for? maintenance of cellular functions such as the (TOP 4 -
A. Metabolism in skeletal muscles during physical maintenace of the cellular electrolyte composition by AUG 2013
activity for rapid and sustain effort the Na/K pump. MED
B. Metabolism in cardiac and respiratory cells to BOARDS;
maintain adequate oxygen and nutrient delivery to all TOPNOTCH
cells of the body MD)
C. Metabolism in nerve cells needed to conduct
electrical impulses and control bodily functions
D. Metabolism in the GIT to extract nutrients needed to
sustain the energy requirements
E. Metabolism in cells to maintain homeostasis such as
the maintenance of the Na/K pump

597 Which precursors of nucleotide synthesis is necessary All are needed for either purine or pyrimidine TIMOTHY MIDTERM 1
for both purine and pyrimidine synthesis? synthesis. For purine synthesis, there is the addition TANG LEE EXAM - FEB
A. Glutamine of glycine and tetrahydrofolate as necessary SAY, MD 2014
B. Aspartic acid components. (TOP 4 -
C. Carbon dioxide AUG 2013
D. All of the above MED
E. None of the above BOARDS;
TOPNOTCH
MD)
598 You are task to elucidate the identity of a heriditary Mitochondrial diseases are always maternally derived TIMOTHY MIDTERM 1
disease X. A father inflicted with the disease does not since the mitochondria of a child all comes from the TANG LEE EXAM - FEB
transmit it to any of his children. An afflicted mother mother. Remember the only contribution of the sperm SAY, MD 2014
transmit it to all her offsprings. The disease most likely is its haploid DNA. (TOP 4 -
shows this characteristic? AUG 2013
A. X-linked MED
B. Autosomal Dominant BOARDS;
C. Autosomal Recessive TOPNOTCH
D. Mitochondrial MD)
E. Variable Penetrance

599 What is known as the Central Dogma of Molecular The central dogma involves DNA replication to form TIMOTHY MIDTERM 1
Biology? copies of DNA, DNA transcription to mRNA. mRNA TANG LEE EXAM - FEB
A. The Genetic Code has been conserved from the early serves as a template for the translation of proteins. SAY, MD 2014
stages of evolutions with only slight changes in the (TOP 4 -
manner of which the code is translated. AUG 2013
B. DNA is replicated semi-conservatively with the MED
daughter DNA strands containing half of the original BOARDS;
strands. TOPNOTCH
C. The Genetic Code is specific, universal, MD)
degenerate/redundant and non-overlapping.
D. Southern blot is used to identify DNA, Northern blot
to identify RNA and Western blot to identify proteins
E. DNA is replicated in the nucleus, transcribed to
mRNA, and then translated to proteins in the ribosomes.
600 In biotechnology, restriction enzymes are enzymes that Restriction enzymes target palindromic sequences or TIMOTHY MIDTERM 1
cleave certain locations in the DNA sequence that helps those sequence that are mirror images of each other .. TANG LEE EXAM - FEB
identify the presence of genetic polymorphisms. They Ex SAY, MD 2014
preferentially target which sites in the DNA molecule? AAAAGTCGACAAAAA (TOP 4 -
A. Sites where there is a high GC content TTTTCAGCTGTTTTT AUG 2013
B. Sites where there is a palindromic sequence MED
C. Sites that have altered DNA bases such as thymidine BOARDS;
dimers TOPNOTCH
D. Sites that have trinucleotide repeats (CAGs, etc.) MD)
E. Sites that are "trash" or those that do not encode
proteins
601 Water is the universal solvent. This is because water: Having a high dielectric constant permits water to RACHELLE FINAL EXAM -
A. is an excellent nucleophile decrease force of attraction between charged and MENDOZA, FEB 2013
B. is amphoteric polar species. Water as a dipole means it has electric MD (TOP 9
C. has high dielectric constant charge distributed asymetrically about its structure. - AUG 2012
D. is a dipole Water as an excellent nucleophile means it can cleave MED
E. All of the above amide, glycoside or ester bonds. Water as an BOARDS;
amphoteric means it acts both as an acid and a base. TOPNOTCH
MD)
602 The following are electron transport chain uncouplers, Uncouplers are agents that allow dissipation of RACHELLE FINAL EXAM -
EXCEPT: hydrogen ions across the inner mitochondrial MENDOZA, FEB 2013
A. Aspirin membrane without production of ATP. These MD (TOP 9
B. Thermogenin substances include aspirin, 2,4 DNP and thermogenin - AUG 2012
C. 2, 4-dinitrophenol (brown fat) MED
D. Oligomycin BOARDS;
E. None of the above TOPNOTCH
MD)

TOPNOTCH MEDICAL BOARD PREP BIOCHEMISTRY SUPEREXAM Page 77 of 92


For inquiries visit www.topnotchboardprep.com.ph or email us at topnotchmedicalboardprep@gmail.com
TOPNOTCH MEDICAL BOARD PREP BIOCHEMISTRY SUPEREXAM
For inquiries visit www.topnotchboardprep.com.ph or email us at topnotchmedicalboardprep@gmail.com
Item QUESTION EXPLANATION AUTHOR TOPNOTCH
# EXAM
603 Enzyme 1 has Km (michaelis constant) of 4, while A greater Km value indicates lesser affinity of the RACHELLE FINAL EXAM -
Enzyme 2 has Km of 2. Assuming that Enzymes 1 and 2 enzyme for a given substrate. Km is substrate MENDOZA, FEB 2013
act on the same substrate, which of the following is concentration at which the speed of reaction is half MD (TOP 9
correct? the maximal velocity (Vmax/2) - AUG 2012
A. Enzyme 1 has greater substrate affinity than Enzyme MED
2 BOARDS;
B. Enzyme 2 has greater substrate affinity than Enzyme 1 TOPNOTCH
C. Enzyme 1 and 2 have the same substrate affinity MD)
D. Affinity cannot be determined by Km
E. None of the above
604 After a high carbohydrate meal, blood glucose level Liver, kidney and heart utilizes the malate-aspartate RACHELLE FINAL EXAM -
increases. In the brain, how many ATPs will be shuttle, thereby producing 38 ATPs over-all (NADH MENDOZA, FEB 2013
produced from 1 molecule of glucose after complete will be converted to 3 ATPs). Skeletal muscle and MD (TOP 9
oxidative glycolysis? brain utilizes the glycerol-phosphate shuttle, thereby - AUG 2012
A. 32 producing only 36 ATPs (NADH will be converted to 2 MED
B. 34 ATPs only). BOARDS;
C. 36 TOPNOTCH
D. 38 MD)
E. Cannot be determined

605 The liver cannot utilize ketones as a source of energy, Thiophorase (also known as succinyl coa - acetoacetyl RACHELLE FINAL EXAM -
due to the absence of which enzyme/s? coa transferase) is needed for tissues to utilize MENDOZA, FEB 2013
A. Thiophorase ketones as source of energy. Liver lacks this enzyme. MD (TOP 9
B. Succinyl CoA - Acetoacteyl CoA transferase - AUG 2012
C. Acetoacetyl - acyl transferase MED
D. A and B BOARDS;
E. B and C TOPNOTCH
MD)

606 Which of the following apoproteins attached to Apo C-II activates lipoprotein lipase. Apo B serves as a RACHELLE FINAL EXAM -
lipoproteins are responsible for activation of lipoprotein receptor ligand for LDL (B 100) or Chylomicrons (B MENDOZA, FEB 2013
lipase? 48). Apo E facilitates uptake of lipoproteins by the MD (TOP 9
A. Apo C-II liver. - AUG 2012
B. Apo B-100 MED
C. Apo B-48 BOARDS;
D. Apo E TOPNOTCH
E. B and C MD)

607 A preterm newborn male was observed to be dyspneic Dipalmitoylphosphatidycholine (aka RACHELLE FINAL EXAM -
after 12 hours of life during your NICU rotation. You dipalmitoylecithin) is the major lung surfactant. MENDOZA, FEB 2013
noted chest indrawing, alar flaring and cyanosis. As the Inadequate levels lead to respiratory distress MD (TOP 9
intern-in-charge, you suggested chest x-ray to be done, syndrome in the newborn, more commonly observed - AUG 2012
revealing "white out lungs." Which of the following among preterms. MED
is/are most probably decreased in this patient? BOARDS;
A. Phosphatidylcholine TOPNOTCH
B. Dipalmitoylphosphatidylcholine MD)
C. Dipalmitoyllecithin
D. A and B
E. B and C

608 Which of the following amino acids lacks an amino group Proline has an imino group in its structure. RACHELLE FINAL EXAM -
in its structure? MENDOZA, FEB 2013
A. Histidine MD (TOP 9
B. Arginine - AUG 2012
C. Tryptophan MED
D. Proline BOARDS;
E. None of the above TOPNOTCH
MD)
609 The following are synthesized from tryptophan, EXCEPT: Tryptophan is the precursor for niacin, serotonin and RACHELLE FINAL EXAM -
A. Melanin melatonin. Melanin is synthesized from tyrosine. MENDOZA, FEB 2013
B. Melatonin MD (TOP 9
C. Niacin - AUG 2012
D. Serotonin MED
E. B and C BOARDS;
TOPNOTCH
MD)
610 Your patient has muscular weakness and degeneration Leber's hereditary optic neuropathy is caused by an RACHELLE FINAL EXAM -
of the optic nerve. Your tentative diagnosis is Leber’s abnormality involving complex III of ETC. This is MENDOZA, FEB 2013
hereditary optic neuropathy. If this is the correct characterized by acute onset of unilateral visual loss MD (TOP 9
diagnosis, which of the following pathways will contain which may become bilateral. - AUG 2012
the MED
defect? BOARDS;
A. tricarboxylic acid cycle TOPNOTCH
B. electron transport chain MD)
C. glycolysis
D. gluconeogenesis
E. pyruvate dehdyrogenase multienzyme complex
611 The eukaryotic DNA has the following The eukaryotic DNA has the following properties: RACHELLE FINAL EXAM -
property/properties: 1)coding regions are often interrupted by intervening MENDOZA, FEB 2013
A. Coding regions are not interrupted by intervening sequences, 2)more than half of the DNA is unique or MD (TOP 9
sequeces nonrepetitive sequences, 3)at least 30% of the - AUG 2012
B. All of the cellular DNA is located within the nucleus genome consist of repitive sequences, 4)1% of cellular MED
C. Contains no repetitive sequences DNA is in mitochondria, 5) replication is bidirectional BOARDS;
D. Replication of double stranded DNA is unidirectional TOPNOTCH

TOPNOTCH MEDICAL BOARD PREP BIOCHEMISTRY SUPEREXAM Page 78 of 92


For inquiries visit www.topnotchboardprep.com.ph or email us at topnotchmedicalboardprep@gmail.com
TOPNOTCH MEDICAL BOARD PREP BIOCHEMISTRY SUPEREXAM
For inquiries visit www.topnotchboardprep.com.ph or email us at topnotchmedicalboardprep@gmail.com
Item QUESTION EXPLANATION AUTHOR TOPNOTCH
# EXAM
E. None of the above MD)

612 Which of the following pairs is INCORRECT? RNA polymerase 1 is used in the transcription of large RACHELLE FINAL EXAM -
A. RNA polymerase I - rRNA rRNAs in the nucleolus, II for mRNAs and III for tRNAs MENDOZA, FEB 2013
B. RNA polymerase II - ssRNA MD (TOP 9
C. RNA polymerase III - tRNA - AUG 2012
D. A and C MED
E. All are incorrect BOARDS;
TOPNOTCH
MD)
613 Which of the following pertains to a set of structure Promoter regions are part of the DNA where RNA RACHELLE FINAL EXAM -
genes coding for a group of proteins required for a polymerase binds to start transcription. The primary MENDOZA, FEB 2013
particular metabolic function along with the regulatory transcript is a linear copy of the transcriptional unit, MD (TOP 9
region that controls the expression of the structural the segment of DNA between specific initiation and - AUG 2012
genes? termination sequences. Rho factor is involved in MED
A. Promoter region termination of DNA transcription by binding to a C- BOARDS;
B. Primary transcript rich region near the 3'-end of the newly synthesized TOPNOTCH
C. Rho factor RNA and migrates along the 5' to 3' direction until MD)
D. Operon termination site is reached.
E. None of the above
614 A sample of DNA-RNA hybrid is to be examined for its Southern blot is utilized for purely DNA sample. RACHELLE FINAL EXAM -
component. The best method to utilize is: Northern blot is utlized for RNA and mixed DNA-RNA MENDOZA, FEB 2013
A. Southern blot samples, while Western blot can be utilized for MD (TOP 9
B. Northern blot protein-containing sample. - AUG 2012
C. Western blot MED
D. Eastern blot BOARDS;
E. None of the above TOPNOTCH
MD)

615 A 6-year old boy was brought to your clinic due to This patient is suffering from mucopolysaccharidoses. RACHELLE FINAL EXAM -
persistent back pain. You noted that he was short for his Morquio's syndrome is the only MENDOZA, FEB 2013
age and has some degree of skeletal dysplasia. Upon mucopolysaccharidoses which does not present with MD (TOP 9
further examination, his mental development was found mental retardation or other CNS involvement. - AUG 2012
to be at par with his age. Suspecting a type of Findings usually include skeletal dysplasia and short MED
mucopolysacchridoses, which of the following is most stature. This is due to deficiency in galactose 6- BOARDS;
likely affecting this patient? sulfatase. TOPNOTCH
A. Morquio's syndrome MD)
B. Hunter's syndrome
C. San fillippo syndrome
D. Sly syndrome
E. None of the above

616 A 65-year old diabetic female was brought to the ER due The very low blood glucose of the patient would RACHELLE FINAL EXAM -
to body weakness, followed by seizures and sudden loss trigger the release of counterregulatory hormones MENDOZA, FEB 2013
of consciousness. History revealed that she was on that are supposed to elevate glucose levels by MD (TOP 9
insulin therapy for 6 years now and has missed several increasing mobilization of glucose from liver and - AUG 2012
meals that day due to a bingo tournament she joined in inducing gluconeogenesis. These counterregulatory MED
the neighborhood. Blood glucose level was 18 mg/dl. hormones include glucagon, epinephrine and cortisol. BOARDS;
Which of the following can be expected in this patient? Since the patient is being maintained on exogenous TOPNOTCH
A. Increased glucagon insulin for sugar control, endogenous insulin secretion MD)
B. Increased epinephrine can be presumed to be decreased. C peptide is
C. Decreased C-peptide produced only in endogenous insulin. Hence, in this
D. All of the above patient, C peptide can be presumed to be decreased,as
E. A and B well.

617 Which of the following enzyme/s involved in heme Lead inhibits ALA dehydratase and ferrochelatase. RACHELLE FINAL EXAM -
synthesis is/are inhibited by lead intoxication, thereby MENDOZA, FEB 2013
causing anemia? MD (TOP 9
A. aminolevulinic acid synthase - AUG 2012
B. aminolevulinic dehydratase MED
C. ferrochelatase BOARDS;
D. A and B TOPNOTCH
E. B and C MD)

618 A 3-month old infant prsented with fair skin, eczema, The patient has phenyketonuria. This is either due to RACHELLE FINAL EXAM -
growth retardation and musty body odor. Which of the deficiency in phenylalanine hydroxylase or decreased MENDOZA, FEB 2013
following should be done to prevent further worsening tetrahydrobipterin cofactor. This sually presents with MD (TOP 9
of his condition? mental retardation, growth retardation, fair skin, - AUG 2012
A. Administration of branched chain amino acid in the eczema and musty body odor. Treatment includes MED
diet elimination of phenylalanine from the diet and BOARDS;
B. Eliminate branched chain amino acid from the diet administer tyrosine-rich diet TOPNOTCH
C. Administer phenylalanine in the diet and eliminate MD)
tyrosine
D. Eliminate phenylalanine from the diet and administer
tyrosine supplement
E. None of the above

TOPNOTCH MEDICAL BOARD PREP BIOCHEMISTRY SUPEREXAM Page 79 of 92


For inquiries visit www.topnotchboardprep.com.ph or email us at topnotchmedicalboardprep@gmail.com
TOPNOTCH MEDICAL BOARD PREP BIOCHEMISTRY SUPEREXAM
For inquiries visit www.topnotchboardprep.com.ph or email us at topnotchmedicalboardprep@gmail.com
Item QUESTION EXPLANATION AUTHOR TOPNOTCH
# EXAM
619 The following amino acids can be converted to ketones, Strictly ketogenic amino acids are: leucine and lysine. RACHELLE FINAL EXAM -
EXCEPT: Ketogenic and glucogenic amino acids are: MENDOZA, FEB 2013
A. Glycine phenylalanine, tyrosine, tryptophan and isoleucine. MD (TOP 9
B. Leucine All others are strictly glucogenic. - AUG 2012
C. Phenylalanine MED
D. Tryptophan BOARDS;
E. B and C TOPNOTCH
MD)
620 A mitochondrion was experimentally divided into its All components of ETC are fixed to the inner RACHELLE FINAL EXAM -
individual parts. On examining the inner membrane, mitochondrial membrane except for: conenzyme Q MENDOZA, FEB 2013
which of the following electron transport chain and cytochrome C. MD (TOP 9
components will NOT be present? - AUG 2012
A. Complex I MED
B. Complex III BOARDS;
C. Cytochrome oxidase TOPNOTCH
D. Cytochome c MD)
E. C and D

621 A 49 year old man with a 10 year history of poorly Aldose reductase catalyzes the breakdown of glucose VON DIAGNOSTIC
controlled diabetes mellitus presents to his physician into sorbitol. Sorbitol is then metabolized to fructose. ANDRE EXAM - AUG
complaining of changes in his vision. Physical In patients with hyperglycemia as in a case of DM, MEDINA, 2012
examination reveals opacities on the lens of the eye. sorbitol accumulation with the cells leads in the rise in MD (TOP 4
Which enzyme most likely contributed to this the intracellular osmolality, causing water movement - FEB 2012
complication? in the cell resulting to osmolar swelling and osmotic MED
A. Adenosine deaminase damage. swelling of the lens fiber can lead into BOARDS;
B. Aldose reductase rupture and cataract formation. TOPNOTCH
C. Galactose-1-phosphate uridyltransferase MD)
D. Hexokinase
E. Glucokinase
622 A woman gives birth to a full term baby. Upon delivery, This is a case of Phenylketonuria (PKU). Patients with VON DIAGNOSTIC
her pediatrician, Dra. Casimiro, noticed that the baby is this kind of disease are unable to convert ANDRE EXAM - AUG
small and has a musty odor. Upon questioning, the phenylalanine to tyrosine due to a deficiency of MEDINA, 2012
woman says that she did not smoke nor drink alcohol phenylalanine hydroxylase. MD (TOP 4
during the course of her pregnancy and she only drinks - FEB 2012
diet soda and water. Which of the following amino acid is MED
most likely to be deficient in the newborn? BOARDS;
A. Alanine TOPNOTCH
B. Tyrosine MD)
C. Phenylalanine
D. Serine
E. B and C

623 Which amino acid would most likely to be found in the Phenylalanine is a non-polar amino acid. VON DIAGNOSTIC
interior of the a protein? ANDRE EXAM - AUG
A. Tyrosine MEDINA, 2012
B. Phenylalanine MD (TOP 4
C. Histidine - FEB 2012
D. Asparagine MED
E. A and C BOARDS;
TOPNOTCH
MD)
624 Water is considered a dipolar molecule because? water is a dipole- a molecule with electrical charge VON DIAGNOSTIC
A. It can act both as a proton donor and a proton distributed asymmetrically it its structure. ANDRE EXAM - AUG
acceptor MEDINA, 2012
B. It has a partial positive and negative charges MD (TOP 4
unequally distibuted in its structure - FEB 2012
C. It is made up of two elements- oxygen and hydrogen MED
D. it can exist in a solid (ice) as well as the liquid state BOARDS;
E. all of the above TOPNOTCH
MD)
625 The higher the oxygen affinity of hemoglobin for oxygen, The higher the affinity of oxygen for hemoglobin the VON DIAGNOSTIC
A. The higher its P50 for oxygen becomes difficult it becomes to give up or to distribute the ANDRE EXAM - AUG
B. The more difficult it is to saturate with oxygen oxygen to the tissues. MEDINA, 2012
C. The more difficult it becomes for oxygen to MD (TOP 4
dissociate from the hemoglobin molecule - FEB 2012
D. A & B MED
E. none of the above BOARDS;
TOPNOTCH
MD)
626 A drug that is a competitive inhibitor of an enzyme a competitive inhibitor competes with a substrate for VON DIAGNOSTIC
A. Increases the apparent Km but does not affect the the active site of the enzyme, in effect increasing the ANDRE EXAM - AUG
Vmax apparent Km. As the substrate concentration is MEDINA, 2012
B. Decreases the apparent Km but does not affect the increased, the substrate, by competing with the MD (TOP 4
Vmax inhibitor, can overcome its inhibitory effects and - FEB 2012
C. Increases Vmax but does not affect the Km eventually the normal Vmax is reached. MED
D. Decreases the Vmax but does not affect the Km BOARDS;
E. Decreases both Vmax and Km TOPNOTCH
MD)
627 What enzyme converts Norepinephrine to Epinephrine? PNMT converts NE to E, with SAM (S adenosyl VON DIAGNOSTIC
A. Dopamine B-hydroxylase methionine) which adds CH3 and serves as a donor ANDRE EXAM - AUG
B. Dopa decarboxylase for the conversion. MEDINA, 2012
C. Norepinephrine hydroxylase MD (TOP 4
D. Epinephrine decarboxylase - FEB 2012
E. Phenylethanolamine -N-Methyl transferase MED
BOARDS;
TOPNOTCH
TOPNOTCH MEDICAL BOARD PREP BIOCHEMISTRY SUPEREXAM Page 80 of 92
For inquiries visit www.topnotchboardprep.com.ph or email us at topnotchmedicalboardprep@gmail.com
TOPNOTCH MEDICAL BOARD PREP BIOCHEMISTRY SUPEREXAM
For inquiries visit www.topnotchboardprep.com.ph or email us at topnotchmedicalboardprep@gmail.com
Item QUESTION EXPLANATION AUTHOR TOPNOTCH
# EXAM
MD)

628 What vitamin is required for the synthesis of niacin from Pyridoxine of B6 is required for the synthesis of niacin VON DIAGNOSTIC
typtophan? from tryptophan. ANDRE EXAM - AUG
A. Thiamine MEDINA, 2012
B. Riboflavin MD (TOP 4
C. Pantothenic acid - FEB 2012
D. Pyridoxine MED
E. Cobalamin BOARDS;
TOPNOTCH
MD)
629 Mr. Dorado was diagnosed to have a deficiency of muscle this is a case of McArdle's disease, muscle VON DIAGNOSTIC
phosphorylase enzyme. An individual with deficiency of phosphorylase deficiency. Muscle glycogen could not ANDRE EXAM - AUG
this enzyme would; be oxidized during exercise. Therfore, lactate level MEDINA, 2012
A. Produce a higher than normal amount of lactate would be low and the person could not tolerate MD (TOP 4
during a brief period of intense exercise like push-ups intense exercise of brief duration and would rely on - FEB 2012
B. Be incapable of performing mild exercises of long fuels from the blood for energy. the person could MED
duration (a 10-mile walk) engage in mild exercises of long duration using these BOARDS;
C. have lower than normal amounts of glycogen in blood fuels. the liver would not be affected because it TOPNOTCH
muscle tissue contains a different phosphorylase enzyme and it MD)
D. be less dependent than normal on blood glucose to could still respond to glucagon by breaking down
supply energy for exercise glycogen.
E. produce normal amounts of blood glucose in
response to increased glucagon
630 Which of the following statements about the conversion Citrate transports acetyl unit from mitochondria to VON DIAGNOSTIC
of glucose to triacylglycerol in the liver is CORRECT? cytosol. NADPH is provided by PPP and MALIC ANDRE EXAM - AUG
A. Malate serves to transport acetyl units across the enzyme. The liver has glycerol kinase, thus blood MEDINA, 2012
mitochondrial membrane glycerol can be used. 2 monoacylglycerol is produced MD (TOP 4
B. Reducing equivalents are provided by the reactions only in intestinal cells. - FEB 2012
of glycolysis MED
C. Reducing equivalents are provided by the malic BOARDS;
enzyme (a decarboxylating malate dehydrogenase) TOPNOTCH
D. The glycerol moiety can be derived from MD)
dihydroxyacetone phosphate (DHAP) but not from blood
glycerol
E. 2-Monoacylglycerol is an intermediate
631 What reaction/pathway generates the greatest amount VON DIAGNOSTIC
of ATP? ANDRE EXAM - AUG
A. Glycolysis MEDINA, 2012
B. Citric acid cycle MD (TOP 4
C. Oxidative Phosphorylation - FEB 2012
D. Adenylate kinase reaction MED
E. None of the above BOARDS;
TOPNOTCH
MD)
632 When it transfers extramitochondrial NADH to the Glycerophosphate shuttle 2 ATPs; malate 3 ATPS VON DIAGNOSTIC
respiratory chain, it generates 2 rather than 3 ATPs ANDRE EXAM - AUG
A. Malate shuttle MEDINA, 2012
B. Glycerophosphate shuttle MD (TOP 4
C. Citrate shuttle - FEB 2012
D. Both A and B MED
E. None of the above BOARDS;
TOPNOTCH
MD)
633 A neonate born 4 hours ago is having a difficulty of Dipalmitoyl phosphatidylcholine is the same as VON DIAGNOSTIC
breathing. The baby was born at 32 weeks AOG. He is lecithin. ANDRE EXAM - AUG
tachypneic and is flaring and grunting. The baby's heart MEDINA, 2012
rate is 120bpm, BP 100/60 mmHg and respiratory rate MD (TOP 4
is 55 cpm. What is lacking in this patient? - FEB 2012
A. Dipalmitoyl phosphatidylcholine MED
B. Lecithin BOARDS;
C. Elastase TOPNOTCH
D. Collagen MD)
E. A and B

634 Which of the following statements about nitrogen because of the sulfur of methionine is used for VON DIAGNOSTIC
metabolism is correct? cysteine synthesis, as cysteine increases in the diet, ANDRE EXAM - AUG
A. Cysteine "spares" methionine; that is ingestion of less methionine is required. Glutamate DH catalyzes MEDINA, 2012
cysteine reduces the need for methionine in the diet the addition of ammonia to alpha ketoglutarate to MD (TOP 4
B. The enzyme glutamate dehydrogenase catalyzes the form glutamate. For creatine synthesis, arginine, not - FEB 2012
transamination of glutamate ornithine, is required. FIGLU is produced during the MED
C. Creatine requires glycine, ornithine, and methionine degradation of histidine. BOARDS;
for the synthesis of its carbon skeleton TOPNOTCH
D. Formiminoglutamate (FIGLU) is an intermediate in MD)
glutamine degradation
E. All of the above

TOPNOTCH MEDICAL BOARD PREP BIOCHEMISTRY SUPEREXAM Page 81 of 92


For inquiries visit www.topnotchboardprep.com.ph or email us at topnotchmedicalboardprep@gmail.com
TOPNOTCH MEDICAL BOARD PREP BIOCHEMISTRY SUPEREXAM
For inquiries visit www.topnotchboardprep.com.ph or email us at topnotchmedicalboardprep@gmail.com
Item QUESTION EXPLANATION AUTHOR TOPNOTCH
# EXAM
635 Which of the following statements concerning Arachidonic acid is cleaved from membrane VON DIAGNOSTIC
metabolism of arachidonic acid is TRUE? phospholipids by phospholipase A2. ANDRE EXAM - AUG
A. It is converted to prostagalndins by a process that is MEDINA, 2012
stimulated by aspirin MD (TOP 4
B. It is coverted to prostaglandins by a process that is - FEB 2012
stimulated by glucocorticoids MED
C. It is produced from thromboxanes and leukotrienes BOARDS;
D. It is derived from palmitate TOPNOTCH
E. it is cleaved from membrane phospholipids by a MD)
phospholipase
636 Which of the following statements is TRUE of de novo In pyrimidine biosynthesis, carbamoyl phosphate VON DIAGNOSTIC
pyrimidine synthesis but not of de novo purine produced from glutamine, C02 and ATP, reacts with ANDRE EXAM - AUG
synthesis? aspartate to form a base which, after oxidation, reacts MEDINA, 2012
A. The base is synthesized while attached to ribose 5 with PRPP to form a nucleotide. MD (TOP 4
phosphate - FEB 2012
B. One carbon fragments are donated by folic acid MED
derivatives BOARDS;
C. Carbamoyl PO4 donates a carbamoyl group TOPNOTCH
D. The entire glycine molecule is incorporated into a MD)
precursor of the base
E. Glutamine donates nitrogen that becomes one of the
atoms in the ring
637 A molecule of palmitate will yield how many ATP? VON DIAGNOSTIC
A. 12 ANDRE EXAM - AUG
B. 38 MEDINA, 2012
C. 39 MD (TOP 4
D. 126 - FEB 2012
E. 129 MED
BOARDS;
TOPNOTCH
MD)
638 Which level of protein structure is involved in the B pleated sheet= secondary structure VON DIAGNOSTIC
pathology of prion diseases? ANDRE EXAM - AUG
A. Primary structure MEDINA, 2012
B. Secondary structure MD (TOP 4
C. Tertiary structure - FEB 2012
D. Quaternary structure MED
E. A and C BOARDS;
TOPNOTCH
MD)
639 Typtophan is an amino acid with the largest side chain tryptophan is a precursor of niacin, melatonin and VON DIAGNOSTIC
containing an indole ring. It is a precursor of what serotonin ANDRE EXAM - AUG
substances? MEDINA, 2012
A. niacin MD (TOP 4
B. melatonin - FEB 2012
C. serotonin MED
D. A and B BOARDS;
E. All of the above TOPNOTCH
MD)
640 Desmolase is the rate limiting enzyme in androgen and desmolase converts cholesterol to pregnenolone, VON DIAGNOSTIC
steroid synthesis. It is responsible for what reaction? which is the rate limiting step ANDRE EXAM - AUG
A. Conversion of progesterone to 11 MEDINA, 2012
deoxycorticosterone MD (TOP 4
B. Conversion of pregnenolone to 17 - FEB 2012
hydroxypregnenolone MED
C. Conversion of cholesterol to pregnenolone BOARDS;
D. Conversion of DHEA to Androstenedione TOPNOTCH
E. None of the above MD)
641 Glycosaminoglycans (GAG's) are long, unbranched All of the choices given are GAG's. However, in terms LITO JAY DIAGNOSTIC
heteropolysaccharide chains that are generally of cell migration and wound repair (specifically on MACARAIG, EXAM - AUG
composed of repeating dissacharide units. Which among synovial fluid and vitreous humor), Hyaluronic Acid is MD (TOP 8 2013
the following GAG's is mainly found on synovial fluid and the best answer. Chondroitin sulfate is present on - FEB 2013
vitreous humor; and plays an important role in cartilage and cornea. Keratan Sulfate and Dermatan MED
permitting cell migration during morphogenesis and sulfate are responsible for corneal transparency. BOARDS;
wound repair? Heparan Sulfate determines the charge-selectiveness TOPNOTCH
A. Hyaluronic Acid (GlcNAc, GLCUA) of the renal glomerulus. MD)
B. Chondroitin Sulfate (GalNAc, GlcUA)
C. Keratan Sulfate (GlcNAc, Gal)
D. Dermatan Sulfate (GalNAc, IdUA)
E. Heparan Sulfate (GalNAc, GlcUA)

642 After acquiring your Physician PRC license, you decided Morquio's Syndrome is characterized by accumulation LITO JAY DIAGNOSTIC
to go on moonlighting. At the OPD, a mother brought her of GAG's in the lysosomes due to galactose-6-sulfatase MACARAIG, EXAM - AUG
4 year old child who presented with short stature, deficiency, and is the only mucopolysaccharidoses MD (TOP 8 2013
hypermobile joints and widely-spaced teeth. Upon chest syndrome that is NOT associated with CNS - FEB 2013
examination, you heard a grade 4/6 systolic murmur. abnormality. MED
You are suspecting a mucopolysaccharidoses syndrome BOARDS;
and upon knowing that the patient has NO CNS TOPNOTCH
abnormality, you already know that the patient is MD)
suffering from?
A. Hurler's Syndrome
B. Hunter's Syndrome
C. Morquio's Syndrome
D. Sanfilippo Syndrome
E. Zellweger Syndrome

TOPNOTCH MEDICAL BOARD PREP BIOCHEMISTRY SUPEREXAM Page 82 of 92


For inquiries visit www.topnotchboardprep.com.ph or email us at topnotchmedicalboardprep@gmail.com
TOPNOTCH MEDICAL BOARD PREP BIOCHEMISTRY SUPEREXAM
For inquiries visit www.topnotchboardprep.com.ph or email us at topnotchmedicalboardprep@gmail.com
Item QUESTION EXPLANATION AUTHOR TOPNOTCH
# EXAM
643 Myoglobin is a heme protein found in heart and skeletal Histidine residues play an essential role in O2-binding LITO JAY DIAGNOSTIC
muscle. It acts as reservoir of oxygen in the said of myoglobin. Glutamine and Alanine are the main MACARAIG, EXAM - AUG
locations and releases oxygen in times of hypoxia. The carriers of Nitrogen from peripheral tissues towards MD (TOP 8 2013
residue of which amino acid is responsible for the liver. Serine and Threonine are the sites for O- - FEB 2013
myoglobin's oxygen-binding capacity? linked glycosylation in the Golgi apparatus. MED
A. Glutamine BOARDS;
B. Histidine TOPNOTCH
C. Alanine MD)
D. Serine
E. Threonine

644 Which among the following amino acids is a weak base, Histidine, Arginine, and Lysine are all basic in nature. LITO JAY DIAGNOSTIC
and therefore has no charge on neutral pH? Arginine and Lysine are positively charged at neutral MACARAIG, EXAM - AUG
A. Histidine pH. Leucine and Isoleucine are branched-chain amino MD (TOP 8 2013
B. Arginine acids. - FEB 2013
C. Lysine MED
D. Leucine BOARDS;
E. Isoleucine TOPNOTCH
MD)

645 On a research laboratory, you are asked to determine the The linear sequence of a protein is called its Primary LITO JAY DIAGNOSTIC
linear sequence of a specific protein. While doing so, you Structure. It is composed by specific amino acids MACARAIG, EXAM - AUG
recalled that the kind of bond you least likely to break is attached to each other by Peptide Bonds. All other MD (TOP 8 2013
the? choices are found on higher levels of protein - FEB 2013
A. Hydrogen Bond structure. MED
B. Ionic Bond BOARDS;
C. Covalent Bond TOPNOTCH
D. Peptide Bond MD)
E. Hydrophobic Bond

646 These are specialized group of proteins required for the SIMILAR TO PREVIOUS BOARD EXAM LITO JAY DIAGNOSTIC
folding of many species of proteins. They can also CONCEPT/PRINCIPLE. Motifs are the non-functional MACARAIG, EXAM - AUG
"rescue" proteins that have become thermodynamically combinations of secondary protein structures. MD (TOP 8 2013
trapped in a misfolded dead-end by unfolding Domains are made of Motifs and are the functional - FEB 2013
hydrophobic regions. forms. Chaperones are the folding-guides and are also MED
A. Domains called the "rescue" proteins. The rest are just nuisance BOARDS;
B. Chaperones choices. TOPNOTCH
C. Motifs MD)
D. Protein-guiding Benders
E. Hydrophilic β-benders

647 Hemoglobin A1C (HbA1C) is being used in clinics to When blood glucose enters the erythocytes, it LITO JAY DIAGNOSTIC
monitor a diabetic patient's compliance in taking his/her glycosylates the ξ-amino group of Lysine residues and MACARAIG, EXAM - AUG
medications. On what amino acid residue does the amino terminals of hemoglobin. MD (TOP 8 2013
glycosylation of erythrocytes occur? - FEB 2013
A. Glycine MED
B. Alanine BOARDS;
C. Valine TOPNOTCH
D. Lysine MD)
E. Leucine

648 α1-antitrypsin is said to be a protective factor against Elastase destroys alveolar walls, resulting to LITO JAY DIAGNOSTIC
developing emphysema. What proteolytic enzyme does emphysema. Α1-antitrypsin regulates this enzyme and MACARAIG, EXAM - AUG
this substance inhibit? serves as a protection. MD (TOP 8 2013
A. Fibrillin Kinase - FEB 2013
B. Desmosinase MED
C. Elastase BOARDS;
D. Elastinase TOPNOTCH
E. Protease MD)

649 In bioenergetics, the measure of heat content of the Joules is the unit of measurement used for describing LITO JAY DIAGNOSTIC
reactants and products is called Enthalpy. What unit is Enthalpy. Joules per Kelvin is used to measure MACARAIG, EXAM - AUG
used to describe it? Entropy. MD (TOP 8 2013
A. Kelvin - FEB 2013
B. Joules MED
C. Joules per Kilogram BOARDS;
D. Joules per Kelvin TOPNOTCH
E. Joules per Fahrenheit MD)

650 The electron transport chain (ETC) is the final common Complex II is the Succinate Dehydrogenase enzyme. LITO JAY DIAGNOSTIC
pathway by which electrons from the different fuels of Aside from accepting FADH2 in the ETC, it also MACARAIG, EXAM - AUG
the body flow and create energy. What complex of ETC is participates in the Kreb's cycle. It converts Succinate MD (TOP 8 2013
also an essential part of the Kreb's Cycle? to Fumarate and yields FADH2 as a reducing - FEB 2013
A. Complex I equivalent. MED
B. Complex II BOARDS;
C. Coenzyme Q TOPNOTCH
D. Complex III MD)
E. Complex IV

TOPNOTCH MEDICAL BOARD PREP BIOCHEMISTRY SUPEREXAM Page 83 of 92


For inquiries visit www.topnotchboardprep.com.ph or email us at topnotchmedicalboardprep@gmail.com
TOPNOTCH MEDICAL BOARD PREP BIOCHEMISTRY SUPEREXAM
For inquiries visit www.topnotchboardprep.com.ph or email us at topnotchmedicalboardprep@gmail.com
Item QUESTION EXPLANATION AUTHOR TOPNOTCH
# EXAM
651 Uncouplers are substances that increase the In the presence of uncouplers, ETC still continue to LITO JAY DIAGNOSTIC
permeability of the inner mitochonadrial membrane to produce energy BUT in the form of HEAT and NOT MACARAIG, EXAM - AUG
protons. Which among the following statements is/are ATP. Uncouplers increase oxygen consumption and MD (TOP 8 2013
true about ETC in the presence of uncouplers? oxidation of NADH. - FEB 2013
A. Continue to produce energy MED
B. Increase ATP synthesis BOARDS;
C. Decrease oxidation of NADH TOPNOTCH
D. choices A and B are both correct MD)
E. choices B and C are both correct

652 Lactose is from? It is a common mistake that medical students believe LITO JAY DIAGNOSTIC
A. Glucose + Glucose that Galactose is the dissacharide form of Lactose MACARAIG, EXAM - AUG
B. Galactose + Glucose (which is the other way around). Lactose is the MD (TOP 8 2013
C. Galactose - Glucose condensation product of Galactose plus Glucose. - FEB 2013
D. Galactose - Fructose Galactose is only a monosaccharide, just like Glucose. MED
E. Galactose + Maltose BOARDS;
TOPNOTCH
MD)
653 Glucose is the main fuel of most of the cells in the body. The brain utilizes Glut-1 and Glut-3 to transport LITO JAY DIAGNOSTIC
Before it can be utilized, some cells need specific glucose inside the cell. Glut-2 is present in the liver, MACARAIG, EXAM - AUG
transporters to mediate its use. Which among the pancreatic B-cells, kidneys, and basement membrane MD (TOP 8 2013
following glucose transporters is/are present in the of the small intestines. - FEB 2013
brain? MED
A. Glut 1 BOARDS;
B. Glut 2 TOPNOTCH
C. Glut 3 MD)
D. choices A and B are both correct
E. choices A and C are both correct
654 Phosphofructokinase 1 (PFK1) is the rate-limiting step During the well-fed state (when there is increased LITO JAY DIAGNOSTIC
for Glycolysis. Which among the following statements Insulin and decreased Glucagon levels), PFK2 converts MACARAIG, EXAM - AUG
is/are true about this enzyme? fructose-6-phosphate (F-6-P) to fructose-2,6- MD (TOP 8 2013
A. Activated by excess ATP bisphosphate (F-2,6-BP). Accordingly, F-2,6-BP - FEB 2013
B. Inhibited by excess Citrate activates PFK1 and glycolysis will proceed to produce MED
C. Active in Starved state energy from the food we eat. In contrary, when there BOARDS;
D. Choices A and B are both correct is excess energy (in the form of ATP and Citrate), PFK- TOPNOTCH
E. Choices A and C are both correct 1 is inhibited. MD)

655 Pyruvate Dehydrogense enzyme deficiency is the most Pyruvate Dehydrogenase enzyme is the link between LITO JAY DIAGNOSTIC
common biochemical cause of congenital lactic acidosis. Glycolysis and Kreb's Cycle. It needs the following co- MACARAIG, EXAM - AUG
Which among the following substances is/are needed by enzymes: 1. Thiamine pyrophosphate, 2. FAD, 3. MD (TOP 8 2013
this enzyme as co-factor/s? NAD+, 4. Coenzyme-A, 5. Lipoic Acid - FEB 2013
A. Thiamine bisphosphate MED
B. FAD BOARDS;
C. NAD+ TOPNOTCH
D. Choices A and B are both correct MD)
E. Choices B and C are both correct

656 Gluconeogenesis pathway is technically the reverse of The steps 10, 3 and 1 of Glycolysis are catalyzed by LITO JAY DIAGNOSTIC
Glycolysis. They just differ in three enzymes. Which Pyruvate Kinase, PFK-1, and Hexokinase/Glucokinase MACARAIG, EXAM - AUG
among the following groups of enzymes is PRESENT in respectively. In Gluconeogenesis, these enzymes are MD (TOP 8 2013
Gluconeogenesis and NOT in the later? subtituted by: PEP carboxykinase + Pyruvate - FEB 2013
A. PEP carboxykinase + Pyruvate Carboxylase, Carboxylase, Fructose-1,6-bisphosphatase, and Glucose- MED
Fructose-1,6-bisphosphatase, Glucose-6-phosphatase 6-phosphatase respectively. BOARDS;
B. PEP carboxykinase + Pyruvate Carboxylase, TOPNOTCH
Fructose-1,6-bisphosphatase, Hexokinase/Glucokinase MD)
C. PEP carboxykinase + Pyruvate Carboxylase, Fructose-
2,6-bisphosphatase, Glucose-6-phosphatase
D. PEP carboxykinase + Pyruvate Carboxylase,
Fructose-2,6-bisphosphatase, Hexokinase/Glucokinase
E. PEP carboxykinase + Pyruvate Carboxykinase,
Fructose-2,6-bisphosphatase, Hexokinase/Glucokinase

657 Cellulose is the primary structure of a plant cell wall. Cellulose is a polysaccharide consisiting of a linear LITO JAY DIAGNOSTIC
Being a stool-former, it is an important part of human chain of several hundred to ten thousand of β(1->4) MACARAIG, EXAM - AUG
diet and is said to be protective from colon cancer. What linkage which cannot be digested by humans. MD (TOP 8 2013
kind of glycosidic bond is present in cellulose that - FEB 2013
humans cannot digest? MED
A. α(1->4) linkage BOARDS;
B. α(1->6) linkage TOPNOTCH
C. β(1->4) linkage MD)
D. β(1->6) linkage
E. none of the choices is true

TOPNOTCH MEDICAL BOARD PREP BIOCHEMISTRY SUPEREXAM Page 84 of 92


For inquiries visit www.topnotchboardprep.com.ph or email us at topnotchmedicalboardprep@gmail.com
TOPNOTCH MEDICAL BOARD PREP BIOCHEMISTRY SUPEREXAM
For inquiries visit www.topnotchboardprep.com.ph or email us at topnotchmedicalboardprep@gmail.com
Item QUESTION EXPLANATION AUTHOR TOPNOTCH
# EXAM
658 During your rotation in the Department of Pediatrics, Glucose-6-phosphate dehydrogenase (G6PD) LITO JAY DIAGNOSTIC
you encountered a 5 year-old patient with a chief deficiency is the most common disease-producing MACARAIG, EXAM - AUG
complaint of loose watery stools accompanied by enzyme abnormality in humans. Precipitating factors MD (TOP 8 2013
undocumented fever. During PE, patient is febrile with involves infection and certain medications. - FEB 2013
macular rashes on the chest (Rose Spots). CBC showed MED
lymphocytosis and the Widal Test turned out to be BOARDS;
positive. You prescribed Chloramphenicol but the TOPNOTCH
mother told you that the patient experienced bleeding in MD)
the past when he took the said medication. What kind of
abnormality are you thinking of?
A. Glucose-6-phosphatase deficiency
B. Glucose-6-phosphate dehydroxylase deficiency
C. Glucose-6-phosphate dehydrogenase deficiency
D. Glucose-6-phosphate dihydrogenase deficiency
E. Glucose-6-phosphatase dehydroxylase deficiency
659 This is a rare neurologic disorder due to abnormal In excess, Phytanic Acid blocks beta oxidation and LITO JAY DIAGNOSTIC
accumulation of Phytanic Acid. causes neurologic symptoms due to improper MACARAIG, EXAM - AUG
A. Zellweger's Syndrome myelinization. This disorder is called Refsum's MD (TOP 8 2013
B. Carnitine palmitoyl transferase I deficiency Disease. - FEB 2013
C. Carnitine palmitoyl transferase II deficiency MED
D. Ornithine transcarbamoylase deficiency BOARDS;
E. Refsum's Disease TOPNOTCH
MD)

660 Which among the following codons is/are "stop There are only three "stop codons". UAA, UAG and LITO JAY DIAGNOSTIC
codon/s"? UGA. MACARAIG, EXAM - AUG
A. UAA MD (TOP 8 2013
B. UGG - FEB 2013
C. UAU MED
D. Choices A and B are both correct BOARDS;
E. Choices A and C are both correct TOPNOTCH
MD)
661 This type of chemical bond is formed between 2 amino Peptide bond is formed between two amino acids and HAZEL MIDTERM 2 -
acidsand destroyed by heating the protein in a strong is the bonds that form the primary structure of KAREN AUG 2013
acid solution for 24 hours. proteins. Phosphodiester bonds exist between RAZ, MD
A. Peptide bond nucleotides in a DNA strand. H-bonds exist between (TOP 6 -
B. Phosphodiester bond water and an electron rich molecule such as sulfur, FEB 2013
C. Hydrogen bond oxygen, nitrogen and fluoride. Disulfide bonds exist MED
D. Disulfide bond between cysteine residues in 3o structures. BOARDS;
E. None of the above TOPNOTCH
MD)

662 Precursor of purine amino acids: De Novo Purine synthesis uses IMP as precursor HAZEL MIDTERM 2 -
A. IMP molecule while Pyrimidine synthesis uses OMP. KAREN AUG 2013
B. OMP RAZ, MD
C. UMP (TOP 6 -
D. dUMP FEB 2013
E. None of the above MED
BOARDS;
TOPNOTCH
MD)
663 Deficiency in Vitamin B6 would lead to decreased Vitamin B6 (Pyridoxine) is a co-factor in several HAZEL MIDTERM 2 -
synthesis of the following except? amino acid biosynthetic pathway including KAREN AUG 2013
A. Niacin Tryptophan, Histidine, Glycine and Glutamate to RAZ, MD
B. GABA produce Niacin, Histamine, Porphyrin and GABA (TOP 6 -
C. Epinephrine respectively. Tetrahydrobiopterin (BH4) is needed in FEB 2013
D. Histamine the synthesis of Epinephrine, to convert Phenylalanine MED
E. None of the above into Tyrosine. BOARDS;
TOPNOTCH
MD)
664 Tyrosine becomes an essential amino acid in the In a normal person, Tyrosine is synthesized form HAZEL MIDTERM 2 -
presence of _________? Phenylalanine. However in the presence of KAREN AUG 2013
A. Phenylketonuria phenylketonuria, Phe intake is restricted, so Tyrosine RAZ, MD
B. Tryptophan deficiency must be derived from the diet for the synthesis of (TOP 6 -
C. Adenosine deaminase deficiency catecholamines. FEB 2013
D. Vit. B6 Deficiency MED
E. None of the above BOARDS;
TOPNOTCH
MD)
665 The following are the only ketogenic amino acids? The only ketogenic amino acids are Leucine and HAZEL MIDTERM 2 -
A. Lysine, Leucine Lysine. Ile, Thr, Phe, Tyr and Trp are both ketogenic KAREN AUG 2013
B. Isoleucine, Lysine and glucogenic. Others are purely glucogenic. RAZ, MD
C. Tyrosine, Methionine (TOP 6 -
D. Leucine, Arginine FEB 2013
E. None MED
BOARDS;
TOPNOTCH
MD)

TOPNOTCH MEDICAL BOARD PREP BIOCHEMISTRY SUPEREXAM Page 85 of 92


For inquiries visit www.topnotchboardprep.com.ph or email us at topnotchmedicalboardprep@gmail.com
TOPNOTCH MEDICAL BOARD PREP BIOCHEMISTRY SUPEREXAM
For inquiries visit www.topnotchboardprep.com.ph or email us at topnotchmedicalboardprep@gmail.com
Item QUESTION EXPLANATION AUTHOR TOPNOTCH
# EXAM
666 Keloid is made up of what type of collagen? CONTROVERSIAL. It's made up of both type I and III, HAZEL MIDTERM 2 -
A. I but more type I rather than III. II) Keloid tissues KAREN AUG 2013
B. II contained 31.6 +/- 2.2 percent type III collagen as RAZ, MD
C. III compared to 21.4 +/- 2.7 percent type III present in (TOP 6 -
D. IV normal human skin dermis. FEB 2013
E. V MED
Alteration of collagen composition and cross-linking BOARDS;
in keloid tissues. TOPNOTCH
Di Cesare PE, Cheung DT, Perelman N, Libaw E, Peng L, MD)
Nimni ME.
Matrix. 1990 Jul;10(3):172-8. PubMed PMID: 2215357
667 Post - transcriptional modification in eukaryotes, except? Co-transcriptional modification is a process in cell HAZEL MIDTERM 2 -
A. Capping of 5' end biology by which, in eukaryotic cells, primary KAREN AUG 2013
B. Polyadenylation of 3' end transcript RNA is converted into mature RNA. RAZ, MD
C. Splicing of introns (TOP 6 -
D. Splicing of exons FEB 2013
E. None of the above MED
BOARDS;
TOPNOTCH
MD)
668 A protein is separated by gel electrophoresis and Southern blot uses DNA as sample and DNA as probe. HAZEL MIDTERM 2 -
antibody is bound to a protein. This process describes? Northern blot uses RNA as sample, DNA as probe and KAREN AUG 2013
A. Southern blot Western blot uses Protein as probe and sample. RAZ, MD
B. Northern blot (TOP 6 -
C. Western blot FEB 2013
D. Southwestern blot MED
E. Northwestern blot BOARDS;
TOPNOTCH
MD)
669 This type of inhibition leads to decreased affinity and Non - competitive inhibition decreases efficacy but HAZEL MIDTERM 2 -
does not affect efficacy? does not affect affinity. Allosteric inhibition is a type KAREN AUG 2013
A. Non - competitive Inhibition wherein the substrate binds to a different site from RAZ, MD
B. Allosteric inhibition the inhibitor. Mixed inhibition leads to deacreased (TOP 6 -
C. Mixed inhibition affinity and decreased efficacy. FEB 2013
D. Competitive inhibition MED
E. None of the above BOARDS;
TOPNOTCH
MD)
670 This process occurs both in the cytoplasm and Processes that occur both in the cytoplasm and HAZEL MIDTERM 2 -
mitochondria: mitochondria includes Heme synthesis, Urea cycle and KAREN AUG 2013
A. Glycolysis Gluconeogenesis. RAZ, MD
B. Heme synthesis (TOP 6 -
C. B-oxidation of fatty acids FEB 2013
D. ketogenesis MED
E. None of the above BOARDS;
TOPNOTCH
MD)
671 Rate Limiting step in de Novo Pyrimidine Synthesis? Glutamine - PRPP amidotransferase - de novo purine HAZEL MIDTERM 2 -
A. Carbamoyl PO4 synthetase II synthesis KAREN AUG 2013
B. Glutamine - PRPP amidotransferase Carnitine acyltransferase I - FA oxidation RAZ, MD
C. Carnitine acyltransferase I Carbamoyl phosphate synthetase I - FA oxidation (TOP 6 -
D. Carbamoyl phosphate synthetase i G6PD - HMP shunt FEB 2013
E. G6PD MED
BOARDS;
TOPNOTCH
MD)
672 Gross ATP production per molecule of glucose during Two enzymes in glycolysis produce ATP through HAZEL MIDTERM 2 -
glycolysis through substrate level phosphorylation? substrate level phosphorylation: PG Kinase and KAREN AUG 2013
A. 2 Pyruvate kinase producing 2 ATPs each, making it a RAZ, MD
B. 4 total of 4 ATPs. (TOP 6 -
C. 6 FEB 2013
D. 8 MED
E. 10 BOARDS;
TOPNOTCH
MD)
673 Main function of Pentose Phosphate Pathway? The pentose phosphate pathway (also called HAZEL MIDTERM 2 -
A. NADPH production the phosphogluconate pathway and the hexose KAREN AUG 2013
B. ATP production monophosphate shunt) is a process that RAZ, MD
C. Removal of nitrogenous waste generates NADPH and pentoses (5-carbon sugars). (TOP 6 -
D. Transport of FA from cytosol to inner mitochondrial ATP production is through Kreb's cycle and ETC, FEB 2013
membrane nitrogenous waste is removed through Urea cycle and MED
E. None of the above FA transport is carnitine shuttle for B-oxidation of FA. BOARDS;
TOPNOTCH
MD)
674 Reactions proceed spontaneously in which of the Spontaneous reactions occur when ΔG is negative, that HAZEL MIDTERM 2 -
following? ΔG is <0. Positive ΔG needs energy for it to occur and if ΔG KAREN AUG 2013
A. = 0 = 0, rate of formation is equal to rate of degradation. RAZ, MD
B. < 0 (TOP 6 -
C. >0 FEB 2013
D. ΔH - TΔS MED
E. None of the above. BOARDS;
TOPNOTCH
MD)

TOPNOTCH MEDICAL BOARD PREP BIOCHEMISTRY SUPEREXAM Page 86 of 92


For inquiries visit www.topnotchboardprep.com.ph or email us at topnotchmedicalboardprep@gmail.com
TOPNOTCH MEDICAL BOARD PREP BIOCHEMISTRY SUPEREXAM
For inquiries visit www.topnotchboardprep.com.ph or email us at topnotchmedicalboardprep@gmail.com
Item QUESTION EXPLANATION AUTHOR TOPNOTCH
# EXAM
675 This lipoprotein transports triglyceride from liver to the LDL transports cholesterol into cells. HDL reverses HAZEL MIDTERM 2 -
periphery. cholesterol transport, transporting it from the KAREN AUG 2013
A. LDL periphery into the liver, chylomicrons transport RAZ, MD
B. HDL dietary TG and cholesterol into tissues while micelles (TOP 6 -
C. VLDL are formed through the biliary saponification of FEB 2013
D. Chylomicrons dietary fat from the intestinal lumen for absorption MED
E. Micelles into the intestinal brush border. BOARDS;
TOPNOTCH
MD)
676 Component of surfactant produced by type II Dipalmitoyl phosphatidylcholine is the major HAZEL MIDTERM 2 -
pneumocytes? component of lung surfactant. Phosphatidyl inositol is KAREN AUG 2013
A. Phosphatidyl inositol a reservoir of arachidonic acid in cellular membranes, RAZ, MD
B. Phosphatidyl choline needed for synthesis of prostaglandins and (TOP 6 -
C. Phospholipase leukotrienes. FEB 2013
D. Glycerophospholipid MED
E. Glycerol BOARDS;
TOPNOTCH
MD)
677 Major reactants for heme synthesis includes: Glutamine, glycine and aspartate are the reactants for HAZEL MIDTERM 2 -
A. Glutamine, glycine, aspartate purine synthesis, aspartate and glutamine for KAREN AUG 2013
B. Aspartate, glutamine pyrimidine synthesis, acetyl CoA for FA synthesis. RAZ, MD
C. 2 acetyl CoA (TOP 6 -
D. Glycine, succinyl CoA FEB 2013
E. None MED
BOARDS;
TOPNOTCH
MD)
678 A 6 - month old presents with recurrent sinus infection, This condition is also known as Severe Combined HAZEL MIDTERM 2 -
chronic cough and pneumonia. CBC showed Immunodeficiency (SCID) which is a genetic disorder KAREN AUG 2013
lymphopenia. This may be caused by a deficiency in characterized by the absence of fuctional T- RAZ, MD
which enzyme? lymphocytes,which results in a defective antibody (TOP 6 -
A. HGPRT response. FEB 2013
B. Adenine deaminase MED
C. G6PD BOARDS;
D. Homogentisic acid oxidase TOPNOTCH
E. HMG CoA synthetase MD)

679 This vitamin deficiency may result secondary to Tryptophan is a precursor for synthesis of Niacin (Vit HAZEL MIDTERM 2 -
decreased absorption of tryptophan, also known as B3). KAREN AUG 2013
Hartnup disease. RAZ, MD
A. Thiamine (TOP 6 -
B. Riboflavin FEB 2013
C. Niacin MED
D. Pyridoxine BOARDS;
E. Folic acid TOPNOTCH
MD)
680 Protein - energy malnutrition which occurs more often Kwashiorkor is a form of malnutrition wherein there HAZEL MIDTERM 2 -
in children >1y/o, with hair and skin changes, edema is adequate intake of calories but with low protein KAREN AUG 2013
and stunted growth. Plasma protein is very low when content, which leads to the above symptoms. RAZ, MD
measured. Marasmus on the other hand is due to lack of calories (TOP 6 -
A. Kwashiorkor but with adequate protein. FEB 2013
B. Marasmus MED
C. Cachexia BOARDS;
D. Anorexia TOPNOTCH
E. None of the above MD)

681 The model proposed by Watson, Crick and Wilkins A. Hydrogen bond (pp. 311, Harper's Biochemistry, MICHELLE MIDTERM 1 -
consists of a double-stranded helical structure of the B 27th ed.) JAY AUG 2013
form of DNA. Several bonds maintain the integrity of the Notes: FRANCISC
DNA molecule. What bond is found in between the • van der Waals and hydrophobic interactions - O, MD (TOP
purine and pyrimidine bases of its respective linear between stacked adjacent base pairs 9 - FEB
molecules holding the two strands together of the • 3'-5'-phosphodiester bridges - connects monomeric 2013 MED
double-stranded helix? units of DNA to become polymeric forms BOARDS;
A. Hydrogen bond TOPNOTCH
B. van der Waals MD)
C. Hydrophobic interactions
D. 3'-5'-phosphodiester bridges
682 In the prokaryotic gene expression, what is the genetic C. Cistron (pp 381, Harper's Biochemistry, 27th ed). MICHELLE MIDTERM 1 -
unit coding for the structure of the subunit of a protein SIMILAR TO PREVIOUS BOARD EXAM JAY AUG 2013
molecule, acting as it does as the smallest unit of genetic CONCEPT/PRINCIPLE. FRANCISC
expression? O, MD (TOP
A. Operon 9 - FEB
B. Intron 2013 MED
C. Cistron BOARDS;
D. Exon TOPNOTCH
MD)

TOPNOTCH MEDICAL BOARD PREP BIOCHEMISTRY SUPEREXAM Page 87 of 92


For inquiries visit www.topnotchboardprep.com.ph or email us at topnotchmedicalboardprep@gmail.com
TOPNOTCH MEDICAL BOARD PREP BIOCHEMISTRY SUPEREXAM
For inquiries visit www.topnotchboardprep.com.ph or email us at topnotchmedicalboardprep@gmail.com
Item QUESTION EXPLANATION AUTHOR TOPNOTCH
# EXAM
683 The occurrence of the 21st amino acid in proteins is Answer: B. Selenocysteine (pp. 243, Harper's MICHELLE MIDTERM 1 -
uncommon; however, it is present at the active site of Biochemistry, 27th ed.) JAY AUG 2013
several human enzymes that catalyze redox reactions. Notes: Keshan Disease = selenium deficiency FRANCISC
Replacement of this amino acid with another can result cardiomyopathy O, MD (TOP
in significantly decreased catalytic activity and might 9 - FEB
impair a kind of human protein which has been 2013 MED
implicated in tumorigenesis, atherosclerosis and Keshan BOARDS;
disease. What is this amino acid? TOPNOTCH
A. Isoleucine MD)
B. Selenocysteine
C. Hydroxyproline
D. Selenoleucine

684 The Kreb's-Henseleit Cycle is a pathway present only in Answer: A. Carbamoyl Phosphate Synthetase-1 MICHELLE MIDTERM 1 -
the liver useful in the removal of nitrogenous waste (Biochemistry, Topnotch Handouts). Notes: This JAY AUG 2013
products in the body. The "bottleneck" reaction and the question can be rephrased into: What is the rate- FRANCISC
metabolite flux of this pathway is reduced when the limiting (bottleneck) enzyme for urea cycle? *Kreb's- O, MD (TOP
catalytic efficiency or quantity of what catalyst is Henseleit Cycle (Urea Cycle/Ornithine Cycle) is NOT 9 - FEB
decreased? synonymous with Kreb's Cycle (TCA). 2013 MED
A. Carbamoyl Phosphate Synthetase-1 BOARDS;
B. Phosphofructokinase-1 TOPNOTCH
C. Isocitrate Dehydrogenase MD)
D. Carnitine Palmitoyl Transferase-1
685 5. The different etiologies of jaundice can be classified Answer: B. Hemolytic Anemia (pp. 292, Harper's MICHELLE MIDTERM 1 -
into prehepatic, hepatic and posthepatic causes. Biochemistry, 27th ed.) JAY AUG 2013
Differentiation of these causes can be evaluated by Notes: The presence of bilirubin in the urine is FRANCISC
determination of bilirubin levels in the serum, urine and sometimes referred to as Choluria. O, MD (TOP
feces. What is the clinical significance of acholuric • Hepatitis and obstruction of CBD = choluric jaundice 9 - FEB
jaundice? • Hemolytic anemia = acholuric jaundice 2013 MED
A. Normal • Normal = (obviously) absent jaundice and absent BOARDS;
B. Hemolytic Anemia urine bilirubin TOPNOTCH
C. Hepatitis MD)
D. Obstructive Jaundice
686 Impairment of oxidation of fatty acids gives rise to Answer: C. Phytanic Acid (pp. 194, Harper's MICHELLE MIDTERM 1 -
several diseases. An example of which is Refsum's Biochemistry, 27th ed.) JAY AUG 2013
disease, a rare neurologic disorder due to a metabolic Notes: FRANCISC
defect resulting in the accumulation of a certain • Jamaican Vomiting Sickness = Hypoglycin (from O, MD (TOP
substance found in dairy products and ruminant fat and Akee Tree) = inactivates medium- and short-chain 9 - FEB
meat. What is this substance thought to have acyl-CoA Dehydrogenase 2013 MED
pathological effects on membrane function, protein • Dicarboxylic Aciduria = w-dicarboxylic acids = lack BOARDS;
prenylation and gene expression? mitochondrial medium-chain acyl-CoA TOPNOTCH
A. Hypoglycin Dehydrogenase MD)
B. Dicarboxylic Acid • Refsum disease = Phytanic Acid
C. Phytanic Acid • Zellweger's (cerebrohepatorenal) syndrome =
D. Polyenoic Acid accum. polyenoic acid in the brain =absence of
peroxisomes in all tissues
• *This topic came out during Feb 2013 Physician
Licensure Exam.
687 There are several types of mutation which involves any Answer: B. Missense (Biochemistry, Topnotch MICHELLE MIDTERM 1 -
permanent heritable change in the DNA base sequence of Handouts) JAY AUG 2013
an organism. What type of mutation causes a possible FRANCISC
decrease in protein function because the new codon O, MD (TOP
specifies for a different amino acid? 9 - FEB
A. Silent 2013 MED
B. Missense BOARDS;
C. Nonsense TOPNOTCH
D. Frame shift MD)

688 Blotting is a method used to study macromolecules like Answer: A. Eastern Blot (pp. 20, Comment Section, MICHELLE MIDTERM 1 -
DNA, RNA and proteins which are separated by gel Rush University Medical Center, Review of Surgery, JAY AUG 2013
electrophoresis and transferred into a carrier. The 5th edition) FRANCISC
macromolecules can then be visualized by specific Notes: Southern Blot - DNA sample, DNA probe; O, MD (TOP
probes or staining methods. Which of the following Northern Blot - RNA or mRNA sample, DNA probe; 9 - FEB
methods is a modification of the Western Blot Technique Western Blot - Protein Sample, Antibody Probe; 2013 MED
by detecting posttranslational modification of proteins? Eastern Blot - Posttranslational modification of BOARDS;
A. Eastern Blot protein (sample); Southwestern Blot - DNA-binding TOPNOTCH
B. Polymerase Chain Reaction proteins (sample). MD)
C. Southwestern Blot
D. Northern Blot
689 Which of the following conditions involve severe Answer: B. Fatal Infantile Mitochondrial Myopathy MICHELLE MIDTERM 1 -
diminution or absence of most oxidoreductases of the (Biochemistry, Topnotch Handouts) JAY AUG 2013
respiratory chain as this affects all complexes of the FRANCISC
ETC? O, MD (TOP
A. MELAS 9 - FEB
B. Fatal Infantile Mitochondrial Myopathy 2013 MED
C. Leber's Hereditary Optic Neuropathy BOARDS;
D. Kearns-Sayre Syndrome TOPNOTCH
MD)

TOPNOTCH MEDICAL BOARD PREP BIOCHEMISTRY SUPEREXAM Page 88 of 92


For inquiries visit www.topnotchboardprep.com.ph or email us at topnotchmedicalboardprep@gmail.com
TOPNOTCH MEDICAL BOARD PREP BIOCHEMISTRY SUPEREXAM
For inquiries visit www.topnotchboardprep.com.ph or email us at topnotchmedicalboardprep@gmail.com
Item QUESTION EXPLANATION AUTHOR TOPNOTCH
# EXAM
690 Immunoglobulins (Ig) play a key role in the defense Answer: C. IgM and secretory IgA (pp 601, Harper’s MICHELLE MIDTERM 1 -
mechanism of the body. Each class of Ig has its own Biochemistry, 27th edition) JAY AUG 2013
distinct property for it to confer its specific function. Notes: FRANCISC
Among the 5 classes, based on its structure, which of the • serum IgA – may exist both as a monomer and a O, MD (TOP
Ig may exist as a dimer? dimer 9 - FEB
A. IgG1 and IgM • secretory IgA – exist as a dimer with a secretory 2013 MED
B. serum IgA and IgD component BOARDS;
C. IgM and secretory IgA • IgM – monomer, dimer or pentamer TOPNOTCH
D. IgD and IgG1 • IgD, IgG, IgE – monomer MD)
691 An adult man suffered from stable angina pectoris for 15 Fabry disease is an X-linked disorder that results from MICHELLE MIDTERM 1 -
yrs. During which time there was progressive heart a deficiency in alpha-galactosidase A. This leads to JAY AUG 2013
failure and repeated pulmonary thromboembolism, on deposition of neutral glycosphingolipids. Most FRANCISC
his death at age 63, autopsy disclosed enormous affected tissues are heart, kidneys and eyes. O, MD (TOP
cardiomyopathy (1100 g), cardiac storage of 9 - FEB
globotriaosylceramide (11 mg lipid/g wet weight) and 2013 MED
restricted cardiocytes. Which of the following lipid BOARDS;
storage disease would result in these clinical findings? TOPNOTCH
A. Fabry disease MD)
B. Gaucher disease
C. Krabbe disease
D. Niemann-Pick disease type 1A
E. Tay-Sachs disease

692 Lipoprotein lipase (LPL) is the endothelial cell- The presence of Apo CII on the surfaces of lipoprotein MICHELLE MIDTERM 1 -
associated enzyme necessary for release of fatty acid particles is necessary for the activation of endothelial JAY AUG 2013
from circulating lipoproteins. Which of the following cell LPL. FRANCISC
apoplipoproteins is required to activate LPL-mediated O, MD (TOP
release of fatty acids from chylomicrons? 9 - FEB
A. Apo A 2013 MED
B. Apo B100 BOARDS;
C. Apo B48 TOPNOTCH
D. Apo CII MD)
E. Apo E

693 Hemochromatosis, a disorder that is the result of excess Hemochromatosis is a disorder in iron metabolism MICHELLE MIDTERM 1 -
iron accumulation, is caused by deficiencies in which of that is characterized by excess iron absorption, JAY AUG 2013
the following proteins? saturation of iron-binding proteins and deposition of FRANCISC
A. Divalent metal transporter-1 (DMT-1) hemosiderin in the tissues. O, MD (TOP
B. Human leukocyte antigen (HLA) complex iron protein 9 - FEB
(HFE) 2013 MED
C. Ferritin BOARDS;
D. Ferroportin TOPNOTCH
E. Transferrin MD)
694 One important function of nitric oxide is the induction of NO is generated from arginine catalyzed ny NOS. MICHELLE MIDTERM 1 -
vascular smooth muscle relaxation in response to Other product is citrulline. JAY AUG 2013
acetylcholine. The production of NO requires which FRANCISC
amino acid? O, MD (TOP
A. Arginine 9 - FEB
B. Asparagines 2013 MED
C. Cysteine BOARDS;
D. Glutamine TOPNOTCH
E. Lysine MD)

695 Hepatocytes deliver ketone bodies to the circulation Succinuylo CoA Acetoacetate CoA transferase is the MICHELLE MIDTERM 1 -
because they lack which of the following enzymes? long name of thiophorase which is an enzyme not JAY AUG 2013
A. Beta-hydroxybutyrate dehdrogenase found in the liver FRANCISC
B. Hydroxymethyglutaryl-CoA-lyase O, MD (TOP
C. Hydroxymethylglutaryl-CoA-synthetase 9 - FEB
D. Succinyl-CoA-acetoacetate-CoA-tranferase 2013 MED
E. Beta-ketothiasolase to hydrolyze acetoacetyl-CoA BOARDS;
TOPNOTCH
MD)
696 The inability to rapidly synthesize DNA during the MICHELLE MIDTERM 1 -
process of erythrocyte maturation leads to abnormally JAY AUG 2013
enlarged erythrocytes. This disorder referred to FRANCISC
macrocytic anemia and is caused by a deficiency in O, MD (TOP
which of the following vitamins? 9 - FEB
A. Ascorbate 2013 MED
B. Biotin BOARDS;
C. Folate TOPNOTCH
D. Niacin MD)
E. Thiamine

697 Hypersensitive individuals have IgE to specific antigens PAF is a unique complex lipid of the plasmalogen MICHELLE MIDTERM 1 -
on the surface of their WBC. When these individuals are family and functions in hypersensitivity rxns, acute JAY AUG 2013
challenged with antigen, the antigen-IgE complexes inflammatory rxns and anaphylactic shock by inc. FRANCISC
induce synthesis and release of which of the following Vasopermeability, vasodilation and O, MD (TOP
physiologically potent lipids? bronchoconstriction. 9 - FEB
A. Arachidonic acid 2013 MED
B. Leukotriene B4 BOARDS;
C. Platelet activating factor (PAF) TOPNOTCH
D. Prostaglandin E2 MD)
E. Thromboxane A2

TOPNOTCH MEDICAL BOARD PREP BIOCHEMISTRY SUPEREXAM Page 89 of 92


For inquiries visit www.topnotchboardprep.com.ph or email us at topnotchmedicalboardprep@gmail.com
TOPNOTCH MEDICAL BOARD PREP BIOCHEMISTRY SUPEREXAM
For inquiries visit www.topnotchboardprep.com.ph or email us at topnotchmedicalboardprep@gmail.com
Item QUESTION EXPLANATION AUTHOR TOPNOTCH
# EXAM
698 Which of the following represents the enzyme deficiency Essential fructosuria is an autosomal recessive MICHELLE MIDTERM 1 -
that leads essential fructosuria? disorder manifesting benign asymtotomatology due to JAY AUG 2013
A. Fructose 1 phosphate aldolase (aldolase B) lack of fructokinase. FRANCISC
B. Fructose 1,6 bis phosphate aldolase (aldolase A) O, MD (TOP
C. Fructokinase 9 - FEB
D. Hexokinase 2013 MED
E. 6PFK-1 BOARDS;
TOPNOTCH
MD)
699 There is but a single enzyme-catalyzed reaction in the Heme is oxidized, w/ heme ring being opened by MICHELLE MIDTERM 1 -
human body known to generate carbon monoxide as one endoplasmic reticulum enzyme, heme oxygenase. JAY AUG 2013
of its products. Which of the following enzymes FRANCISC
represents the CO-producing reaction? O, MD (TOP
A. Biliverdin reductase 9 - FEB
B. Coproporphyrinogen oxidase 2013 MED
C. Heme oxygenase BOARDS;
D. Protoporphyrin oxidase TOPNOTCH
E. Uroporphysin decarboxylase MD)

700 Obesity, genetic profile and aging all contribute to the The overall level of significant influence is at the level MICHELLE MIDTERM 1 -
development of Type II DM. Which is the most important of skeletal muscle sensitivity to the action of insulin. JAY AUG 2013
additive factor for these 3 conditions in the development FRANCISC
of type II DM? O, MD (TOP
A. Inc.hepatic gluconeogenesis 9 - FEB
B. Inc. Pancreatic glucagon secretion 2013 MED
C. Impaired renal clearance of glucose BOARDS;
D. Inc. Adipose tissue activity TOPNOTCH
E. Muscle resistance to insulin MD)

TOPNOTCH MEDICAL BOARD PREP BIOCHEMISTRY SUPEREXAM Page 90 of 92


For inquiries visit www.topnotchboardprep.com.ph or email us at topnotchmedicalboardprep@gmail.com
TOPNOTCH MEDICAL BOARD PREP BIOCHEMISTRY SUPEREXAM
For inquiries visit www.topnotchboardprep.com.ph or email us at topnotchmedicalboardprep@gmail.com
Item # KEY 88 B 177 A 266 B 355 A
ANSWER 89 A 178 E 267 D 356 B
1 D 90 B 179 E 268 E 357 C
2 A 91 C 180 B 269 B 358 D
3 A 92 B 181 B 270 D 359 C
4 D 93 D 182 D 271 D 360 D
5 A 94 C 183 E 272 D 361 D
6 B 95 C 184 C 273 A 362 D
7 C 96 A 185 E 274 D 363 D
8 B 97 E 186 D 275 C 364 C
9 A 98 C 187 E 276 B 365 D
10 B 99 C 188 C 277 B 366 A
11 A 100 A 189 A 278 A 367 A
12 C 101 D 190 C 279 D 368 B
13 D 102 E 191 C 280 C 369 D
14 A 103 A 192 D 281 B 370 B
15 A 104 B 193 B 282 E 371 D
16 A 105 C 194 A 283 C 372 D
17 C 106 B 195 B 284 A 373 B
18 B 107 D 196 A 285 C 374 C
19 D 108 C 197 E 286 A 375 D
20 C 109 A 198 D 287 D 376 A
21 D 110 A 199 C 288 C 377 D
22 B 111 D 200 D 289 B 378 B
23 B 112 B 201 A 290 E 379 E
24 A 113 E 202 C 291 D 380 E
25 D 114 D 203 B 292 C 381 D
26 E 115 E 204 B 293 B 382 C
27 D 116 B 205 A 294 B 383 D
28 C 117 A 206 C 295 E 384 B
29 E 118 D 207 C 296 A 385 A
30 D 119 B 208 D 297 D 386 C
31 B 120 C 209 D 298 E 387 C
32 A 121 D 210 C 299 C 388 A
33 C 122 B 211 A 300 D 389 B
34 B 123 C 212 B 301 A 390 A
35 D 124 D 213 C 302 B 391 A
36 B 125 A 214 D 303 D 392 C
37 A 126 A 215 E 304 A 393 E
38 A 127 E 216 C 305 C 394 B
39 C 128 A 217 B 306 D 395 A
40 C 129 D 218 D 307 D 396 A
41 E 130 A 219 C 308 A 397 E
42 E 131 D 220 B 309 B 398 C
43 A 132 A 221 C 310 D 399 D
44 B 133 C 222 B 311 A 400 C
45 B 134 A 223 A 312 B 401 C
46 D 135 C 224 A 313 C 402 E
47 C 136 B 225 B 314 D 403 A
48 B 137 A 226 A 315 C 404 B
49 B 138 C 227 B 316 B 405 D
50 A 139 E 228 D 317 A 406 C
51 C 140 C 229 B 318 D 407 D
52 B 141 A 230 B 319 A 408 B
53 A 142 C 231 A 320 C 409 A
54 B 143 B 232 A 321 A 410 D
55 E 144 C 233 B 322 D 411 C
56 A 145 D 234 C 323 D 412 C
57 D 146 E 235 C 324 D 413 D
58 A 147 B 236 A 325 A 414 B
59 C 148 D 237 C 326 E 415 C
60 D 149 B 238 A 327 D 416 A
61 B 150 A 239 A 328 D 417 B
62 E 151 A 240 B 329 C 418 B
63 B 152 D 241 B 330 E 419 D
64 C 153 B 242 A 331 D 420 E
65 A 154 D 243 D 332 C 421 B
66 B 155 A 244 A 333 C 422 C
67 D 156 B 245 B 334 C 423 A
68 D 157 D 246 C 335 C 424 D
69 B 158 D 247 D 336 B 425 C
70 C 159 D 248 E 337 D 426 A
71 E 160 A 249 C 338 A 427 B
72 A 161 B 250 D 339 D 428 D
73 C 162 A 251 E 340 B 429 D
74 E 163 A 252 B 341 C 430 D
75 D 164 D 253 D 342 A 431 A
76 D 165 C 254 B 343 B 432 B
77 E 166 A 255 A 344 C 433 A
78 A 167 C 256 A 345 B 434 C
79 B 168 C 257 B 346 B 435 C
80 E 169 E 258 C 347 D 436 B
81 B 170 B 259 B 348 A 437 C
82 D 171 A 260 D 349 C 438 D
83 B 172 D 261 B 350 D 439 B
84 B 173 B 262 A 351 D 440 E
85 D 174 C 263 C 352 B 441 E
86 A 175 D 264 B 353 B 442 C
87 D 176 E 265 A 354 A 443 C
TOPNOTCH MEDICAL BOARD PREP BIOCHEMISTRY SUPEREXAM Page 91 of 92
For inquiries visit www.topnotchboardprep.com.ph or email us at topnotchmedicalboardprep@gmail.com
TOPNOTCH MEDICAL BOARD PREP BIOCHEMISTRY SUPEREXAM
For inquiries visit www.topnotchboardprep.com.ph or email us at topnotchmedicalboardprep@gmail.com
444 D 533 A 622 B
445 B 534 B 623 B
446 B 535 D 624 B
447 D 536 B 625 C
448 C 537 A 626 A
449 B 538 C 627 E
450 A 539 B 628 D
451 A 540 A 629 E
452 B 541 E 630 C
453 C 542 D 631 C
454 A 543 C 632 B
455 D 544 D 633 E
456 D 545 C 634 A
457 C 546 E 635 E
458 B 547 E 636 C
459 B 548 E 637 E
460 E 549 C 638 B
461 A 550 E 639 E
462 E 551 D 640 C
463 B 552 C 641 A
464 B 553 C 642 C
465 A 554 E 643 B
466 B 555 D 644 A
467 A 556 A 645 D
468 C 557 E 646 B
469 B 558 A 647 D
470 A 559 B 648 C
471 D 560 B 649 B
472 A 561 A 650 B
473 B 562 D 651 A
474 D 563 C 652 B
475 C 564 C 653 E
476 C 565 A 654 B
477 A 566 A 655 E
478 B 567 D 656 A
479 B 568 C 657 C
480 C 569 C 658 C
481 B 570 A 659 E
482 C 571 A 660 A
483 C 572 A 661 A
484 A 573 A 662 A
485 C 574 B 663 C
486 C 575 C 664 A
487 C 576 C 665 A
488 A 577 A 666 A
489 D 578 B 667 D
490 B 579 A 668 C
491 A 580 D 669 D
492 A 581 D 670 B
493 C 582 C 671 A
494 D 583 D 672 B
495 D 584 E 673 A
496 C 585 E 674 B
497 D 586 D 675 C
498 D 587 A 676 B
499 C 588 C 677 D
500 B 589 D 678 B
501 A 590 C 679 C
502 D 591 A 680 A
503 C 592 A 681 A
504 B 593 A 682 C
505 D 594 B 683 B
506 E 595 D 684 A
507 B 596 E 685 B
508 D 597 D 686 C
509 A 598 D 687 B
510 B 599 E 688 A
511 E 600 B 689 B
512 D 601 C 690 C
513 D 602 D 691 A
514 B 603 B 692 D
515 E 604 C 693 B
516 A 605 D 694 A
517 D 606 A 695 D
518 A 607 E 696 C
519 E 608 D 697 C
520 C 609 A 698 C
521 D 610 B 699 C
522 C 611 E 700 E
523 B 612 B
524 C 613 D
525 A 614 B
526 D 615 A
527 D 616 D
528 D 617 E
529 C 618 D
530 A 619 A
531 C 620 D
532 B 621 B
TOPNOTCH MEDICAL BOARD PREP BIOCHEMISTRY SUPEREXAM Page 92 of 92
For inquiries visit www.topnotchboardprep.com.ph or email us at topnotchmedicalboardprep@gmail.com

Вам также может понравиться